0% found this document useful (0 votes)
45 views183 pages

Pastest-Basic Sciences PDF

Uploaded by

abdul wakeel
Copyright
© © All Rights Reserved
We take content rights seriously. If you suspect this is your content, claim it here.
Available Formats
Download as PDF, TXT or read online on Scribd
0% found this document useful (0 votes)
45 views183 pages

Pastest-Basic Sciences PDF

Uploaded by

abdul wakeel
Copyright
© © All Rights Reserved
We take content rights seriously. If you suspect this is your content, claim it here.
Available Formats
Download as PDF, TXT or read online on Scribd
You are on page 1/ 183

Elzohry MRCP Questions Bank (Part 1) – 2013 (For my personal use)

[ Q: 1269 ] PasTest 2009 - Basic 1- Kappa light chains in the urine (Bence Jones
Science protein)
A patient presents with eczema, 2- Normal IgG and IgM
thrombocytopenia and recurrent infections. 3- Reduced plasma viscosity
What is the most likely diagnosis? 4- Elevated b 2-glycoprotein-1
1- Wiskott-Aldrich syndrome 5- Elevated serum creatinine
2- Hyper-IgE syndrome
3- Gaucher's disease Answer & Comments

4- IgA deficiency Answer: 1- Kappa light chains in the urine


(Bence Jones protein)
5- Malignancy
The finding of light chains in urine is
Answer & Comments suggestive of a malignant process. Although
low levels of IgG and IgM suggest malignancy,
Answer: 1- Wiskott-Aldrich syndrome values in the normal range do occur,
This X-linked disease, characterised by especially during the early stages of the
eczema, thrombocytopenia and repeated process. Renal impairment has no
infections, is caused by mutations in the WASP discriminatory value. Elevated serum b 2-
gene. The WASP protein is expressed in cells microglobulin levels have prognostic
of all haematopoietic lineages. It may serve a significance in myeloma, but serum b 2-
cytoskeletal organising role for signalling glycoprotein-1 is associated with the
elements that are particularly important in antiphospholipid antibody syndrome. Plasma
platelets and T cells. The platelets are small viscosity is elevated particularly when
and have a shortened half-life. Affected male paraproteins of the IgM and also IgA classes
infants often present with bleeding and most are present.
do not survive childhood, dying of
complications of bleeding, infection or [ Q: 1271 ] PasTest 2009 - Basic
lymphoreticular malignancy. The Science
immunological defects include low serum
In the investigation of a woman presenting
concentrations of IgM, while IgA and IgG are
with jaundice and itching, which one of the
normal and IgE is frequently increased. The
following would most strongly support the
number and class distribution of B
diagnosis of primary biliary cirrhosis (PBC)?
lymphocytes are usually normal.
1- Raised serum IgM
[ Q: 1270 ] PasTest 2009 - Basic 2- Positive antinuclear antibodies
Science 3- Antimitochondrial antibodies (M2 pattern)
You identify an IgAk paraprotein in an elderly 4- Raised serum IgA
male and need to decide whether this is part
5- Anti-smooth muscle antibodies
of an underlying malignancy such as myeloma,
or a monoclonal gammopathy of
underdetermined significance. Answer & Comments

Which of the following tests most supports a Answer: 3- Antimitochondrial antibodies (M2
malignant process? pattern)

Dr. Khalid Yusuf El-Zohry – Sohag Teaching Hospital (01118391123)


Ref MRCPass OE OE 2012 PasTest 2009 PassMedicine 2009 PasTest Exam ReviseMRCP 586
Elzohry MRCP Questions Bank (Part 1) – 2013 (For my personal use)

M2 AMA is highly indicative of PBC. A fourth branchial arches, and therefore the
polyclonal increase in IgM (sometimes thymus and parathyroid, fail to develop. As a
associated with elevated IgG) is typical but not result, cardiac outflow tract anomalies occur
specific for PBC. Raised serum IgA and anti- (interrupted aortic arch, truncus arteriosus
smooth muscle antibodies would not support and teratology of Fallot).
a diagnosis of PBC.
[ Q: 1273 ] PasTest 2009 - Basic
[ Q: 1272 ] PasTest 2009 - Basic Science
Science
A 22-year-old woman with partial
Which of the following diseases is correctly lipodystrophy presents with a 3- month
matched to the immunodeficiency? history of increasing swelling of her legs,
1- Ataxia-telangiectasia - absent NBT which is now up to her knees. Urinalysis shows
(neutrophil nitroblue tetrazolium) heavy proteinuria but no haematuria. A
reduction diagnosis of nephrotic syndrome is therefore
made.
2- Bruton's disease - impaired phagocytosis
Which investigation is most likely to help in the
3- Chronic granulomatous disease (CGD) - definition of the underlying renal pathology?
hypogammaglobulinaemia
1- Serum immunoglobulins
4- Chèdiak-Higashi - reduced IgA levels
2- Complement studies
5- DiGeorge syndrome - absent T-cell function
3- Antineutrophil cytoplasmic antibodies
(ANCA)
Answer & Comments
4- Antiglomerular basement membrane
Answer: 5- DiGeorge syndrome - absent T-cell antibodies (anti-GBM)
function
5- Antistreptolysin titre (ASOT)
Ataxia-telangiectasia presents in childhood
with cerebellar ataxia, impaired cell-mediated Answer & Comments
immunity and productions of antibody. It is an
autosomal-recessive condition. Bruton's Answer: 2- Complement studies
disease is an X-linked Nephrotic syndrome in an individual with
hypogammaglobulinaemia due to absent partial lipodystrophy is likely to be
mature B cells. Recurrent pyogenic infections mesangiocapillary nephritis with C3 nephritic
occur once maternal antibody levels fall. CGD factor. The latter is an autoantibody, and
(chronic granulomatous disease) is caused by interacts with the complement pathway to
a failure of intracellular killing (no respiratory give a low C3 with a normal C4 level. Serum
burst). There are various types. Screening is by immunoglobulins are likely to show reduced
the nitroblue tetrazolium (NBT) test. IgG and raised IgM levels, which is typical of
Ch‫©أ‬diak-Higashi is a disorder affecting the nephrotic syndrome. Anti-GBM antibodies
neutrophil chemotaxis and is inherited as an are likely to be negative as the patient is
autosomal-recessive. DiGeorge syndrome young and there is no haematuria or
consists of hypoparathyroidism pulmonary involvement.
(hypocalcaemic convulsions in newborns),
cardiac anomalies, abnormal facies and absent
T-cell function (impaired cell-mediated
immunity). In this condition the third and

Dr. Khalid Yusuf El-Zohry – Sohag Teaching Hospital (01118391123)


Ref MRCPass OE OE 2012 PasTest 2009 PassMedicine 2009 PasTest Exam ReviseMRCP 587
Elzohry MRCP Questions Bank (Part 1) – 2013 (For my personal use)

[ Q: 1274 ] PasTest 2009 - Basic Answer & Comments


Science
Answer: 3- Patch testing
A 16-year-old youth presents with lethargy
Patch testing is the classical method for
and abdominal bloating. He has lost 5 kg in
investigating contact dermatitis (which is T-
weight over the last 6 months. Examination
lymphocyte mediated). Skin-prick testing
confirms a thin teenager with obvious pallor.
investigates IgE-mediated reactions, typically
Which is the most appropriate serological test to aeroallergens and food. Skin prick testing
to investigate possible malabsorption? with latex extracts is sensitive, specific, and
1- IgG antireticulin antibodies rapid; however, it carries the risk of
anaphylaxis . The atopy patch test is a
2- IgG antiendomysial antibodies
3- IgA antigliadin antibodies
4- IgA enterocyte antibodies
5- IgA antiendomysial antibodies

Answer & Comments


Answer: 5- IgA antiendomysial antibodies

Small-bowel biopsy is the ‘gold standard'


investigation for coeliac disease. However,
serological tests such as IgA antiendomysial
antibodies are the most discriminating of the
above choices. The newer test becoming
increasingly available is IgA
antitransglutaminase. IgG antiendomyseal
antibodies can be of value in the diagnosis of
coeliac disease in IgA-deficient individuals, ie
they will be negative in the IgA
antiendomyseal antibody test.

[ Q: 1275 ] PasTest 2009 - Basic


Science
A man presents with an eczematous rash on
his hands suggestive of contact dermatitis,
possibly related to wearing latex gloves at
work. He requests confirmatory tests.
Which of the following tests is most likely to
be helpful establishing the diagnosis?
1- Atopy patch testing
2- Skin-prick testing
3- Patch testing
4- Intradermal testing
5- Direct exposure tests to gloves

Dr. Khalid Yusuf El-Zohry – Sohag Teaching Hospital (01118391123)


Ref MRCPass OE OE 2012 PasTest 2009 PassMedicine 2009 PasTest Exam ReviseMRCP 588
Elzohry MRCP Questions Bank (Part 1) – 2013 (For my personal use)

predisposing to thrombosis) are a [ Q: 1278 ] PasTest 2009 - Basic


consequence of renal loss of these substances Science
(in the former case, because it is largely
A patient presents with facial
protein-bound). LDL cholesterol (and
abnormalities that may include abnormal ears,
triglyceride) concentrations are frequently
a shortened philtrum, micrognathia and
elevated: the increased lipoprotein
hypertelorism.
contributes to an increased beta-globulin
band. Which cells is this patient lacking?
1- T lymphocytes
[ Q: 1277 ] PasTest 2009 - Basic 2- B lymphocytes
Science
3- Erythrocytes
A 72-year-old man presents with weight loss,
4- Melanocytes
low-grade fever, mononeuritis and
hypertension. You consider the diagnosis of 5- Leucocytes
polyarteritis nodosa (PAN) and request certain
immunological investigations. Answer & Comments
Which one of the following would best support Answer: 1- T lymphocytes
your clinical diagnosis?
This classic example of isolated T-cell
1- Positive c-ANCA
deficiency results from maldevelopment of
2- Positive p-ANCA thymic epithelial elements derived from the
3- ANCA-negative third and fourth pharyngeal pouches. The
gene defect has been mapped to chromosome
4- Positive ANA
22q11 in most patients with the DiGeorge
5- Positive thyroid microsomal antibodies syndrome, and to chromosome 10p in others.
Signs of defective development of organs
Answer & Comments dependent on cells of embryonic neural crest
origin include: congenital cardiac defects,
Answer: 3- ANCA-negative particularly those involving the great vessels;
In PAN (medium-sized arteritis), the ANCA hypocalcaemic tetany, due to failure of
(anti-neutrophil cytoplasmic antibody) is parathyroid development; and absence of a
usually negative. Perinuclear (p)-ANCA and normal thymus. Facial abnormalities may
cytoplasmic (c)-ANCA are associated with the include abnormal ears, a shortened philtrum,
small-vessel vasculitides, namely microscopic micrognathia and hypertelorism. Serum
polyangiitis and Wegener’s granulomatosus, immunoglobulin concentrations are
respectively. A positive antinuclear antibody frequently normal, but antibody responses,
(ANA) in a 72-year-old man is unlikely to be particularly of IgG and IgA isotypes, are usually
relevant, and positive thyroid autoantibodies impaired. T-cell levels are reduced, whereas B-
should lead to thyroid function testing, cell levels are normal. Affected individuals
hypothyroidism may be a coincidental finding. usually have a small, histologically normal
Sometimes a positive ANCA is reported with a thymus located near the base of the tongue or
negative proteinase 3 (PR3)-ANCA (PR3) and in the neck, allowing most patients to develop
negative myeloperoxidase (MPO)-ANCA. This functional T cells in numbers that may or may
result can occur in various inflammatory and not be adequate for host defence.
infective conditions and is not necessarily
associated with vasculitis.

Dr. Khalid Yusuf El-Zohry – Sohag Teaching Hospital (01118391123)


Ref MRCPass OE OE 2012 PasTest 2009 PassMedicine 2009 PasTest Exam ReviseMRCP 589
Elzohry MRCP Questions Bank (Part 1) – 2013 (For my personal use)

[ Q: 1279 ] PasTest 2009 - Basic 4- IgG


Science 5- IgM
A 22-year-old woman presents to
Accident & Emergency with an acute Answer & Comments
hereditary angioedema involving the larynx.
Answer: 4- IgG
What is the optimum immediate
management? IgG is the major serum immunoglobin, while
IgA is the predominant immunoglobulin in
1- Antihistamines
secretions and so protects mucous
2- IV hydrocortisione membranes.
3- Fresh frozen plasma
4- C1 inhibitor concentrate [ Q: 1281 ] PasTest 2009 - Basic
Science
5- Recombinant C1 inhibitor
Immunoglobulin structure - which of the
following regions forms the antigen binding
Answer & Comments
site?
Answer: 4- C1 inhibitor concentrate
1- The heavy chain
This condition is due to deficiency of C1 2- The light chain
inhibitor. This causes uncontrolled activation
3- The constant region of one heavy and one
of the classical complement pathway,
light chain
resulting in angioedema. Patients often
present with orofacial swelling, which can 4- The hinge region
cause respiratory obstruction and abdominal 5- The variable region of one heavy and one
pain. They do not usually get urticaria or light chain
pruritis (this is not a type-I hypersensitivity
reaction). Hereditary angioedema is more
Answer & Comments
refractory to the use of subcutaneous
epinephrine, antihistamines, and steroids. C4 Answer: 5- The variable region of one heavy
levels fall during an acute attack, while C3 and one light chain
levels may remain stable. It needs to be
The antigen binding site is situated within the
treated with the inhibitor, present in fresh
variable region of the immunoglobulin
frozen plasma or C1 inhibitor concentrate;
structure, made up of both the heavy and light
concentrate is the preferred option.
chains, specifically within the hypervariable
Recombinant preparations are not yet in
regions.
routine clinical use.

[ Q: 1282 ] PasTest 2009 - Basic


[ Q: 1280 ] PasTest 2009 - Basic
Science
Science
Which immunoglobulin can fix complement via
Which of the following immunoglobulin
the alternative pathway?
isotypes has the highest concentration in
serum? 1- IgA
1- IgA 2- IgM
2- IgD 3- IgG
3- IgE 4- IgE

Dr. Khalid Yusuf El-Zohry – Sohag Teaching Hospital (01118391123)


Ref MRCPass OE OE 2012 PasTest 2009 PassMedicine 2009 PasTest Exam ReviseMRCP 590
Elzohry MRCP Questions Bank (Part 1) – 2013 (For my personal use)

5- IgD Which of the following diseases is most closely


associated with HLA B5?
Answer & Comments 1- Dermatitis herpetiformis
Answer: 1- IgA 2- Behçet's syndrome

IgA is unusual in that it can fix complement via 3- Grave's disease


the alternative pathway. IgG and IgM can fix 4- Addison's disease
complement via the classical pathway through
5- Sjögren's syndrome
the Fc portion of the immunoglobulin.

Answer & Comments


[ Q: 1283 ] PasTest 2009 - Basic
Science Answer: 2- Behçet's syndrome

A medical SHO is required to give a blood The major histocompatibility complex (MHC)
sample to check his HepB status. He received is a cluster of genes located on the short arm
a course of vaccinations nine months ago. of chromosome 6, and it codes for a series of
What is his blood test likely to show? molecules known as the ‘human leucocyte
antigens' (HLA). HLA molecules are distributed
1- Anti-HBeAb throughout the body, and it is through
2- Anti-HBsAb differential HLA expression that cells are
classified as ‘self' or ‘non-self'. Class-I
3- Anti-HBsAb + anti-HBcAb
molecules (subtypes A, B and C) are expressed
4- HBsAg + HBcAg on all cell types except erythrocytes and
5- IgM to HBcAg trophoblasts. They interact with CD8-positive
T cells and are involved in driving cytotoxic
Answer & Comments reactions.

Answer: 2- Anti-HBsAb Behçet's syndrome is associated with HLA B5


positivity, as is polycystic kidney disease and
Surface and core antigens (HBsAg, HBcAg) are ulcerative colitis. Both HLA-B5 and HLA-B51
detectable during acute infection. HbeAg confer an increased risk of developing the
(envelope) is a good marker of high infectivity, Behçet's syndrome and are more common in
while anti-HbeAg suggests a patient who is persons of Japanese and Mediterranean
less infective. Acute infection is also implied descent, with an incidence 3-6 times that of
by IgM to HbcAg, while IgG to HBcAg suggests the regular population. All the other stems
a previous infection. Viral clearance and mentioned in this question are associated
recovery correlate with the disappearance of with the B8-DR3 subtype, as are autoimmune
antigens and the appearance of antibodies. hepatitis, idiopathic membranous
Previous vaccination is suggested by the glomerulonephritis, myasthenia gravis and
presence of only anti-HBsAb. systemic lupus erythematosus.

[ Q: 1284 ] PasTest 2009 - Basic [ Q: 1285 ] PasTest 2009 - Basic


Science Science
You are reviewing a man who has paid Which of the following statements concerning
privately for HLA tissue typing. He is found to hyperacute rejection after renal
be HLA B5-positive on typing.

Dr. Khalid Yusuf El-Zohry – Sohag Teaching Hospital (01118391123)


Ref MRCPass OE OE 2012 PasTest 2009 PassMedicine 2009 PasTest Exam ReviseMRCP 591
Elzohry MRCP Questions Bank (Part 1) – 2013 (For my personal use)

Low-titre antinuclear antibodies are often a [ Q: 1292 ] PasTest 2009 - Basic Science
normal finding in middle-aged and older
Susceptiblity to Pneumocystis jiroveci can be
women.
associated with high levels of:
1- IgM
[ Q: 1291 ] PasTest 2009 - Basic
Science 2- IgG

A young woman with known systemic lupus 3- IgA


erythematosus, complicated by an 4- IgE
autoimmune neutropenia, is referred for
5- IgD
consideration of her therapeutic options. The
initial choice of treatment would have been
azathioprine; however, monitoring of the drug Answer & Comments
therapy is likely to be highly problematic since Answer: 1- IgM
this patient is neutropenic. A cytotoxic agent
selective for lymphocytes is therefore Hyper-IgM syndrome, or CD40 ligand
preferable, to circumvent such problems. deficiency, is an X-linked condition presenting
in a similar way to X-linked
Which agent best fits her requirements?
agammaglobulinaemia with recurrent
1- Mycophenolate mofetil sinpulmonary disease. Levels of IgG, IgA and
2- Sirolimus IgE are undetectable, but there are normal or
high levels of IgM and IgD. This is due to the
3- Chlorambucil
role of CD40 in B-cell maturation and isotype
4- Ciclosporin switching. This is a T-cell defect and affected
5- Thalidomide individuals are also susceptible to P. jiroveci
pneumonia. They can develop chronic
cryptosporidial infection, leading to sclerosing
Answer & Comments
cholangitis and liver failure. They have an
Answer: 1- Mycophenolate mofetil increased risk of malignancy, particularly
abdominal cancers.
Azathioprine is a cytotoxic drug that functions
unselectively by blocking purine metabolism.
Mycophenolate mofetil is an antipurine drug [ Q: 1293 ] PasTest 2009 - Basic
but, because other populations of cells have a Science
salvage pathway and hence bypass its effects, A 20-year-old woman complains of an
this drug is selective for lymphocytes. immediate intense itching in her throat when
Sirolimus is a non-calcineurin-dependent, T- eating apples, but says that she can eat
cell inhibitor; in contrast, ciclosporin is a cooked ones. She mentions that she has
calcineurin-dependent, T-cell inhibitor. allergic rhinitis but the current symptoms
Neither sirolimus nor ciclosporin are cytotoxic occur in April and May and not the typical
drugs. Thalidomide functions as an June/July period associated with grass pollen.
antiangiogenesis agent and also blocks
What is the most likely diagnosis?
tumour-necrosis factor-alpha (TNF-a ).
Chlorambucil is an unselective alkylating agent 1- Birch-associated oral allergy syndrome
and would therefore be contraindicated in this 2- Food intolerance
patient because of her neutropenia.
3- Allergy to the wax coating on apples
4- Latex allergy

Dr. Khalid Yusuf El-Zohry – Sohag Teaching Hospital (01118391123)


Ref MRCPass OE OE 2012 PasTest 2009 PassMedicine 2009 PasTest Exam ReviseMRCP 594
Elzohry MRCP Questions Bank (Part 1) – 2013 (For my personal use)

5- Salicylate sensitivity onset that usually exceeds 24 hours.


Complement-mediated (type-2
Answer & Comments hypersensitivity) and immune complex (type-3
hypersensitivity) reactions typically produce
Answer: 1- Birch-associated oral allergy vascular damage, and therefore bullae and
syndrome petechiae would predominate in the skin.
The birch pollen-induced oral allergy
syndrome occurs with stoned fruits, apples, [ Q: 1295 ] PasTest 2009 - Basic
carrots and potatoes. However, this only Science
happens with the raw form as cooking
During the last trimester, IgG is actively
denatures the allergen. The birch-tree pollen
transported across the placenta to supply
season is usually in April/May, giving the
passive immunity to the fetus.
typical rhinitis symptoms. Actual throat
swelling is unusual. Immediate symptoms Which disease occurring during pregnancy is
(minutes) are untypical of food intolerance most likely to lead to the neonate having low
and the wax coating on apples is not a cause immunoglobulin levels and hence being prone
of allergies. Latex allergy can be associated to bacterial infections?
with certain foods such as bananas, avocado, 1- Intestinal lymphangiectasia
kiwi and melon, but this allergen is heat-
2- Systemic lupus erythematosus
stable. Most apples contain a considerable
amount of salicylate, which can induce 3- Myasthenia gravis
urticaria in aspirin-sensitive individuals; 4- Ulcerative colitis
however, this is not usually associated with
5- Prematurity
pharyngeal itching.

Answer & Comments


[ Q: 1294 ] PasTest 2009 - Basic
Science Answer: 1- Intestinal lymphangiectasia
A 42-year-old, atopic, health-care worker Typically, intestinal lymphangiectasia has its
presents with red weals and itchy hands onset during pregnancy, and is sometimes
within 20 minutes of wearing latex gloves. diagnosed when the neonate is investigated
Which of the following mechanisms is most for bacterial infection. Although systemic
likely to be relevant? lupus erythematosus with severe nephrotic
syndrome could also produce
1- Contact dermatitis hypogammaglobulinaemia, this would be a
2- Complement-mediated rare occurrence. Myasthenia gravis due to the
3- Immune complex-mediated transfer of IgG across the placenta can result
in neonatal myasthenia. Significant protein
4- Delayed-type hypersensitivity loss from the gut in ulcerative colitis is highly
5- IgE-mediated sensitivity unusual. Prematurity does reduce the time for
placental transfer of IgG whilst the fetus is in
Answer & Comments utero and hence produces a so-called
‘physiological hypogammaglobulinaemia', but
Answer: 5- IgE-mediated sensitivity this is not a disease.
This is an immediate hypersensitivity reaction
and hence will be IgE-mediated. Delayed-type
hypersensitivity, eg contact dermatitis, has an

Dr. Khalid Yusuf El-Zohry – Sohag Teaching Hospital (01118391123)


Ref MRCPass OE OE 2012 PasTest 2009 PassMedicine 2009 PasTest Exam ReviseMRCP 595
Elzohry MRCP Questions Bank (Part 1) – 2013 (For my personal use)

[ Q: 1296 ] PasTest 2009 - Basic [ Q: 1297 ] PasTest 2009 - Basic Science


Science
An 82-year-old woman, who is not on drug
A 48-year-old man went up in a hot therapy, presents with a bullous skin rash on
air balloon without sufficient clothes to keep her arms.
him warm. In the evening he presents with Which of the following tests is most likely to
acute renal failure and a purpuric/vasculitic supply the definitive diagnosis?
rash on his legs.
1- Search for anti-skin antibodies in blood
Which of the following investigations is likely
to be most helpful in identifying the process 2- Complement studies
leading to the above? 3- Antineutrophil cytoplasmic antibodies
1- Cold agglutinins (ANCA)

2- Cryoglobulins 4- Skin biopsy for examination by


immunofluorescence
3- Cryofibrinogens
5- Immunoglobulins
4- Plasma viscosity
5- Paroxysmal cold haemoglobinuria Answer & Comments
Answer: 4- Skin biopsy for examination by
Answer & Comments
immunofluorescence
Answer: 2- Cryoglobulins
This is a case of pemphigoid. Both direct and
Cryoglobulins are cold precipitable indirect immunofluorescence studies on a skin
immunoglobulins that are present in blood. biopsy would help make the diagnosis. The
Those that precipitate in the above manner finding of linear IgG, and often C3 deposits at
are usually monoclonal IgM proteins present the dermoepidermal junction (basement
in high concentrations. Such monoclonal membrane) in a perilesional skin biopsy would
proteins are usually associated with support the diagnosis of pemphigoid and help
lymphoproliferative disorders, eg distinguish it from pemphigus, in which the
Waldenstrom's macroglobulinaemia or mixed IgG deposits are on cell surfaces
essential cryoglobulinaemia (which can be ("intraepidermal"). ANCA would be typically
associated with hepatitis C). Because they are negative and serum/plasma complement
slow to solubilise once they have undergone studies would be normal. Anti-skin antibodies
cold precipitation, the cryoglobulins clog up can be detected in the serum of patients with
the capillaries of the skin and kidneys. pemphigus and are reported to correlate with
disease activity. Serum immunoglobulins
Cold agglutinins typically occur with
would be unremarkable.
mycoplasma infections and, like
hyperviscosity, cause cerebral symptoms.
Cryofibrinogens rarely cause renal failure. [ Q: 1298 ] PasTest 2009 - Basic
Paroxysmal cold haemoglobinuria would be Science
confirmed by finding haemoglobin in the A pregnant woman in her second trimester
patient's urine. presents with a recent onset of a bullous rash
on her lower anterior abdominal wall. She has
no other clinical features.
Which of the following bullous conditions is
the most likely diagnosis?

Dr. Khalid Yusuf El-Zohry – Sohag Teaching Hospital (01118391123)


Ref MRCPass OE OE 2012 PasTest 2009 PassMedicine 2009 PasTest Exam ReviseMRCP 596
Elzohry MRCP Questions Bank (Part 1) – 2013 (For my personal use)

1- Pemphigus remaining 15%. C4 levels usually remain low


2- Pemphigoid foliaceus even between attacks, whereas C3 levels
remain normal. The measurement of C4 is
3- Pemphigoid gestationis therefore helpful in identifying those 15% of
4- Drug-associated rash patients who have a normal amount of C1
5- Flea bites inhibitor which functions poorly but gives
normal levels when measured. Paraproteins
can occasionally bind C1 inhibitor to create a
Answer & Comments functional deficiency. ACE inhibitors may
Answer: 3- Pemphigoid gestationis exacerbate the angioedema in HAE and hence
should be avoided. Urticaria is never a feature
As the rash is in the typical site and its usual of HAE. A drug history would be useful but
onset occurs during the second and third would not identify the HAE condition.
trimesters, this woman is likely to have
pemphigoid gestationis. A perilesional skin
[ Q: 1300 ] PasTest 2009 - Basic
biopsy showing linear C3 at the
Science
dermoepidermal junction would confirm the
diagnosis. Drug-associated rashes and flea Regarding the clinical physiology of the
bites would be part of the differential adrenal gland in Cushing's disease, which of
diagnosis of any bullous disorder, but they are the following pertains?
less likely in the case presented here.
1- The zona glomerulosa of the cortex is
Pemphigus affects mucous membranes in
predominantly responsible for sex steroid
approximately 50% of cases.
production
2- The zona fasciculata is predominantly
[ Q: 1299 ] PasTest 2009 - Basic
controlled by ACTH and is often
Science
hypertrophied
In a patient with suspected hereditary angio- 3- The zona reticularis is predominantly
oedema (HAE), which of the following is most responsible for mineralocorticoid
likely to be helpful in identifying this clinical production
condition?
4- About 15% of glucocorticoid production
1- Persistently low C3 levels, including takes place in the adrenal medulla
between attacks
5- The zona fasciculata is primarily responsible
2- Persistently low C4 levels, including for mineralocorticoid production
between attacks
3- Search for a paraprotein Answer & Comments
4- Drug history of taking ACE inhibitors
Answer: 2- The zona fasciculata is
5- Presence of urticaria with angio- predominantly controlled by ACTH and is
oedematous swellings often hypertrophied

The zona glomerulosa of the cortex is


Answer & Comments predominantly responsible for
Answer: 2- Persistently low C4 levels, including mineralocorticoid production, the zona
between attacks fasciculata for glucocorticoid production and
the zona reticularis for sex corticoid
A lack of C1 inhibitor occurs in approx 85% of production. The adrenal medulla originates
cases of HAE and a functional deficiency in the

Dr. Khalid Yusuf El-Zohry – Sohag Teaching Hospital (01118391123)


Ref MRCPass OE OE 2012 PasTest 2009 PassMedicine 2009 PasTest Exam ReviseMRCP 597
Elzohry MRCP Questions Bank (Part 1) – 2013 (For my personal use)

2- Cytotoxic T cells are activated before 4- The normal colloid oncotic pressure is 70
natural killer cells during the course of mmHg
infection 5- Albumin is indicated
3- Viruses stimulate the non-immune cells
that they infect to produce interferon-g Answer & Comments
4- Non-enveloped viruses are susceptible to
Answer: 1- Certain intravenous solutions,
damage by complement
which would be hypo-osmolar, have dextrose
5- Influenza virus can avoid antibody added to ensure they are iso-osmolar
recognition by mutational changes in its
nucleocapsid proteins Hypovolaemia is an important factor
contributing to shock and tissue hypoxia;
therefore, all patients with sepsis require
Answer & Comments supplemental fluids. The amount and rate of
Answer: 1- IgA can offer protection at mucosal infusion are guided by an assessment of the
surfaces patient's volume and cardiovascular status.
Patients withs suspected septic shock require
Natural killer cells are activated faster than an initial crystalloid fluid challenge of 20-30
cytotoxic T cells. Infected non-immune cells ml/Kg (1-2 l) over a period of 30-60 minutes.
produce interferon-a and - b, whereas Fluid resuscitation is continued until heart
interferon-g is produced by T cells. Influenza rate, urine output, and blood pressure are
virus mutates its surface neuraminidase and satisfactory and/or the pulmonary capillary
haemagglutinin to avoid antibody recognition. wedge pressure exceeds 18 mmHg. Patients
Enveloped viruses are susceptible to with septic shock often require a total of 4-6 l
complement attack. or more of crystalloid resuscitation; this need
for relatively large volumes to restore
[ Q: 1304 ] PasTest 2009 - Basic intravascular volume is a disadvantage of
Science crystalloid solutions. Colloids on the other
hand are much more expensive and colloid
A 76-year-old patient is admitted to the acute
resuscitation (with albumin or hetastarch) has
admission unit with septic shock. Pulse is 106
not previously been shown in meta-analyses
and BP 90/40 mmHg. Urinary catheterisation
to have any benefit over isotonic crystalloid
produces 75 ml of concentrated urine.
resuscitation (isotonic sodium chloride
Which of the following principles applies to the solution or lactated Ringer solution).
choice of an appropriate intravenous fluid for Hartmann's solution contains sodium,
resuscitation? potassium, chloride, calcium and lactate. It is
1- Certain intravenous solutions, which would not possible to store bicarbonate in solution
be hypo-osmolar, have dextrose added to with calcium; instead lactate is used which is
ensure they are iso-osmolar metabolised to bicarbonate by the liver.
2- Hartmann's solution contains sodium,
potassium, chloride, calcium and [ Q: 1305 ] PasTest 2009 - Basic
bicarbonate Science

3- An advantage of crystalloid solutions is that Which of the following takes place during
relatively small volumes have to be infused inspiration?
to restore an intravascular volume deficit 1- The diaphragm drops by 10 cm during
normal breathing

Dr. Khalid Yusuf El-Zohry – Sohag Teaching Hospital (01118391123)


Ref MRCPass OE OE 2012 PasTest 2009 PassMedicine 2009 PasTest Exam ReviseMRCP 599
Elzohry MRCP Questions Bank (Part 1) – 2013 (For my personal use)

Dr. Khalid Yusuf El-Zohry – Sohag Teaching Hospital (01118391123)


Ref MRCPass OE OE 2012 PasTest 2009 PassMedicine 2009 [(Pa)-8(sT)-3(E2i.61 37.824 6/F1 11 37.824 6/F1 11 37.824 6/F1.104
Elzohry MRCP Questions Bank (Part 1) – 2013 (For my personal use)

stage. A large pulsatile increase in GnRH 2- Low-frequency waves are detected in the
occurs just before ovulation. scala vestibuli
3- The scala media is filled with perilymph
[ Q: 1308 ] PasTest 2009 - Basic
4- The scala media contains the organ of Corti
Science
5- Normal hearing frequency only ranges from
A 24-year-old woman visits her GP 20 to 2000 Hz
complaining that she is tired and lethargic. He
arranges some routine blood tests including
Answer & Comments
FBC, U&E, LFT, TFT, viscosity and
immunoglobulins. The only abnormality is Answer: 4- The scala media contains the organ
elevated IgE levels to 1.2 times the upper limit of Corti
of normal.
Low-frequency waves are detected in the
Raised IgE levels are a normal finding in what scala tympani, high-frequency waves are
% of the population? detected in the scala vestibuli. Both contain
1- 2% perilymph. The scala media is filled with
potassium-rich endolymph. Normal hearing
2- 5%
frequency ranges from 20 to 20,000 Hz.
3- 2.5%
4- 1.25% [ Q: 1310 ] PasTest 2009 - Basic
5- 10% Science
Whilst treating a diabetic patient who is
Answer & Comments controlled with insulin you perform
fundoscopy and find new vessel disease in the
Answer: 3- 2.5%
macula lutea.
Reference ranges for laboratory variables are Which of the following is true of ocular
calculated based on sample values from a physiology?
series of normal volunteers. IgE levels are
normally distributed, and abnormal values are 1- The macula is rich in rods
said to lie outside 2 standard deviations either 2- This corresponds with the optic papilla
side of the mean. In a normal distribution 95% 3- The central portion of the macula is known
of values lie within 2 standard deviations as the fovea centralis
either side of the mean. As such 2.5% are
lower than 2 standard deviations less than the 4- As it corresponds with the blind spot,
mean, and 2.5% are greater than 2 standard retinopathy will have less functional effect
deviations above the mean. on acuity
5- There is a direct relationship between
[ Q: 1309 ] PasTest 2009 - Basic proliferative retinopathy and the frequency
Science of cataracts

A 41-year-old patient complains of tinnitus.


Answer & Comments
Which of the following statements is correct
regarding the clinical physiology of the ear? Answer: 3- The central portion of the macula
is known as the fovea centralis
1- High-frequency waves are detected in the
scala tympani The macula is rich in cones. The optic papilla is
the opening of the optic nerve and

Dr. Khalid Yusuf El-Zohry – Sohag Teaching Hospital (01118391123)


Ref MRCPass OE OE 2012 PasTest 2009 PassMedicine 2009 PasTest Exam ReviseMRCP 601
Elzohry MRCP Questions Bank (Part 1) – 2013 (For my personal use)

corresponds with the blind spot. The fovea concentration of carbon dioxide in the blood
centralis has the highest acuity. There is no rises, a condition known as ‘hypercapnia', the
direct relationship between proliferative central and peripheral chemoreceptors
retinopathy and the frequency of cataracts. stimulate the inspiratory areas to stimulate
the rate and depth of breathing. This
[ Q: 1311 ] PasTest 2009 - Basic hyperventilation causes more carbon dioxide
Science to be exhaled until levels return to normal.

Which of the following statements is most


[ Q: 1312 ] PasTest 2009 - Basic
accurate with regards to the chemoreceptor
Science
control of breathing?
1- The aortic body is located in the root of the Which of the following physiological
aorta characteristics relates to the lining of the
respiratory tract?
2- When the CO2 concentration of plasma
falls, the central chemoreceptors stimulate 1- About 1 litre of mucus is produced every
the inspiratory area of the brain day

3- Central chemoreceptors are sensitive to the 2- The cilia are under the control of a
H+ content of the CSF physiological motor, dynein

4- The carotid bodies are located on the 3- The mucociliary escalator moves at 0.2
external carotid artery cm/minute

5- Central chemoreceptors are sensitive to the 4- The bronchioles have cartilage in their wall
O2 content of the CSF 5- The bronchioles have diameters up to 5 mm

Answer & Comments Answer & Comments


Answer: 3- Central chemoreceptors are Answer: 2- The cilia are under the control of a
sensitive to the H+ content of the CSF physiological motor, dynein

The chemoreceptors that regulate respiration About 100 ml of mucus is produced every day.
are located both centrally and peripherally. The cilia are under the control of a
Normally control is exercised by the central physiological motor, dynein (which is absent
receptors located in the medulla, which in Kartagener's syndrome). The mucociliary
respond to the CSF hydrogen ion escalator moves at 2 cm/minute. The
concentration, in turn determined by CO2, bronchioles do not have cartilage in their wall
which diffuses freely across the blood-brain (which distinguishes them from bronchi). The
barrier from the arterial blood. The response bronchioles can be up to 1 mm in diameter.
is both quick and sensitive to small changes in
arterial CO2 (PaCO2). Whilst central [ Q: 1313 ] PasTest 2009 - Basic
chemoreceptors are therefore (indirectly) Science
sensitive to levels of carbon dioxide,
peripheral chemoreceptors are also highly In estimating the physiological clearance of 10
sensitive to oxygen. There are three clusters ml of an intravenous substance which has
of peripheral chemoreceptors: the aortic body been administered at 10 mg/ml, the plasma
located in the wall of the arch of the aorta, concentration at equilibration is 15 mg/litre,
and the two nodular carotid bodies in the left the urine concentration is 150 mg/litre and
and right common carotid arteries. When the the subject produces 1440 ml of urine during a

Dr. Khalid Yusuf El-Zohry – Sohag Teaching Hospital (01118391123)


Ref MRCPass OE OE 2012 PasTest 2009 PassMedicine 2009 PasTest Exam ReviseMRCP 602
Elzohry MRCP Questions Bank (Part 1) – 2013 (For my personal use)

24h collection. The substance is not actively 4- Only about 0.15% of oxygen is carried in
secreted or absorbed by the kidney. solution in the plasma
What is the clearance of the substance? 5- Carbon dioxide is less water-soluble than
1- 1 ml/min oxygen

2- 10 ml/min
Answer & Comments
3- 0.1 ml/min
Answer: 1- Gas exchange can occur in the final
4- 100 ml/min seven branches of the bronchoalveolar tree
5- Cannot say from the information given
Gas exchange can occur in the final seven
branches of the bronchoalveolar tree (the
Answer & Comments respiratory zone). The first 16 branches of the
Answer: 2- 10 ml/min bronchial tree are collectively known as the
conducting zone. The equilibration of gases
Clearance is calculated using the formula (U ‫—أ‬ takes about 0.25 s in the resting lung. Only
V)/P where U = urine concentration in mg/ml, about 1.5% of oxygen is carried in solution in
V = urine production in ml/min, P = plasma the plasma. Carbon dioxide is more water-
concentration in mg/ml. soluble than oxygen, between 5 and 10% of
Using the values here, this gives: CO2 is carried in dissolved form.

U=150 mg/l = 0.15mg/ml [ Q: 1315 ] PasTest 2009 - Basic


V=1440 ml/24hours =1440ml/1440 minutes Science
=1ml/min Blood is circulated through the arteries by
P 15mg/l =0.015 mg/ml which of the following physiological
mechanisms?
Therefore (UxV) = 0.15
1- Suction during cardiac diastole
So clearance = (UxV)/P = 0.15/0.015 = 2- Negative pressure during inspiration
10ml/min
3- Maintenance of diastolic pressure in the
The bolus size of the substance is irrelevant to arteries and arterioles
the clearance.
4- Contraction of external muscles
5- Residual pressure from blood flow through
[ Q: 1314 ] PasTest 2009 - Basic
the capillaries
Science
Pulmonary gas exchange occurs under which Answer & Comments
of the following physiological principles?
Answer: 3- Maintenance of diastolic pressure
1- Gas exchange can occur in the final seven
in the arteries and arterioles
branches of the bronchoalveolar tree
2- The first 12 branches of the bronchial tree Suction during cardiac diastole, negative
are collectively known as the conducting pressure during inspiration, contraction of
zone external muscles and residual pressure from
blood flow through the capillaries all promote
3- The equilibration of gases takes about 2.5 s venous blood flow. Ventricular systole and
in the resting lung recoil of the elastic aorta are additional
mechanisms of arterial circulation.

Dr. Khalid Yusuf El-Zohry – Sohag Teaching Hospital (01118391123)


Ref MRCPass OE OE 2012 PasTest 2009 PassMedicine 2009 PasTest Exam ReviseMRCP 603
Elzohry MRCP Questions Bank (Part 1) – 2013 (For my personal use)

[ Q: 1316 ] PasTest 2009 - Basic 2- Antibodies to ABO blood groups are IgG,
Science whereas antibodies to Rhesus antigens are
IgM
Which of the following is true
concerning complement activation? 3- Antibodies to ABO blood groups are IgA,
whereas antibodies to Rhesus antigens are
1- IgG and IgE are the main antibody classes
IgG
involved in classical pathway activation
4- Antibodies to Rhesus antigens are IgD,
2- C1q binds to the Fab regions of antigen-
whereas anti-ABO blood groups are IgM
complexed IgG antibodies
5- Antibodies to Rhesus antigens are IgE,
3- The alternative, but not the classical C3,
whereas anti-ABO blood groups are IgG
convertase enzyme involves C3b
4- Elevated serum C3dg is a good marker of Answer & Comments
complement activation
Answer: 1- Antibodies to ABO blood groups
5- The membrane-attack complex involves
are IgM, whereas antibodies to Rhesus
polymerisation of C7
antigens are IgG

Answer & Comments IgG antibodies to Rhesus antigens can cross


the placenta during the last trimester,
Answer: 3- The alternative, but not the whereas ABO antibodies are IgM and hence
classical C3, convertase enzyme involves C3b cannot cross the placenta. The function of
IgG and IgM are the main antibody classes serum IgD is unknown. The transplacental
that activate the classical pathway. C1q binds passage of immunoglobulin only applies to
to the Fc rather than the Fab fragment that IgG.
binds antibody (Fc, crystallisable fragment;
Fab antigen-binding fragment). It is C9 that [ Q: 1318 ] PasTest 2009 - Basic
polymerises in the membrane-attack complex. Science
Clotting of blood can lead to complement
A 25-year-old woman with systemic lupus
activation, and hence complement conversion
erythematosus (SLE) has dsDNA antibodies
products must be measured on plasma and
and a grade III glomerulonephritis.
not serum.
Which is the most likely immunopathological
[ Q: 1317 ] PasTest 2009 - Basic process?
Science 1- Activation of the alternative complement
pathway
Haemolytic disease of the newborn is typically
restricted to the presence of Rhesus antigens 2- Type-II hypersensitivity reaction
on red cells rather than ABO antigens. 3- Type-I hypersensitivity reaction
Predominantly, such anti-Rh antibodies cross
4- Activation of the classical complement
the placenta during the third trimester.
pathway
Which of the following statements best
5- A complement deficiency
explains the background physiology?
1- Antibodies to ABO blood groups are IgM,
Answer & Comments
whereas antibodies to Rhesus antigens are
IgG Answer: 4- Activation of the classical
complement pathway

Dr. Khalid Yusuf El-Zohry – Sohag Teaching Hospital (01118391123)


Ref MRCPass OE OE 2012 PasTest 2009 PassMedicine 2009 PasTest Exam ReviseMRCP 604
Elzohry MRCP Questions Bank (Part 1) – 2013 (For my personal use)

Activation of the classical complement intermittent diarrhoea. As an infant he had


pathway occurs in SLE due to the large many episodes of otitis media and at the age
number of dsDNA and other immune of 10 years was hospitalised with tonsilitis. His
complexes that form and fix complement. mother has coeliac disease and his sister
They deposit in the kidneys and other organs, autoimmune haemolytic anaemia. His full
where they attract other components of the blood count is normal.
immune system that cause tissue damage. What is his most likely diagnosis?
Complement consumption is common in
active disease. SLE could therefore also be 1- Coeliac disease
described as a type-III hypersensitivity 2- Systemic lupus erythematosus
reaction. 3- Selective IgA deficiency
4- X-linked agammaglobulinaemia
[ Q: 1319 ] PasTest 2009 - Basic
Science 5- Wiskott-Aldrich syndrome

A 15-year-old boy presents with fever, rash


Answer & Comments
and arthralgia. He has ++ protein and + blood
on urinalysis. Investigations reveal the Answer: 3- Selective IgA deficiency
following: erythrocyte sedimentation rate 32;
C-reactive protein 12; full blood count normal; The history of invasive infection and gut
and U+Es normal. Antistreptolysin-O titres are involvement and the family history of immune
raised. cytopenia are suggestive of selective IgA
deficiency (SIgAD) which is the most common
What immunological phenomenon is taking of the primary antibody deficiencies. Patients
place? with SIgAD have a 10-fold increased risk of
1- Type-I hypersensitivity reaction coeliac disease and there may be a positive
history of SIgAD, coeliac or auto-immune
2- Type-II hypersensitivity reaction
disease in first degree relatives. The
3- Type-III hypersensitivity reaction serological prevalence of IgAD varies 100-fold
4- Type-IV hypersensitivity reaction among populations, with individuals of Arab
and Spanish origin having a particularly high
5- Cryoglobulinaemia
incidence. The more heterogeneous related
immune disorder, known as common variable
Answer & Comments immune deficiency, (involving low levels of
Answer: 3- Type-III hypersensitivity reaction both IgA and IgG, and sometimes also IgM)
could present in a similar fashion and would
He has a poststreptococcal need to be excluded here; this too is
glomerulonephritis, an immune complex associated with auto-immune disease (in
disease. Other infectious causes of immune contrast to X-linked agammaglobulinaemia).
complex disease include hepatitis B and C, Indeed some cases of IgAD may progress to
cytomegalovirus and malaria. common variable immune deficiency and a
common genetic substrate has been
[ Q: 1320 ] PasTest 2009 - Basic suspected. X-linked agammaglobulinaemia is
Science characterised early-onset
agammaglobulinaemia and absence of B cells,
A 13-year-old boy of Spanish descent has
with abnormalities in the Btk gene (also
suffered from recurrent sinus infections for
known as Bruton agammaglobulinaemia). This
the past 2 years and has a history of
tends to present with more severe and

Dr. Khalid Yusuf El-Zohry – Sohag Teaching Hospital (01118391123)


Ref MRCPass OE OE 2012 PasTest 2009 PassMedicine 2009 PasTest Exam ReviseMRCP 605
Elzohry MRCP Questions Bank (Part 1) – 2013 (For my personal use)

Answer & Comments condition also being the first example of


enzyme replacement therapy (initially using
Answer: 1- Latex allergy
red cell transfusions which contain ADA) in
Given current public awareness of peanut clinical medicine, and later being the first
allergy, these tend now to be avoided at disease to be treated by gene therapy,
children’s parties and hence peanut allergy is although with only partial success. ADA has an
less likely than latex allergy here. Most people important role in the intermediate pathways
make the assumption that party balloons are of purine metabolism. Purine-nucleoside
made of some form of plastic, but most are phosphorylase (PNP) is also active in this
still produced from some form of latex. pathway, but is a much rarer cause of SCID. It
Allergic contact dermatitis would have a less is interesting that deficiencies in both these
acute presentation, and it C-1 esterase enzymes predominantly affect lymphocytes,
deficiency is less likely than latex allergy. despite their presence in most other cells of
Corticosteroids, anti-histamines, nebulised the body.
salbutamol or adrenaline, with fluid
resuscitation if required form the mainstay of [ Q: 1325 ] PasTest 2009 - Basic
therapy. Given the severity of her reaction she Science
should be patch or RAST tested to confirm the
diagnosis. You review a 16-year-old boy who has a
history of Type 1 diabetes. He is doing a
project at his school on aspects of the human
[ Q: 1324 ] PasTest 2009 - Basic
immune system and asks about the function
Science
of T lymphocytes and how they interact with
Cells from a patient with severe combined other cells in the body.
immunodeficiency disease (SCID) lack the Which of the following fits best with the action
adenosine deaminase enzyme. of the immune system?
This will have a direct effect on? 1- CD4 T cells interact with macrophages via
1- synthesis of deoxynucleoside diphosphates MHC class I
(dNDPs) 2- CD4 T cells interact with B cells via MHC
2- degradation of guanine nucleotides class II
3- purine salvage 3- CD8 T cells interact with any cell via MHC
class II
4- de novo synthesis of purine nucleotides
4- CD8 T cells only interact with detritic cells
5- degradation of adenine nucleotides
via MHC class I

Answer & Comments 5- Only cells of the immune system express


MHC class I
Answer: 3- purine salvage

Lymphocytes need efficient salvage and Answer & Comments


interconversion pathways for purines and Answer: 2- CD4 T cells interact with B cells via
pyrimidines during rapid bursts of MHC class II
proliferation, particularly in the lymphoid
germinal centres and fetal thymus. Adenosine Macrophages expressing MHC class II are
deaminase (ADA) deficiency was the first activated when they come into contact with
established cause of severe combined CD4 T cells, and B cells are activated,
immunodeficiency disease (SCID), the upregulating antibody production when they

Dr. Khalid Yusuf El-Zohry – Sohag Teaching Hospital (01118391123)


Ref MRCPass OE OE 2012 PasTest 2009 PassMedicine 2009 PasTest Exam ReviseMRCP 607
Elzohry MRCP Questions Bank (Part 1) – 2013 (For my personal use)

Answer & Comments condition also being the first example of


enzyme replacement therapy (initially using
Answer: 1- Latex allergy
red cell transfusions which contain ADA) in
Given current public awareness of peanut clinical medicine, and later being the first
allergy, these tend now to be avoided at disease to be treated by gene therapy,
children’s parties and hence peanut allergy is although with only partial success. ADA has an
less likely than latex allergy here. Most people important role in the intermediate pathways
make the assumption that party balloons are of purine metabolism. Purine-nucleoside
made of some form of plastic, but most are phosphorylase (PNP) is also active in this
still produced from some form of latex. pathway, but is a much rarer cause of SCID. It
Allergic contact dermatitis would have a less is interesting that deficiencies in both these
acute presentation, and it C-1 esterase enzymes predominantly affect lymphocytes,
deficiency is less likely than latex allergy. despite their presence in most other cells of
Corticosteroids, anti-histamines, nebulised the body.
salbutamol or adrenaline, with fluid
resuscitation if required form the mainstay of [ Q: 1325 ] PasTest 2009 - Basic
therapy. Given the severity of her reaction she Science
should be patch or RAST tested to confirm the
diagnosis. You review a 16-year-old boy who has a
history of Type 1 diabetes. He is doing a
project at his school on aspects of the human
[ Q: 1324 ] PasTest 2009 - Basic
immune system and asks about the function
Science
of T lymphocytes and how they interact with
Cells from a patient with severe combined other cells in the body.
immunodeficiency disease (SCID) lack the Which of the following fits best with the action
adenosine deaminase enzyme. of the immune system?
This will have a direct effect on? 1- CD4 T cells interact with macrophages via
1- synthesis of deoxynucleoside diphosphates MHC class I
(dNDPs) 2- CD4 T cells interact with B cells via MHC
2- degradation of guanine nucleotides class II
3- purine salvage 3- CD8 T cells interact with any cell via MHC
class II
4- de novo synthesis of purine nucleotides
4- CD8 T cells only interact with detritic cells
5- degradation of adenine nucleotides
via MHC class I

Answer & Comments 5- Only cells of the immune system express


MHC class I
Answer: 3- purine salvage

Lymphocytes need efficient salvage and Answer & Comments


interconversion pathways for purines and Answer: 2- CD4 T cells interact with B cells via
pyrimidines during rapid bursts of MHC class II
proliferation, particularly in the lymphoid
germinal centres and fetal thymus. Adenosine Macrophages expressing MHC class II are
deaminase (ADA) deficiency was the first activated when they come into contact with
established cause of severe combined CD4 T cells, and B cells are activated,
immunodeficiency disease (SCID), the upregulating antibody production when they

Dr. Khalid Yusuf El-Zohry – Sohag Teaching Hospital (01118391123)


Ref MRCPass OE OE 2012 PasTest 2009 PassMedicine 2009 PasTest Exam ReviseMRCP 607
Elzohry MRCP Questions Bank (Part 1) – 2013 (For my personal use)

interact with CD4 T cells. CD8 T cells interact [ Q: 1327 ] PasTest 2009 - Basic Science
via MHC class I and this MHC class can be
Where is the antibody targeted in Lambert-
expressed on any cell in the body. T cell
Eaton syndrome?
interactions are regulated by expression of
different MHC antigens, resulting in the 1- Anticholinesterase
association between a number of 2- Mitochondria
autoimmune diseases and specific MHC
3- Sodium channels
subtypes.
4- Potassium channels
[ Q: 1326 ] PasTest 2009 - Basic 5- Voltage-gated calcium channels
Science
Where is the immune defect in chronic Answer & Comments
lymphocytic leukaemia situated? Answer: 5- Voltage-gated calcium channels
1- Complement
Lambert-Eaton myasthenic syndrome (LEMS)
2- Immunoglobulin G (IgG) is a rare condition where muscle weakness is
3- Macrophages the result of abnormal acetylcholine (ACh)
release at the neuromuscular junction. It was
4- Mast cells
first described by Lee McKendre Eaton, an
5- B cells American neurologist, with Edward H.
Lambert and E. D. Rooke. Antibodies against
Answer & Comments presynaptic voltage-gated calcium channels
(VGCC) are believed to play an important role
Answer: 5- B cells in the pathogenesis by inhibiting the calcium
Chronic lymphocytic leukemia (CLL) is a currents that trigger ACh release. A serum test
malignancy of B lymphocytes. In about 50% of for voltage-gated calcium channel antibodies
cases, a diagnosis of CLL is incidental (to some is now available.
other condition); it requires a white blood cell
count and peripheral smear, with definitive [ Q: 1328 ] PasTest 2009 - Basic
diagnosis by flow cytometric Science
immunophenotype.
A dental practice nurse is admitted for an
The median survival after diagnosis is about 7 elective operation. During induction with a
years. One interesting aspect of CLL is that it general anaesthetic she develops tachycardia,
results in defective antigen presentation with rash and diffuse wheeze. She had mentioned
an acquired T cell dysfunction, and manifests that she has had rashes when assisting with
as both an alloimmune and autoimmune anaesthetics at work.
defect. Abnormal antibodies are produced in What is the likely diagnosis?
about 30% of patients, and they are produced
not by the malignant cells but by healthy 1- Systemic mastocytosis
polyclonal cells. These antibodies are typically 2- Anaphylaxis
target restricted for blood cell antigens. 3- Pseudoallergy
4- Serum sickness
5- Contact dermatitis

Dr. Khalid Yusuf El-Zohry – Sohag Teaching Hospital (01118391123)


Ref MRCPass OE OE 2012 PasTest 2009 PassMedicine 2009 PasTest Exam ReviseMRCP 608
Elzohry MRCP Questions Bank (Part 1
Elzohry MRCP Questions Bank (Part 1) – 2013 (For my personal use)

Dr. Khalid Yusuf El-Zohry – Sohag Teaching Hospital (01118391123)


Elzohry MRCP Questions Bank (Part 1) – 2013 (For my personal use)

3- X-linked recessive be inherited in autosomal recessive or


4- Autosomal recessive dominant fashion.

5- Chromosomal non-dysjunction
[ Q: 1340 ] PasTest 2009 - Basic
Science
Answer & Comments
A couple aged in their early 30s come to the
Answer: 4- Autosomal recessive genetics clinic. They have one 6-year-old son
This boy has Wolfram syndrome, a rare who has gross obesity and mild mental
progressive neurodegenerative disorder impairment, diagnosed as Prader Willi
characterised by diabetes insipidus, diabetes syndrome and they are concerned about the
mellitus, optic atrophy and sensorineural possibility of further children being affected.
deafness (DIDMOAD). Patients may also suffer Which of the following most accurately
from red-green colour blindness. describes the mode of inheritance of Prader
Heterozygotes are thought to have an Willi?
increased risk of Type 1 diabetes. Carrier
1- Autosomal recessive
frequency is around 1:350, leading to a
prevalence of 1:770,000. A variety of 2- Autosomal dominant
mutations in the WFS1 gene are thought to be 3- X-linked recessive
responsible.
4- X-linked dominant
5- Non-mendelian
[ Q: 1339 ] PasTest 2009 - Basic
Science
Answer & Comments
Which of the following disorders may have an
autosomal recessive mode of inheritance? Answer: 5- Non-mendelian
1- Achondroplasia Prader Willi occurs because of deletion or
2- Glucose 6-phosphate dehydrogenase disruption of genes on the proximal arm of
deficiency chromosome 15, (15q11-13). It is the first
human disorder to be attributed to genetic
3- Huntington's disease imprinting, where genes are expressed
4- Ehlers-Danlos syndrome Type IV differently based on which parent they came
5- Haemophilia A from. Around 70% of cases occur due to
deletion of 15q11-13, 28% due to maternal
disomy. Prevalence of the condition is
Answer & Comments
estimated as being as high as 1:8000 in rural
Answer: 4- Ehlers-Danlos syndrome Type IV Sweden, and as low as 1:25000 in other areas.
Features include morbid obesity, hyperphagia,
Ehlers-Danlos syndrome is a autosomal hypogonadism and mental impairment.
recessive heritable disorder of connective
tissue with easy bruising, joint hypermobility
[ Q: 1341 ] PasTest 2009 - Basic
(loose joints), skin laxity, and weakness of
Science
tissues. There are a number of different types
of Ehlers-Danlos syndrome (EDS) which share In which of the following genetic diseases is
the foregoing features but can be categorised DNA analysis useful?
into at least nine different types. Type IV may
1- Adult polycystic kidney disease
2- Down's syndrome

Dr. Khalid Yusuf El-Zohry – Sohag Teaching Hospital (01118391123)


Ref MRCPass OE OE 2012 PasTest 2009 PassMedicine 2009 PasTest Exam ReviseMRCP 613
Elzohry MRCP Questions Bank (Part 1) – 2013 (For my personal use)

one of the most common single gene [ Q: 1337 ] PasTest 2009 - Basic
mutation disorders and has a prevalence said Science
to be between 1 in 3000 and 1 in 10,000. It
Which of the following should be
carries an autosomal dominant inheritance
considered in the management of haemophilia
pattern with a relatively high clinical
A?
variability. Two prominent systems of clinical
classification, (Berlin and Ghent) are used to 1- The infant is protected at birth due to
characterise the disorder. maternal transfer of factor VIII
2- Desmopressin may be useful
[ Q: 1336 ] PasTest 2009 - Basic 3- A factor VIII concentration < 10% causes
Science severe disease
A 19-year-old man from a family of travellers 4- Most cases are the result of new mutations
presents to the Emergency department. He
5- von Willebrand factor levels are reduced
has suffered a sudden deterioration in vision.
Additional past medical history of note
includes bilateral shoulder dislocation. On Answer & Comments
examination he is tall and thin with a high Answer: 2- Desmopressin may be useful
arched palate. He appears to have suffered a
lens dislocation Because factor VIII is not transferred through
the placenta, bleeding may occur in severe
Which of the following genes is most likely to
cases. People with mild to moderate
be abnormal?
haemophilia respond to desmopressin
1- Fibrillin-1 sufficiently to cover minor procedures like
2- Collagen Type II tooth extraction. Less than a 2% level is
associated with severe disease, which entails
3- Collagen Type III
spontaneous bleeding into joints and muscles.
4- Collagen Type IV Most cases are the result of X-linked recessive
5- Collagen Type V transmission. Probably one-third of cases may
result from a new mutation. von Willebrand
factor levels are normal.
Answer & Comments
Answer: 1- Fibrillin-1 [ Q: 1338 ] PasTest 2009 - Basic
This man has Marfan’s, his late presentation Science
to medical services perhaps being explained A 17-year-old boy with a history of type 1
by the fact that he is from a travelling family. diabetes mellitus, sensorineural high tone
The history of joint and lens dislocation, and deafness and colour blindess presents for
his Marfanoid habitus is strongly supportive of review in the genetics clinic. You learn from
the diagnosis. Marfan’s occurs due to a defect his mother that other members of the family
in the fibrillin-1 gene, which codes for fibrillin, are affected, including a daughter aged 21
an essential building block of microfibrils, who also suffers from Type 1 diabetes.
which are a component of the suspensory
Which of the following represents the usual
ligament of the lens and other tissues with
mode of inheritance for the likely genetic
elasticity such as the aorta. Collagen gene
syndrome?
defects are associated with Ehlers Danlos
syndrome. 1- Autosomal dominant
2- X-linked

Dr. Khalid Yusuf El-Zohry – Sohag Teaching Hospital (01118391123)


Ref MRCPass OE OE 2012 PasTest 2009 PassMedicine 2009 PasTest Exam ReviseMRCP 612
Elzohry MRCP Questions Bank (Part 1) – 2013 (For my personal use)

3- X-linked recessive be inherited in autosomal recessive or


4- Autosomal recessive dominant fashion.

5- Chromosomal non-dysjunction
[ Q: 1340 ] PasTest 2009 - Basic
Science
Answer & Comments
A couple aged in their early 30s come to the
Answer: 4- Autosomal recessive genetics clinic. They have one 6-year-old son
This boy has Wolfram syndrome, a rare who has gross obesity and mild mental
progressive neurodegenerative disorder impairment, diagnosed as Prader Willi
characterised by diabetes insipidus, diabetes syndrome and they are concerned about the
mellitus, optic atrophy and sensorineural possibility of further children being affected.
deafness (DIDMOAD). Patients may also suffer Which of the following most accurately
from red-green colour blindness. describes the mode of inheritance of Prader
Heterozygotes are thought to have an Willi?
increased risk of Type 1 diabetes. Carrier
1- Autosomal recessive
frequency is around 1:350, leading to a
prevalence of 1:770,000. A variety of 2- Autosomal dominant
mutations in the WFS1 gene are thought to be 3- X-linked recessive
responsible.
4- X-linked dominant
5- Non-mendelian
[ Q: 1339 ] PasTest 2009 - Basic
Science
Answer & Comments
Which of the following disorders may have an
autosomal recessive mode of inheritance? Answer: 5- Non-mendelian
1- Achondroplasia Prader Willi occurs because of deletion or
2- Glucose 6-phosphate dehydrogenase disruption of genes on the proximal arm of
deficiency chromosome 15, (15q11-13). It is the first
human disorder to be attributed to genetic
3- Huntington's disease imprinting, where genes are expressed
4- Ehlers-Danlos syndrome Type IV differently based on which parent they came
5- Haemophilia A from. Around 70% of cases occur due to
deletion of 15q11-13, 28% due to maternal
disomy. Prevalence of the condition is
Answer & Comments
estimated as being as high as 1:8000 in rural
Answer: 4- Ehlers-Danlos syndrome Type IV Sweden, and as low as 1:25000 in other areas.
Features include morbid obesity, hyperphagia,
Ehlers-Danlos syndrome is a autosomal hypogonadism and mental impairment.
recessive heritable disorder of connective
tissue with easy bruising, joint hypermobility
[ Q: 1341 ] PasTest 2009 - Basic
(loose joints), skin laxity, and weakness of
Science
tissues. There are a number of different types
of Ehlers-Danlos syndrome (EDS) which share In which of the following genetic diseases is
the foregoing features but can be categorised DNA analysis useful?
into at least nine different types. Type IV may
1- Adult polycystic kidney disease
2- Down's syndrome

Dr. Khalid Yusuf El-Zohry – Sohag Teaching Hospital (01118391123)


Ref MRCPass OE OE 2012 PasTest 2009 PassMedicine 2009 PasTest Exam ReviseMRCP 613
Elzohry MRCP Questions Bank (Part 1) – 2013 (For my personal use)

3- Huntington's disease 5- X-linked dominant


4- Hyperobstructible coronary myopathy
Answer & Comments
5- Klinefelter's syndrome
Answer: 5- X-linked dominant
Answer & Comments Males and females with autosomal recessive
Answer: 3- Huntington's disease Alport’s syndrome are often equally severely
affected when family histories are examined.
Single-gene disorder (also called Mendelian or In x-linked Alport’s females may present with
monogenic) is caused by changes or mutations dipstick positive haematuria and hearing
that occur in the DNA sequence of one gene. impairment, but are much less likely than
Genes code for proteins, the molecules that males to suffer from gross haematuria and
carry out most of the work, perform most life increased risk of progression to renal failure.
functions, and even make up the majority of Males with x-linked Alport’s, as described
cellular structures. When a gene is mutated so here, are affected more severely, with
that its protein product can no longer carry approximately 90% of them progressing to
out its normal function, a disorder can result. renal failure by age 40 yrs. The condition is
There are more than 6000 known single-gene characterised by sensorineural deafness,
disorders, which occur in about 1 out of every haematuria, proteinuria, and progressive renal
200 births. Some examples are cystic fibrosis, impairment, and abnormalities of the ocular
sickle cell anaemia, Marfan's syndrome, lens, coupled with dot and fleck retinopathy.
Huntington's disease, and hereditary
haemochromatosis.
[ Q: 1343 ] PasTest 2009 - Basic
Science
[ Q: 1342 ] PasTest 2009 - Basic
Science A male patient age 33 presents with
depression, weight loss and choreiform
The father of a 12-year-old girl attends the movements. He informs you that his father
genetics clinic with his new partner. The 12- had similar symptoms aged 50, his
year-old girl suffers from mild sensorineural grandfather at aged 75, and both deteriorated
hearing loss and dipstick positive haematuria. in terms of mobility and mental state and
He wants to start a new family with his second eventually died.
partner. You want to advise him of the
chances of further children being affected by What is the phenomenon described here
the disease. He tells you that out of other known as?
affected relatives, males seem much more 1- Increased penetrance
severely affected than females, with a number 2- Autsomal dominance
of them having required renal replacement
therapy before the age of 40 years. 3- Anticipation

Given the history above, what is the most 4- X-linked dominant characteristics
likely mode of inheritance? 5- Mitochondrial characteristics
1- Autosomal dominant
Answer & Comments
2- Autosomal recessive
3- Polysomal Answer: 3- Anticipation

4- X-linked recessive The phenomenon described here is known as


genetic anticipation, where successive

Dr. Khalid Yusuf El-Zohry – Sohag Teaching Hospital (01118391123)


Ref MRCPass OE OE 2012 PasTest 2009 PassMedicine 2009 PasTest Exam ReviseMRCP 614
Elzohry MRCP Questions Bank (Part 1) – 2013 (For my personal use)

generations present with symptoms at an would be detectable by microscopy. For


earlier age. It is known to occur in differences less than this, RFLPs will be useful.
Huntingdon's disease, the probable inherited
illness seen here, although it occurs in [ Q: 1345 ] PasTest 2009 - Basic
affected males, whereas affected females do Science
not seem to show traits reflecting
anticipation. Genetic anticipation is also seen Sickle-cell anaemia is characterised by which
in myotonic dystrophy, Friedrich's ataxia and of the following?
fragile X syndrome. 1- Occurrence due to the substitution of
alanine for valine on position 6 of the b-
[ Q: 1344 ] PasTest 2009 - Basic globin gene
Science 2- Presence of 70-90% HbS, 5-10% HbA and 2-
Which of the following is a feature of 20% HbF on electrophoresis
restriction fragment length polymorphisms 3- Resistance to falciparum malaria
(RFLPs)? 4- Absent reticulocytosis
1- They utilise restriction exonucleases 5- Vaso-occlusive crises
2- They are infrequently used in linkage
analysis Answer & Comments
3- They may be used to diagnose Huntington's
Answer: 5- Vaso-occlusive crises
disease
4- They use Western blotting technique Sickle-cell disease is an autosomal recessive
disorder caused by a substitution of the amino
5- They are used in linkage which involves a acid glutamine for valine on position 6.
difference of more than 1 million base Haemoglobin electrophoresis demonstrates
pairs of DNA no HbA, 2-20% HbF and the predominance of
HbS. Individuals with sickle cell trait are
Answer & Comments relatively resistant to lethal effects of
falciparum malaria from early childhood.
Answer: 3- They may be used to diagnose
Patients with sickle cell anaemia do not have a
Huntington's disease
correspondingly greater resistance to
RFLPs may be used to diagnose Huntington's falciparum malaria. Microcytic hypochromic
disease. Enzymes from bacteria called anaemia occurs in this condition with marked
restriction endonucleases are used in RFLPs. reticulocytosis (10-20%). Vaso-occlusive crises
These are known to cleave at specific sites, are characteristic. These occur due to sickling,
resulting in specific fragments of the DNA precipitated by hypoxia, acidosis, dehydration
material being analysed. The relative position and infection.
of different genes on a chromosome is called
‘linkage'. By using RFLPs, a linkage map may [ Q: 1346 ] PasTest 2009 - Basic
be created when the genes are too close to be Science
seen on microscopy. Tracking of the disease
gene through successive generations may be Which of the following is a feature of the
done with RFLPs. Southern blotting is the Sézary disease?
technique used in RFLP analysis. A difference 1- B-cell lymphoma
of more than a million base pairs of DNA
2- Macrophage disorder
3- Natural killer cell deficiency

Dr. Khalid Yusuf El-Zohry – Sohag Teaching Hospital (01118391123)


Ref MRCPass OE OE 2012 PasTest 2009 PassMedicine 2009 PasTest Exam ReviseMRCP 615
Elzohry MRCP Questions Bank (Part 1) – 2013 (For my personal use)

4- Neutrophil disease Antibodies such as those to cholera,


5- T-cell malignancy diphtheria and tetanus toxin can play a major
role in protective immunity in these
infections. Cellular immunity is essential in
Answer & Comments
protection against intracellular bacteria, eg
Answer: 5- T-cell malignancy mycobacteria. Phagocytes interact directly but
weakly with bacteria, or strongly if they are
S‫©أ‬zary syndrome is a T (helper)-cell complement-opsonised. Endotoxin activates
malignancy. There are variants, for example macrophages by binding to CD14.
mycosis fungoides, that affect the skin and
may begin as an eczematous-type reaction but
[ Q: 1348 ] PasTest 2009 - Basic
which can progress to plaques and ulcers.
Science
S‫©أ‬zary syndrome is a combination of
leukaemia and a skin disorder, usually A 27-year-old Jewish man attends the clinic
erythroderma. Lymphadenopathy and with his wife. They are keen to start a family
hepatosplenomegaly are found. These but are worried because his brother
disorders occur most commonly in males and developed jaundice during teenage years and
present around middle-age. Prognosis is was given a diagnosis of Dubin Johnson
worse than for CLL. syndrome. He asks for advice as to the chance
of his children inheriting the condition.
[ Q: 1347 ] PasTest 2009 - Basic Which of the following most accurately
Science represents the mode of inheritance for Dubin
Which of the following statements is true Johnson syndrome?
concerning immunity to bacteria? 1- X-linked recessive
1- Antibodies to secreted bacterial products 2- Autosomal dominant
play no protective role 3- Autosomal recessive
2- Bacteria opsonised by antibodies and 4- X-linked dominant
complement are more effectively
phagocytosed than those opsonised by 5- Occurs by new mutation only
antibodies alone
3- Humoral rather than cellular immunity is Answer & Comments
predominant in protection against all types Answer: 3- Autosomal recessive
of bacteria
Dubin Johnson is a rare cause of conjugated
4- Phagocytes cannot engulf bacteria in the hyperbilirubinaemia with a prevalence of
absence of antibodies around 1:1300 in Iranian Jews, the population
5- Endotoxin induces shock mainly through with the highest prevalence. The cause is a
the activation of T cells genetic mutation of the human canalicular
multispecific organic anion transporter
Answer & Comments (cMOAT) protein, also known as the multi-
drug resistance protein. It results in impaired
Answer: 2- Bacteria opsonised by antibodies transport of non-bile salt organic anions which
and complement are more effectively results in conjugated hyperbilirubinaemia.
phagocytosed than those opsonised by Over the long term the condition follows a
antibodies alone benign course.

Dr. Khalid Yusuf El-Zohry – Sohag Teaching Hospital (01118391123)


Ref MRCPass OE OE 2012 PasTest 2009 PassMedicine 2009 PasTest Exam ReviseMRCP 616
Elzohry MRCP Questions Bank (Part 1) – 2013 (For my personal use)

[ Q: 1349 ] PasTest 2009 - Basic The history of primary hyperparathyroidism


Science and a pituitary tumour in this patient is
diagnostic of multiple endocrine neoplasia
The 3' to 5' exonuclease activity
type 1 (MEN1). This is an autosomal dominant
possessed by some DNA polymerases that
disorder that is caused by mutations in the
enables the enzyme to replace
MEN1 gene on 11q13. It is characterised by
misincorporated nucleotide is called what?
tumours of the parathyroid glands, pituitary
1- Proofreading and pancreatic islet cells. About 90% of
2- Replication patients develop parathyroid hyperplasia or
adenoma. Pituitary tumours are seen in about
3- Recombination
60% of patients. They include prolactinomas,
4- Retrotransposition growth hormone-producing tumours and
5- Splicing ACTH-producing tumours. Between 50-75% of
patients with MEN1 develop pancreatic 0.455 in(atient )-4
Answer & Comments
Answer: 1- Proofreading

Retrotransposition is transposition via an RNA


intermediate (transposition is the movement
of a genetic element from one site to another
in a DNA molecule). Splicing is the removal of
introns from the primary transcript of a
discontinuous gene.

[ Q: 1350 ] PasTest 2009 - Basic


Science
A 25-year-old woman presents with primary
hyperparathyroidism. It emerges that she has
been treated previously for a prolactinoma.
What is your diagnosis?
1- Familial parathyroid hyperplasia
2- Multiple endocrine neoplasia type 1
3- Neurofibromatosis type 1
4- Polyglandular autoimmune syndrome type
1
5- von Hippel-Lindau syndrome

Answer & Comments


Answer: 2- Multiple endocrine neoplasia type
1

Dr. Khalid Yusuf El-Zohry – Sohag Teaching Hospital (01118391123)


Ref MRCPass OE OE 2012 PasTest 2009 PassMedicine 2009 PasTest Exam ReviseMRCP 617
Elzohry MRCP Questions Bank (Part 1) – 2013 (For my personal use)

[ Q: 1355 ] PasTest 2009 - Basic mother, he will be haemophilic because the Y


Science chromosome cannot counteract the defective
gene located on his X chromosome.
A young pregnant girl would like to
know the risk of having a child with genetic
abnormalities. Her father and two brothers [ Q: 1356 ] PasTest 2009 - Basic
are affected by haemophilia, which is Science
inherited as an X-linked disease. She is herself Which enzyme breaks base pairs in a double-
completely asymptomatic. She knows from stranded DNA molecule?
her last ultrasound test that her baby is a boy.
1- Helicase
What is the chance for her son to be
2- Nuclease
genetically affected by the same condition?
3- Phosphodiesterase
1- 1 in 2
4- Restriction endonuclease
2- 1 in 4
5- Telomerase
3- 1 in 6
4- 1 in 8
Answer & Comments
5- 1 in 16
Answer: 1- Helicase

Answer & Comments Nucleases are enzymes that degrade nucleic


acid molecules. Phosphodiesterases are a type
Answer: 1- 1 in 2
of nuclease. Restriction endonucleases cut
Males are hemizygous, receiving their only X DNA molecules at a limited number of specific
chromosome from their mother. Females are nucleotide sequences. Telomerases are
heterozygous, inheriting X chromosomes from enzymes that maintain the ends of eukaryotic
both parents. If a female has a defective gene chromosomes by synthesising telomeric
on one of her two X chromosomes, she will be repeat sequences.
protected from its effects by the normal gene
on her second X chromosome. If a male has a [ Q: 1357 ] PasTest 2009 - Basic
mutant X and a normal Y chromosome, he will Science
be affected by a X-linked disease. A son,
whose mother has two normal alleles, will not In a patient with nickel-associated contact
be affected by haemophilia even if the father dermatitis which of the following statements
has the disease and the defective gene. A is true?
daughter of the same parents will be a 1- Mediated by mast cells
heterozygous carrier. A heterozygous carrier 2- Mediated by IgE
mother and a normal father pass the gene for
haemophilia on to possibly one-half of their 3- Skinprick testing is the best way to establish
children. Half the daughters will be carriers the sensitising antigen
and half the sons will be haemophilic. The rest 4- Systemic cytokine release induces skin
of the siblings will be normal. Daughters, as inflammation
long as one parent is genotypically normal,
5- Elimination of the responsible agent is the
can only be carriers. The normal gene on the
most important goal
second X chromosome counteracts the defect,
and the daughters do not suffer from the trait.
If a son receives the defective gene from his

Dr. Khalid Yusuf El-Zohry – Sohag Teaching Hospital (01118391123)


Ref MRCPass OE OE 2012 PasTest 2009 PassMedicine 2009 PasTest Exam ReviseMRCP 620
Elzohry MRCP Questions Bank (Part 1) – 2013 (For my personal use)

Dr. Khalid Yusuf El-Zohry – Sohag Teaching Hospital (01118391123)


Ref MRCPass
Elzohry MRCP Questions Bank (Part 1) – 2013 (For my personal use)

[ Q: 1355 ] PasTest 2009 - Basic mother, he will be haemophilic because the Y


Science chromosome cannot counteract the defective
gene located on his X chromosome.
A young pregnant girl would like to
know the risk of having a child with genetic
abnormalities. Her father and two brothers [ Q: 1356 ] PasTest 2009 - Basic
are affected by haemophilia, which is Science
inherited as an X-linked disease. She is herself Which enzyme breaks base pairs in a double-
completely asymptomatic. She knows from stranded DNA molecule?
her last ultrasound test that her baby is a boy.
1- Helicase
What is the chance for her son to be
2- Nuclease
genetically affected by the same condition?
3- Phosphodiesterase
1- 1 in 2
4- Restriction endonuclease
2- 1 in 4
5- Telomerase
3- 1 in 6
4- 1 in 8
Answer & Comments
5- 1 in 16
Answer: 1- Helicase

Answer & Comments Nucleases are enzymes that degrade nucleic


acid molecules. Phosphodiesterases are a type
Answer: 1- 1 in 2
of nuclease. Restriction endonucleases cut
Males are hemizygous, receiving their only X DNA molecules at a limited number of specific
chromosome from their mother. Females are nucleotide sequences. Telomerases are
heterozygous, inheriting X chromosomes from enzymes that maintain the ends of eukaryotic
both parents. If a female has a defective gene chromosomes by synthesising telomeric
on one of her two X chromosomes, she will be repeat sequences.
protected from its effects by the normal gene
on her second X chromosome. If a male has a [ Q: 1357 ] PasTest 2009 - Basic
mutant X and a normal Y chromosome, he will Science
be affected by a X-linked disease. A son,
whose mother has two normal alleles, will not In a patient with nickel-associated contact
be affected by haemophilia even if the father dermatitis which of the following statements
has the disease and the defective gene. A is true?
daughter of the same parents will be a 1- Mediated by mast cells
heterozygous carrier. A heterozygous carrier 2- Mediated by IgE
mother and a normal father pass the gene for
haemophilia on to possibly one-half of their 3- Skinprick testing is the best way to establish
children. Half the daughters will be carriers the sensitising antigen
and half the sons will be haemophilic. The rest 4- Systemic cytokine release induces skin
of the siblings will be normal. Daughters, as inflammation
long as one parent is genotypically normal,
5- Elimination of the responsible agent is the
can only be carriers. The normal gene on the
most important goal
second X chromosome counteracts the defect,
and the daughters do not suffer from the trait.
If a son receives the defective gene from his

Dr. Khalid Yusuf El-Zohry – Sohag Teaching Hospital (01118391123)


Ref MRCPass OE OE 2012 PasTest 2009 PassMedicine 2009 PasTest Exam ReviseMRCP 620
Elzohry MRCP Questions Bank (Part 1) – 2013 (For my personal use)

Answer & Comments [ Q: 1359 ] PasTest 2009 - Basic Science


Answer: 5- Elimination of the responsible You review a 28-year-old man with a family
agent is the most important goal history of early thyroid carcinoma and
phaeochromocytoma. Your patient has been
This is caused by T lymphocytes reacting with
searching on the internet and has found
haptens conjugated to skin proteins. Patch
information about the RET proto-oncogene.
testing is the classical in vivo mechanism for
ascertaining the sensitising antigen and is read The proto-oncogene RET causes which thyroid
at 48-96 hours. Skinprick testing investigates cancer?
IgE-mediated reactions and is typically read at 1- Papillary
15 minutes using a positive histamine control
2- Medullary
and a negative solution of diluent. Local
cytokine release produces the local response 3- Follicular
typical of these contact reactions. 4- Anaplastic
5- Lymphoma
[ Q: 1358 ] PasTest 2009 - Basic
Science
Answer & Comments
A 16-year-old girl presents with primary
Answer: 2- Medullary
amenorrhoea. On examination, her height is
145 cm and weight 45 kg. Mutations of the RET proto-oncogene on
What is the most likely diagnosis? chromosome 10 are associated with multiple
endocrine neoplasia (MEN) 2a and 2b. MEN2a
1- Crohn's disease is associated with phaeochromocytoma,
2- Cystic fibrosis Cushing's syndrome, medullary thyroid
3- McCune-Albright syndrome carcinoma, and parathyroid hyperplasia.
MEN2b is similar, but patients have a
4- Iron deficiency anaemia marfanoid habitus and intestinal and visceral
5- Turner's syndrome ganglioneuromas, but not
hyperparathyroidism. Rarely, it may occur in
Answer & Comments isolation without other tumours. Screening for
medullary thyroid carcinoma is with the
Answer: 5- Turner's syndrome pentagastrin and the calcium infusion test
Crohn's disease and cystic fibrosis are with measurement of calcitonin. Where there
associated with slowing of growth but not is a family history of MEN2, total
primary amenorrhoea. In polyostotic fibrous thyroidectomy in childhood to prevent the
dysplasia (McCune-Albright syndrome), development of carcinoma is usually
precocious puberty causes premature closure recommended. Prior to consideration of
of the epiphyses and short stature but thyroidectomy, screening for co-existent
menstruation is not affected. Chromosomal phaeochromocytoma is essential, so that
analysis will establish the diagnosis of true alpha-blockade can be considered, otherwise
Turner's syndrome (45 XO). Mosaic forms of the risk of on-table hypertensive crisis is high.
Turner's syndrome may not have these
features. [ Q: 1360 ] PasTest 2009 - Basic
Science
A man who has autosomal recessively
inherited common variable immunodeficiency

Dr. Khalid Yusuf El-Zohry – Sohag Teaching Hospital (01118391123)


Ref MRCPass OE OE 2012 PasTest 2009 PassMedicine 2009 PasTest Exam ReviseMRCP 621
Elzohry MRCP Questions Bank (Part 1) – 2013 (For my personal use)

would like to start a family. His partner does 4- Sickle cell disease
not have the disease. 5- Phenylketonuria
What is the percentage that his children will
inherit his disease? Answer & Comments
1- 100%
Answer: 2- α1-antitrypsin deficiency (A1AT)
2- <5%
Alpha-1-antitrypsin deficiency (α1-antitrypsin
3- 25% deficiency, A1AD or Alpha-1) is a genetic
4- 50% disorder caused by defective production of
alpha-1-antitrypsin (A1AT), leading to
5- 0%
decreased A1AT activity in be blood and lungs,
and deposition of excessive abnormal A1AT
Answer & Comments
Answer: 2- <5%

Most patients with common variable


immunodeficiency present as sporadic cases,
although reports exist of familial cases with
various inheritance modes, including
autosomal dominant with variable
penetrance, autosomal recessive, of X-linked.
Prevalence is around 1 : 50,000. All children of
the patient will carry at least one gene for
common variable immunodeficiency, but the
chance of his partner carrying one copy of the
gene is low. Patients commonly present with a
history of recurrent infections and often have
associated autoimmune phenomena such as
rheumatoid arthritis, vitiligo, haemolytic
anaemia or neutropenia. Patients have an
increased risk of B-cell lymphoma. Twenty-
year survival rate is around 64% for man
patients and 67% for female patients.

[ Q: 1361 ] PasTest 2009 - Basic


Science
A 25-year-old adopted man consults for
review. He is interested in having children and
is concerned about the possibility of
inheritance of genetic disease.
Which of the following is most prevalent in
northern Caucasians?
1- Cystic fibrosis
2- α1-antitrypsin deficiency (A1AT)
3- Congenital hypothyroidism

Dr. Khalid Yusuf El-Zohry – Sohag Teaching Hospital (01118391123)


Ref MRCPass OE OE 2012 PasTest 2009 PassMedicine 2009 PasTest Exam ReviseMRCP 622
Elzohry MRCP Questions Bank (Part 1) – 2013 (For my personal use)

[ Q: 1362 ] PasTest 2009 - Basic What kind of genetic deficiency do those


Science patients have?
A 16-year-old boy presented with 1- Nucleotide excision repair
absence of pubertal development. He had no 2- Protein kinase
other symptoms of note. On examination he
3- Base excision repair
was tall, his voice was unbroken, testes were
undescended and there was only scanty pubic 4- Mismatch repair
hair. The remainder of the examination was 5- Ligase
unremarkable.
What is the most likely diagnosis? Answer & Comments
1- Klinefelter's syndrome Answer: 1- Nucleotide excision repair
2- Turner's syndrome
Xeroderma pigmentosum is an inherited skin
3- Kallmann's syndrome disorder characterised by photosensitivity
4- Testicular feminisation with severe sunburn in infancy, the
development of numerous pigmented spots
5- Hypopituitarism
resembling freckles, larger atrophic lesions
associated with telangiectasis, and multiple
Answer & Comments solar keratoses. Transmitted in an autosomal
Answer: 1- Klinefelter's syndrome recessive manner, xeroderma pigmentosa
involves a defect in nucleotide excision repair
Klinefelter's syndrome affects around 1 in (NER), leading to deficient repair of DNA
1000 males and is a chromosome disorder damaged by UV radiation and chromosome
associated with 47XXY being the most breakage. Individuals with this disease
common chromosomal disorder associated develop multiple malignant cutaneous
with male hypogonadism and infertility. neoplasms at an early age, and may suffer
Phenotypical changes include poor sexual from severe ophthalmic and neurological
development, small or undescended testes, abnormalities.
scanty pubic hair, gynaecomastia and
infertility. Infertility and gynaecomastia are [ Q: 1364 ] PasTest 2009 - Basic
the 2 most common symptoms that lead to
Science
diagnosis. The disease involves both the loss
of Leydig cells and seminiferous tubular A 35-year-old man presents with a one-year
dysgenesis. Androgen replacement restores history of choreiform movements, personality
secondary sexual characteristics, but no changes and mild memory loss. His father died
treatment is possible for the infertility. with similar problems at the age of 50.
Confirmation of the disorder is by What is the most likely diagnosis?
chromosomal analysis. Kallman's syndrome is
1- Alzheimer's disease
associated with anosmia in both male and
female patients. 2- Creutzfeldt-Jakob disease
3- Hallervorden-Spatz disease
[ Q: 1363 ] PasTest 2009 - Basic 4- Huntington's disease
Science
5- Wilson's disease
Patients with xeroderma pigmentosum have a
high risk of developing skin cancer.

Dr. Khalid Yusuf El-Zohry – Sohag Teaching Hospital (01118391123)


Ref MRCPass OE OE 2012 PasTest 2009 PassMedicine 2009 PasTest Exam ReviseMRCP 623
Elzohry MRCP Questions Bank (Part 1) – 2013 (For my personal use)

Answer & Comments 1- They are usually found in association with


Waldenstrom's macroglobulinaemia
Answer: 4- Huntington's disease
2- They are commonly found in the serum of
The most likely diagnosis in this patient is myeloma patients with normal renal
Huntington's disease (HD)or Huntington's function
chorea. This is an autosomal dominant
disorder that usually presents in the third or 3- They are found in isolation in 20-30% of
fourth decade of life with personality change, cases of myeloma
dementia and chorea. It is a progressive 4- They are not associated with IgE- or IgD-
neurodegenerative disorder leading to death secreting myelomas
about 15 years after onset. It is caused by the 5- They are rarely associated with renal
expansion of a cytosine-adenine-guanine tubular damage
(CAG) triplet repeat in the HD gene on 4p16.
Normal repeat numbers are 9-35 and affected
Answer & Comments
individuals have 36 ³‫ آ‬repeats.
Answer: 3- They are found in isolation in 20-
Patients with Alzheimer's disease usually
30% of cases of myeloma
present after the age of 50 years with slowly
progressive dementia. Chorea is not a feature Light chains are toxic to renal tubules. They
of this condition. Patients with Creutzfeld- are uncommon in association with
Jakob disease (CJD) present with progressive Waldenstrom's macroglobulinaemia, and not
dementia and usually die within 2 years of found in the serum of patients with myeloma
onset of the disease. Only a small proportion because they are cleared rapidly in those with
of patients with Alzheimer's and CJD have normal renal function. All patients with light
familial disease and familial forms are chains in their serum without IgG, IgM or IgA
inherited in an autosomal dominant manner. paraproteins should be checked for IgE and
IgD. Both blood and urine must be sent to the
Hallervorden-Spatz disease is an autosomal
laboratory in the search for paraproteins,
recessive neurodegenerative disorder that
since 20-30% of cases will be missed if only
usually presents in childhood or adolescence.
blood is sent. But the diagnostic clue is often
It is characterised by pyramidal and
the finding of low levels of the non-
extrapyramidal signs, dementia, optic atrophy
paraprotein classes of immunoglobulin as the
and the ‘eye-of-the tiger' sign on magnetic
malignant clone in myeloma suppresses the
resonance imaging (MRI) scan of the brain due
normal production of immunoglobulins.
to the deposition of iron in the basal ganglia.
Wilson's disease is also an autosomal
recessive disorder. Most adult patients with [ Q: 1366 ] PasTest 2009 - Basic
Wilson's disease present with a movement Science
disorder, personality changes and a Kayser- A 60-year-old man presents to the emergency
Fleischer ring on clinical or slit-lamp team with dramatic swelling of his tongue and
examination. lips. He has hypertension and type-2 diabetes
mellitus. His drug therapy has been
[ Q: 1365 ] PasTest 2009 - Basic unchanged for 5 years and comprises
Science lisinopril, low-dose aspirin and metformin.
Concerning monoclonal free light chains, Which of the following diagnoses is most
which of the following statements is true? likely?

Dr. Khalid Yusuf El-Zohry – Sohag Teaching Hospital (01118391123)


Ref MRCPass OE OE 2012 PasTest 2009 PassMedicine 2009 PasTest Exam ReviseMRCP 624
Elzohry MRCP Questions Bank (Part 1) – 2013 (For my personal use)

1- Acquired angioedema related to a This disorder is due to low levels of the C1


paraprotein inhibitor of the complement system and is one
2- Idiopathic angioedema of the commonest complement deficiencies.
Low levels of the C1 inhibitor allow C1 to act
3- Hereditary angioedema on C4 and C2. This in turn produces kinin-like
4- ACE inhibitor-associated angioedema products that cause the angio-oedema. Low
5- Salicylate-induced angioedema levels of C4 are found during an attack. C3
levels are normal. Membrane-attack complex
deficiencies leave patients particularly
Answer & Comments susceptible to neisserial infection. In a few
Answer: 4- ACE inhibitor-associated cases C1 inhibitor levels are normal but
angioedema defective. The skin lesions are not itchy, unlike
allergic urticaria. Painful intestinal
ACE inhibitor-associated angioedema is the involvement can occur. Triggers include stress,
commonest cause of these swellings involving infection and menstruation. Danazol may be
the face and tongue in this age group; it often used in treatment. Acquired C1 inhibitor
begins several years after starting an ACE deficiency may be associated with
inhibitor, especially ACE-1 inhibitors. A normal lymphoproliferative disease and infection.
C4 level found during an attack excludes C1-
inhibitor deficiency. Hereditary angioedema
[ Q: 1368 ] PasTest 2009 - Basic
would be unusual at this age, but the CI
Science
inhibitor should be checked nevertheless.
Salicylate-/aspirin-associated angioedema is In a patient with anaphylaxis, which of the
more likely to coexist with urticaria. Idiopathic following should be given to inhibit the
angioedema is a possibility, but a trial of an important late-phase reaction?
ACE inhibitor is essential.
1- Antihistamines
2- Epinephrine
3- Leukotriene inhibitor
[ Q: 1367 ] PasTest 2009 - Basic
Science 4- Hydrocortisone
5- NSAID
Which of the following is associated with
hereditary angio-oedema?
Answer & Comments
1- Low levels of C1 inhibitor
Answer: 4- Hydrocortisone
2- High levels of C1 inhibitor
3- C3 deficiency Hydrocortisone blocks the generation of
leukotrienes and prostaglandins, and hence
4- High levels of C4 complement during an
prevents the late-phase reaction often
attack
characterised by asthma. It should be given
5- Deficiency of the membrane-attack intravenously/intramuscularly at a dose of
complex (C5b-9) 100-200 mg. None of the other agents listed
above affect this aspect of anaphylaxis.
Answer & Comments Approximately 30% of deaths related to
anaphylaxis occur as a consequence of this
Answer: 1- Low levels of C1 inhibitor late-phase reaction.

Dr. Khalid Yusuf El-Zohry – Sohag Teaching Hospital (01118391123)


Ref MRCPass OE OE 2012 PasTest 2009 PassMedicine 2009 PasTest Exam ReviseMRCP 625
Elzohry MRCP Questions Bank (Part 1) – 2013 (For my personal use)

[ Q: 1369 ] PasTest 2009 - Basic Answer & Comments


Science
In a 20-year-old man, recurrent
and/or severe infections with which of the
following organisms would make you suspect
immunodeficiency associated with
hypogammaglobulinaemia?
1- Haemophilus influenzae
2- Chlamydia Psitacci
3- Herpes simplex virus
4- Candida spp
5- Pneumocystis jiroveci

Answer & Comments


Answer: 1- Haemophilus influenzae

Infections with Haemophilus influenzae and


Streptococcus pneumoniae occur with this
condition, not Streptococcus pyogenes.
Pneumocystis jiroveci (formerly P. carinii),
viral and candida infections are associated
with T-cell deficiencies, including HIV
infection.

[ Q: 1370 ] PasTest 2009 - Basic


Science
Which of the following statements in relation
to skin disease is true?
1- Eye involvement is unusual in benign
mucous membrane pemphigoid
2- The rash of herpes gestationis most
commonly affects the chest
3- Pemphigus is associated with subepidermal
bullae
4- Epidermolysis bullosa acquisita is strongly
associated with HLA-DR4
5- Nasal swabs should be routinely checked in
patients with recurrent staphylococcal
abscesses

Dr. Khalid Yusuf El-Zohry – Sohag Teaching Hospital (01118391123)


Ref MRCPass OE OE 2012 PasTest 2009 PassMedicine 2009 PasTest Exam ReviseMRCP 626
Elzohry MRCP Questions Bank (Part 1) – 2013 (For my personal use)

1- Granuloma formation is uncommon


2- It is likely to be associated with a history of
asthma and atopy over a few years before
the diagnosis is made
3- May be characterised by basophilia
4- Is associated with polyneuritis
5- pANCA is positive in > 80% of cases

Answer & Comments


Answer: 2- It is likely to be associated with a
history of asthma and atopy over a few years
before the diagnosis is made

Some 50% of cases of Churg-Strauss syndrome


are pANCA-positive and patients usually have
an eosinophilia. Mononeuritis, often
multiplex, is CA

Dr. Khalid Yusuf El-Zohry – Sohag Teaching Hospital (01118391123)


Ref MRCPass OE OE 2012 PasTest 2009 PassMedicine 2009 PasTest Exam ReviseMRCP 627
Elzohry MRCP Questions Bank (Part 1) – 2013 (For my personal use)

[ Q: 1375 ] PasTest 2009 - Basic 1- Carcinoma of the prostate


Science 2- Multiple myeloma
Anti-Ro antibodies in isolation 3- Rheumatoid arthritis
(negative ANA) occur in which of the following
4- Monoclonal gammopathies of
conditions?
undetermined significance (MGUS)
1- Sjögren's syndrome
5- Waldenstrom's macroglobulinaemia
2- Sicca syndrome
3- Systemic lupus erythematosus (SLE) Answer & Comments
4- Scleroderma Answer: 4- Monoclonal gammopathies of
5- Polymyositis undetermined significance (MGUS)

The incidence of monoclonal gammopathies


Answer & Comments of undetermined significance (MGUS)
Answer: 3- Systemic lupus erythematosus increases with age, from 1% of persons aged
(SLE) 25 years to 4% of patients > 70 years. Many
cases are seemingly benign. However, up to
ANA-negative lupus is uncommon. ANA- 25% progress to a Bcell malignancy or
negative SLE patients are usually Ro-positive myeloma, which may not become clinically
and have skin rashes with or without apparent until after 20 years. The course is
photosensitivity. In Sjögren's syndrome, Ro impossible to predict. Laboratory evaluation
and La antibodies commonly coexist. Sicca usually shows low M-protein levels in serum
syndrome is not typically associated with (< 3 g/dl) or urine (< 300 mg/24 hours) that
autoantibodies. are stable over time, normal levels of other
serum immunoglobulins, and no lytic bone
[ Q: 1376 ] PasTest 2009 - Basic lesions or Bence-Jones proteinuria; bone
Science marrow shows only mild plasmacytosis. No
treatment is recommended. Patients should
A 74-year-old-man is seen in the preoperative
be observed for clinical and immunochemical
assessment clinic prior to a right total hip changes every 4-6 months. The small M-band,
replacement scheduled in eight weeks time. absent lytic lesions and negative Bence-Jones
He had a recent chest infection and suffered
proteinuria would probably exclude multiple
chronic joint pain and prostatism for many myeloma as a possible diagnosis in a fit elderly
years. On examination there were few chest
man. The diagnosis of Waldenstr‫¶أ‬m's
signs and the musculoskeletal examination
macroglobulinaemia is established by
revealed evidence of severe arthritis in the
demonstrating a typical M spike on serum
right hip and base of the thumb. The prostate
protein electrophoresis that proves to be IgM
is firm and enlarged. It was thought that he
by immunoelectrophoresis or immunofixation.
was fit to have the operation. However, the
Similar blood abnormalities are also
blood tests showed raised immunoglobulins
associated with rheumatoid arthritis and
and the immune electrophoresis identified an
carcinoma of the prostate. In these
M-protein of the IgG type at 2.5 g/dl. Further
circumstances, serum M components may
tests reveal no lytic bone lesions on skeletal
represent unusual antibody responses to
survey and no Bence-Jones proteinuria. The
protracted antigenic stimuli. However, the
bone profile and the prostate specific antigen
clinical features in this case are not suggestive
were within normal limits.
of any of these conditions.
The MOST likely underlying disease is?

Dr. Khalid Yusuf El-Zohry – Sohag Teaching Hospital (01118391123)


Ref MRCPass OE OE 2012 PasTest 2009 PassMedicine 2009 PasTest Exam ReviseMRCP 628
Elzohry MRCP Questions Bank (Part 1) – 2013 (For my personal use)

[ Q: 1377 ] PasTest 2009 - Basic Reiter's syndrome (B27)


Science
HLA-CPsoriasis (Cw6)
Absent immune deposits on
HLA-DRGoodpasture's syndrome (DR2)
immunohistochemical analysis of renal tissue
is characteristic of which one of the following Multiple sclerosis (DR2)
renal disorders?
Addison's disease(DR3)
1- Systemic lupus erythematosus
Rheumatoid disease (DR4)
2- Henoch-Schönlein nephritis
3- Goodpasture's disease
[ Q: 1379 ] PasTest 2009 - Basic
4- Wegener's granulomatosis Science
5- Berger's disease A 36-year-old woman with an eight month
history of Raynaud's phenomenon presents to
Answer & Comments the emergency room with new onset
precordial chest pain. Physical examination
Answer: 4- Wegener's granulomatosis
reveals a pericardial friction rub and her CPK is
Wegener's granulomatosis is a primary small elevated 5 times above the upper normal
vessels vasculitis which involves the kidneys limit, but the MB isoenzyme is negative. The
and causes glomerulonephritis with crescent immunology profile reveals a positive ANA
formation. It is distinguished from other test at 1:640 with a speckled staining pattern.
causes of glomerulonephritis by the absence The MOST appropriate immunology test at
of immune deposits on immunohistochemical this stage is?
analysis.
1- Parasagittal cerebral rheumatoid nodule

[ Q: 1378 ] PasTest 2009 - Basic 2- Anti-ribonucleoprotein (anti-RNP antibody)


Science 3- Anti-centromere antibody
Which of the following is associated with the 4- Rheumatoid factor
correct disease? 5- Anti-neutrophil cytoplasmic antibody
1- HLA DR4 - ankylosing spondylitis (ANCA)
2- HLA B27 - Behçet's disease
Answer & Comments
3- HLA B5 - haemochromatosis
4- HLA A3 - multiple sclerosis Answer: 2- Anti-ribonucleoprotein (anti-RNP
antibody)
5- HLA Cw6 - psoriasis
The patient's clinical features are highly
Answer & Comments suggestive of mixed connective tissue disease
(MCTD). This diagnosis has been applied to a
Answer: 5- HLA Cw6 - psoriasis particular subset of patients with overlapping
clinical features of lupus, scleroderma, and
Regions of HLA are found on chromosome 6:
myositis. An immune response to U1-RNP is
HLA-AHaemochromatosis (A3) the additional defining serological feature of
MCTD.
HLA-BBehçet's syndrome (B5)

Ankylosing spondylitis (B27)

Dr. Khalid Yusuf El-Zohry – Sohag Teaching Hospital (01118391123)


Ref MRCPass OE OE 2012 PasTest 2009 PassMedicine 2009 PasTest Exam ReviseMRCP 629
Elzohry MRCP Questions Bank (Part 1) – 2013 (For my personal use)

[ Q: 1380 ] PasTest 2009 - Basic 2- Good IgG antibody responses to


Science immunisations
Immunological investigations in a 3- The presence of existing antibody
patient with renal disease are important in the responses to past infections
diagnostic work-up, which of the following 4- Normal IgG subclasses
statements is correct?
5- Normal peripheral blood lymphocyte
1- Henoch-Schönlein purpura is associated subpopulations
with IgG in the mesangium
2- SLE is typically associated with sparse Answer & Comments
deposits of IgG and complement in the
Answer: 2- Good IgG antibody responses to
glomeruli
immunisations
3- C3 nephritic factor is associated with
mesangiocapillary glomerulonephritis type The most common type of primary
I immunodeficiency (>50% of cases) involves
being deficient in antibody production.
4- Minimal-change glomerulonephritis is
Selective IgG deficiencies include the
associated with hypocomplementaemia
decreased production of IgA and/or the
5- Antiglomerular basement-membrane various IgG subclasses and impaired antibody
antibodies are associated with responses to poly
Goodpasture's disease

Answer & Comments


Answer: 5- Antiglomerular basement-
membrane antibodies are associated with
Goodpasture's disease

Minimal-change glomerulonephritis is not


associated with complement activation. C3
nephritic factor is associated with
mesangiocapillary glomerulonephritis type 2,
not type 1. SLE nephritis is an immune
complex problem; hence deposits of IgG and
complement are prolific. Henoch-Schönlein
purpura is typically associated with IgA in the
mesangium. Hence immunological
examination of both a blood and renal biopsy
will help to define the underlying process.

[ Q: 1381 ] PasTest 2009 - Basic


Science
Which of the following definitely excludes IgG
subclass deficiency?
1- Normal serum immunoglobulins

Dr. Khalid Yusuf El-Zohry – Sohag Teaching Hospital (01118391123)


Ref MRCPass OE OE 2012 PasTest 2009 PassMedicine 2009 PasTest Exam ReviseMRCP 630
Elzohry MRCP Questions Bank (Part 1) – 2013 (For my personal use)

Dr. Khalid Yusuf El-Zohry – Sohag Teaching Hospital (01118391123)


Ref
Elzohry MRCP Questions Bank (Part 1) – 2013 (For my personal use)

[ Q: 1385 ] PasTest 2009 - Basic 4- Classical polyarteritis nodosa only affects


Science small arteries
In patients with systemic lupus 5- Behçet's disease is a postcapillary venulitis
erythematosus which of the following
statements is correct? Answer & Comments
1- Up to 80% of patients have anti-dsDNA Answer: 5- Behçet's disease is a postcapillary
antibodies venulitis
2- Patients with anti-dsDNA antibodies are
Takayasu's arteritis affects large arteries,
less likely to have renal disease
whereas both Churg-Strauss syndrome and
3- 10% of patients can be antinuclear Wegener's granulomatosis affect small
antibody-negative arteries. Classical PAN affects medium-sized
4- Ro positivity is associated with Raynaud's arteries.
disease
5- Beta-blockeres should be avoided in [ Q: 1387 ] PasTest 2009 - Basic
patients with SLE Science
A 40-year-old man presents with a purpuric
Answer & Comments rash on his lower limbs. You request
cryoglobulin screening. The report indicates a
Answer: 1- Up to 80% of patients have anti-
moderate amount of a mixed cryoglobulin,
dsDNA antibodies
having both a monoclonal and a polyclonal
Up to 80% of patients with SLE will have anti- component.
double stranded DNA antibodies at some With what are these typically associated?
point during their illness. Patients with anti-
1- Hepatitis B
dsDNA antibodies are more likely to have
renal disease. Less than 5% of patients with 2- Hepatitis C
lupus are ANA-negative. Drugs that induce 3- Chronic lymphocytic leukaemia
lupus do not need to be avoided in the
4- Chronic myeloid leukaemia
idiopathic type of lupus.
5- Active SLE
[ Q: 1386 ] PasTest 2009 - Basic
Science Answer & Comments

When taking a history from a patient with Answer: 2- Hepatitis C


suspected vasculitis, clinical determination of
The cryoglobulin associated with hepatitis C
the size of vessel involved is important in the
typically contains a monoclonal IgM k with
identification of the underlying vasculitic
rheumatoid factor activity that binds to
condition.
polyclonal IgG, hence the term ‘mixed
Which of the following statements is true? cryoglobulin'. The cryoglobulin in CLL usually
1- Takayasu's arteritis mainly affects small represents the monoclonal protein produced
blood vessels by the malignant lymphocyte clone. The
cryoglobulin in hepatitis B and lupus reflects
2- Churg-Strauss syndrome mainly affects
the immune complexes in the condition and is
large blood vessels
typically small and polyclonal. Cryoglobulins
3- Wegener's granulomatosis mainly affects are not associated with CML.
large blood vessels

Dr. Khalid Yusuf El-Zohry – Sohag Teaching Hospital (01118391123)


Ref MRCPass OE OE 2012 PasTest 2009 PassMedicine 2009 PasTest Exam ReviseMRCP 632
Elzohry MRCP Questions Bank (Part 1) – 2013 (For my personal use)

[ Q: 1388 ] PasTest 2009 - Basic Answer & Comments


Science
Answer: 2- Acetylcholine receptors
In a patient presenting acutely with
Myasthenia is characterised by fatigability of
severe abdominal pain presumed to be
striated muscle. It is an autoimmune disease
related to previously diagnosed hereditary
with antibodies against acetylcholine
angioneuritic oedema, which of the following
receptors. There are at least four subgroups of
treatment strategies would be optimal?
myasthenia which differ in age and sex
1- Start/increase the dose of danazol spread, systemic muscle/ocular muscle/both,
2- Start tranexamic acid thymic involvement, antibody titres, HLA
associations and treatment. Antibodies to
3- Give an intravenous C1-inhibitor
acetylcholine esterase would, in theory,
concentrate
increase the amount of available
4- Give intravenous fresh-frozen plasma acetylcholine. Myelin is not a primary target
5- Give intramuscular adrenaline for autoantibodies in this disease. Tensilon
(epinephrine), 0.3 or 0.5 mg (edrophonium) has short-acting
anticholinesterase activity and is used to aid
diagnosis of the disease. Electromyography
Answer & Comments
(EMG) may be helpful in diagnosis, and
Answer: 3- Give an intravenous C1-inhibitor around 90% of patients have raised titres of
concentrate anti-acetylcholine receptor antibodies.
Intravenous C1-inhibitor concentrate, 1000-
1500 units given intravenously over 20-30 [ Q: 1390 ] PasTest 2009 - Basic
minutes, is the optimal treatment. Fresh- Science
frozen plasma, although a traditional Which of the following adverse food reactions
treatment, has the potential to aggravate the is mediated by IgE-dependent mechanisms
condition as it replaces all the complement and hence can be ascertained by skinprick
components not just the C1 inhibitor. testing?
Adrenaline will be ineffective. Danazol and
tranexamic acid have an onset of activity that 1- Monosodium glutamate in Chinese food
is days not minutes, and hence is 2- Scombroid fish poisoning
inappropriate for acute management. 3- Sulphites on prepacked salads
4- Salicylate-induced urticaria
[ Q: 1389 ] PasTest 2009 - Basic
Science 5- Kiwi fruit

Antibodies to which of the following are found


Answer & Comments
in patients with myasthenia gravis?
1- Acetylcholine esterase Answer: 5- Kiwi fruit

2- Acetylcholine receptors Kiwi fruit is a member of the latex-associated


foods and adverse reactions to this fruit are
3- Myelin
mediated by IgE. All the others are examples
4- Striated muscle of food intolerance, so that detailed history-
5- Tensilon taking is essential to making the correct
diagnosis. Scombroid-fish poisoning causes
immediate diffuse redness, diarrhoea and

Dr. Khalid Yusuf El-Zohry – Sohag Teaching Hospital (01118391123)


Ref MRCPass OE OE 2012 PasTest 2009 PassMedicine 2009 PasTest Exam ReviseMRCP 633
Elzohry MRCP Questions Bank (Part 1) – 2013 (For my personal use)

vomiting following the consumption of fish beneficial, which of the following is used for
such as tuna, mackerel and mahi mahi. this indication?
Monosodium glutamate causes abdominal 1- Cyclophosphamide
bloating and vomiting - the so-called ‘Chinese
restaurant syndrome'. Sulphites on prepacked 2- Dapsone
salads causing asthma is called the ‘Salad-bar 3- Methotrexate
syndrome'. 4- Pentoxifylline
5- Thalidomide
[ Q: 1391 ] PasTest 2009 - Basic
Science
Answer & Comments
Selective IgA deficiency is often discovered
incidentally in the investigation of a patient, Answer: 5- Thalidomide
however you will need to explain to the Both methotrexate and pentoxifylline have
patient the clinical significance of such a weak anti-TNF activity, but thalidomide has
finding. clinically significant effects. The problems with
Hence which of the following statements is thalidomide therapy include a potentially
relevant? irreversible neuropathy, teratogenesis,
drowsiness, constipation and weight gain.
1- Predisposition to Staphylococcus aureus
Thalidomide and its derivatives are also used
infection
in the management of leprosy, HIV, myeloma
2- An increased risk of colonic carcinoma and melanoma.
3- IgG2 deficiency leading to recurrent
bacterial infections [ Q: 1393 ] PasTest 2009 - Basic
4- A reduced risk of allergy Science

5- An increased risk of viral infections In an elderly patient found to have a large


IgM-kappa paraprotein, which of the following
Answer & Comments helps to decide that it is related to
Waldenstrom's macroglobulinaemia?
Answer: 3- IgG2 deficiency leading to
1- Light chains in the urine
recurrent bacterial infections
2- No isotype suppression (normal IgG and IgA
IgA deficiency predisposes to pernicious levels)
anaemia and hence gastric carcinoma. The
possibility of IgG2 deficiency should always be 3- Recurrent infections
investigated in IgA-deficient individuals with a 4- Hyperviscosity
history of recurrent bacterial infections, but 5- Thrombocytosis
Staph. aureus is the exception. Allergy and
autoimmune conditions are more common in
IgA-deficient individuals. Answer & Comments
Answer: 2- No isotype suppression (normal
[ Q: 1392 ] PasTest 2009 - Basic IgG and IgA levels)
Science
Recurrent infections are associated with a low
In patients with severe oral and genital serum IgG, not IgA and IgM. Isotype
ulceration, therapy with agents having suppression is more a feature of myeloma, as
clinically significant anti-TNF- a activity can be are light chains in the urine. Hyperviscosity

Dr. Khalid Yusuf El-Zohry – Sohag Teaching Hospital (01118391123)


Ref MRCPass OE OE 2012 PasTest 2009 PassMedicine 2009 PasTest Exam ReviseMRCP 634
Elzohry MRCP Questions Bank (Part 1) – 2013 (For my personal use)

can occur with IgA myeloma as well as in Answer & Comments


patients with Waldenstrom's
Answer: 5- Latex
macroglobulinaemia. Marrow involvement in
myeloma can lead to thrombocytopenia not Latex can induce allergy through IgE bound to
thrombocytosis. mast cells. All the others induce histamine
release via their direct effects on mast cells,
[ Q: 1394 ] PasTest 2009 - Basic except for scombroid fish poisoning which is
Science related to the heat-stable toxin in
tuna/mackerel/mahi mahi, etc.
Regarding B cells and plasma cells, which of
the following is true?
[ Q: 1396 ] PasTest 2009 - Basic
1- B cells and plasma cells have surface-bound Science
IgG
In a healthcare worker with a proven natural
2- B cells and plasma cells have surface MHC rubber/latex allergy, which of the following
class II foods is most commonly associated with this
3- Plasma cells can undergo somatic condition?
hypermutation
1- Bananas
4- B cells can undergo isotype switching 2- Melons
5- Plasma cells can undergo isotype switching 3- Potatoes
4- Tomatoes
Answer & Comments
5- Chestnuts
Answer: 4- B cells can undergo isotype
switching
Answer & Comments
B cells have surface IgG and MHC class II,
Answer: 1- Bananas
undergo somatic hypermutation and isotype
switching (ie switching through the All the above are latex-associated foods, but
immunoglobulin classes). Plasma cells are fully bananas are the commonest. Other common
differentiated cells from B cells, and hence implicated foods include avocados and kiwi
lack these features. fruits. Anyone presenting with possible latex
problems should be asked about problems
[ Q: 1395 ] PasTest 2009 - Basic with the above foods, as well as issues that
Science occur when blowing up balloons, genital
itching after using condoms, problems
Only IgE-mediated allergic reactions can be following vaginal examinations and visits to
formally tested by skinprick testing. the dentist. The latter questions are central to
Adverse reactions to which of the following the clinical diagnosis of latex allergy.
substances can be tested in this manner?
1- Morphine [ Q: 1397 ] PasTest 2009 - Basic
Science
2- Radiocontrast media
3- Scombrotoxins High titres of antithyroid microsomal and
antithyroglobulin antibodies would suggest
4- Colloid plasma expanders which of the following diagnoses in a patient
5- Latex presenting with a complaint of tiredness?

Dr. Khalid Yusuf El-Zohry – Sohag Teaching Hospital (01118391123)


Ref MRCPass OE OE 2012 PasTest 2009 PassMedicine 2009 PasTest Exam ReviseMRCP 635
Elzohry MRCP Questions Bank (Part 1) – 2013 (For my personal use)

Dr. Khalid Yusuf El-Zohry – Sohag Teaching Hospital (01118391123)


Ref MRCPass
Elzohry MRCP Questions Bank (Part 1) – 2013 (For my personal use)

Answer & Comments with defective gluconeogenesis from the


options given above. Glycolysis consumes
Answer: 4- Vitamin B12
glucose: the activity of the glycolytic pathway
No significant absorption of nutrients takes is reduced during starvation, to conserve
place in the stomach. However, because of glucose, with complementary increases in the
the lack of secretion of pepsin, and hence production and oxidation of ketones and fatty
reduced activation of pancreatic proenzymes, acids. Abnormalities of the tricarboxylic acid
and the fact that the ability to eat normal cycle are very rare and do not cause
amounts of food may be greatly decreased, hypoglycaemia.
patients who have had total gastrectomies
may require general nutritional [ Q: 1401 ] PasTest 2009 - Basic
supplementation, eg with proprietary high- Science
energy, high-protein liquids. However, the
absorption of vitamin B12, although it takes A 32-year-old man is referred to a
place in the terminal ileum, is critically dermatologist with soft, fleshy lumps over his
dependent on the availability of intrinsic elbows. Examination also reveals that he has
factor, which is only secreted by the parietal yellow fat deposits in his palmar creases. His
(oxyntic) cells of the stomach. (fasting) serum cholesterol concentration is
12.4 mmol/l, triglyceride 14.2 mmol/l.

[ Q: 1400 ] PasTest 2009 - Basic Which of the following groups of lipoprotein


Science particles is likely to be present in excess?
1- Chylomicrons
A 39-year-old man presents with recurrent
episodes of early morning dizziness, which 2- Intermediate-density lipoproteins (IDL)
resolve rapidly when he has his breakfast. 3- Lipoprotein (a)
Which of the following metabolic pathways is 4- Low-density lipoproteins (LDL)
most likely to be functioning abnormally?
5- Very low-density lipoproteins (VLDL)
1- Gluconeogenesis
2- Glycolysis Answer & Comments
3- Glycogen synthesis Answer: 2- Intermediate-density lipoproteins
4- Ketogenesis (IDL)
5- The tricarboxylic acid cycle (citric acid cycle, The clinical description and elevation of
Krebs' cycle) cholesterol and triglyceride concentrations to
approximately equal levels is typical of familial
Answer & Comments dysbetalipoproteinaemia (also known as
remnant dyslipidaemia, broad beta disease,
Answer: 1- Gluconeogenesis
type-III hyperlipidaemia). This condition is
The history is typical of hypoglycaemia. Blood caused by an inherited abnormality of the
glucose concentration is maintained in the remnant receptor responsible for the removal
fasting state through glucose release from of IDL from the circulation by the liver.
glycogen (glycogenolysis) and gluconeogenesis
Chylomicrons contain little cholesterol: they
(glucose synthesis from lactate and other
are the carriers of dietary lipids in the blood.
precursors). Given that his dizziness occurs in
LDL are the principal carriers of cholesterol.
the morning, this is consistent with the fasting
VLDL contain predominantly triglyceride.
state and thus most likely to be associated
Excesses of each of these can manifest as

Dr. Khalid Yusuf El-Zohry – Sohag Teaching Hospital (01118391123)


Ref MRCPass OE OE 2012 PasTest 2009 PassMedicine 2009 PasTest Exam ReviseMRCP 637
Elzohry MRCP Questions Bank (Part 1) – 2013 (For my personal use)

xanthomas but not of the type described here. attacks, the urine darkens on standing but
Lipoprotein (a) is a modified form of LDL and there is no jaundice. Quinine-like antimalarials
appears to be an independent risk factor for do not precipitate acute attacks of porphyria
coronary disease. in susceptible patients.

[ Q: 1402 ] PasTest 2009 - Basic [ Q: 1403 ] PasTest 2009 - Basic


Science Science
A 21-year-old man is alarmed to find that his A 25-year-old man, previously rather
urine has become unusually dark 2 days sedentary in his habits but alarmed by his
before he is to leave on a backpacking holiday increasing weight, is persuaded by his
in South-East Asia . He consults his GP, who girlfriend to join a gym, and begins an exercise
observes that he is slightly jaundiced. programme. Although there are several
Deficiency of which of the following enzymes similar people enrolled in the programme, he
could best explain these clinical features? finds that he is unable to keep up with them
because of muscle pain, particularly in his legs.
1- Bilirubin glucuronyltransferase Within a few minutes of resting, however, the
2- Glucose 6-phosphate dehydrogenase pains resolve. Investigation revealed a slight
fall in blood lactate concentration during
3- Glucose 6-phosphatase
exercise.
4- Hydroxymethylbilane synthase
Which of the following is most likely to be the
5- Pyruvate kinase cause of his complaint?
1- Decreased muscle glucose uptake
Answer & Comments
2- Decreased hepatic glycogenolysis
Answer: 2- Glucose 6-phosphate
3- Decreased muscle glycogenolysis
dehydrogenase
4- Impaired hepatic gluconeogenesis
Deficiency of red cell glucose 6-phosphate
dehydrogenase is a common inherited 5- Premature atherosclerosis
disorder, which renders red cells susceptible
to increases in oxidative stress. Antimalarial Answer & Comments
drugs are among several that can cause
Answer: 3- Decreased muscle glycogenolysis
haemolysis by increasing oxidative stress in
such patients, although many different An ischaemic cause for his muscle pain is
mutations in the enzyme have been excluded by the failure of a rise in lactate
described, and the range of clinical severity is concentration during exercise. Although
wide. Glucose 6-phosphatase deficiency is the muscle can take up glucose from the blood,
cause of type-1 glycogen storage disease. during intense activity it is partly dependent
on glucose supplied by the breakdown of
Pyruvate kinase deficiency is a rare cause of
muscle glycogen. The enzyme responsible is
haemolytic anaemia. Decreased activity of
muscle phosphorylase, and the history is
bilirubin glucuronyltransferase can cause
typical of McArdle's disease - ie an inherited
unconjugated hyperbilirubinaemia (eg
deficiency of this enzyme.
Gilbert's syndrome, Crigler-Najjar disease);
because the excess bilirubin is unconjugated,
there is no bilirubin in the urine. Deficiency of
porphobilinogen deaminase is the cause of
acute intermittent porphyria. During acute

Dr. Khalid Yusuf El-Zohry – Sohag Teaching Hospital (01118391123)


Ref MRCPass OE OE 2012 PasTest 2009 PassMedicine 2009 PasTest Exam ReviseMRCP 638
Elzohry MRCP Questions Bank (Part 1) – 2013 (For my personal use)

[ Q: 1404 ] PasTest 2009 - Basic [ Q: 1405 ] PasTest 2009 - Basic


Science Science
When taking a history from a 61-year-old A 70-year-old man presents in an acute
woman with disseminated sclerosis, you confusional state. His plasma sodium
ascertain that she is intermittently incontinent concentration is 108 mmol/l, his creatinine is
for liquid stools. 106 micromol/l, urea of 5.0 mmol/l.
Which of the following pertains to the clinical Which of the following findings would be most
physiology of faecal continence? suggestive of the hyponatraemia being due to
1- Voluntary faecal continence is controlled by inappropriate secretion of vasopressin
the internal anal sphincter (antidiuretic hormone, ADH)?

2- Involuntary faecal continence is controlled 1- Plasma osmolality of 230 mmol/kg


by the external anal sphincter 2- Plasma urea concentration 2.5 mmol/l
3- Anal ultrasound is widely used to assess 3- Urine sodium concentration of 50 mmol/l
sphincter function 4- Urine osmolality 510 mosmol/kg
4- The commonest cause of sphincter 5- Plasma albumin concentration of 28 g/l
weakness is pudendal neuropathy
5- Incontinence may be associated with rectal Answer & Comments
prolapse
Answer: 4- Urine osmolality 510 mosmol/kg
Answer & Comments Since plasma osmolality is largely determined
by sodium concentration, a low osmolality
Answer: 5- Incontinence may be associated
would be expected in any patient with
with rectal prolapse
hyponatraemia, unless this is related to the
Involuntary faecal continence is controlled by presence of a high concentration of another
the internal anal sphincter. Voluntary faecal osmotically active substance (e.g. glucose) or
continence is controlled by the external anal to the replacement of plasma water by lipid or
sphincter. Anal ultrasound is widely used to protein. Low plasma urea and albumin
assess sphincter structure, anal manometry is concentrations reflect dilution as a result of
used to assess function. The commonest water overload, irrespective of the cause.
cause of sphincter weakness is sphincter Natriuresis is characteristic of inappropriate
damage and atrophy, although pudendal secretion of ADH, but can also occur when
neuropathy may contribute. Faecal hyponatraemia is due to sodium depletion -
incontinence may be associated with for example, in cerebral salt-wasting, with
constipation, due to overflow. Around 50% of which inappropriate secretion of ADH is often
patients with rectal prolapse may have some confused. The finding of a urinary osmolality
degree of incontinence. The latter can be higher than that of plasma implies defective
graded as incontinence to flatus, liquid stool water excretion, and is characteristic of
or solid stool. Ability to defer defecation by inappropriate secretion of ADH.
more than 15 minutes, use of constipating
medication and the regular usage of pads or [ Q: 1406 ] PasTest 2009 - Basic
anal plugs are also important points to elicit Science
when taking a clinical history.
Some 4 hours after sustaining major trauma in
a road traffic accident, a 22-year-old man, not

Dr. Khalid Yusuf El-Zohry – Sohag Teaching Hospital (01118391123)


Ref MRCPass OE OE 2012 PasTest 2009 PassMedicine 2009 PasTest Exam ReviseMRCP 639
Elzohry MRCP Questions Bank (Part 1) – 2013 (For my personal use)

known to have diabetes, is found to have a Which of the following types of acid-base
high blood glucose concentration. disturbance is most likely?
Increased secretion of which of the following 1- Acute respiratory acidosis
substances is most likely to be responsible? 2- Chronic, compensated respiratory acidosis
1- Adrenaline (epinephrine) 3- Exacerbation of chronic respiratory acidosis
2- Cortisol 4- Mixed respiratory acidosis and metabolic
3- C-reactive protein alkalosis
4- Growth hormone 5- Severe metabolic acidosis
5- Insulin
Answer & Comments
Answer & Comments Answer: 3- Exacerbation of chronic respiratory
Answer: 1- Adrenaline (epinephrine) acidosis

During the metabolic response to trauma, The patient is acidotic but the elevated
there is increased secretion of bicarbonate concentration is incompatible
catecholamines, cortisol, glucagon and growth with a metabolic acidosis. The high p(CO2)
hormone. The first three of these tend to indicates that there is a respiratory acidosis.
increase blood glucose concentration; Were this to be acute, the bicarbonate
catecholamines, cortisol and glucagon act concentration would be normal or only
directly, whereas growth hormone appears to marginally elevated and a higher hydrogen-ion
potentiate the action of cortisol and opposes concentration would be expected (hydrogen-
the action of insulin. Adrenaline and glucagon ion concentration increases by approximately
act most rapidly, by stimulating 5.5 nmol/l for each increase in p(CO2) by 1
glycogenolysis; cortisol tends to act more kPa in an acute disturbance.) Were
slowly, through the stimulation of compensation to be complete, it would be
gluconeogenesis. Insulin is a hypoglycaemic expected that the patient would be less
hormone. C-reactive protein is a marker of acidotic, and the bicarbonate concentration
inflammation, but does not affect glucose higher.
homeostasis. The degree of the The data are compatible with both an acute
neuroendocrine stress response is related to exacerbation of a chronic respiratory acidosis
the severity of the trauma and the time lapse and a mixed respiratory acidosis and
before treatment (fluid replacement, pain metabolic alkalosis, but the clinical setting
relief etc.) is instituted. Hyperglycaemia can makes the former the more likely.
persist for many hours and even days in some
cases.
[ Q: 1408 ] PasTest 2009 - Basic
Science
[ Q: 1407 ] PasTest 2009 - Basic
Science A 19-year-old man presents with clinical
features suggestive of malabsorption.
A 71-year-old man with known chronic
Which of the following investigations has the
obstructive pulmonary disease is admitted to
potential to indicate a single cause for this?
A&E with severe shortness of breath. Blood
gas analysis shows: arterial [H+] 55 nmol/l (pH 1- Breath hydrogen measurement
7.26), p(CO2) 9.4 kPa, p(O2) 9.1 kPa, derived 2- Faecal fat excretion
[HCO3- ] 31 mmol/l.
3- Lactose tolerance test

Dr. Khalid Yusuf El-Zohry – Sohag Teaching Hospital (01118391123)


Ref MRCPass OE OE 2012 PasTest 2009 PassMedicine 2009 PasTest Exam ReviseMRCP 640
Elzohry MRCP Questions Bank (Part 1) – 2013 (For my personal use)

4- Pancreolauryl test (comparison of the 2- Deficiency of apolipoprotein C-II


absorption of fluorescein and fluorescein 3- Deficiency of LDL (low-density lipoprotein)
dilaurate) receptors
5- Xylose tolerance test 4- Deficiency of lipoprotein lipase
5- Increased synthesis of VLDL (very low-
Answer & Comments
density lipoprotein)
Answer: 3- Lactose tolerance test

The measurement of faecal fat excretion (now Answer & Comments


regarded as obsolete by many physicians) is a Answer: 5- Increased synthesis of VLDL (very
test for malabsorption, but it does not low-density lipoprotein)
distinguish between its possible causes. The
pancreolauryl test can be abnormal in Hypertriglyceridaemia occurs frequently in
pancreatic or biliary disease. Breath hydrogen association with a high alcohol intake, and is
excretion is increased with increased bacterial due to the increased synthesis of triglyceride
activity in the gut, but this can be a cause of for incorporation into VLDL in the liver. This is
malabsorption (colonisation of the small gut thought to be as a result of the altered redox
causes deconjugation of bile salts) and also a state consequent on alcohol metabolism,
consequence (fermentation of unabsorbed which favours fat synthesis.
nutrients in the large intestine). The Lipoprotein lipase is responsible for the
absorption of xylose may be decreased in clearance of VLDL and chylomicrons from the
various small intestinal diseases. plasma: it is activated by apolipoprotein C-II.
The lactose tolerance test, in which the rise in Lipoprotein lipase deficiency and
blood glucose following a test dose of lactose apolipoprotein C-II deficiency are rare,
is compared with that following a test dose of inherited causes of severe
glucose, is, however, specific for lactase hypertriglyceridaemia, which usually present
deficiency (though definitive diagnosis is by in childhood. LDL receptors are involved in the
measurement of the enzyme in a biopsy transport of cholesterol from the plasma to
sample). the interior of cells: deficiency is the cause of
familial hypercholesterolaemia. Pancreatic
lipase is normally secreted into the gut.
[ Q: 1409 ] PasTest 2009 - Basic
Although high plasma activities are found in
Science
pancreatitis, this does not affect the
A 47-year-old man with a long history of metabolism of lipoproteins.
alcohol abuse is admitted to hospital with
acute abdominal pain and is diagnosed [ Q: 1410 ] PasTest 2009 - Basic
clinically as having acute pancreatitis. This is Science
confirmed by finding a high serum amylase
activity and by the results of an ultrasound A 49-year-old postmenopausal woman of
scan. He has not had a similar illness before. Southern Asian origin complains of muscle
His serum is seen to be lipaemic: serum weakness. She is found to have
triglyceride concentration is 26 mmol/l. hypocalcaemia, and X-ray examination reveals
two Looser's zones in her left upper femur.
Which of the following is the most likely cause
of the hypertriglyceridaemia? A defect in which of the following physiological
processes is most likely to be the cause of her
1- Decreased pancreatic lipase secretion illness?

Dr. Khalid Yusuf El-Zohry – Sohag Teaching Hospital (01118391123)


Ref MRCPass OE OE 2012 PasTest 2009 PassMedicine 2009 PasTest Exam ReviseMRCP 641
Elzohry MRCP Questions Bank (Part 1) – 2013 (For my personal use)

1- Absorption of calcium from the gut 1- Conversion of testosterone to


2- Osteoblastic activity dihydrotestosterone

3- Osteoclastic activity 2- FSH (follicle-stimulating hormone) secretion

4- Parathyroid hormone secretion 3- Prolactin secretion

5- Renal excretion of calcium 4- Testosterone secretion


5- The androgen receptor
Answer & Comments
Answer & Comments
Answer: 1- Absorption of calcium from the gut
Answer: 5- The androgen receptor
The findings in this woman suggest
osteomalacia, and the most important reason The combination of female phenotype on
for the impaired mineralisation of bone is external examination with 46XY karyotype is
reduced intestinal calcium absorption typical of complete androgen insensitivity
consequent on vitamin D deficiency. The syndrome, which is caused by a mutation that
decreased availability of calcium to mineralise prevents the binding of androgens to the
bone leads to increased osteoblastic activity androgen receptor.
(and hence increased osteoid formation).
A failure of testosterone secretion alone in a
Hypocalcaemia causes increased parathyroid
46XY individual usually results in an
hormone secretion (secondary
ambiguous phenotype; this may also occur
hyperparathyroidism), which stimulates renal
with a failure of conversion of testosterone to
calcium reabsorption (hence reduced
dihydrotestosterone (DHT) (5a-reductase
excretion). Thus, while this woman may have
deficiency): the development of the male
increased osteoblastic activity and increased
external genitalia in utero is largely dependent
PTH secretion, both these are secondary to
on DHT. However, in this condition,
vitamin D deficiency and decreased intestinal
masculinisation may occur at puberty as a
absorption of calcium.
result of the increased secretion of
Osteoporosis (postmenopausal osteoporosis is testosterone. Neither an abnormal secretion
due to increased osteoclastic activity) is not of FSH or prolactin causes this presentation.
associated with hypocalcaemia.
[ Q: 1412 ] PasTest 2009 - Basic
[ Q: 1411 ] PasTest 2009 - Basic Science
Science
A 20-year-old man is referred to the
A 17-year-old phenotypic female with no endocrinology clinic because of delayed
clinical evidence of dysmorphism is referred to puberty. On examination, he has a
the Endocrine Clinic for the investigation of preadolescent body habitus and no evidence
primary amenorrhoea. On examination, breast of the development of secondary sexual
development is normal, the external genitalia characteristics. Serum testosterone, LH
are normal female in appearance but pubic (luteinising hormone) and FSH (follicle-
hair is sparse. Her karyotype is found to be stimulating hormone) concentrations are all in
46XY. the prepubertal range. The LH and FSH
A defect in which of the following hormonal concentrations increase only slightly following
processes is most likely to be the abnormality a single injection of GnRH (gonadotrophin-
causing this presentation? releasing hormone) but a normal response is

Dr. Khalid Yusuf El-Zohry – Sohag Teaching Hospital (01118391123)


Ref MRCPass OE OE 2012 PasTest 2009 PassMedicine 2009 PasTest Exam ReviseMRCP 642
Elzohry MRCP Questions Bank (Part 1) – 2013 (For my personal use)

elicited after GnRH is given daily for seven most appropriate to provide his energy
consecutive days. requirements?
Which of the following is most likely to be the 1- Dextrose 50% 500 ml, lipid emulsion 20%
cause of his delayed puberty? 500 ml
1- A hypothalamic disorder 2- Dextrose 50% 500 ml, lipid emulsion 20%
2- A pituitary disorder 750 ml

3- Klinefelter's syndrome 3- Dextrose 50% 500 ml, lipid emulsion 20%


1000 ml
4- Mumps orchitis in childhood
4- Dextrose 50% 750 ml, lipid emulsion 20%
5- Seminiferous tubular dysfunction 500 ml
5- Dextrose 50% 1000 ml, lipid emulsion 20%
Answer & Comments
500 ml
Answer: 1- A hypothalamic disorder
Answer & Comments
The combination of low testosterone and low
gonadotrophin concentration suggests a Answer: 4- Dextrose 50% 750 ml, lipid
defect at the level of the pituitary or emulsion 20% 500 ml
hypothalamus.
Dextrose 50% and 20% lipid emulsions each
In Klinefelter's syndrome (46XXY) and provide about 478 J (2000 kcal)/l. His
testicular failure due to mumps orchitis, the requirements would thus be met by a
lack of negative feedback leads to increased combined total of about 1250 ml of these
gonadotrophin concentrations. In fluids. It is usually recommended that lipid
seminiferous tubule dysfunction, the plasma should provide not more than half the total
FSH concentration tends to be elevated; energy requirements: in the combination in
spermatogenesis is reduced or absent but option B, it would provide 60%; in D, 40%. The
masculinisation is normal. The lack of former is too high. The other combinations
response to a single injection of GnRH, but either provide too much or too little energy.
normal response following chronic
stimulation, suggests that the mechanism for [ Q: 1414 ] PasTest 2009 - Basic
the secretion of FSH and LH by the pituitary is Science
intact. Chronic understimulation as a result of
a lack of GnRH causes decreased pituitary A 25-year-old man is admitted to hospital with
responsiveness to the hypothalamic hormone, persistent vomiting. He is clinically dehydrated
which can be restored by chronic stimulation. and hypotensive. His serum sodium
concentration is 124 mmol/l, potassium 4.9
mmol/l, urea 9.8 mmol/l, creatinine 96 m
[ Q: 1413 ] PasTest 2009 - Basic
mol/l. Urine sodium concentration in a
Science
specimen passed on admission is 62 mmol/l.
A 24-year-old man is being provided with
Which of the following is the most likely cause
long-term parenteral nutrition for Crohn's
of the hyponatraemia?
disease. His energy requirement is estimated
to be approximately 597 J (2500 kcal)/24 h. 1- Adrenal failure

Which of the following combinations of lipid 2- Cerebral salt wasting


emulsion and dextrose solutions would be 3- Gastrointestinal fluid loss
4- Low sodium intake

Dr. Khalid Yusuf El-Zohry – Sohag Teaching Hospital (01118391123)


Ref MRCPass OE OE 2012 PasTest 2009 PassMedicine 2009 PasTest Exam ReviseMRCP 643
Elzohry MRCP Questions Bank (Part 1) – 2013 (For my personal use)

5- Syndrome of inappropriate antidiuresis alpha-1-antitrypsin (A1AT), leading to


(SIAD) decreased A1AT activity in be blood and lungs,
and deposition of excessive abnormal A1AT
Answer & Comments protein in liver cells. The responsible genetic
defect in AAT deficiency affects 1 in 3000-
Answer: 1- Adrenal failure 5000 individuals. Fortunately, not every
Natriuresis in a dehydrated, hyponatraemic individual with AAT deficiency develops
patient suggests that there is uncontrolled clinically significant disease. There are several
renal loss of sodium, such as occurs in adrenal genotypes that may produce alpha-1-
failure. Cerebral salt wasting can also cause antitrypsin deficiency (of varying severity and
dehydration and hyponatraemia due to clinical presentation). The production of the
excessive natriuresis, but typically occurs alpha-1-antiprotease is controlled by a pair of
following a head injury or brain surgery. genes at the protease inhibitor (Pi) locus.
Hyponatraemia and dehydration due to Nearly 24 variants of the alpha-1-antiprotease
gastrointestinal fluid loss or sodium deficiency molecule have been identified, and all are
due to a low intake should lead to renal inherited as codominant alleles. The most
conservation of sodium. Although SIAD is an common (90%) allele is M (PiM), and
important cause of hyponatraemia and homozygous individuals (MM) produce normal
sodium excretion may be high, the amounts of alpha-1. The most common form
hyponatraemia is due to water excess and of AAT deficiency is associated with allele Z, or
patients are not dehydrated. homozygous PiZ (ZZ). Serum levels of AAT in
these patients are about 10-15% of normal
serum levels. Serum levels greater than 11
[ Q: 1415 ] PasTest 2009 - Basic
mmol/L appear to be protective. Emphysema
Science
develops in most (but not all) individuals with
Which of the following statements is true in serum levels less than 9 mmol/L. Around 4%
the epidemiology of disease associated with a- of people of northern European descent carry
1-antitrypsin deficiency? the PiZ allele and between 1 in 625 and 1 in
2000 are estimated to by homozygous. This
1- Smoking is not a major risk factor
contrasts with cystic fibrosis (the most
2- There is an increased incidence of hyper- common lethal disease inherited by the white
reactive airways in adult life population), which has a prevalence of 1 per
3- It is an autosomal dominant disorder with 3200. Emphysema results from the
low penetrance uncontrolled action of the proteases on the
lung tissue. There is accelerated age-related
4- The disorder is an indication for liver
decline in the FEV1, which is exacerbated by
transplantation in a child
smoking. Smoking is the greatest risk factor
5- Most cases present during the neonatal for the development of panacinar emphysema
period in adults with a-1-antitrypsin deficiency. In
non-smokers this may never develop or occur
Answer & Comments later in life. Jaundice and cirrhosis occur
because the hepatocytes are unable to
Answer: 4- The disorder is an indication for
secrete the protein. Progressive liver damage
liver transplantation in a child
in an infant in the neonatal period is an
Alpha-1-antitrypsin deficiency (α1-antitrypsin indication for liver transplant.
deficiency, A1AD or Alpha-1) is a genetic
disorder caused by defective production of

Dr. Khalid Yusuf El-Zohry – Sohag Teaching Hospital (01118391123)


Ref MRCPass OE OE 2012 PasTest 2009 PassMedicine 2009 PasTest Exam ReviseMRCP 644
Elzohry MRCP Questions Bank (Part 1) – 2013 (For my personal use)

[ Q: 1416 ] PasTest 2009 - Basic The pathogenesis of Parkinson's disease is


Science multifactorial, characterised by a progressive
death of heterogeneous populations of
Which one of the following is higher at the
neurones particularly in the substantia nigra,
apex of the lung than at the base when a
resulting in a regional loss of the
person is standing?
neurotransmitter dopamine. A 60-70% loss of
1- V/Q ratio neurones occurs prior to the emergence of
2- Ventilation symptoms.

3- paCO2
[ Q: 1418 ] PasTest 2009 - Basic
4- Compliance Science
5- Blood flow
The secretion of growth hormone is increased
by?
Answer & Comments
1- Hyperglycaemia
Answer: 1- V/Q ratio
2- Exercise
The alveoli at the apex of the lung are larger 3- Somatostatin
than those at the base so their compliance is
4- Growth hormone
less. Because of the reduced compliance, less
inspired gas goes to the apex than to the base. 5- Free fatty acids
Also, because the apex is above the heart
level, less blood flows through the apex than Answer & Comments
through the base. However, the reduction in
air flow is less than the reduction in blood Answer: 2- Exercise
flow, so that the V/Q ratio at the top of the Growth hormone (GH) is synthesised, stored,
lung is greater than it is at the bottom. The and secreted by the endocrine cells of the
increased V/Q ratio at the apex makes PaCO2 anterior pituitary. Its release is stimulated by
lower and PaO2 higher at the apex than they growth hormone-releasing hormone and
are at the base . inhibited by somatostatin. Numerous factors
serve as a stimulus for GH release, including
[ Q: 1417 ] PasTest 2009 - Basic hypoglycaemia (e.g. insulin administration),
Science moderate to severe exercise, stress due to
emotional disturbances, illness, and fever, and
The primary neurochemical disturbance in
dopamine agonists such as bromocriptine.
idiopathic Parkinson's disease involves
1- Noradrenaline
[ Q: 1419 ] PasTest 2009 - Basic
2- Dopamine Science
3- Gamma-aminobutyric acid (GABA) A 65-year-old smoker with a history of
4- Substance P transient ischaemic attacks presents with loss
of co-ordination of his right side. On
5- Adrenaline
examination there is decreased skin sensation
in the right half of his face and the left half of
Answer & Comments his body. In addition he has a right Horner
Answer: 2- Dopamine syndrome.
Which vessel is most likely to be involved?

Dr. Khalid Yusuf El-Zohry – Sohag Teaching Hospital (01118391123)


Ref MRCPass OE OE 2012 PasTest 2009 PassMedicine 2009 PasTest Exam ReviseMRCP 645
Elzohry MRCP Questions Bank (Part 1) – 2013 (For my personal use)

1- Basilar artery One of the most common lesions at this site is


2- Posterior inferior cerebellar artery carpal tunnel syndrome, in which the median
nerve is compressed as it passes deep to the
3- Middle cerebral artery flexor retinaculum.
4- Internal carotid artery
The usual presentation is with
5- Posterior cerebral artery acroparaesthesias. This consists of numbness,
tingling and burning sensations felt in the
Answer & Comments hand and fingers; the pain sometimes radiates
up the forearm as far as the elbow or even as
Answer: 2- Posterior inferior cerebellar artery
high as the shoulder or root of the neck.
Involvement of the posterior inferior Although the paraesthesias are sometimes
cerebellar artery is called Wallenberg's restricted to the radial fingers, they may affect
syndrome, with ipsilateral cerebellar ataxia all the digits as some fibres from the median
through infarction of the inferior part of the nerve are distributed to the fifth finger
cerebellum. In addition, it causes a through a communication with the ulnar
combination of deficits through infarction of nerve in the palm.
the dorsolateral medulla, decreased skin
The attacks of pain and paraesthesias are
sensation in the ipsilateral half of the face and
most common at night and often wake the
the contralateral half of the body, ipsilateral
patient from sleep. They are then relieved by
Horner's syndrome, ipsilateral weakness of
shaking the hand. The hand tends to feel
the soft palate, larynx and pharynx and also
numb and useless on waking in the morning
causes rotatory vertigo.
but recovers after it has been used for some
minutes. The symptoms may recur during the
[ Q: 1420 ] PasTest 2009 - Basic day following use, or at other times if the
Science patient sits with the hands immobile.
A 49-year-old woman has been complaining of Such symptoms of acroparaesthesias may
a tingling feeling in her right hand at night and persist for many years without the
is under the impression that her hand is appearance of symptoms of median nerve
swollen although there is no obvious oedema. damage. In other patients, weakness of the
In the last few days she has noticed numbness thenar muscles develops, particularly with
in her right index finger and the tip of her abduction of the thumb, and is associated
thumb, especially while working. with atrophy of the lateral aspect of the
Which nerve is most likely to be responsible for thenar eminence. Sensory loss may appear
her symptoms? over the tips of the median innervated fingers.
1- Ulnar nerve
[ Q: 1421 ] PasTest 2009 - Basic
2- Median nerve
Science
3- Radial nerve
A 21-year-old man takes advantage of his
4- Nerve root C7 employer's health benefits and undergoes a
5- Nerve root C8 private health screen. Urinalysis is positive for
protein on two occasions, once at the health
Answer & Comments screen and a later repeated sample at the GP.
His serum urea and creatinine concentrations
Answer: 2- Median nerve are normal.

Dr. Khalid Yusuf El-Zohry – Sohag Teaching Hospital (01118391123)


Ref MRCPass OE OE 2012 PasTest 2009 PassMedicine 2009 PasTest Exam ReviseMRCP 646
Elzohry MRCP Questions Bank (Part 1) – 2013 (For my personal use)

Which of the following would be the most


important investigations to perform next?
1- Creatinine clearance
2- Measurement of urine protein immediately
on getting out of bed in the morning
3- Mid-stream urine for culture and sensitivity
4- Urine protein electrophoresis
5- 24-hour urine protein excretion

Answer & Comments


Answer: 2- Measurement of urine protein
immediately on getting out of bed in the
morning

Proteinuria can occur because of glomerular


damage, tubular damage (hence the
decreased absorption of the small amount of
low molecular weight proteins that are
normally filtered by the glomeruli), an
increased concentration of low molecular
weight proteins in the blood (eg
immunoglobulin light chains in myeloma -
Bence Jones - protein) and secretion of
proteins into the urine (eg in urinary tract
infection). Tubular proteins and Bence Jones
protein can be detected by electrophoresis of
concentrated urine. However, particularly in a
young person, orthostatic proteinuria should
be excluded before proceeding to further
investigation.

In this benign condition, protein may be


present in the urine when the individual is
erect, but not when they have been supine for
a period.

Dr. Khalid Yusuf El-Zohry – Sohag Teaching Hospital (01118391123)


Ref MRCPass OE OE 2012 PasTest 2009 PassMedicine 2009 PasTest Exam ReviseMRCP 647
Elzohry MRCP Questions Bank (Part 1) – 2013 (For my personal use)

The metabolic alkalosis secondary to a loss of Grade II shock15-30% (750 ml - 1.5 l) blood-
gastric acid is a hypochloraemic alkalosis. volume loss, systolic blood pressure is usually
There is predominant loss of H+ and chloride normal but a tachycardia is present
ions. The alkalosis is perpetuated by the
Grade III shock30-40% (1.5-2 litres) loss,
hypochloraemia, which prevents renal
hypotension, tachycardia and fall in urine
excretion of the excess bicarbonate since its
output seen
proximal tubular reabsorption (with sodium) is
enhanced. Provision of adequate chloride ions Grade IV shock> 40% (> 2 l) blood-volume loss,
allows the excess bicarbonate to be excreted anuria and severe shock observed
and corrects the alkalosis. Dextrose 5%
contains no chloride and dextrose saline [ Q: 1425 ] PasTest 2009 - Basic
contains insufficient for this purpose. Twice Science
normal saline is occasionally used for treating
severe hyponatraemia but has no place in this A 24-year-old man, a new immigrant to the UK
clinical situation. Ringer's lactate is from Eastern Europe, who had worked in his
inappropriate, since the metabolism of the own country as a demolition contractor,
lactate that it contains to bicarbonate would undergoes a medical examination and is found
exacerbate the alkalosis. to have glycosuria. His blood lead
concentration is high.
[ Q: 1424 ] PasTest 2009 - Basic Which of the following findings would suggest
Science that he has a generalised disorder of proximal
tubular function?
You are called to see a 56-year-old man 2 h
after a cardiac catheterisation. He is actively 1- Albuminuria
bleeding from his catheter site and his 2- Hyperkalaemia
dressings and bedclothes are soaked with
3- Increased serum urea concentration
blood.
4- Inability to acidify the urine
Which of the following statements is true?
5- Metabolic acidosis
1- Grade I shock applies with up to a 20% loss
of circulating blood volume
Answer & Comments
2- Loss of 2 litres of blood is consistent with
normal systolic blood pressure Answer: 5- Metabolic acidosis
3- The pulse can remain normal in patients Bicarbonate is normally totally reabsorbed
with grade I shock from the glomerular filtrate in the proximal
4- Anuria is pathognomonic of grade III shock convoluted tubule. Decreased bicarbonate
absorption causes metabolic acidosis (renal
5- Grade IV shock is seen with a 30% loss of
tubular acidosis-type 2); however, the urine
circulating blood volume
can be acidified as the plasma bicarbonate
concentration (and hence the bicarbonate
Answer & Comments load for reabsorption) falls and distal tubular
Answer: 3- The pulse can remain normal in hydrogen-ion secretion is intact. The
patients with grade I shock proteinuria associated with proximal tubular
damage typically affects mainly low molecular
Grade I shockLoss of up to 15% (750ml) of weight proteins that are filtered by the
blood volume; blood pressure is normal but glomerulus. There is no reduction in
there may be a slight tachycardia glomerular filtration and hence no increase in

Dr. Khalid Yusuf El-Zohry – Sohag Teaching Hospital (01118391123)


Ref MRCPass OE OE 2012 PasTest 2009 PassMedicine 2009 PasTest Exam ReviseMRCP 648
Elzohry MRCP Questions Bank (Part 1) – 2013 (For my personal use)

serum urea concentration. Decreased


proximal tubular potassium reabsorption
leads to hypokalaemia (unusually in acidosis,

Dr. Khalid Yusuf El-Zohry – Sohag Teaching Hospital (01118391123)


Ref MRCPass OE OE 2012 PasTest 2009 PassMedicine 2009 PasTest Exam ReviseMRCP 649
Elzohry MRCP Questions Bank (Part 1) – 2013 (For my personal use)

aetiologies for his renal failure and suggest Answer & Comments
that it is longstanding?
Answer: 1- Rise in pH
1- Anaemia
All the above shift the dissociation curve to
2- Small kidneys on ultrasound examination the right, with the exception of a rise in pH.
3- High serum parathyroid hormone
concentration [ Q: 1430 ] PasTest 2009 - Basic
4- Hypotension Science
5- Hyperuricaemia You have performed a liver biopsy, and shortly
after the procedure the patient develops pain
Answer & Comments on the tip of his right shoulder.

Answer: 2- Small kidneys on ultrasound Which nerve is most likely to be responsible for
examination his pain?
1- Right phrenic nerve
The kidneys are usually decreased in size in
chronic renal failure, except when this is due 2- Axillary nerve
to amyloid or polycystic disease. Kidneys jare 3- Right vagus
usually normal in size in advanced diabetic
4- Right sympatheticus
nephropathy, whereas affected kidneys are
initially enlarged from hyperfiltration. 5- Intercostobrachial nerve
Hypertension can be both a cause and a
consequence of renal failure. Uric acid Answer & Comments
nephropathy can cause renal failure but
hyperuricaemia is usual in chronic renal Answer: 1- Right phrenic nerve
failure, due to decreased excretion. Anaemia The phrenic nerve on both sides originates
(due to decreased erythropoietin secretion) from the ventral rami of thethird to fifth
and secondary hyperparathyroidism cervical nerves. It passes inferiorly down the
(decreased calcitriol secretion causes neck to the lateral border of the scalenus
hypocalcaemia) are features, not causes, of anterior, then it passes medially across the
chronic renal failure. border of scalenus anterior parallel to the
internal jugular vein that lies inferomedially.
[ Q: 1429 ] PasTest 2009 - Basic
The right phrenic nerve pierces the diaphragm
Science
in its tendinous portion just slightly lateral to
The oxygen-haemoglobin dissociation curve is the inferior vena caval foramen. It then forms
shifted to the left by which of the following three branches on the inferior surface of the
factors? diaphragm: anterior, lateral and posterior.
1- Rise in pH These ramify out in a radial manner from the
point of perforation to supply all but the
2- Rise in 2,3-DPG (2,3-diphosphoglycerate) periphery of the muscle.
3- Rise in plasma temperature
4- Rise in blood CO2 content [ Q: 1431 ] PasTest 2009 - Basic
Science
5- Fall in plasma bicarbonate concentration
A patient on enteral nutrition develops
constipation.

Dr. Khalid Yusuf El-Zohry – Sohag Teaching Hospital (01118391123)


Ref MRCPass OE OE 2012 PasTest 2009 PassMedicine 2009 PasTest Exam ReviseMRCP 650
Elzohry MRCP Questions Bank (Part 1) – 2013 (For my personal use)

What could explain the underlying clinical [ Q: 1433 ] PasTest 2009 - Basic
physiology? Science
1- Hyperosmolar feed You are researching a new agent for the
2- Bacterial contamination management of hypoxia related to acute
pneumonia. Part of the assessment includes
3- Low feed temperature
changes in the pulmonary vasculature in
4- Inadequate fluid replacement response to hypoxia.
5- Reduced intestinal absorptive capacity When considering acute hypoxia, which of the
following is true of the pulmonary
Answer & Comments vasculature?

Answer: 4- Inadequate fluid replacement 1- An area of lung affected by pneumonia is


likely to experience vasodilatation
Hyperosmolar feed, bacterial contamination,
2- An area of lung unaffected by pneumonia is
low feed temperature, too rapid or irregular
likely to experience vasodilatation
administration, lactose intolerance, reduced
intestinal absorptive capacity can all explain 3- An area of lung unaffected by pneumonia is
diarrhoea. likely to experience vasoconstriction
4- Increased cardiac output promotes
[ Q: 1432 ] PasTest 2009 - Basic pulmonary vasoconstriction
Science 5- Hypothermia promotes pulmonary
A 56-year-old woman sustains a myocardial vasoconstriction
infarction. ST elevation and Q waves are
present in leads V4-V6 , I and AVL. Answer & Comments
Which of the following aspects of the heart is Answer: 2- An area of lung unaffected by
most likely to have been involved in the pneumonia is likely to experience
infarct? vasodilatation
1- Anterior
When an area of lung is affected by hypoxia,
2- Anterolateral but CO2 is not raised, a process of shunting
3- Anteroseptal occurs, where vasodilatation occurs in areas
of lung where gas exchange is better, and
4- Inferior vasoconstriction restricts the blood supply to
5- Lateral the poorly oxygenated areas of the lung. This
in effect increases the functional part of the
Answer & Comments lung available for gas exchange. Increased
cardiac output, hypothermia, and a number of
Answer: 2- Anterolateral pharmacological agents such as nitrous oxide
This combination suggests an anterolateral and calcium antagonists inhibit this process.
infarct. Purely anterior infarcts tend to involve
the chest leads only (typically V2-V5), [ Q: 1434 ] PasTest 2009 - Basic
anteroseptal V1-V3, lateral infarcts chest leads Science
only (I, II, AVL) and inferior infarcts II, III and
The mean lower oesophageal sphincter
AVF.
pressure is reduced in which of the following
situations?

Dr. Khalid Yusuf El-Zohry – Sohag Teaching Hospital (01118391123)


Ref MRCPass OE OE 2012 PasTest 2009 PassMedicine 2009 PasTest Exam ReviseMRCP 651
Elzohry MRCP Questions Bank (Part 1) – 2013 (For my personal use)

1- Grade III oesophagitis skeletal deterioration. Acid phosphatase


2- Grade IV oesophagitis post-laparoscopic activity is often high in serum.
Nissens fundoplication
[ Q: 1436 ] PasTest 2009 - Basic
3- Achalasia
Science
4- Nutcracker oesophagus
An otherwise well, 62-year-old woman with
5- Diffuse oesophageal spasm
hypertension is referred to an outpatient clinic
for assessment. Despite treatment with a
Answer & Comments thiazide diuretic, a beta-adrenergic antagonist
Answer: 1- Grade III oesophagitis (beta-blocker) and an angiotensin-converting
enzyme (ACE) inhibitor, her blood pressure is
Grade III oesophagitis, whether or not 158/92 mmHg. Investigations show serum
associated with a sliding hiatus hernia, has sodium concentration is 138 mmol/l, serum
reduced mean lower sphincter pressures. The potassium concentration 3.2 mmol/l, serum
other conditions would have an increased creatinine concentration 108 mmol/l.
pressure, together with differing motility
What is the most likely cause of the
patterns.
hypokalaemia?
1- A low dietary potassium intake
[ Q: 1435 ] PasTest 2009 - Basic
Science 2- Excessive secretion of aldosterone

Which enzyme in serum is increased in 3- The ACE inhibitor


Gaucher's disease? 4- The beta-adrenergic antagonist
1- Alkaline transferase (ALT) 5- The thiazide diuretic
2- Aspartate transaminase (AST)
Answer & Comments
3- Lactate dehydrogenase (LDH)
4- Acid phosphatase Answer: 5- The thiazide diuretic

5- Alkaline phosphatase (ALP) Most normal diets contain adequate amounts


of potassium. Increased aldosterone secretion
Answer & Comments causes increased potassium excretion and
hypokalaemia, but aldosteronism is an
Answer: 4- Acid phosphatase uncommon cause of hypertension. ACE
Gaucher's disease, a rare, inherited, and inhibitors and β-blockers tend to raise plasma
potentially fatal disorder, is characterised by potassium concentration. Thiazides cause
decreased levels of the enzyme hypokalaemia through increasing the delivery
glucocerebrosidase. Deficiency of of sodium to the distal part of the distal
glucocerebrosidase leads to the accumulation convoluted tubule where sodium is
of the lipid glucocerebroside within the reabsorbed and potassium excreted: the
lysosomes of the monocyte macrophage sodium depletion caused by the diuretic
system. Lipid-engorged cells with eccentric stimulates this exchange through increasing
nuclei, known as Gaucher cells, constitute the the production of aldosterone.
primary defect in Gaucher's disease. Gaucher
cells lead to the displacement of healthy,
normal cells in bone marrow,
hepatosplenomegaly, organ dysfunction, and

Dr. Khalid Yusuf El-Zohry – Sohag Teaching Hospital (01118391123)


Ref MRCPass OE OE 2012 PasTest 2009 PassMedicine 2009 PasTest Exam ReviseMRCP 652
Elzohry MRCP Questions Bank (Part 1) – 2013 (For my personal use)

[ Q: 1437 ] PasTest 2009 - Basic into the mitochondria is accomplished via an


Science acyl-carnitine intermediate, which itself is
generated by the action of carnitine
Which of the following statements about
acyltransferase I, an enzyme that resides in
prostaglandin synthesis is correct?
the outer mitochondrial membrane. The acyl-
1- It is activated by glucocorticoids carnitine molecule is then transported into
2- It is produced by lipoxygenase the mitochondria where carnitine
acyltransferase II catalyses the regeneration of
3- It is activated by aspirin
the fatty acyl-CoA molecule. Deficiencies in
4- It is mediated by cyclooxygenase carnitine lead to an inability to transport fatty
5- It causes vasoconstriction acids into the mitochondria for oxidation. This
can occur in newborns and particularly in pre-
term infants. Carnitine deficiencies are also
Answer & Comments
found in patients undergoing haemodialysis or
Answer: 4- It is mediated by cyclooxygenase exhibiting organic aciduria. Carnitine
deficiencies may manifest systemic
Prostaglandins (PGs) are biosynthesised from
symptomatology or may be limited to only
straight-chain C20 fatty acids, arachidonic acid
muscles. Symptoms can range from mild
being the precursor. The responsible enzyme
occasional muscle cramping to severe
is cyclooxygenase, leukotrienes are formed by
weakness or even death. Treatment is by oral
lipoxygenase. Glucocorticoids and aspirin
carnitine administration.
inhibit the prostaglandin synthesis.
Prostaglandins lead to vasodilatation. The
amounts of PGs in the tissues are very low, [ Q: 1439 ] PasTest 2009 - Basic
indicating that they are biosynthesised Science
immediately before hormone-stimulated A patient who had a myocardial infarction 6
release. PGs seem to be metabolised by all months ago is diagnosed as having gingival
body tissues. hypertrophy.
Which drug is most likely to be responsible for
[ Q: 1438 ] PasTest 2009 - Basic this?
Science
1- Atorvastatin
Fatty acids transported from adipose tissue to
2- Isosorbide mononitrate
the muscle.
3- Aspirin
Which enzyme is essential for this process?
4- Amlodipine
1- Carnitine acyltransferase I
5- Atenolol
2- Xanthine oxidase
3- Arginine hydroxylase Answer & Comments
4- Phosphoribosyl pyrophosphate synthetase
Answer: 4- Amlodipine
5- Cyclooxygenase
Drug-induced gingival hypertrophy occurs as a
side-effect of some systemic medications. The
Answer & Comments
pharmacological agents mainly associated
Answer: 1- Carnitine acyltransferase I with gingival overgrowth are:

Oxidation of fatty acids occurs in the


mitochondria. The transport of fatty acyl-CoA

Dr. Khalid Yusuf El-Zohry – Sohag Teaching Hospital (01118391123)


Ref MRCPass OE OE 2012 PasTest 2009 PassMedicine 2009 PasTest Exam ReviseMRCP 653
Elzohry MRCP Questions Bank (Part 1) – 2013 (For my personal use)

phenytoin, a drug used for the management [ Q: 1441 ] PasTest 2009 - Basic
of epilepsy, and other anticonvulsants such as Science
sodium valproate, phenobarbital, vigabatrin
A patient has the following urea and
ciclosporin an immunosuppressant drug used electrolytes results: Sodium 140 mmol/l
to reduce organ transplant rejection Potassium 4 mmol/l Chloride 105 mmol/l
Bicarbonate 20mmol/l Calculate the anion
calcium-channel blockers (nifedipine,
gap.
verapamil, diltiazem, oxodipine, amlodipine),
a group of antihypertensive drugs. 1- 19 meq/l
2- 5 meq/l
Other drugs, such as antibiotics
(erythromycin) and hormones, have also been 3- 10 meq/l
also associated with this side-effect. 4- 30 meq/l
5- 0 meq/l
[ Q: 1440 ] PasTest 2009 - Basic
Science
Answer & Comments
A 20-year-old female presents with the
Answer: 1- 19 meq/l
following blood gases: pH 7.48 p(CO2) 3.9
mmHg HCO3- 22 mmol/l H+ 35mmol/l Anion gap = ([Na+] + [K+]) - ([Cl-] + [HCO3-])
What is the most likely reason for these (all units mmol/l).
results? Normal range is 8-16 meq/l
1- Amitriptyline
2- Cushing's syndrome [ Q: 1442 ] PasTest 2009 - Basic
Science
3- Hepatic failure
4- Pregnancy In relation to the nutritional physiology of
patients, which of the following would
5- Diazepam overdose
represent appropriate nitrogen requirements
(g N/kg per day) and calorie requirements
Answer & Comments (kcal/kg per day)?
Answer: 4- Pregnancy 1- Reduced food intake: nitrogen requirement
0.3 g N/kg per day, calorie requirement 35
Respiratory alkalosis results from
kcal/kg per day
hyperventilation which is manifested by
excess elimination of CO2 from the blood and 2- Moderate injury: nitrogen requirement 0.15
a rise in the blood pH. Examples of specific g N/kg per day, calorie requirement 25
causes are listed below: kcal/kg per day
3- Moderate sepsis: nitrogen requirement 0.3
catastrophic central nervous system (CNS)
g N/kg per day, calorie requirement 15
event (CNS haemorrhage)
kcal/kg per day
drugs (salicylates, progesterone) 4- Severe injury: nitrogen requirement 0.3 g
pregnancy (especially the third trimester) N/kg per day, calorie requirement 35
kcal/kg per day
decreased lung compliance (interstitial lung
5- Severe sepsis: nitrogen requirement 0.2 g
disease)
N/kg/day, calorie requirement 15
anxiety kcal/kg/day

Dr. Khalid Yusuf El-Zohry – Sohag Teaching Hospital (01118391123)


Ref MRCPass OE OE 2012 PasTest 2009 PassMedicine 2009 PasTest Exam ReviseMRCP 654
Elzohry MRCP Questions Bank (Part 1) – 2013 (For my personal use)

Answer & Comments 2- Diffusion capacity increased, FEV1/FVC


normal, total lung capacity increased
Answer: 4- Severe injury: nitrogen
requirement 0.3 g N/kg per day, calorie 3- Diffusion capacity normal, FEV1/FVC
requirement 35 kcal/kg per day reduced, total lung capacity reduced

Usual ranges for: 4- Diffusion capacity decreased, FEV1/FVC


normal, total lung capacity normal
reduced food intake: nitrogen requirement
5- Diffusion capacity decreased, FEV1/FVC
0.15-0.2 g N/kg per daycalorie requirement
increased, total lung capacity increased
25-30 kcal/kg per day

moderate injury/sepsis: nitrogen requirement Answer & Comments


0.2-0.3 g N/kg per daycalorie requirement 30-
35 kcal/kg per day Answer: 1- Diffusion capacity decreased,
FEV1/FVC normal, total lung capacity reduced
severe injury/sepsis: nitrogen requirement
0.3-0.35 g N/kg per daycalorie requirement Diffusion capacity is characteristically
35-40 kcal/kg per day decreased in restrictive lung disorders.
FEV1/FVC reduced would be seen in
obstructive airways disease, which would be
[ Q: 1443 ] PasTest 2009 - Basic
reversible in asthma and irreversible in COPD.
Science In restrictive conditions FEV1/FVC ratio is
What is the importance of the P1 receptor? normal or increased. Total lung capacity is
reduced in restrictive lung disease, whilst it is
1- Inducing apoptosis
normal or increased in obstructive airways
2- Increasing cytotoxic drug excretion from disease.
cells
3- Increasing cytotoxic drug metabolite [ Q: 1445 ] PasTest 2009 - Basic
clearance Science
4- Calcium channel activator What is the mechanism of action of
5- Potassium channel activator erythropoietin when used as a performance-
enhancing drug?
Answer & Comments 1- Improvement of renal function
Answer: 1- Inducing apoptosis 2- Increase in muscle mass

P1 receptors are involved in mast cell function 3- Improvement of blood pressure control
and eosinophil apoptosis. 4- Improvement of exercise tolerance
5- Reduction in pain
[ Q: 1444 ] PasTest 2009 - Basic
Science Answer & Comments
Which of the following respiratory physiology Answer: 4- Improvement of exercise tolerance
tests would be consistent with a diagnosis of
moderately established cryptogenic fibrosing Erythropoietin is used as a performance-
alveolitis? enhancing drug. The hormone regulates the
red cell mass in the human body. By increasing
1- Diffusion capacity decreased, FEV1/FVC
red cell mass and therefore the oxygen-
normal, total lung capacity reduced
carrying capacity of the blood, more oxygen is

Dr. Khalid Yusuf El-Zohry – Sohag Teaching Hospital (01118391123)


Ref MRCPass OE OE 2012 PasTest 2009 PassMedicine 2009 PasTest Exam ReviseMRCP 655
Elzohry MRCP Questions Bank (Part 1) – 2013 (For my personal use)

delivered to exercising muscles. Benefits of 1- Leptin


even a short course of the substance can last 2- Somatostatin
for weeks, but it can be detected in urine only
for a few days after the most recent injection. 3- CCK
Twenty-six young cyclists have died after using 4- Secretin
erythropoietin since it was introduced into the 5- Gherelin
world of competitive cycling in the late 1980s.

Answer & Comments


[ Q: 1446 ] PasTest 2009 - Basic
Science Answer: 5- Gherelin

A patient undergoes respiratory function Gherelin is a hormone produced in the fundus


tests. of the stomach and in the pancreas, the levels
of which increase before meals and decrease
Which of the following are normal readings for
afterwards. Receptors for gherelin are found
a 70-kg man?
in the arcuate nucleus and the hypothalamus.
1- Peak expiratory flow of 376 l/min Leptin is involved in satiety, somatostatin, CCK
2- Total lung capacity of 3.5 litres and secetin are all hormones which have been
investigated as potential weight loss agents.
3- Functional residual capacity of 3.5 litres
4- Tidal volume of 250 ml
[ Q: 1448 ] PasTest 2009 - Basic
5- Inspiratory reserve volume of 2 litres Science
Glucokinase exists in brain, pancreas and liver,
Answer & Comments and responds differently in different locations.
Answer: 5- Inspiratory reserve volume of 2 In the brain, response is governed merely by
litres different glucose levels, while in liver
glucokinase activity increases after meals.
Normal readings for such a patient would be:
Downregulation of glucokinase activity in the
peak expiratory flow520-700 l/min liver is an example of which of the following?
total lung capacity 5-6.5 litres 1- Affinity

functional residual capacity2-3 litres 2- Specificity


3- Co-activation
tidal volume500-700 ml
4- Stereoisomerism
[ Q: 1447 ] PasTest 2009 - Basic 5- Co-repression
Science
You are asked to review a 15-year-old boy Answer & Comments
with abnormal weight gain and learning Answer: 5- Co-repression
difficulties. As part of your assessment you
arrange for an assessment of peptide Glucokinase activity in the liver is
hormones to assess if a deficiency may be downregulated by binding with glucokinase
involved in the pathogenesis of his condition. regulatory protein (GKRP). Co-repression is
the term given to the fact that by binding to
Elevated levels of which of the following may GKRP, the action of glucokinase is
be involved in the pathogenesis of his downregulated or repressed. Small levels of
condition?

Dr. Khalid Yusuf El-Zohry – Sohag Teaching Hospital (01118391123)


Ref MRCPass OE OE 2012 PasTest 2009 PassMedicine 2009 PasTest Exam ReviseMRCP 656
Elzohry MRCP Questions Bank (Part 1) – 2013 (For my personal use)

glucose lead to the release of glucokinase 5- Increase in renal sodium excretion


from GKRP. In addition, insulin acts via
SREBP1c to upregulate production of Answer & Comments
glucokinase, and a number of other promoter
genes are also involved. Mutations in the Answer: 1- Increase of aquaporin-2 in
glucokinase coding gene which lead to loss of collecting duct
function are responsible for MODY 2, (one of Aquaporin-2 expression is increased in
the forms of maturity onset diabetes of the response to vasopressin over a number of
young). Co-activation would be the opposite days. Increased aquaporin-2 expression then
of this, where the presence of a co-factor leads to a reduction in free water excretion
leads to up-regulation of activity. Affinity and and so can protect against extreme
specificity are to do with the level of binding dehydration. Even so, this woman has an
to glucose and how specific glucokinase may elevated sodium, creatinine and urea, and is in
be for glucose or other sugars, and pre-renal failure. Lithium, used to treat bipolar
steroisomerism implies that differing raceimic disorder is thought to decrease expression of
forms of the enzyme may have different aquaporin-2, and hence lead to nephrogenic
glucose activation thresholds. Non of these diabetes insipidus in some patients.
factors are relevant here.

[ Q: 1450 ] PasTest 2009 - Basic


[ Q: 1449 ] PasTest 2009 - Basic Science
Science
A patient on total parenteral nutrition (TPN)
A 19-year-old woman is found in the desert regimen presents with drowsiness and
following an accident 7 days earlier. She is abnormal serum electrolytes.
severely dehydrated, drowsy and confused.
What is the most likely cause?
On examination she looks very dry and has a
BP of 90/65 mmHg. 1- Hypocalcaemia

Investigations: 2- Hypercalcaemia

Hb 14.9 g/dl 3- Hypernatraemia

WCC 6.0 x 109/L 4- Hypophosphataemia

PLT 190 x 109/L 5- Hypomagnesaemia

Na+ 145 mmol/l


Answer & Comments
K+ 5.4 mmol/l
Answer: 4- Hypophosphataemia
Creatinine 198 μmol/l
Administering carbohydrate lowers serum
Urea 21.0 mmol/l
phosphate by stimulating the release of
Which of the following is the adaptive insulin, which moves phosphate and glucose
mechanism which has prevented her from into cells. This so-called refeeding syndrome
dying from dehydration? occurs when starving or chronically
1- Increase of aquaporin-2 in collecting duct malnourished patients are re-fed or given
intravenous (iv) glucose. Phosphate deficiency
2- Decrease in ADH
commonly impairs neurological function,
3- Reduction in GFR which may be manifested by confusion,
4- Decrease in BP seizures, and coma. Peripheral neuropathy
and ascending motor paralysis, similar to

Dr. Khalid Yusuf El-Zohry – Sohag Teaching Hospital (01118391123)


Ref MRCPass OE OE 2012 PasTest 2009 PassMedicine 2009 PasTest Exam ReviseMRCP 657
Elzohry MRCP Questions Bank (Part 1) – 2013 (For my personal use)

Guillain–Barré syndrome, may also occur. to the accumulation of pyrimidine nucleotides


Weakness of skeletal or smooth muscle is the in red cells, which in turn reduces the stability
most common clinical manifestation of of the cell membrane (and is seen on a blood
phosphate deficiency. It can involve any film as basophilic stippling).
muscle group, alone or in combination,
ranging from ophthalmoplegia to proximal [ Q: 1452 ] PasTest 2009 - Basic
myopathy, to dysphagia or ileus. Respiratory Science
insufficiency may occur in some patients with
severe hypophosphataemia, particularly when Which one of the following porphyrias can be
the underlying cause is malnourishment. both acquired and inherited?
Impaired cardiac contractility occurs, leading 1- Acute intermittent porphyria
to generalised signs of myocardial depression.
2- Congenital erythropoietic porphyria
The hypophosphataemic myocardium also has
a reduced threshold for ventricular 3- Erythropoietic protoporphyria
arrhythmias. 4- Porphyria cutanea tarda (cutaneous hepatic
porphyria)
[ Q: 1451 ] PasTest 2009 - Basic 5- Variegate porphyria
Science
Which one of the following features in an Answer & Comments
adult patient presenting with porphyrinuria
Answer: 4- Porphyria cutanea tarda
would most suggest lead poisoning rather
(cutaneous hepatic porphyria)
than acute intermittent porphyria as a cause?
1- Abdominal pain The porphyrias are a group of inherited
metabolic diseases in which the metabolism of
2- Anaemia porphyrins, the precursors of haem, is
3- Foot drop disturbed. They are classified as acute
4- Hypertension (neurological), in which porphyrin precursors
are produced in excess, and chronic, in which
5- Seizures photosensitising porphyrins are produced in
excess. Most are inherited as autosomal-
Answer & Comments dominant conditions, except for congenital
erythropoietic porphyria (autosomal
Answer: 2- Anaemia
recessive). Although in some instances
Abdominal pain, motor neuropathies and porphyria cutanea tarda is inherited
seizures can be features of both lead (autosomal dominant), most cases are
poisoning and acute intermittent porphyria sporadic, and occur in patients with liver
(although encephalopathy is more common in diseases, especially alcohol-related.
lead poisoning in children than in adults).
Autonomic neuropathy in acute intermittent [ Q: 1453 ] PasTest 2009 - Basic
porphyria can cause hypertension but this is Science
not a feature of lead poisoning. Anaemia
occurs only in lead poisoning: it is due to What is the medical treatment of choice for
inhibition of ferrocheletase (the activity of this the majority of patients with carcinoid
enzyme is normal in AIP) and a decrease in red syndrome?
cell lifespan. This is also a result of enzyme 1- Conventional chemotherapy
inhibition (pyrimidine 5'-nucleotidase) leading
2- Cyproheptadine

Dr. Khalid Yusuf El-Zohry – Sohag Teaching Hospital (01118391123)


Ref MRCPass OE OE 2012 PasTest 2009 PassMedicine 2009 PasTest Exam ReviseMRCP 658
Elzohry MRCP Questions Bank (Part 1) – 2013 (For my personal use)

3- Interferon-a 2- Racial variant


4- Methysergide 3- Recent exercise
5- Octreotide 4- Statin treatment
5- Thiazide treatment
Answer & Comments
Answer: 5- Octreotide Answer & Comments

Although conventional chemotherapy (eg with Answer: 2- Racial variant


5-fluorouracil and streptozotocin) may lead to Myocardial infarction is excluded by the
a partial remission in some patients, it is not normal troponin concentration, and the
widely used for this indication. Infusion of 5- creatine kinase would have been expected to
fluorouracil into the hepatic artery may be have fallen to normal 4 days after an MI, as
valuable in some patients. Interferon-a is would an elevated CK due to exercise.
effective at controlling symptoms (and may Thiazides are not reported as increasing CK
have an antitumour effect) in some patients. activity, but moderate increases occur in some
Partial serotonin antagonists such as patients on statin treatment. Black people
cyproheptadine and methysergide may frequently have CK activities up to three times
sometimes be helpful. Octreotide, an the normal levels seen in Caucasians.
analogue of somatostatin, reduces the
secretion of hormones by the tumour and
[ Q: 1455 ] PasTest 2009 - Basic
appears to reduce the sensitivity of tissues to
Science
them, and is regarded as the first-line medical
treatment for the majority of patients. A young man is admitted to the emergency
Surgical debulking may be appropriate prior to room unconscious and hyperventilating. His
medical treatment, but surgery is rarely breath smells of alcohol.
curative, as by the time patients develop the
Which of the following findings would suggest
carcinoid syndrome the primary tumour has
a specific cause for his condition?
usually spread to the liver.
1- Blood glucose concentration of 12 mmol/l
[ Q: 1454 ] PasTest 2009 - Basic 2- Ketonuria
Science 3- Metabolic acidosis
A 54-year-old Afro-Caribbean man consults his 4- Serum calcium concentration (corrected)
family doctor because of the chest discomfort 1.62 mmol/l
he first noticed 4 days ago after a session
5- Serum sodium concentration 131 mol/l
digging in his garden, but which he says is now
resolving. He has previously been well, but is
being treated with a statin for Answer & Comments
hypercholesterolaemia, and a thiazide and a Answer: 4- Serum calcium concentration
calcium-channel antagonist for hypertension. (corrected) 1.62 mmol/l
Serum creatine kinase activity is 425 U/l
(normal up to 150 U/l); serum troponin-T Patients with diabetic ketoacidosis sometimes
concentration is normal. have only moderately elevated blood glucose
concentrations, but mild hyperglycaemia can
What is the most likely explanation for the occur in any stressed patient. Ketosis can
elevated creatine kinase? occur in DKA, alcoholic ketoacidosis and with
1- Myocardial infarction prolonged fasting. Many drugs can cause

Dr. Khalid Yusuf El-Zohry – Sohag Teaching Hospital (01118391123)


Ref MRCPass OE OE 2012 PasTest 2009 PassMedicine 2009 PasTest Exam ReviseMRCP 659
Elzohry MRCP Questions Bank (Part 1) – 2013 (For my personal use)

metabolic acidosis. Mild hyponatraemia is also [ Q: 1457 ] PasTest 2009 - Basic


a non-specific finding. Severe hypocalcaemia, Science
particularly in this clinical context, is highly
In metabolic alkalosis associated with
suggestive of poisoning with ethylene glycol.
prolonged nasogastric aspiration in
This is oxidised to various organic acids,
postoperative ileus, what is the most
including oxalic acid, which combines with
important cause of the acid-base disturbance?
calcium to produce insoluble calcium oxalate:
oxalate crystals are often present in the urine. 1- Hypoventilation
The only other poison that causes 2- Increased renal bicarbonate reabsorption
hypocalcaemia is hydrofluoric acid.
3- Loss of gastric acid

[ Q: 1456 ] PasTest 2009 - Basic 4- Potassium depletion


Science 5- Secondary aldosteronism

An 8-year-old child is admitted to hospital


having ingested some of her mother's iron Answer & Comments
tablets (ferrous fumarate) 4 hours earlier. Her Answer: 3- Loss of gastric acid
serum iron concentration is 182 mmol/l. Plain
abdominal X-ray is unremarkable. Loss of unbuffered gastric acid is the cause of
the metabolic alkalosis seen under these
What would be the most useful therapeutic
circumstances if there is inadequate
measure?
replacement of the fluid lost with intravenous
1- Gastric lavage with desferrioxamine physiological saline. Increased renal
2- Induction of vomiting bicarbonate reabsorption (needed to allow
adequate renal sodium reabsorption in the
3- Intravenous infusion of desferrioxamine
presence of hypochloraemia), potassium
4- Oral activated charcoal depletion (gastric secretions contain about 10
5- Whole bowel irrigation mmol/l of potassium) and secondary
aldosteronism (a result of extracellular fluid
loss) all help to maintain the alkalosis, but
Answer & Comments
they do not cause it. Hypoventilation is a
Answer: 3- Intravenous infusion of compensatory change: on its own,
desferrioxamine hypoventilation causes carbon dioxide
retention and a respiratory acidosis.
Gastric lavage should only be considered in
patients with iron poisoning who present
within an hour of taking the tablets. Inducing [ Q: 1458 ] PasTest 2009 - Basic
vomiting after this time is ineffective, and, Science
anyway, iron toxicity often causes vomiting. A young woman is admitted to A&E having
Activated charcoal poorly adsorbs inorganic taken an aspirin overdose. The plasma
poisons such as iron. Whole bowel irrigation concentration of salicylate is 550 mg/l (4.0
may be helpful if undissolved tablets remain in mmol/l).
the gut (they are radio-opaque), as is more
What clinical feature would most suggest that
likely to occur when slow-release iron
another drug has been taken in addition to
preparations have been ingested. However,
aspirin?
the standard treatment is an intravenous
infusion of desferrioxamine, which chelates 1- Coma
iron and facilitates its urinary excretion. 2- Hyperventilation

Dr. Khalid Yusuf El-Zohry – Sohag Teaching Hospital (01118391123)


Ref MRCPass OE OE 2012 PasTest 2009 PassMedicine 2009 PasTest Exam ReviseMRCP 660
Elzohry MRCP Questions Bank (Part 1) – 2013 (For my personal use)

3- Sweating usually a rapid one), it would be normal. The


4- Tinnitus hydrogen-ion concentration is too low to be
accounted for by a respiratory acidosis alone:
5- Vomiting there must therefore be a metabolic acidosis
in addition (as the low bicarbonate
Answer & Comments concentration also indicates).
Answer: 1- Coma
[ Q: 1460 ] PasTest 2009 - Basic
Hyperventilation, nausea and vomiting, Science
sweating and tinnitus frequently occur in mild
to moderate salicylate poisoning (plasma A young man is admitted to A&E having taken
salicylate < 700 mg/l (5.1 mmol/l)). Confusion an overdose of an unknown drug. On
and delirium may occur with more severe examination, he is hyperventilating and has a
poisoning, but it is rare for patients to be tachycardia. Results of arterial blood gas
unconscious. analysis (breathing room air) are: [H+] 30
nmol/l (pH 7.56); p(CO2) 3.5 kPa; p(O2) 14.2
kPa; [HCO3-] 21 mmol/l.
[ Q: 1459 ] PasTest 2009 - Basic
Science Which of the following drugs is he most likely
to have taken?
A 24-year-old, unconscious man is admitted to
A&E. No history is available. The results of 1- A benzodiazepine
arterial blood gas analysis are: [H+] 80 nmol/l 2- A tricyclic antidepressant
(pH 7.1), p(CO2) 7.0 kPa, p(O2) 8.2 kPa, [HCO3-] 3- Cocaine
17.1 mmol/l.
4- Paracetamol (acetaminophen)
These results indicate which one of the
following acid-base disturbances? 5- Theophylline

1- Metabolic acidosis with respiratory


compensation Answer & Comments

2- Mixed metabolic and respiratory acidosis Answer: 5- Theophylline

3- Respiratory acidosis The blood gas data indicate an acute


4- Respiratory acidosis with metabolic respiratory alkalosis. This is particularly
alkalosis associated with poisoning with a salicylate and
theophylline, both of which are respiratory
5- Uncompensated metabolic acidosis stimulants. Benzodiazepines and tricyclic
antidepressants may cause respiratory
Answer & Comments depression in overdose, and hence respiratory
acidosis. Cocaine can cause a tachycardia (and
Answer: 2- Mixed metabolic and respiratory
ventricular arrhythmias and acute myocardial
acidosis
infarction) but not, usually, hyperventilation.
The high hydrogen-ion concentration (low pH) Paracetamol poisoning does not usually cause
indicates acidosis. The elevated p(CO2) an acid-base disturbance unless significant
indicates a respiratory component; in liver damage occurs, when there may be
compensated metabolic acidosis, p(CO2) is metabolic or, sometimes, respiratory,
reduced; in an uncompensated metabolic alkalosis.
acidosis (a very unusual situation, since the
respiratory response to a metabolic acidosis is

Dr. Khalid Yusuf El-Zohry – Sohag Teaching Hospital (01118391123)


Ref MRCPass OE OE 2012 PasTest 2009 PassMedicine 2009 PasTest Exam ReviseMRCP 661
Elzohry MRCP Questions Bank (Part 1) – 2013 (For my personal use)

[ Q: 1461 ] PasTest 2009 - Basic 5- Vertical banded gastroplication


Science
A 41-year-old man has a body mass index of Answer & Comments
38 kg/m2. He is normotensive and a non- Answer: 5- Vertical banded gastroplication
smoker. A fasting blood glucose concentration
is normal. Both sibutramine and orlistat have been
shown to induce and maintain a greater
His mortality in comparison with an otherwise
weight loss than diet alone, but a patient's
comparable individual of normal body weight
weight often plateaus before adequate weight
is increased primarily because of an increased
loss has occurred. Orlistat is only licensed for
risk of which one of the following?
use for 1 year in the UK, and sibutramine for 2
1- Carcinoma of colon years. Energy-deficient diets, particularly if
2- Cardiovascular disease coupled with increased exercise, are effective,
but the lost weight is almost invariably
3- Chronic respiratory disease
regained, as it is after jaw-wiring and milk
4- Hypertension feeding. Surgery offers the best chance of
5- Type-2 diabetes mellitus achieving long-term weight loss, the results
from vertical banded gastroplication
combined with a by-pass procedure being
Answer & Comments
even better than those with gastroplication
Answer: 2- Cardiovascular disease alone.

The risk of all these conditions is increased in


severe obesity, diabetes being the disease [ Q: 1463 ] PasTest 2009 - Basic
most strongly associated. Up to 75% of new Science
cases of type-2 diabetes are thought to be Which of the following is the most important
related to obesity. In terms of mortality, causative factor in the development of obesity
however, the most significant association is in the majority of patients?
with cardiovascular disease - both coronary
1- Energy intake in excess of expenditure
heart disease and cerebrovascular disease.
2- Genetic predisposition
[ Q: 1462 ] PasTest 2009 - Basic 3- Insulin resistance
Science 4- Intrauterine malnutrition
A 34-year-old woman with a body mass index 5- Leptin deficiency
of 44 kg/m2 seeks medical help for her
obesity.
Answer & Comments
Which one of the following treatments offers
Answer: 1- Energy intake in excess of
her the highest probability of achieving a long-
expenditure
term reduction in weight?
1- An energy-deficient diet (600 kcal/day (~ The ultimate cause of obesity is always an
143 J/day) less than requirements) for 6 intake of energy in excess of expenditure, but
months many factors govern both intake and
expenditure. There is undoubtedly a genetic
2- Jaw-wiring and milk feeding for 3 months
predisposition in some individuals.
3- Treatment with orlistat for 12 months Intrauterine malnutrition may be important in
4- Treatment with sibutramine for 12 months others. Leptin deficiency is a very rare cause

Dr. Khalid Yusuf El-Zohry – Sohag Teaching Hospital (01118391123)


Ref MRCPass OE OE 2012 PasTest 2009 PassMedicine 2009 PasTest Exam ReviseMRCP 662
Elzohry MRCP Questions Bank (Part 1) – 2013 (For my personal use)

of obesity: more frequently, obese individuals [ Q: 1465 ] PasTest 2009 - Basic


are resistant to the actions of leptin. Insulin Science
resistance is probably a consequence of
A 55-year-old diabetic patient complains of
obesity, not a cause.
soreness of his tongue, he has also noticed
white plaques on his tongue.
[ Q: 1464 ] PasTest 2009 - Basic
Which antibody in the saliva is the most
Science
important host defence in preventing further
In deciding whether to provide nutritional spreading of the disease?
support to a malnourished patient either
1- IgA
enterally or parenterally, which of the
following is the most important 2- IgG
consideration? 3- IgM
How long nutritional support is likely to be 4- IgD
required
5- IgE
1- The patient's nutritional requirements
2- Whether the small intestine is functioning Answer & Comments
normally
Answer: 1- IgA
3- Whether the patient can swallow
IgA is a dimeric immunoglobulin with a half-
4- Whether there is a risk of aspiration of
life of 5 days; it is predominantly found in
gastric contents
secretions such as tears, saliva, colostrum,
5- sweat and mucus. Due to its extensive
secretion it is often seen as the body's major
Answer & Comments antibody. It protects the host by preventing
organisms attaching and penetrating epithelial
Answer: 2- Whether the small intestine is cells. IgAs do not have complement receptors.
functioning normally

The paramount consideration in deciding [ Q: 1466 ] PasTest 2009 - Basic


whether to provide nutritional support Science
enterally or parenterally is whether the small
Which of the following statements is most
intestine is functioning normally. If it is, then
consistent with the Crigler-Najjar syndrome?
enteral nutrition should be attempted into the
stomach, or, if there is a significant risk of 1- Autosomally inherited, severe conjugated
aspiration or delayed gastric emptying, into hyperbilirubinaemia
the jejunum. In the short term, this can be 2- Autosomally inherited, severe
achieved by a nasogastric or nasojejunal tube; unconjugated hyperbilirubinaemia
in the long term, a percutaneous gastrostomy
3- Autosomal-recessive, mild unconjugated
or jejunostomy may be appropriate. Both
hyperbilirubinaemia
enteral and parenteral nutrition can be
continued long term if necessary. The 4- X-linked, severe conjugated
patient's nutritional requirements only hyperbilirubinaemia
become a limiting factor if they cannot be 5- X-linked, severe unconjugated
absorbed fully from the gut, in which case hyperbilirubinaemia
parenteral supplementation may be required.

Dr. Khalid Yusuf El-Zohry – Sohag Teaching Hospital (01118391123)


Ref MRCPass OE OE 2012 PasTest 2009 PassMedicine 2009 PasTest Exam ReviseMRCP 663
Elzohry MRCP Questions Bank (Part 1) – 2013 (For my personal use)

Answer & Comments [ Q: 1468 ] PasTest 2009 - Basic


Science
Answer: 2- Autosomally inherited, severe
unconjugated hyperbilirubinaemia Which of the following patterns of serum lipids
is most characteristic of diabetes mellitus?
This is a rare syndrome that can either be
autosomal-recessive (type 1) or autosomal- 1- Elevated LDL-cholesterol, elevated HDL-
dominant (type 2). Severe unconjugated cholesterol, elevated triglycerides
hyperbilirubinaemia results from an absence 2- Elevated LDL-cholesterol, elevated HDL-
(type 1) or decrease (type 2) of glucuronyl cholesterol, normal triglycerides
transferase. As, in effect, it is a prehepatic
3- Elevated LDL-cholesterol, low HDL-
problem, liver histology is normal. Only those
cholesterol, normal triglycerides
with the type-2 disease can survive to
adulthood. There is no available treatment 4- Normal LDL-cholesterol, elevated HDL-
except liver transplantation. cholesterol, elevated triglycerides
5- Normal LDL-cholesterol, low HDL-
[ Q: 1467 ] PasTest 2009 - Basic cholesterol, elevated triglycerides
Science
Which of the following is the initial treatment Answer & Comments
of choice for the hyponatraemia in the Answer: 5- Normal LDL-cholesterol, low HDL-
majority of patients with the syndrome of cholesterol, elevated triglycerides
inappropriate [secretion of] ADH (antidiuretic
hormone, vasopressin)? The LDL-cholesterol concentration is usually
normal or only slightly elevated in patients
1- Intravenous infusion of hypertonic saline with diabetes (unless they have another cause
2- Intravenous infusion of isotonic saline of hypercholesterolaemia) even though the
3- Oral demeclocycline LDL particles are abnormal, being denser,
richer in triglyceride and thus more
4- Oral frusemide atherogenic. HDL-cholesterol is typically low,
5- Restriction of water intake and triglycerides are elevated. Achievement of
good glycaemic control may reverse these
Answer & Comments abnormalities in type-1 diabetes (HDL-
cholesterol may even be increased). However,
Answer: 5- Restriction of water intake treatment may ameliorate, but usually does
Restriction of water intake is the treatment of not normalise, the abnormalities in type-2
choice for asymptomatic, hyponatraemic diabetes.
patients with SIADH: if the water intake is less
than the water output, the plasma sodium [ Q: 1469 ] PasTest 2009 - Basic
concentration will increase. If patients are Science
symptomatic, then hypertonic saline may be
A 55-year-old woman is diagnosed with type 2
appropriate, but it should always be given
diabetes mellitus. Her weight is 76 kg, body
cautiously; frusemide is sometimes given at
mass index 34 kg/m2. After 3 months' trial of
the same time. Demeclocycline can also be
dietary modification, she has lost 2 kg in
used, but is potentially nephrotoxic. Neither
weight, but her Hb A1c, which was 10.2% at
isotonic saline nor frusemide on its own are
diagnosis, is 9.6%.
appropriate.
The most appropriate treatment would now
be which of the following?

Dr. Khalid Yusuf El-Zohry – Sohag Teaching Hospital (01118391123)


Ref MRCPass OE OE 2012 PasTest 2009 PassMedicine 2009 PasTest Exam ReviseMRCP 664
Elzohry MRCP Questions Bank (Part 1) – 2013 (For my personal use)

1- Acarbose 3- The result of the OGTT confirms a diagnosis


2- A sulphonylurea of diabetes

3- A thiazolidinedione 4- The result of the OGTT indicates impaired


glucose tolerance
4- Insulin
5- The result of the OGTT indicates that he is
5- Metformin at increased risk of microvascular disease

Answer & Comments Answer & Comments


Answer: 5- Metformin Answer: 4- The result of the OGTT indicates
Metformin, which acts principally by impaired glucose tolerance
increasing the sensitivity to insulin, is the The combination of a fasting venous plasma
treatment of choice in obese patients with glucose concentration of < 7.0 mmol/l and a 2-
type 2 diabetes, provided that the serum h value of 7.8 mmol/l or greater but < 11.1
creatinine concentration is not elevated (there mmol/l is diagnostic of impaired glucose
is an increased risk of lactic acidosis with tolerance. However, this is not associated with
metformin in renal impairment). an increased risk of microvascular
Sulphonylureas are valuable oral complications (although the risk of
hypoglycaemic agents, but may cause weight macrovascular complications is increased). A
gain. Thiazolidinediones are not yet licensed diagnosis of diabetes requires a fasting
for monotherapy in the UK, but may be useful glucose concentration of 7.0 mmol/l or more,
alternatives to metformin in the future. and/or a 2-h value of 11.1 mmol/l or more. In
Acarbose, an a-glucosidase inhibitor, is a weak the absence of osmotic symptoms, a single
hypoglycaemic agent and is used only in blood glucose measurement should never be
combination with other agents. This patient used to diagnose diabetes; the presence of
may well come to require insulin, but oral glycosuria may prompt investigations for
treatment should be tried first. diabetes, but does not contribute to diagnosis.

[ Q: 1470 ] PasTest 2009 - Basic [ Q: 1471 ] PasTest 2009 - Basic


Science Science
A 42-year-old man consulted his family doctor A 38-year-old woman with type-1 diabetes
because of a 2-3 month history of lethargy reports at her regular clinic review that she
and feeling generally unwell. The history was has been experiencing frequent episodes of
otherwise unremarkable. His urine tested hypoglycaemia during the previous 6 months,
positive for glucose, and a random venous despite reducing her overall dose of insulin by
plasma glucose concentration was 8.3 mmol/l. almost 50%. Her Hb A1c in the clinic is 5.6%,
The family doctor arranged an oral glucose having been 6.8 % a year previously. She also
tolerance test: glucose concentration at says she has lost some weight and that her
baseline 5.6 mmol/l, 9.3 mmol/l at 120 periods are becoming scanty and irregular,
minutes. and wonders if she is having an early
Which of the following statements is correct? menopause. Gonadotrophin concentrations
1- Diabetes could have been diagnosed on the are in the normal early follicular range.
random glucose value alone What is the most likely cause of the reduction
2- The combination of the random glucose in her insulin requirement?
and glycosuria are diagnostic of diabetes 1- Addison's disease

Dr. Khalid Yusuf El-Zohry – Sohag Teaching Hospital (01118391123)


Ref MRCPass OE OE 2012 PasTest 2009 PassMedicine 2009 PasTest Exam ReviseMRCP 665
Elzohry MRCP Questions Bank (Part 1) – 2013 (For my personal use)

2- Hypopituitarism 3- Serum lactate concentration 4.3 mmol/l


3- Increased exercise 4- Serum triglyceride concentration 12.2
4- Spontaneous regression of diabetes mmol/l

5- Weight loss 5- White cell count of 15 x 109/l

Answer & Comments Answer & Comments

Answer: 2- Hypopituitarism Answer: 1- Serum amylase activity of 1244 U/l


(upper limit of normal, 150 U/l)
Type-1 diabetes is irreversible. Exercise,
weight loss and decreased production of Patients in DKA often have elevated serum
counter-regulatory hormones can all result in amylase activities as a result of decreased
decreased insulin requirements. The latter can renal excretion of the enzyme, but a level this
occur with Addison's disease (resulting in a high is very suggestive of pancreatitis.
decreased secretion of cortisol) and Hypertriglyceridaemia is common in
hypopituitarism (decreased secretion of uncontrolled diabetes. An increased white cell
growth hormone and cortisol). Autoimmune count can occur in any acute illness and does
Addison's disease occurs more frequently in not specifically indicate infection. The serum
patients with type-1 diabetes than in non- creatinine concentration is often slightly
diabetic subjects, as does autoimmune raised in patients with DKA (even in the
ovarian failure. However, the evidence of absence of diabetic nephropathy) as a result
decreasing ovarian function without an of a decreased glomerular filtration rate.
increase in gonadotrophin concentrations Decreased tissue perfusion as a result of
makes hypopituitarism the most likely cause dehydration often leads to an element of
here. The lower limit of detection in many lactic acidosis in addition to the ketoacidosis.
gonadotrophin assays includes the lower limit
of the normal follicular range, so that frankly [ Q: 1473 ] PasTest 2009 - Basic
low concentrations, such as may occur in Science
hypopituitarism, may not be distinguishable
A 75-year-old man with prostatism has a
from low-normal values.
serum prostate-specific antigen (PSA)
concentration of 15 ng/l.
[ Q: 1472 ] PasTest 2009 - Basic
Which of the following statements is true with
Science
regard to this result?
A 42-year-old man with type-1 diabetes 1- It could be explained by prostatitis
mellitus, which was diagnosed 23 years ago, is
admitted to hospital with diabetic 2- It is diagnostic of malignancy
ketoacidosis. He is complaining of abdominal 3- It is likely to be invalidated if he underwent
pain. a digital rectal examination 48 h before the
Which of the following findings on admission blood sample was taken
would most suggest an acute event has 4- It is prognostically highly significant
preciptated his DKA? 5- It is unremarkable in a man of this age
1- Serum amylase activity of 1244 U/l (upper
limit of normal, 150 U/l) Answer & Comments
2- Serum creatinine concentration 140 m
Answer: 1- It could be explained by prostatitis
mol/l

Dr. Khalid Yusuf El-Zohry – Sohag Teaching Hospital (01118391123)


Ref MRCPass OE OE 2012 PasTest 2009 PassMedicine 2009 PasTest Exam ReviseMRCP 666
Elzohry MRCP Questions Bank (Part 1) – 2013 (For my personal use)

Prostatitis and acute urinary retention can dilution of the plasma by the excess water,
both result in increased serum PSA but it could also suggest chronic liver disease,
concentrations; digital rectal examination has another cause of chronic hyponatraemia.
a minor, and only transient effect. This level is
certainly compatible with malignancy, but is [ Q: 1475 ] PasTest 2009 - Basic
not diagnostic of it. In general, the higher the Science
PSA, the greater the likelihood of malignancy,
but some patients with malignancy have Which of the following is an acute porphyria?
normal levels (often taken as less than 4 ng/l, 1- Congenital erythropoietic porphyria
but are actually age-dependent). The absolute
2- Erythropoietic protoporphyria
PSA concentration correlates poorly with
prognosis in prostatic cancer. 3- Porphyria cutanea tarda
4- Variegate porphyria
[ Q: 1474 ] PasTest 2009 - Basic 5- None of the above
Science
In a patient with chronic hyponatraemia Answer & Comments
(sodium concentration 112 mmol/l), which of
Answer: 4- Variegate porphyria
the following findings would most suggest a
diagnosis of the syndrome of inappropriate The acute porphyrias include acute
[secretion of] antidiuretic hormone (SIADH)? intermittent porphyria and variegate
1- Normal cortisol response to ACTH porphyria. These are rare genetic errors of
haem biosynthesis. Accumulation of porphyrin
2- Plasma albumin concentration 28 g/l precursors (porphobilinogen and d-
3- Plasma osmolality 248 mOsmol/kg aminolaevulinic acid) cause neuronal and
4- Urinary osmolality 350 mOsmol/kg visceral crises. The acute intermittent type is
inherited as an autosomal-dominant, although
5- Urinary sodium concentration < 20 mmol/l penetrance is low and around one-third of
cases are due to mutations. The urinary
Answer & Comments porphobilinogen level is raised during an acute
attack, but only 50% of samples will turn red
Answer: 4- Urinary osmolality 350 mOsmol/kg
on standing. Faecal levels of porphyrin are
The most significant finding is that the urine is normal in patients with the acute intermittent
concentrated relative to plasma. This is type, unlike the variegate form. Variegate
inappropriate, in that a low plasma osmolality porphyria is also inherited as an autosomal-
should suppress ADH secretion and lead to the dominant and is associated with
formation of a maximally dilute urine. The low photosensitivity causing blistering.
plasma osmolality reflects the hyponatraemia,
since sodium is the principal determinant of [ Q: 1476 ] PasTest 2009 - Basic
extracellular fluid osmolality. A low urine Science
sodium excretion would suggest extrarenal
sodium depletion as a cause of Which of the following findings would most
hyponatraemia. Addison's disease, another suggest that fluid loss from the body was
cause of chronic hyponatraemia, is excluded primarily hypotonic (water depletion) rather
by the cortisol response to ACTH, as it should than isotonic (sodium depletion)?
be before SIADH is diagnosed. The low plasma 1- Hyponatraemia
albumin concentration could just reflect
2- Increased haematocrit

Dr. Khalid Yusuf El-Zohry – Sohag Teaching Hospital (01118391123)


Ref MRCPass OE OE 2012 PasTest 2009 PassMedicine 2009 PasTest Exam ReviseMRCP 667
Elzohry MRCP Questions Bank (Part 1) – 2013 (For my personal use)

3- Postural hypotension normal losses, although potassium depletion


4- Production of a highly concentrated urine is a recognised feature of starvation. ACE
inhibitors tend to raise the plasma potassium
5- Tachycardia concentration. Whereas renal tubular
acidoses types 1 and 2 are associated with
Answer & Comments hypokalaemia, type 4 (hyporeninaemic
hypoaldosteronism, most frequently seen in
Answer: 4- Production of a highly
patients with diabetes) is associated with
concentrated urine
hyperkalaemia. Primary aldosteronism causes
Hypotonic fluid loss is shared by the whole- hypertension and may cause hypokalaemia.
body water compartment, whereas isotonic However, hypokalaemia is not always present
loss initially affects only the extracellular in primary aldosteronism; this condition is
compartment. The inevitable increase in now recognised to be commoner than it used
plasma osmolality with hypotonic fluid loss to be when the diagnosis was often only
stimulates the secretion of vasopressin and considered in patients with hypokalaemia.
results in the production of a concentrated Nevertheless, it probably accounts for only
urine. Decreased extracellular fluid volume about 5% of cases of hypertension. In
causes hypotension and tachycardia, and, contrast, hypokalaemia is a common side-
except with blood loss, an increase in the effect of thiazide treatment.
haematocrit. The plasma sodium
concentration may fall, but is elevated with [ Q: 1478 ] PasTest 2009 - Basic
hypotonic fluid loss. Science
A woman, aged 55, presents with features
[ Q: 1477 ] PasTest 2009 - Basic
consistent with Cushing's syndrome. She is
Science
taking no medication. Her basal cortisol and
A 62-year-old woman with type-2 diabetes plasma ACTH levels are significantly raised.
being treated with a thiazide, β-blocker and ACTH levels have failed to suppress in
ACE inhibitor for hypertension is found to response to a low dose dexamethasone
have a serum potassium concentration of 3.1 suppression test.
mmol/l. Her blood pressure is 156/94 mmHg; What is the most likely diagnosis?
serum creatinine concentration is 115 mmol/l.
1- Adrenal tumour
What is the most likely cause of the
hypokalaemia? 2- Carney's syndrome

1- Low dietary potassium intake 3- Cushing's disease

2- Primary aldosteronism 4- Depression

3- Renal tubular acidosis type 4 5- Ectopic ACTH-secreting tumour

4- The ACE inhibitor


Answer & Comments
5- The thiazide diuretic
Answer: 3- Cushing's disease
Answer & Comments A raised adrenocorticotrophic hormone
(ACTH) level with a raised cortisol implies the
Answer: 5- The thiazide diuretic
problem is caused by excess ACTH production,
Most people who are eating normally have a otherwise negative feedback would suppress
sufficient dietary potassium intake to balance ACTH. A low ACTH level would therefore be

Dr. Khalid Yusuf El-Zohry – Sohag Teaching Hospital (01118391123)


Ref MRCPass OE OE 2012 PasTest 2009 PassMedicine 2009 PasTest Exam ReviseMRCP 668
Elzohry MRCP Questions Bank (Part 1) – 2013 (For my personal use)

expected in patients with an adrenal tumour. 5- Renal impairment


Carney's syndrome comprises atrial myxoma
and freckles with high cortisol levels Answer & Comments
independent of ACTH. Pituitary tumours
producing ACTH and adrenal stimulation are Answer: 1- Heparin treatment
the cause of Cushing's disease. While ectopic Approximately 20 mmol/24 h is the minimum
ACTH-secreting tumours are associated with obligatory potassium output, while the typical
significantly raised ACTH and cortisol levels, potassium requirements for patients on
it's unusual to develop classic cushingoid parenteral feeding are 40-80 mmol/24 h.
features. Failure of the dexamethasone Pseudohyperkalaemia is hyperkalaemia
suppression test can occur in patients with occurring as a result of a loss of potassium
depression, but cushingoid features are not from white cells and platelets during clotting,
expected. usually seen in patients with high white cell or
platelet counts. Typically, the plasma
[ Q: 1479 ] PasTest 2009 - Basic potassium concentration is significantly lower
Science than the serum potassium concentration in
this condition. The elevated urea may be due
A 45-year-old woman with type-2 diabetes is
to an excessive provision of amino acids, but
making an apparently good recovery 7 days
neither it nor the creatinine level suggest
after a partial resection of the small intestine
sufficient renal impairment to cause such a
following trauma sustained in a stabbing
severe hyperkalaemia. Incipient adrenal
incident. She is receiving parenteral nutrition
failure could have been made overt by the
with additional 'normal' saline and, because of
stress of surgery, but this is uncommon. The
a history of deep vein thrombosis some 10
heparin is more likely to be responsible:
years previously, is on prophylactic heparin.
heparin inhibits aldosterone secretion by the
Before her admission she was well, with no
adrenal cortex, leading to impaired renal
ongoing medical problems and taking no
potassium excretion, particularly in patients
regular medication. Serum electrolyte results
with diabetes or those who are acidotic.
are as follows: sodium 129 mmol/l, potassium
6.5 mmol/l, bicarbonate 24 mmol/l, urea 8.5
mmol/l, creatinine 120 mmol/l, glucose 10.2 [ Q: 1480 ] PasTest 2009 - Basic
mmol/l. Her potassium concentration has Science
risen over the past 3 days. The potassium A 54-year-old man with a long history of
content of the parenteral feed has been excessive alcohol ingestion develops a
reduced from 60 to 20 mmol/24 h during this blistering skin eruption. His urine tests
period. Urine output is appropriate to her positive for porphyrins.
fluid input. Her red cell, white cell and platelet
counts are all normal. What is the most likely diagnosis?

What is the most likely cause of the 1- Acute intermittent porphyria


hyperkalaemia? 2- Cutaneous hepatic porphyria
1- Heparin treatment 3- Hepatoerythropoietic porphyria
2- Overprovision of potassium in the 4- Hereditary coproporphyria
parenteral feed 5- Variegate porphyria
3- Primary adrenal failure (Addison's disease)
4- Pseudohyperkalaemia

Dr. Khalid Yusuf El-Zohry – Sohag Teaching Hospital (01118391123)


Ref MRCPass OE OE 2012 PasTest 2009 PassMedicine 2009 PasTest Exam ReviseMRCP 669
Elzohry MRCP Questions Bank (Part 1) – 2013 (For my personal use)

Answer & Comments inflammatory condition (including other liver


diseases). Serum iron concentration is normal
Answer: 2- Cutaneous hepatic porphyria
in approximately 25% of patients with HH and
Cutaneous hepatic porphyria (porphyria can be elevated in healthy individuals or
cutanea tarda) can be inherited, but is more people with secondary iron overload. Total
frequently acquired, and is seen most iron-binding capacity reflects the transferrin
frequently in association with excessive concentration, which is usually normal in
alcohol intake, with or without liver disease. haemochromatosis. Moreover, although
Both acute intermittent porphyria and transferrin saturation is typically high in HH, it
hereditary coproporphyria are acute can also be increased in other iron overload
porphyrias, and do not cause photosensitivity. conditions and in liver disease. Molecular
Variegate porphyria is characterised by both genetic analysis and demonstration of
acute attacks and photosensitisation. It is less homozygosity for the C282Y mutation, or of
common than cutaneous hepatic porphyria compound heterozygosity for C282Y and
and is always genetically determined. H63D, can detect asymptomatic individuals at
Hepatoerythropoietic porphyria is a very rare, risk of developing clinical haemochromatosis.
inherited porphyria, which is the homozygous
form of inherited cutaneous hepatic [ Q: 1482 ] PasTest 2009 - Basic
porphyria. Science
A 44-year-old pharmacist presents with a
[ Q: 1481 ] PasTest 2009 - Basic
history of recurrent episodes of faintness,
Science
sweating and tremor, occurring particularly in
In an adult patient with cirrhosis, which of the the late morning or late afternoon. An
following findings is the most reliably insulinoma is suspected. The patient is
diagnostic of hereditary haemochromatosis as admitted for 3 days' consecutive overnight
the cause? fasting with blood glucose measurements in
the morning. She remains asymptomatic
1- Liver biopsy
during her admission and the lowest blood
2- Serum ferritin concentration glucose concentration is 4.4 mmol/l. On
3- Serum iron concentration discharge from hospital, she reports that her
symptoms have recurred.
4- Serum total iron-binding capacity
What would be the most useful investigation
5- Transferrin saturation
to establish the diagnosis?

Answer & Comments 1- Measurement of plasma b -


hydroxybutyrate
Answer: 1- Liver biopsy
2- Measurement of glycated haemoglobin (Hb
In hereditary haemochromatosis (HH), the A1c)
excess iron is primarily found in parenchymal 3- Prolonged glucose tolerance test
cells, whereas with secondary iron overload,
accumulation tends to be in Kupffer cells. 4- Simultaneous measurement of glucose and
C-peptide when symptomatic
Liver biopsy can demonstrate this, allows
assessment of liver damage and is of 5- Simultaneous measurement of glucose and
prognostic value. Serum ferritin insulin when symptomatic
concentrations are almost always markedly
elevated, but elevations can occur in any

Dr. Khalid Yusuf El-Zohry – Sohag Teaching Hospital (01118391123)


Ref MRCPass OE OE 2012 PasTest 2009 PassMedicine 2009 PasTest Exam ReviseMRCP 670
Elzohry MRCP Questions Bank (Part 1) – 2013 (For my personal use)

Answer & Comments Answer & Comments


Answer: 4- Simultaneous measurement of Answer: 2- Gilbert's syndrome
glucose and C-peptide when symptomatic
Dubin-Johnson, Rotor and Gilbert's syndromes
The occurrence of hypoglycaemia other than are all inherited disorders of bilirubin
in response to an obvious stimulus or in the metabolism. However, in the first two, there is
fasting state is suggestive of factitious a defect in the secretion of bilirubin from the
hypoglycaemia (covert administration of liver and the bilirubin that accumulates in the
insulin). This will be detectable by measuring plasma is conjugated, water-soluble and thus
C-peptide during hypoglycaemia. C-peptide is is excreted in the urine.
co-secreted with endogenous insulin but is
Infectious mononucleosis can cause hepatitis
not present in commercially produced insulin.
and jaundice but an elevated transaminase
Measurement of insulin will not distinguish
activity would be expected. Hereditary
the two. Prolonged glucose tolerance tests are
spherocytosis is a chronic haemolytic disorder
seldom informative in the investigation of
due to a defect in the red cell membrane
hypoglycaemia and certainly not in this
(most frequently in spectrin, a structural
context. Plasma b-hydroxybutyrate
protein). It can present with a wide range of
concentrations tend to be low in
severity, from jaundice at birth to
hypoglycaemia caused by insulin (whether
asymptomatic anaemia or jaundice in adults,
endogenous or exogenous: insulin suppresses
but is much less common (approximately
lipolysis). Measurements of glycated
1:5000 in Northern Europeans) than Gilbert's
haemoglobin have no part to play in the
syndrome (approximately1:20).
diagnosis of hypoglycaemia, and in diabetes
are only used for monitoring.
[ Q: 1484 ] PasTest 2009 - Basic
Science
[ Q: 1483 ] PasTest 2009 - Basic
Science Insulin binds to a 400-kDa glycoprotein, which
straddles the cell membrane of many cells and
A 21-year-old male medical student who has
exerts much of its peripheral effects through
been feeling non-specifically unwell for
action at this receptor.
several days is noticed to have slightly icteric
sclerae by his girlfriend and has liver function The expression of which glucose transporter
tests performed. The results of these are (GLUT) is upregulated by insulin binding at this
normal apart from a serum bilirubin receptor site?
concentration of 44 mmol/l (3-17). His urine 1- GLUT-1
does not contain bilirubin.
2- GLUT-2
Which of the following is the most likely
3- GLUT-4
diagnosis?
4- GLUT-5
1- Dubin-Johnson syndrome
5- GLUT-3
2- Gilbert's syndrome
3- Hereditary spherocytosis
Answer & Comments
4- Infectious mononucleosis
Answer: 3- GLUT-4
5- Rotor syndrome
GLUT-4 is a 400-kDa glucoprotein coded for on
the short arm of chromosome 19. Insulin

Dr. Khalid Yusuf El-Zohry – Sohag Teaching Hospital (01118391123)


Ref MRCPass OE OE 2012 PasTest 2009 PassMedicine 2009 PasTest Exam ReviseMRCP 671
Elzohry MRCP Questions Bank (Part 1) – 2013 (For my personal use)

binding at the cell membrane-receptor site glycine or taurine which increases their
results in tyrosine kinase activation and solubility. Intestinal bacteria then convert
initiation of a cascade of intracellular them into secondary bile acids. The bile salt
reactions. One of these is the migration of the pool is small (around 2.5-5 g) and is recycled
GLUT-4 receptor to the cell surface. GLUT-4 is 6-8 times per day via the enterohepatic
the channel through which glucose is taken up circulation, this causes reabsorption of 95% of
into muscle and adipose tissue cells following bile acids. Total faecal loss is around 10-20%
stimulation of the insulin receptor. of the bile acid pool in any one day.

The GLUT-1 receptor enables basal non-


insulin-stimulated glucose uptake into cells, [ Q: 1486 ] PasTest 2009 - Basic
GLUT-2 transports glucose into beta-cells Science
(allowing for glucose sensing), GLUT-3 enables You are reviewing a young woman who
non-insulin-mediated glucose uptake into presents with intermittent abdominal pain,
brain neurones. depression and anxiety. She has a diagnosis of
acute intermittent porphyria.
[ Q: 1485 ] PasTest 2009 - Basic Which of the following statements best fits the
Science production of haem?
You are asked to review a 45-year-old obese 1- Increased haem levels increase the activity
woman who is jaundiced. of delta-ALA synthetase
Which of the following best fits the 2- Protoporphyrin is formed as the
metabolism of bile acids? penultimate step before haem formation
1- Concentration in the gall-bladder is around 3- Defects of uroporphyrinogen co-synthetase
500 mmol/l cause acute intermittent porphyria
2- Their production is catalysed by cholesterol 4- Defects of ferrocheletase cause variegate
7a-hydroxylase porphyria
3- Around 30% of the bile acid pool is lost in 5- Protoporphyrinogen oxidase catalyses the
faeces each day production of haem
4- 80% of bile salts are reabsorbed during
each cycle Answer & Comments
5- The total bile salt pool in the liver is around Answer: 2- Protoporphyrin is formed as the
10 g penultimate step before haem formation

The production of haem is a complex process.


Answer & Comments
It begins with glycine and succinyl CoA and
Answer: 2- Their production is catalysed by proceeds, via a number of intermediate steps,
cholesterol 7a-hydroxylase penultimately to protoporphyrin and then to
haem. Delta-ALA synthetase activity is the
Bile salts are synthesised in hepatocytes from
rate-limiting step in this process, and
cholesterol via a process catalysed by
increased haem levels provide negative
cholesterol 7a-hydroxylase. They are then
feedback on the activity of this enzyme.
excreted in bile and pass into the duodenum.
Bile acids exist in the gall-bladder at a Due to the number of enzymes involved in
concentration of between 100 and 300 haem production, a number of inherited
mmol/l. The two primary bile acids (cholic and defects have been identified. These are
chenodeoxycholic acid) are conjugated with classified as hepatic porphyrias when the

Dr. Khalid Yusuf El-Zohry – Sohag Teaching Hospital (01118391123)


Ref MRCPass OE OE 2012 PasTest 2009 PassMedicine 2009 PasTest Exam ReviseMRCP 672
Elzohry MRCP Questions Bank (Part 1) – 2013 (For my personal use)

excess production of porphyrins occurs in the gamma-glutamyltransferase 82 U/l (11-51),


liver, or erythropoietic porphyrias when the albumin 36 g/l, total protein 82 g/l.
excess production occurs in the bone marrow. Which of the following is the most likely
They can also be classified as acute or non- diagnosis?
acute according to their clinical presentation.
Acute intermittent porphyria occurs due to an 1- Alcoholic cirrhosis
abnormality of porphobilinogen deaminase. 2- Carcinoma of the head of the pancreas
3- Cholangiocarcinoma
[ Q: 1487 ] PasTest 2009 - Basic
4- Primary biliary cirrhosis
Science
5- Primary sclerosing cholangitis
A 55-year-old, chronic heavy smoker is
brought to A&E with a 2-day history of
Answer & Comments
polyuria, polydipsia, nausea and altered
sensorium. On examination, he is lethargic Answer: 4- Primary biliary cirrhosis
and confused. A chest X-ray shows a round
shadow in the right mid-zone with enlarged The high alkaline phosphatase concentration
hilar lymph nodes on the right side. An ECG is suggests cholestatic jaundice. Alcoholic
normal except for a narrowed QT interval. cirrhosis is common but is unlikely (though not
excluded) by the only slightly elevated g-
What is the most likely metabolic abnormality glutamyltransferase. Cholangiocarcinoma is a
in this case? rare tumour. Carcinoma of the head of the
1- Hypernatraemia pancreas frequently presents in this manner
(though weight loss is often present also), but
2- Hyperkalaemia
the slight elevation in total protein with low-
3- Hypercalcaemia normal albumin suggests a high globulin
4- Hyperphosphataemia concentration, which suggests autoimmune
liver disease. Primary sclerosing cholangitis is
5- Hypokalaemia
a possibility but is commoner in men than
women (3:1), and in 75% of cases is associated
Answer & Comments with inflammatory bowel disease. Primary
Answer: 3- Hypercalcaemia biliary cirrhosis is more common in women.

This man most probably has bronchial


[ Q: 1489 ] PasTest 2009 - Basic
carcinoma with bony metastases resulting in
Science
hypercalcaemia.
A 40-year-old woman patient has been
[ Q: 1488 ] PasTest 2009 - Basic referred to hospital with jaundice. On
Science examination there is hepatosplenomegaly.
She does not take any medication nor drinks
A 52-year-old woman undergoes investigation alcohol. Her sister, who is 10 years older, had
for jaundice. She first noticed this symptom 2 similar problems.
months ago, but for 4 months prior to that,
Following serum testing, antibodies to which
she had been experiencing generalised
antigen are most likely to be detected in her
pruritus. The results of liver function tests are
serum?
as follows: serum bilirubin 325 mmol/l,
aspartate aminotransaminase 55 U/l (15-42), 1- Double-stranded DNA
alkaline phosphatase 436 U/l (80-150), 2- Proteinase 3

Dr. Khalid Yusuf El-Zohry – Sohag Teaching Hospital (01118391123)


Ref MRCPass OE OE 2012 PasTest 2009 PassMedicine 2009 PasTest Exam ReviseMRCP 673
Elzohry MRCP Questions Bank (Part 1) – 2013 (For my personal use)

3- Mitochondria furthermore, the ceruloplasmin concentration


4- T cells can be decreased because of decreased
hepatic synthesis in liver disease. Hepatic
5- Immunoglobulins copper concentration as measured in a biopsy
sample is invariably increased in Wilson's
Answer & Comments disease, but it can also be increased in chronic
cholestasis from other causes. Urinary copper
Answer: 3- Mitochondria
excretion is usually increased in Wilson's
Over 95% of patients with primary biliary disease (reflecting the decreased binding in
cirrhosis have antibodies to mitochondria, plasma by ceruloplasmin) and is further
with the dominant autoantibody response increased following the administration of
being directed against two components penicillamine. The abnormal gene in Wilson's
(dihydrolipoamide acetyltransferase (E2) and disease is termed ATP7B; HFE is the
E3-binding protein) of the pyruvate haemochromatosis gene.
dehydrogenase complex (PDC). The loss of
tolerance to these autoantigens is an early [ Q: 1491 ] PasTest 2009 - Basic
event in this progressive disease, with Science
antimitochondrial antibodies (AMA) being
detectable in serum before abnormalities in You are reviewing a man with familial
liver function and long before the onset of hypertriglyceridaemia who has presented to
symptoms. the surgeons with pancreatitis. He asks about
why he suffers from this condition.
[ Q: 1490 ] PasTest 2009 - Basic Which of the following best describes the
Science metabolism of VLDL (predominant triglyceride)
particles?
A 20-year-old woman presents with tremor
and dysarthria, she is also noticed to be 1- They are synthesised postprandially in the
slightly jaundiced; clinically, Wilson's disease is small intestine
considered to be the most likely diagnosis. 2- They are formed from IDL particles
Which of the following investigations will most 3- They bind to the hepatic LDL receptor
reliably confirm this diagnosis? 4- They are synthesised continuously in the
1- Identification of a mutation in the HFE gene liver
2- Measurement of copper in a liver biopsy 5- They take up cholesterol from cells
3- Serum ceruloplasmin concentration
Answer & Comments
4- Serum copper concentration
5- Urinary copper excretion after Answer: 4- They are synthesised continuously
penicillamine challenge in the liver

Very low-density lipoprotein (VLDL) particles


Answer & Comments are synthesised continuously in the liver and
contain most of the body's triglyceride
Answer: 5- Urinary copper excretion after
production and a smaller amount of
penicillamine challenge
cholesterol. VLDL particles provide the main
Although serum ceruloplasmin and copper source of energy during prolonged periods of
concentrations are low in many patients with fasting. VLDL particles have apoprotein C-II on
Wilson's disease, this is not a universal finding; their surface, this allows them to bind to

Dr. Khalid Yusuf El-Zohry – Sohag Teaching Hospital (01118391123)


Ref MRCPass OE OE 2012 PasTest 2009 PassMedicine 2009 PasTest Exam ReviseMRCP 674
Elzohry MRCP Questions Bank (Part 1) – 2013 (For my personal use)

capillary endothelium and for triglycerides to (the most frequent presenting feature of the
be progressively removed by lipoprotein condition), Paget's disease is the more likely.
lipase. This leaves a particle depleted of
triglyceride and apoprotein C-II, known as an [ Q: 1493 ] PasTest 2009 - Basic
IDL (intermediate-density lipoprotein) particle. Science
Most familial hypertriglyceridaemia is A 36-year-old man presents to his family
polygenic in origin, and due to a modest doctor complaining of excessive sweating.
excess in the circulating concentration of VLDL Thyroid function tests are performed: serum
particles. There may be a history of retinal TSH concentration is normal, but the
vein thrombosis or recurrent pancreatitis in concentrations of both free thyroxine and free
individuals affected by the familial condition. triiodothyronine are elevated.
Which of the following is the most likely
[ Q: 1492 ] PasTest 2009 - Basic explanation for these results?
Science
1- A TSH-secreting pituitary tumour
A 71-year-old-man is given a health check by
2- Graves' disease
his doctor. He has no complaints, apart from
pain in his hip, which he attributes to arthritis. 3- Self-administration of thyroxine
He has blood taken for a panel of routine 4- The presence of heterophilic antibodies in
biochemical and haematological tests. The the patient's serum
only abnormality is a serum alkaline
5- Thyroid hormone resistance
phosphatase activity of 822 U/l.
Which of the following is the most likely cause
Answer & Comments
of this abnormality in his case?
Answer: 1- A TSH-secreting pituitary tumour
1- Metastatic carcinoma
2- Osteoarthritis These results are unusual: patients with
thyrotoxicosis from, for example, Graves'
3- Osteomalacia
disease would be expected to have
4- Osteoporosis unmeasureable TSH concentration (the high
5- Paget's disease concentrations of thyroid hormones should
suppress TSH secretion). Heterophilic
antibodies (antibodies in the patient's serum
Answer & Comments
that react with animal-derived antibodies
Answer: 5- Paget's disease used in the immunoassay) can cause bizarre
results, but this patient has a classic symptom
Elevated alkaline phosphatase activity is
of thyrotoxicosis. Because of this, thyroid
usually an indicator of bone or cholestatic
hormone resistance is also unlikely. If he were
hepatobiliary disease. The fact that other liver
self-administering thyroid hormones, TSH
function tests are normal militates against
secretion should be suppressed.
(though does not exclude) the latter. An
increase in alkaline phosphatase from bone TSH-secreting pituitary tumours are
reflects increased osteoblastic activity: this is uncommon, but are a recognised cause of
not a feature of osteoporosis or osteoarthritis hyperthyroidism. They sometimes co-secrete
and alkaline phosphatase is not elevated in growth hormone, excessive secretion of which
these conditions. Increases are seen in is also a cause of sweating.
metastatic bone disease and in Paget's, but
given that the patient is well apart from pain

Dr. Khalid Yusuf El-Zohry – Sohag Teaching Hospital (01118391123)


Ref MRCPass OE OE 2012 PasTest 2009 PassMedicine 2009 PasTest Exam ReviseMRCP 675
Elzohry MRCP Questions Bank (Part 1) – 2013 (For my personal use)

[ Q: 1494 ] PasTest 2009 - Basic Which of the following investigations is most


Science important to determine the cause of the
anaemia?
A 22-year-old man presents with a long
history of intermittent abdominal discomfort 1- Measurement of red cell folate
and diarrhoea. On examination, he has a body concentration
mass index of 19 kg/m2 and is clinically 2- Measurement of serum ferritin
anaemic. Coeliac disease is suspected. concentration
Which of the following investigations will most 3- Measurement of serum TSH (thyroid-
reliably diagnose this condition? stimulating hormone) concentration
1- Detection of antigliadin antibodies in serum 4- Measurement of serum Vitamin B12
2- Detection of endomysial antibodies in concentration
serum 5- Reticulocyte count
3- Detection of tissue transglutaminase
antibodies in serum Answer & Comments
4- Microscopic examination of a small bowel Answer: 3- Measurement of serum TSH
biopsy specimen (thyroid-stimulating hormone) concentration
5- Xylose absorption test
The macrocytic anaemia due to folate or
vitamin B12 deficiency is usually
Answer & Comments megaloblastic. However, several conditions
Answer: 4- Microscopic examination of a small can cause a normoblastic macrocytic anaemia,
bowel biopsy specimen including hypothyroidism (although this can
also coexist with pernicious anaemia),
The detection of the typical appearance of excessive alcohol ingestion (although
subtotal villous atrophy on a small bowel megaloblastic changes are sometimes
biopsy is regarded as very important for present), liver disease and conditions
confirming clinical coeliac disease. However, associated with an increased reticulocyte
detection of tissue transglutaminase count (reticulocytes are bigger than normal
antibodies has high specificity and sensitivity mature red cells. (It may also be noted that
and is widely used as a screening test. Tissue reticulocytes contain remnant nuclear
transglutaminase is the antigen detected by material, which gradually disappears with
endomysial antibodies. Antigliadin antibodies their maturation from reticulocyte to red cell.)
are less sensitive. The xylose absorption test is Given this patient's symptom yet only a
a non-specific test for intestinal slightly low haemoglobin, the most important
malabsorption. diagnosis to consider is hypothyroidism, which
often has an insidious onset, or is discovered
[ Q: 1495 ] PasTest 2009 - Basic when it is still subclinical.
Science
A 65-year-old man consults his family doctor [ Q: 1496 ] PasTest 2009 - Basic
complaining of feeling 'tired all the time'. His Science
haemoglobin is 11.5 g/dl (13-17); mean cell A 72-year-old woman is found to have a
volume (MCV) is 105 fl (82-102). Bone marrow serum calcium concentration of 3.12 mmol/l.
examination does not indicate megaloblastic
changes.

Dr. Khalid Yusuf El-Zohry – Sohag Teaching Hospital (01118391123)


Ref MRCPass OE OE 2012 PasTest 2009 PassMedicine 2009 PasTest Exam ReviseMRCP 676
Elzohry MRCP Questions Bank (Part 1) – 2013 (For my personal use)

Which of the following clinical features, if XYY syndrome. The combination of tall
present, would most direct you towards a stature, gynaecomastia and infertility due to
specific cause? azoospermia is diagnostic of Klinefelter's
1- Bone pain syndrome. Almost all males with Klinefelter
syndrome are azoospermic. Other features of
2- Hilar lymphadenopathy this condition include reduced facial hair,
3- Polyuria tendency to obesity, and small atrophic testes.
Mild learning difficulties can be seen in some
4- Short QT interval
patients with Klinefelter's syndrome but
5- Ureteric colic mental retardation is not a feature of this
condition. Chromosome analysis will show 47
Answer & Comments chromosomes, with two X and one Y
chromosome (47, XXY).
Answer: 2- Hilar lymphadenopathy
Patients with Homocystinuria have tall
Bone pain can occur with hypercalcaemia
stature, learning difficulties, lens dislocation
secondary to malignancy or
osteoporosis and recurrent arterial
hyperparathyroidism. Polyuria is a feature of
thrombosis. Features of Marfan's syndrome
severe hypercalcaemia, irrespective of the
include tall stature, arachnodactyly, scoliosis,
cause. A short QT interval is also a feature of
joint laxity, lens dislocation, aortic root
hypercalcaemia. Ureteric colic is particularly
dilatation, dural ectasia, skin striae, and
associated with primary hyperparathyroidism,
recurrent pneumothorax. The XYY syndrome is
but is not specific to this cause. The presence
characterised by tall stature, mild learning
of hilar lymphadenopathy in a patient with
difficulties and behavioural problems. Most
hypercalcaemia should raise a suspicion that
males with this condition have normal fertility,
the latter is due to sarcoid (in which the
although a small proportion of patients will
granulomas secrete calcitriol, 1,25-
have infertility due to azoospermia.
dihydroxycholecalciferol).

[ Q: 1498 ] PasTest 2009 - Basic


[ Q: 1497 ] PasTest 2009 - Basic
Science
Science
A 45-year-man sustains a myocardial
What is the most likely diagnosis in a 30-year-
infarction. His serum cholesterol
old man with tall stature, gynaecomastia and
concentration is 9.6 mmol/l.
azoospermia?
Which of the following physical signs would
1- Cystic fibrosis
most suggest a diagnosis of familial
2- Homocystinuria hypercholesterolaemia?
3- Klinefelter's syndrome 1- Arcus senilis
4- Marfan's syndrome 2- Eruptive xanthomas
5- XYY syndrome 3- Palmar xanthomas
4- Tendon xanthomas
Answer & Comments
5- Xanthelasmas
Answer: 3- Klinefelter's syndrome

Tall stature is a feature of Homocystinuria, Answer & Comments


Klinefelter syndrome, Marfan's syndrome and Answer: 4- Tendon xanthomas

Dr. Khalid Yusuf El-Zohry – Sohag Teaching Hospital (01118391123)


Ref MRCPass OE OE 2012 PasTest 2009 PassMedicine 2009 PasTest Exam ReviseMRCP 677
Elzohry MRCP Questions Bank (Part 1) – 2013 (For my personal use)

Arcus senilis, periorbital xanthelasmas and found. Magnetic resonance imaging shows
tendon xanthomas can all occur in patients changes in the central nervous system, but the
with hypercholesterolaemia, although arcus longer term clinical implications are uncertain.
senilis and, less frequently, xanthelasmas, can
Plasma ornithine values range from 400 to
occur in individuals with normal plasma
1000 mmol/l (normal 75 m mol/l) with high
cholesterol concentrations. Tendon
concentrations in cerebrospinal fluid and the
xanthomas (particularly Achilles tendons and
aqueous humour. Between 400 and 900
extensor tendons on the back of the hand) are
mg/day is excreted with increased amounts of
specific to familial hypercholesterolaemia.
arginine and lysine (competitive inhibition of
Eruptive xanthomas are characteristic of
reabsorption). The activity of ornithine-d-
severe hypertriglyceridaemia; palmar
aminotransferase is low in liver and skeletal
xanthomas (typically in the palmar creases,
muscle. Most affected patients have less than
but sometimes more extensive) are typical of
1% of the normal activity in fibroblasts.
familial dysbetahyperlipoproteinaemia
(remnant dyslipidaemia, broad beta disease). The clinical picture and the amino acid defects
are an adequate means of diagnosis. Enzyme
[ Q: 1499 ] PasTest 2009 - Basic assays can be used to confirm the diagnosis.
Science
[ Q: 1500 ] PasTest 2009 - Basic
A 19-year-old patient presents with gradual
Science
worsening myopia and decreased night vision.
On examination there is atrophy of the retina. Antibodies to which of the following are most
What enzyme deficiency is this patient most frequently present in the serum of patients
likely to have? with type-1 diabetes at diagnosis?

1- Lipoprotein lipase 1- Cocksackievirus

2- Acetyl-coenzyme A acetyltransferase 2- Glucagon

3- Ornithine aminotransferase 3- Insulin

4- Argininosuccinic acid synthetase 4- Islet cells

5- Glutathione synthetase 5- Cytomegalovirus

Answer & Comments Answer & Comments

Answer: 3- Ornithine aminotransferase Answer: 4- Islet cells

Deficiency of ornithine-d-aminotransferase Islet-cell antibodies (usually directed against


causes atrophy of the choroid and retina, glutamic acid decarboxylase, GAD) are present
beginning as a small yellowish spot and in the serum of the majority of patients with
increasing to a circular lesion edged with type-1 diabetes at the time of diagnosis,
pigment giving an atypical retinitis pigmentosa although the titre usually declines rapidly
appearance. Children present with myopia thereafter, and they persist in only 10-15% of
and decreased night vision, which progresses patients. Antibodies to insulin may develop
to blindness in middle life. Cataracts also during treatment, although less frequently
develop but the optic discs, cornea and iris now than when animal insulin preparations
remain normal. A few patients develop mild were widely used. The presence of antibodies
proximal muscle weakness. Microscopic to cocksackie virus in children developing
abnormalities of skeletal muscle fibres are type-1 diabetes has been suggested to imply a

Dr. Khalid Yusuf El-Zohry – Sohag Teaching Hospital (01118391123)


Ref MRCPass OE OE 2012 PasTest 2009 PassMedicine 2009 PasTest Exam ReviseMRCP 678
Elzohry MRCP Questions Bank (Part 1) – 2013 (For my personal use)

causal relationship between infection and the [ Q: 1502 ] PasTest 2009 - Basic
development of diabetes. There is an Science
association between congenital rubella and
A 35-year-old woman is referred by her GP
type-1 diabetes, but the basis of the
because she has been feeling unwell and has
association is unclear.
pain, stiffness and tenderness in her joints
which have gradually worsened over the last 6
[ Q: 1501 ] PasTest 2009 - Basic months. She has a detectable rheumatoid
Science factor.
A 25-year-old patient presents with yellow What is the most likely structure that these
papules on the extensor surfaces of the arms, antibodies are targeted against?
legs, buttocks and back. His triglycerides are
1- Double-stranded DNA
grossly elevated. On questioning he admits
that there is a family history. 2- Tumour necrosis factor-alpha

What kind of deficiency is the patient most 3- Complement


likely to have? 4- Interleukin
1- Apolipoprotein A 5- Fc portion of immunoglobulins
2- Apolipoprotein B48
3- Lipoprotein lipase Answer & Comments

4- LDL receptor Answer: 5- Fc portion of immunoglobulins


5- Acetyl-coenzyme A acetyltransferase The discovery of rheumatoid factor in the
(ACAT) blood of patients with rheumatism over half a
century ago led to the immunological
Answer & Comments hypothesis of disease pathogenesis. Since
rheumatoid factor is an autoantibody directed
Answer: 3- Lipoprotein lipase against epitopes on the constant domains of
Once the chylomicron has acquired the the Fc portion of IgG1, the concept that
apolipoprotein, apoC-II, it is capable of rheumatoid arthritis is an autoimmune
activating lipoprotein lipase. This enzyme is disease gained credibility. In the case of
located on the vascular endothelium of tissues rheumatoid arthritis, rheumatoid factor
with a high requirement for triglycerides, such complexes are present in synovial fluids; and
as skeletal and cardiac muscle (for energy), IgG-producing B cells, whose rearranged
adipose tissue (for storage) and lactating immunoglobulin gene sequences implicate
mammary gland (for milk). Lipoprotein lipase antigen stimulation, are present in inflamed
releases triglycerides from the core of the synovium.
chylomicron by hydrolysing them to fatty B cells in rheumatoid joints also synthesise
acids and glycerol, which are taken up by the antibodies to some cartilage components such
tissues locally. In this way the circulating as collagen type II, although these are not
chylomicron becomes progressively smaller. disease-specific.
Its triglyceride content decreases and it
becomes relatively richer in cholesterol and
[ Q: 1503 ] PasTest 2009 - Basic
protein.
Science
A patient receives too many infusions after an
operation resulting in a 20% increase in his

Dr. Khalid Yusuf El-Zohry – Sohag Teaching Hospital (01118391123)


Ref MRCPass OE OE 2012 PasTest 2009 PassMedicine 2009 PasTest Exam ReviseMRCP 679
Elzohry MRCP Questions Bank (Part 1) – 2013 (For my personal use)

blood volume. Protein C acts to inactivate the active forms of


What is the physiological process that is most the procoagulant cofactors, factors Va and
likely to correct this abnormality? VIIIa. Protein C is a vitamin K-dependent
serine protease structurally similar to factors
1- Reduced activity of arterial pressure VII, IX and X. Thrombin activates protein C
sensors when bound to thrombomodulin, a protein
2- Increased activity of renal sympathetic which acts like an endothelial-cell receptor for
nerves thrombin. Symptomatic manifestations of
3- Aldosterone release protein C deficiency are similar to those of
antithrombin III deficiency. Deep venous
4- Atrial natriuretic peptide (ANP) release thrombosis, with or without pulmonary
5- Venous dilatation embolism, occurs in 50% of patients by the
age of 30 to 45 years, depending on the study
Answer & Comments population.

Answer: 4- Atrial natriuretic peptide (ANP)


[ Q: 1505 ] PasTest 2009 - Basic
release
Science
The atria contain granulated cells that release A 70-year-old patient has become more
peptides, atrial natriuretic peptide (ANP), in forgetful, in that he has lost his day-to-day
response to stretch. This natriuretic agent also memory and is unable to learn new tasks.
relaxes the peripheral vasculature and thereby
opposes the actions of the sympathetic and What kind of pathological deposits are most
renin-angiotensin systems. likely to be present in his brain?
1- b2-Microglobulin
[ Q: 1504 ] PasTest 2009 - Basic 2- Transthyretin
Science
3- Amyloid
A 25-year-old patient suffered recurrent deep 4- Immunoglobulins
vein thromboses and also one pulmonary
embolism. She was extensively investigated 5- Acute-phase proteins
and diagnosed with protein C deficiency.
Answer & Comments
What pathological process is most likely to be
responsible for her venous Answer: 3- Amyloid
thromboembolisms?
A variety of amyloid plaques are observed in
1- Reduced degradation of factors Va and VIIIa Alzheimer's disease. Diffuse amyloid plaques
2- Reduced factor Xa complex have a loose accumulation of b/A4 amyloid
3- Reduced inhibition of tissue-factor without surrounding abnormal neurites, and
are considered to be precursors to neuritic
expression
plaques. The mature neuritic plaque consists
4- Reduced protein S of a dense core of b/A4 amyloid surrounded
5- Reduced synthesis of antithrombin III by a halo and ring of abnormal neurites:
before this stage the plaque is a loose
Answer & Comments accumulation of b/A4 amyloid surrounded by
abnormal neurites. A hypermature plaque has
Answer: 1- Reduced degradation of factors Va a dense core of b/A4 amyloid surrounded by
and VIIIa

Dr. Khalid Yusuf El-Zohry – Sohag Teaching Hospital (01118391123)


Ref MRCPass OE OE 2012 PasTest 2009 PassMedicine 2009 PasTest Exam ReviseMRCP 680
Elzohry MRCP Questions Bank (Part 1) – 2013 (For my personal use)

reactive astrocytes but without abnormal What is the most likely pathological
neurones. mechanism?
1- Bicarbonate loss due to ascites
[ Q: 1506 ] PasTest 2009 - Basic
2- Reduced urea synthesis
Science
3- Increased gastric acid production
You suspect that a 48-year-old man is
4- Reduced bicarbonate secretion from the
suffering from Cushing's syndrome.
pancreas
At what time of day is a random cortisol test
most likely to be abnormal? 5- Reduced lactate formation in skeletal
muscle
1- 0900 h
2- 1200 h Answer & Comments
3- 2400 h Answer: 2- Reduced urea synthesis
4- 1700 h
Urea production is an important feature of
5- 2000 h hepatic metabolism. The production of each
molecule of urea (ultimately from ammonium
Answer & Comments and carbon dioxide) is accompanied by the
generation of two protons. Ureagenesis is
Answer: 3- 2400 h
therefore a potential acidifying mechanism.
Plasma cortisol levels in normal individuals Most of the protons produced in ureagenesis
show a circadian rhythm. Levels are highest in are neutralised by the bicarbonate generated
the early morning, and fall to their lowest during the oxidation of the carbon skeleton of
levels during sleep at around midnight. In amino acids. Normally, however, a slight
patients with Cushing's syndrome the 0900 h excess of protons is produced that has to be
cortisol level may be within the normal range, eliminated by the kidneys.
but midnight levels are usually not
Urea synthesis and accompanying proton
suppressed. However, isolated venous blood
production are negatively regulated by
tests for cortisol levels are not the best way to
acidosis, which constitute another acid-base
detect Cushing's syndrome. A 24-h urine
regulatory system intrinsic to the liver.
collection with analysis of cortisol levels is a
much better early screening test for Cushing's.
[ Q: 1508 ] PasTest 2009 - Basic
A reverse circadian rhythm is seen for Science
melatonin, with levels at their highest during
periods of sleep. Synthetic melatonin is widely A 34-year-old immigrant of African origin is
used by business travellers for relief of ‘jet- prescribed ciprofloxacin for an infection. He
lag'. The best example of a longer and more suffers problems with acute haemolysis. You
complex human biological rhythm is the 28- suspect G6PD deficiency.
day menstrual cycle. Given this, what is the likely underlying cause
of haemolysis?
[ Q: 1507 ] PasTest 2009 - Basic 1- Increased levels of NADPH
Science
2- Decreased levels of NADP
A patient with liver cirrhosis develops 3- A defect on chromosome 29
metabolic alkalosis.
4- Reduced levels of ATP

Dr. Khalid Yusuf El-Zohry – Sohag Teaching Hospital (01118391123)


Ref MRCPass OE OE 2012 PasTest 2009 PassMedicine 2009 PasTest Exam ReviseMRCP 681
Elzohry MRCP Questions Bank (Part 1) – 2013 (For my personal use)

5- Reduced levels of NADPH Answer & Comments


Answer: 5- Factor VIII
Answer & Comments
Deficiency of either factor VIII (haemophilia A)
Answer: 5- Reduced levels of NADPH or factor IX (haemophilia B), which together
Glucose 6-phosphate dehydrogenase (G6PD) make up the factor VIIIa/factor IXa intrinsic
plays a vital role in the hexose tenase enzymatic complex, results in the
monophosphate pathway. It plays a role in the clinical phenotype commonly known as
oxidisation of glucose 6-phosphate to 6- haemophilia. Haemophilia principally presents
phosphoglycerate. The reaction is needed in with haematoma formation, easy bruising and
red blood cells as it provides the only source bleeding at the site of venepuncture during
of NADPH, which in turn plays a role in the toddler period.
conjunction with glutathione to protect the The disease exists in severe, moderate and
red blood cell from oxidative stress. mild forms. These are classified as such on the
Drugs that may cause an acute haemolytic basis of a clinical laboratory blood coagulation
reaction in people with G6PD deficiency test, which is performed to assess the level of
include aspirin, antimalarials, antibacterials functional coagulant protein (per cent activity
(including ciprofloxacin and sulphonamides) of factor VIII or factor IX). The pathological
and other drugs such as vitamin K, probenecid problem in both haemophilia A, factor VIII
and quinidine. Blood count is normal between deficiency and haemophilia B, factor IX
attacks of haemolysis, but during an attack the deficiency (also called ‘Christmas disease') is
blood film may show irregularly contracted the inability to form a functional tenase
cells, bite cells, blister cells, Heinz bodies and complex to activate factor X to factor Xa.
a reticulocytosis. Treatment is withdrawal of The clinical features of haemophilia
the offending drug, treatment of any predominantly include bleeding into joints and
underlying infection and blood transfusion soft tissues. The incidence of central nervous
where required. system bleeding has dramatically decreased
with concentrate therapy. The life expectancy
[ Q: 1509 ] PasTest 2009 - Basic of people with severe haemophilia had
Science increased from 11 years at the beginning of
the twentieth century to approximately 60
A 15-year-old youth with haemophilia A has
years in the early 1980s, before the
suffered recurrent bleeding episodes into his
devastating effects of blood-borne viral
joints. As a consequence he has arthropathies
disease again shortened average life
in his knees, elbows and wrists.
expectancy.
What is the most likely coagulation deficiency
causing his bleeding tendency?
[ Q: 1510 ] PasTest 2009 - Basic
1- Thromboxane Science
2- Factor X You are reviewing a man with significant early
3- Protein C arterial disease. The brachial artery response
postocclusion is severely diminished and
4- Factor IX
confirms a diagnosis of endothelial
5- Factor VIII dysfunction.

Dr. Khalid Yusuf El-Zohry – Sohag Teaching Hospital (01118391123)


Ref MRCPass OE OE 2012 PasTest 2009 PassMedicine 2009 PasTest Exam ReviseMRCP 682
Elzohry MRCP Questions Bank (Part 1) – 2013 (For my personal use)

The successful production of which compound 4- Partial 11b -hydroxylase deficiency


is best linked to vascular smooth muscle cell 5- None of the above
relaxation?
1- Angiotensin II Answer & Comments
2- Nitrous oxide
Answer: 3- Partial 21-hydroxylase deficiency
3- Endothelin I
She has congenital adrenal hyperplasia (CAH).
4- Nitric oxide During the first months of life, adrenal
5- Serotonin metabolism changes to the adult pattern. The
most common type of CAH is 21-hydroxylase
Answer & Comments deficiency (autosomal-recessive) affecting
around 1 in 10,000 Caucasians. As a result,
Answer: 4- Nitric oxide cortisol levels are low and therefore trigger
ACTH release and adrenal hyperplasia. Salt-
Nitric oxide (NO) is a rapidly diffusible gas with
a half-life of only a few seconds. It is formed in losing crises can occur with very low cortisol
endothelial cells via the metabolism of levels. Progesterone cannot be metabolised as
arginine and catalysed by nitric oxide synthase the enzyme deficiency increases the
(NOS). NO has immunological and manufacture of 17a-hydroxyprogesterone and
inflammatory functions, and also androgens, which then cause
antiatherosclerotic, antithrombotic effects. It virilisation/precocious sexual development.
also contributes to smooth muscle cell Late-onset disease can be due to partial
enzyme deficiency. 11b-Hydroxylase
relaxation and the control of blood pressure.
Impaired brachial artery relaxation after a deficiencies can lead to hypertension.
period of occlusion is a sign of endothelial
dysfunction and associated with increased [ Q: 1512 ] PasTest 2009 - Basic
cardiovascular risk. This, at least in part, is due Science
to endothelial damage and reduced NO You are asked to assess the cardiovascular risk
production; in turn, reduced NO production status of a man with the insulin-resistance
brings about an increased susceptibility to
syndrome.
further endothelial damage.
Which of the following pathophysiological
Angiotensin II, endothelin I and serotonin are changes are most strongly associated with
all vasoconstrictive agents. increased insulin resistance?
1- Hypotension
[ Q: 1511 ] PasTest 2009 - Basic
2- Increased levels of Plasminogen activator
Science
inhibitor-1 PAI-1
A teenager presents with excess hair and
3- Decreased platelet aggregation
amenorrhoea. She is normotensive. Her
prolactin levels are normal but she has a 4- Improved endothelial function
raised 17a-hydroxyprogesterone level. 5- Increased HDL levels
What is her diagnosis?
1- Complete 21-hydroxylase deficiency Answer & Comments

2- Complete 11b-hydroxylase deficiency Answer: 2- Increased levels of Plasminogen


activator inhibitor-1 PAI-1
3- Partial 21-hydroxylase deficiency

Dr. Khalid Yusuf El-Zohry – Sohag Teaching Hospital (01118391123)


Ref MRCPass OE OE 2012 PasTest 2009 PassMedicine 2009 PasTest Exam ReviseMRCP 683
Elzohry MRCP Questions Bank (Part 1) – 2013 (For my personal use)

Put simply, increased insulin resistance in an Answer & Comments


individual means that they require higher
Answer: 5- Increased glycogenolysis and
circulating levels of insulin to deal with a given
gluconeogenesis
glucose load compared to a normal subject.
Increased insulin resistance is associated with Glucagon is produced by pancreatic islet cells
the clinical insulin-resistance (metabolic) and its main action is on the liver to promote
syndrome consisting of a number of features glycogenolysis and gluconeogenesis. It also
including abnormal glucose tolerance, increases lipolysis in adipose tissue and
hypertension, low HDL cholesterol and increases ketone body production from fatty
abdominal obesity. In addition, there is an acids. The actions of glucagon on adipose
associated procoagulant state, with increased tissue are mediated by cyclic AMP to stimulate
levels of plasminogen activator inhibitor (PAI- lipolysis, producing free fatty acids that can
1), associated in clinical studies with an act as a major alternative energy source.
increased risk of myocardial infarction. Insulin Catecholamines act in a similar way to
resistance is also associated with raised glucagon, but in addition have effects on
inflammatory markers such as monocyte muscle. Insulin promotes the synthesis of
chemoattractant protein-1 (MCP-1) and glycogen, protein and fat, inhibiting lipolysis
tumour necrosis factor alpha (TNF-a), for and gluconeogenesis.
example.

Clinical studies have proved an association [ Q: 1514 ] PasTest 2009 - Basic


between insulin resistance and cardiovascular Science
risk. However, the value of pharmacological A 17-year-old adolescent complains of
agents that lower insulin resistance in intermittent face swelling. It varies in severity
reducing cardiovascular risk remains to be but sometimes he has difficulty breathing. His
demonstrated. brother has similar symptoms.
What protein is most likely to be responsible
[ Q: 1513 ] PasTest 2009 - Basic for his condition?
Science
1- Interleukin-1
An 81-year-old, nursing-home resident is
2- Interferon-gamma
admitted to hospital in an unconscious state.
His blood sugar is measured as 1.5 mmol/l 3- Complement C3
(normal 3-6 mmol/l). You administer 4- C1 esterase inhibitor
glucagon.
5- Interleukin-6
Which of the following best describes one of
the main actions of glucagon?
Answer & Comments
1- Decreased ketone body production from
fatty acids Answer: 4- C1 esterase inhibitor

2- Increased lipogenesis in adipose tissue Angioedema in the absence of urticaria is


caused by actual or functional C1 esterase
3- Decreased glycogenolysis
inhibitor deficiency. This may be hereditary
4- Decreased gluconeogenesis with an autosomal-dominant mode of
5- Increased glycogenolysis and inheritance, or acquired related to
gluconeogenesis lymphoproliferative disorders. A family
history, the absence of pruritus, the
prominence of abdominal symptoms and

Dr. Khalid Yusuf El-Zohry – Sohag Teaching Hospital (01118391123)


Ref MRCPass OE OE 2012 PasTest 2009 PassMedicine 2009 PasTest Exam ReviseMRCP 684
Elzohry MRCP Questions Bank (Part 1) – 2013 (For my personal use)

recurrent attacks suggest the hereditary small left homonymous hemianopia with
cause. C1 esterase inhibitor concentrate or partial sparing of central vision.
fresh-frozen plasma is used to treat Which of the following is the most likely
recalcitrant cases. anatomical site of the lesion responsible?
1- Right occipital cortex
[ Q: 1515 ] PasTest 2009 - Basic
Science 2- Left occipital cortex

A 50-year-old man with a long-standing 3- Right optic radiation


history of alcohol abuse was admitted to 4- Left optic radiation
hospital because he was difficult to rouse. On 5- Optic chiasm
examination he is confused and ataxic.
Examination of the eyes reveals nystagmus
Answer & Comments
and ophthalmoplegia.
Deficiency of which vitamin is most likely to Answer: 1- Right occipital cortex
have caused his symptoms? In terms of the neuroanatomy of the visual
1- Vitamin A pathway, a homonymous visual field defect
indicates a lesion posterior to the optic
2- Vitamin B1
chiasm. Unilateral occipital lobe infarction due
3- Vitamin B6 to posterior cerebral artery occlusion
4- Vitamin B12 produces a contralateral homonymous
hemianopia. However, a particular
5- Folic acid
phenomenon occurs when cortex itself is
lesioned. Vision at the fovea is spared,
Answer & Comments perhaps because there is such a large
Answer: 2- Vitamin B1 representation of the fovea in the cortex
(possible bilateral representation), or perhaps
A common cause of the amnesic syndrome is due to overlapping blood supply (collateral
the Wernicke-Korsakoff syndrome resulting vascular supply to the occipital pole from
from thiamine (vitamin B1) deficiency in posterior branches of the middle cerebral
association with chronic alcoholism or, artery). The loss of vision is not a complete
occasionally, malnutrition or malabsorption. hemifield, then, but a notched hemifield. This
The Wernicke phase of this disorder is phenomenon is called macular sparing. It is
characterised by confusion, nystagmus, thus typical of occipital cortex lesions.
abducent and conjugate gaze palsies
(ophthalmoplegia) as well as ataxia. These
[ Q: 1517 ] PasTest 2009 - Basic
features are commonly accompanied by
Science
peripheral neuropathy. Prompt treatment
with thiamine replacement is vital in order to You are asked to review an 80-year-old
avert a chronic and disabling amnesic disorder woman who has suffered previous
(the Korsakoff syndrome). neurological damage. You note that she has a
lower homonymous quadrantanopia affecting
[ Q: 1516 ] PasTest 2009 - Basic the temporal side of the left visual field and
Science the nasal side of the right visual field.
Where is the most likely site of her brain
Following an apparent transient ischaemic
lesion?
attack, a patient is demonstrated to have a

Dr. Khalid Yusuf El-Zohry – Sohag Teaching Hospital (01118391123)


Ref MRCPass OE OE 2012 PasTest 2009 PassMedicine 2009 PasTest Exam ReviseMRCP 685
Elzohry MRCP Questions Bank (Part 1) – 2013 (For my personal use)

1- Optic chiasm Answer & Comments


2- Temporal lobe Answer: 1- Cerebellopontine angle
3- Left parietal lobe
A complete fifth nerve lesion causes unilateral
4- Right parietal lobe sensory loss on the face, tongue and buccal
5- Optic nerve mucosa. When motor fibres are damaged
there is deviation of the jaw to the side of the
lesion as the mouth is opened. Loss of the
Answer & Comments
corneal reflex may be an early indication of a
Answer: 4- Right parietal lobe fifth nerve lesion.

Retinal lesions result in a paracentral scotoma, Brainstem lesions involving the fifth nerve
mononuclear field loss results from an optic nuclei may include brainstem glioma, multiple
nerve lesion. Lesions at the chiasm cause sclerosis, brainstem infarction or
bitemporal hemianopia (seen with pituitary syringobulbia. Lesions at the cerebellopontine
macroadenomas, for example). Proximal optic angle resulting in fifth nerve damage may
tract lesions result in a homonymous include acoustic neuroma, meningioma and
hemianopia. More distal upper lesions secondary tumour deposits. Within the
resulting from temporal lobe damage result in cavernous sinus, the trigeminal ganglion may
an upper homonymous quadrantanopia, be compressed by a pituitary tumour
parietal lobe damage results in a lower extending into the sinus, internal carotid
homonymous quadrantanopia. Widespread artery aneurysm, cavernous sinus thrombosis
diffuse bilateral occipital lobe damage may or secondary tumour. The trigeminal ganglion
cause cortical blindness, the so-called ‘Anton's may also be affected by herpes zoster
syndrome'. This unfortunate condition results infection. Prognosis for the recovery of
in patients having an inability to see, but they trigeminal nerve function is dependent on the
have little or no insight into the fact. underlying cause.

[ Q: 1518 ] PasTest 2009 - Basic [ Q: 1519 ] PasTest 2009 - Basic


Science Science

A 54-year-old woman presents with A 32-year-old, 6 ft 7 inch (2.01 m) man is


longstanding tinnitus and evidence of a admitted via ambulance from Heathrow
unilateral fifth (trigeminal nerve) palsy. Her airport after a flight from Australia. He is
MRI scan shows evidence of an acoustic ataxic with double vision and complains of
neuroma. facial numbness. On examination there is left-
sided facial numbness, nystagmus, ataxia and
Where is compression of the trigeminal nerve
Horner 's syndrome. There appears to be
most likely to be occurring?
some loss of pain and temperature sensation
1- Cerebellopontine angle on the right-hand side.
2- Cavernous sinus Which artery is most likely to be occluded in
3- Brainstem this case?
4- Skull base 1- Middle cerebral artery
5- Trigeminal ganglion 2- Posterior inferior cerebellar artery
3- Anterior inferior cerebellar artery
4- Labyrinthine artery

Dr. Khalid Yusuf El-Zohry – Sohag Teaching Hospital (01118391123)


Ref MRCPass OE OE 2012 PasTest 2009 PassMedicine 2009 PasTest Exam ReviseMRCP 686
Elzohry MRCP Questions Bank (Part 1) – 2013 (For my personal use)

5- Posterior communicating artery and C7 mediate the movements of the


shoulder and elbow, and C7 the movements
Answer & Comments of the elbow and wrist. The radial nerve is
required for normal positioning of the arm. A
Answer: 2- Posterior inferior cerebellar artery proximal ulnar nerve lesion affects the small
This is the lateral medullary syndrome muscles of the hand and wrist flexion but not
(Wallenberg's syndrome), which is the the positioning of the arm.
commonest brainstem infarction syndrome
'Economy class syndrome' as described is [ Q: 1521 ] PasTest 2009 - Basic
more commonly related to deep vein Science
thrombosis. This could potentially result in
An 82-year-old woman with atrial fibrillation
embolic stroke if a patent foramen ovale was
develops a sudden arterial occlusion of her
present and detailed echo (preferably trans-
right arm due to a brachial embolism.
oesophageal) would be required as part of his
investigation. In men of this height, Marfan's Which statement pertaining to the arterial
syndrome may be worth considering, which system of the upper limb best accords with
might contribute to the risk of vertebrobasilar usual clinical findings?
dissection. 1- The brachial artery bifurcates into the ulnar
and radial arteries at the level of the head
[ Q: 1520 ] PasTest 2009 - Basic of the radius
Science 2- The brachial artery is crossed by the median
Following a rock-climbing accident in which a nerve immediately above the elbow
foothold gave way, leaving him suspended by 3- A large single brachial vein accompanies the
one arm, a young man develops weakness of artery on its medial side
his right hand. He can manoeuvre his arm into
4- Profunda brachii arises from the brachial
any position but cannot use the hand
artery a hand's breadth above the elbow
effectively.
5- A brachial artery embolus is especially
What structure is most likely to have been
serious because of the poor collateral
damaged in this accident?
circulation around the elbow joint
1- The C6 nerve root
2- The C7 nerve root Answer & Comments
3- The T1 nerve root Answer: 1- The brachial artery bifurcates into
4- The ulnar nerve the ulnar and radial arteries at the level of the
head of the radius
5- The radial nerve
The median nerve crosses from lateral to
Answer & Comments medial at the mid-humerus. The artery is
accompanied by two vena comitantes and
Answer: 3- The T1 nerve root gives off its profunda branch near the upper
He has sustained an injury to the brachial end of the humeral shaft, where it
plexus, affecting the lowest roots (C8, T1), accompanies the radial nerve. As with all
which provides the motor supply to the joints, there is an excellent circulation around
intrinsic muscles of the hand and the long the elbow joint.
flexors and extensors of the fingers. This
deformity is known as Klumpke's paralysis. C6

Dr. Khalid Yusuf El-Zohry – Sohag Teaching Hospital (01118391123)


Ref MRCPass OE OE 2012 PasTest 2009 PassMedicine 2009 PasTest Exam ReviseMRCP 687
Elzohry MRCP Questions Bank (Part 1) – 2013 (For my personal use)

[ Q: 1522 ] PasTest 2009 - Basic 2- Weakness of hip extension


Science 3- Weakness of knee flexion
A patient has been diagnosed with a fast- 4- Weakness of ankle plantar flexion
growing pituitary adenoma. Magnetic
5- Weakness of wrist flexion
resonance image (MRI) scanning reveals
suprasellar extension.
Answer & Comments
Which structure is most likely affected?
Answer: 3- Weakness of knee flexion
1- Abducens nerve
2- Hypothalamus Typically, in a pyramidal tract lesion, the
weakness affects the extensors in the arms
3- Oculomotor nerve
and the flexors in the lower limb. As a result,
4- Third ventricle because of the reduced opposition of the
5- Optic chiasm other muscle groups, the arm tends to be held
across the body with the elbow and wrist
flexed while the leg is held in extension with a
Answer & Comments
foot drop. Note that ankle plantar flexion
Answer: 5- Optic chiasm actually causes extension of the foot.

The pituitary gland occupies the sella turcica,


which is a cup-shaped depression in the [ Q: 1524 ] PasTest 2009 - Basic
basisphenoid bone. The roof of the sella is Science
formed by the diaphragma sella, a fold of Following a stroke, a patient is found to have a
dura, which is perforated to allow passage of left homonymous inferior quadrantic visual
the pituitary stalk. Above the diaphragma lie field defect.
the suprasellar cistern, the optic chiasm and
Which one of the following structures is most
the anterior cerebral arteries. The lateral walls
likely to have been affected by the stroke?
of the pituitary fossa are formed by the
cavernous sinuses which contain the internal 1- The left optical nerve
carotid arteries and cranial nerves 3, 4, the 2- The right occipital cortex
first and second divisions of 5, and 6. Behind
3- The left frontal lobe
the sella is the pontine cistern containing the
basilar artery. The cavernous sinus, pituitary 4- The right parietal lobe
gland, and stalk and median eminence all 5- The right internal capsule
show significant enhancement after the
administration of intravenous contrast
Answer & Comments
medium.
Answer: 4- The right parietal lobe
[ Q: 1523 ] PasTest 2009 - Basic The optic nerve passes from the retina
Science through the optic foramen accompanied by
During the assessment of a stroke, a middle- the ophthalmic artery. At the optic chiasma
aged man undergoes detailed neurological the fibres from the nasal retinae cross,
examination. whereas fibres from the temporal retinae
remain uncrossed. The optic tract thus formed
Which of the following physical signs would
passes to the lateral geniculate body, where
most suggest a pyramidal lesion?
its fibres synapse and pass to the occipital
1- Weakness of forearm flexion (visual) cortex. However while passing

Dr. Khalid Yusuf El-Zohry – Sohag Teaching Hospital (01118391123)


Ref MRCPass OE OE 2012 PasTest 2009 PassMedicine 2009 PasTest Exam ReviseMRCP 688
Elzohry MRCP Questions Bank (Part 1) – 2013 (For my personal use)

through the temporal and parietal lobes, the be upper motor neurone weakness. Sensory
optic radiations separate, with the lower root compression typically causes pain in the
bundle of fibres (carrying information from dermatome supplied by the root; peripheral
the superior visual fields) passing through the polyneuropathy is usually bilateral.
temporal lobe, and upper fibres (carrying
information from the inferior visual fields) [ Q: 1526 ] PasTest 2009 - Basic
going through the parietal lobe. Thus, at these Science
anatomical sites a quadrantanopia is a
possible consequence of localised damage. A A 24-year-old woman attends the neurological
temporal lobe lesion produces a contralateral clinic for review of multiple sclerosis,
superior homonymous quadrantanopia, and a diagnosed 2 years before. She had presented
parietal lobe lesion (as in this case) produces a with blurring of vision and mild pain in her left
contralateral inferior homonymous eye, which had resolved over a period of 3
quadrantanopia. This corresponds to lesion 5 months and had not recurred. On examination
as shown on the diagram below. now, the following observations are made:
light shone in the left eye causes constriction
of the left and right pupils; light shone into the
[ Q: 1525 ] PasTest 2009 - Basic
right eye causes constriction of the right and
Science
left pupils but when the light is shone back
A 36-year-old man presents to his GP with a into the left eye, the left pupil dilates slightly.
feeling of numbness in his left leg. On Which of the following is the most likely site of
examination, he has decreased position sense the lesion responsible?
and light touch and vibration sensation
affecting his left leg to the upper part of the 1- Left ciliary ganglion
thigh. No other neurological deficit is 2- Left oculomotor nerve
demonstrable.
3- Left optic nerve
Which of the following is the most likely cause
4- Right ciliary ganglion
of this presentation?
5- Right optic nerve
1- Left dorsal column lesion
2- Left spinothalamic tract lesion Answer & Comments
3- Peripheral polyneuropathy
Answer: 3- Left optic nerve
4- Partial section of the spinal cord
The abnormal response is on the left; the right
5- Sensory root compression pupil shows a normal direct and consensual
response. The normal constriction of the left
Answer & Comments pupil indicates that the efferent pathway
(involving the Edinger-Westphal nucleus,
Answer: 1- Left dorsal column lesion
oculomotor nerve and ciliary ganglion) is
The dorsal columns carry ipsilateral intact. The defect is in the afferent pathway
proprioreception, light touch and vibration on the left, which involves fibres in the optic
sensation. Spinothalamic tract lesions cause nerve. The sign demonstrated is a relative
contralateral loss of pain and temperature afferent pupillary defect (RAPD), implying
sensation. Partial section of the cord tends to partial damage only to the afferent pathway:
cause contralateral pain and temperature if all function is lost, neither the direct reflex
sensation and ipsilateral loss of the modalities nor consensual reflex (constriction of the right
carried in the dorsal columns; there may also

Dr. Khalid Yusuf El-Zohry – Sohag Teaching Hospital (01118391123)


Ref MRCPass OE OE 2012 PasTest 2009 PassMedicine 2009 PasTest Exam ReviseMRCP 689
Elzohry MRCP Questions Bank (Part 1) – 2013 (For my personal use)

pupil in response to light shone into the left 2- The normal portal vein pressure is 8-12
eye) will be present. mmHg
3- The portal vein supplies 75% of liver blood
[ Q: 1527 ] PasTest 2009 - Basic flow
Science
4- The caudate lobe of the liver does not have
You review a 39-year-old sportsman who it's own branch of the hepatic vein
complains of knee pain. Arthroscopy reveals 5- Only the portal vein enters the liver via the
damage to the cartilage. porta hepatis
Which of the following stems best describes a
property of hyaline cartilage? Answer & Comments
1- It has a blood supply from small arterioles Answer: 3- The portal vein supplies 75% of
2- It is rich in type 1 collagen liver blood flow
3- Chondrocytes secrete collagen only The blood supply of the liver is around 25% of
4- It is avascular the resting cardiac output and arises from two
main vessels. The hepatic artery, a branch of
5- Pressure from normal joint loading
the coeliac axis supplies 25% of the total liver
accelerates damage to cartilage
blood flow. Autoregulation of blood flow by
the hepatic artery ensures constant liver
Answer & Comments blood flow. The portal vein drains most of the
Answer: 4- It is avascular gastrointestinal (GI) tract and the spleen and
constitutes 75% of liver blood flow. Normal
Hyaline cartilage forms the articular surface portal pressure is 5-8 mmHg, but blood flow
and is avascular, relying on diffusion from increases after meals. Both vessels enter the
synovial fluid for nutrients. It is rich in type II liver via the porta hepatis. The caudate lobe
collagen and forms a meshwork containing receives an independent blood supply from
proteoglycan molecules that retain water. the hepatic portal vein and artery and its
Intermittent pressure from joint loading is branch of the hepatic vein drains directly into
essential to maintain normal cartilage the inferior vena cava.
function. Chondrocytes secrete proteoglycans
and collagen and are embedded in the
[ Q: 1529 ] PasTest 2009 - Basic
cartilage. They migrate to the joint surface
Science
along with the matrix that they produce.
You are asked to see a 45-year-old man who is
[ Q: 1528 ] PasTest 2009 - Basic haemodynamically compromised and plan to
Science insert a right subclavian line.
He has a body mass index (BMI) of 38, where
You review a 54-year-old man with a history of
is the correct position for central venous
alcoholism; you are concerned that there may
cannulation?
be evidence of portal hypertension.
1- 1 cm under the mid-point of the clavicle
Which of the following stems best describes
and 0.5 cm laterally
blood flow to the liver?
2- 2 cm under the mid-point of the clavicle
1- The hepatic artery supplies 75% of the total
and 1 cm laterally
liver blood flow
3- 2.5 cm under the mid-point of the clavicle
and 2 cm laterally

Dr. Khalid Yusuf El-Zohry – Sohag Teaching Hospital (01118391123)


Ref MRCPass OE OE 2012 PasTest 2009 PassMedicine 2009 PasTest Exam ReviseMRCP 690
Elzohry MRCP Questions Bank (Part 1) – 2013 (For my personal use)

4- 0.5 cm under the mid-point of the clavicle This patient has acromegaly. Had the illness
and 1 cm laterally started before epiphyseal fusion, he would
5- 1 cm under the mid-point of the clavicle have had gigantism. Measuring the level of
and 1 cm laterally basal growth hormone is not an appropriate
diagnostic test because of variations in
concentration, although levels are often high.
Answer & Comments
Biochemical diagnosis is made by measuring
Answer: 2- 2 cm under the mid-point of the growth-hormone levels in response to a
clavicle and 1 cm laterally glucose challenge. In normal patients, the
concentration of growth hormone should fall.
In obese patients, the standard position for Prolactin is structurally similar to growth
right subclavian central venous cannulation is hormone. Levels of prolactin increase during
2 cm under the mid-point of the clavicle and 1 sleep and in response to estrogen and stress,
cm laterally. In thin patients the standard as well as during suckling in breast-feeding
position for insertion is 1 cm under the mid- women.
point of the clavicle and 0.5 cm laterally.
Jugular vein cannulation is now the preferred
[ Q: 1531 ] PasTest 2009 - Basic
choice for central venous catheterisation, as
Science
insertion under ultrasound guidance is
associated with a much lower rate of Intervertebral disc prolapse in the lumbar
complications than subclavian insertion. The spine most often affects the L4/l5 and L5/S1
major hazard of the subclavian approach is discs.
arterial puncture, as the artery lies close to
In a man presenting with acute back pain
the vein.
following an episode of lifting a heavy weight,
reduced force of which of the following
[ Q: 1530 ] PasTest 2009 - Basic movements would most suggest an L5/S1 (L5
Science root) rather than an L4/5 disc lesion (L4 root)?
A 45-year-old man presents with prognathism 1- Ankle plantar flexion
and interdental separation.
2- Eversion of the foot
Which of the following is the most appropriate
3- Extension of great toe
investigation?
4- Inversion of the foot
1- Fasting glucose test including growth
hormone measurement 5- Knee extension

2- Glucose tolerance test including growth


hormone measurement Answer & Comments

3- Basal growth-hormone measurement Answer: 3- Extension of great toe

4- Serum prolactin measurement In the lumbar spine (in contrast to the cervical
5- Thyroid function test spine) nerve roots emerge below their
respective vertebrae: thus an L4/5 disc lesion
would be expected to affect the L4 root and
Answer & Comments
an L5/S1 disc lesion to affect the L5 nerve
Answer: 2- Glucose tolerance test including root. Knee extension is mediated by L2, 3 and
growth hormone measurement 4; ankle dorsiflexion by L4 and 5; inversion of
the foot by L4 alone; eversion of the foot by
S1, and ankle plantar flexion by S1 and 2.

Dr. Khalid Yusuf El-Zohry – Sohag Teaching Hospital (01118391123)


Ref MRCPass OE OE 2012 PasTest 2009 PassMedicine 2009 PasTest Exam ReviseMRCP 691
Elzohry MRCP Questions Bank (Part 1) – 2013 (For my personal use)

Although L5 contributes to hip abduction and The internal capsule comprises an anterior
extension, knee flexion and ankle dorsiflexion, limb, between the head of the caudate
weakness in the clinical situation of L5 root nucleus and the lentiform nucleus, and the
compression is often minimal because of the posterior limb between the latter and the
contribution of other roots to these thalamus. The junction between the limbs is
movements. Weakness tends to be maximal in the genu, (‘knee'). The lentiform nucleus itself
extension of the toes, particularly the great comprises an outer putamen and an inner
toe. This is due to impairment of extensor globus pallidus. Infarction of this region is
hallucis longus, a branch of the deep peroneal often due to occlusion of a perforating branch
nerve (origin L4, 5 and S1 but predominantly of the middle cerebral artery.
L5 for extensor hallucis longus), which
contributes to ankle dorsiflexion and is [ Q: 1533 ] PasTest 2009 - Basic
responsible for dorsiflexion of the great toe. Science
A 50-year-old woman has a suspected right
[ Q: 1532 ] PasTest 2009 - Basic
renal artery stenosis. A transfemoral
Science
aortogram has been performed.
A 52-year-old hypertensive man presents with Which one of the following is likely to be
a right hemisphere stroke. Neuroimaging correct?
demonstrates the presence of a recent
infarction of the right internal capsule. 1- The femoral artery at the groin is situated
halfway between the anterior superior iliac
Which anatomical features of this region spine and the pubic tubercle
should be borne in mind?
2- The catheter passes through the common
1- The anterior limb of the internal capsule lies femoral artery into first the external iliac
between the tail of the caudate nucleus artery and then the aorta at its bifurcation
and the lentiform nucleus
3- The right renal artery also gives off the right
2- Medial to the posterior limb lies the septum ovarian and suprarenal arteries
lucidum
4- The right and left renal arteries lie in the
3- The lentiform nucleus itself comprises an transpyloric plane at the level of the first
outer globus pallidus and an inner lumbar vertebra
putamen
5- The aorta passes through the diaphragm at
4- The junction of the anterior and posterior the level of the tenth thoracic vertebra
limbs of the internal capsule is termed ‘the
crus'
Answer & Comments
5- The internal capsule receives its arterial
supply from the lenticulostriate vessels Answer: 4- The right and left renal arteries lie
derived from the roots of the middle and in the transpyloric plane at the level of the
anterior cerebral arteries first lumbar vertebra

The femoral artery at the groin constantly lies


Answer & Comments halfway between the anterior superior iliac
spine and the midline (the pubic symphysis).
Answer: 5- The internal capsule receives its
The catheter continues proximally through the
arterial supply from the lenticulostriate
external and then the common iliac artery to
vessels derived from the roots of the middle
reach the aorta. The aorta gives off the
and anterior cerebral arteries
suprarenal and the gonadal (ovarian or

Dr. Khalid Yusuf El-Zohry – Sohag Teaching Hospital (01118391123)


Ref MRCPass OE OE 2012 PasTest 2009 PassMedicine 2009 PasTest Exam ReviseMRCP 692
Elzohry MRCP Questions Bank (Part 1) – 2013 (For my personal use)

testicular) arteries as separate branches, [ Q: 1535 ] PasTest 2009 - Basic


respectively above and below the origins of Science
the renal arteries on each side. The aorta gives
A young man sustains a skull-base fracture at
off its renal branches at L1 and passes through
the middle cranial fossa which injures his right
the diaphragm at T12.
abducent (VI) nerve.

[ Q: 1534 ] PasTest 2009 - Basic Which signs are most likely to be present on
clinical examination?
Science
1- There is ptosis on the right side
A patient presents with mononeuritis
multiplex affecting the oculomotor nerve (III). 2- The pupil on the right side is constricted
and fails to respond to light
What clinical feature is likely to be present on
examination? 3- The right eyelid is numb

1- Ptosis of the upper eyelid on the affected 4- The patient is unable to deviate his right
side eye medially

2- Constricted pupil on the affected side 5- The patient is unable to deviate his right
eye laterally
3- Inability to laterally deviate the eye on that
side
Answer & Comments
4- Decreased sweating of the face on the
affected side Answer: 5- The patient is unable to deviate his
right eye laterally
5- A light shone into the affected eye fails to
produce constriction of the pupil on the The abducent nerve innervates the lateral
opposite side rectus muscle of the eye exclusively; the sole
effect of damage to this nerve is that the
Answer & Comments patient is unable to abduct (laterally deviate)
the eye.
Answer: 1- Ptosis of the upper eyelid on the
affected side
[ Q: 1536 ] PasTest 2009 - Basic
The oculomotor nerve supplies the levator Science
palpebrae superioris - paralysis of which
A dental surgeon carries out a block of the
results in ptosis (lid lag). It supplies
inferior alveolar nerve by infiltrating local
parasympathetic fibres to the constrictor
anaesthetic at the mandibular foramen.
pupillae, so the pupil widely dilates if this
nerve is damaged. The nerve supplies all the Which clinical feature may result from this
extrinsic muscles of the eyeball apart from the procedure?
lateral rectus (abducent nerve, VI) and 1- Numbness of the lower lip on the injected
superior oblique (trochlear nerve, IV). side
Abduction of the eye is thus retained.
2- Ineffective block for the incisor teeth
The nerve plays no part in the supply to the 3- Numbness of the side of the tongue
facial skin (trigeminal nerve, V), and it is the
optic nerve (II) that mediates the pathway of 4- Inability of the patient to clench his jaws
pupillary constriction to the opposite eye 5- Transient weakness of the facial muscles on
when a light is shone into the eye on the the injected side
affected side.

Dr. Khalid Yusuf El-Zohry – Sohag Teaching Hospital (01118391123)


Ref MRCPass OE OE 2012 PasTest 2009 PassMedicine 2009 PasTest Exam ReviseMRCP 693
Elzohry MRCP Questions Bank (Part 1) – 2013 (For my personal use)

Answer & Comments The levator palpebrae superioris is supplied by


the oculomotor nerve, so the patient can still
Answer: 1- Numbness of the lower lip on the
raise his upper lid. The chorda tympani fibres,
injected side
which transmit taste from the anterior two-
The inferior alveolar nerve, a branch of the thirds of the tongue, pass from the lingual
mandibular division of the trigeminal nerve nerve to the facial nerve just below the skull,
(V), traverses the inferior alveolar, or dental, and therefore remain intact in peripheral
canal of the mandible to supply all the teeth injuries of the facial nerve. Some bilateral
of that hemimandible; all the teeth on that innervation of the facial musculature exists at
side are therefore anaesthetised. The mental an upper motor neurone level but not at
branch of the nerve emerges through the lower motor neurone level, so that forehead
mental foramen to supply the lower lip, which sparing would not be expected here, given
becomes numb in a successfully performed that complete division of the nerve is deemed
block. The muscles of the tongue, of necessary.
mastication and of facial expression are not
affected. [ Q: 1538 ] PasTest 2009 - Basic
Science
[ Q: 1537 ] PasTest 2009 - Basic
A patient undergoes excision of the left
Science
submandibular salivary gland for sialectasia.
A patient undergoes a radical parotidectomy Unfortunately, his hypoglossal (XII) nerve on
for a malignant parotid tumour, at which time that side is damaged.
it is found necessary to perform a total What is the most likely outcome?
division of the left facial (VII) nerve.
1- There is numbness of the posterior one-
Postoperatively, which is the most likely third of the tongue
sequel?
2- On protruding the tongue, it deviates
1- Preservation of left sided frown in all cases towards the right
2- Numbness over the cheek on the left side 3- The uvula deviates towards the left
3- Ptosis of the upper eyelid on the left side 4- All the intrinsic muscles of the left side of
4- Loss of taste sensation over the anterior the tongue are paralysed
two-thirds of the tongue on the left side 5- The genioglossus muscle is spared
5- Tendency for food and fluids to collect in
the buccal sulcus after meals Answer & Comments
Answer: 4- All the intrinsic muscles of the left
Answer & Comments
side of the tongue are paralysed
Answer: 5- Tendency for food and fluids to
The hypoglossal nerve supplies all the muscles
collect in the buccal sulcus after meals
of the tongue but none of the palate (the
The facial nerve supplies all the muscles palatoglossus muscle, supplied by the vagus
needed for facial expression including the nerve, is a muscle of the palate). It has no
occipitofrontalis, which wrinkles the forehead. sensory component. The genioglossus muscle
A distressing feature is paralysis of the protrudes the tongue; when it is paralysed,
buccinator muscle, which acts to empty the the muscle on the opposite side is unaffected
buccal sulcus during mastication. There are no and deviates the tongue towards the affected
cutaneous sensory fibres in the facial nerve. side.

Dr. Khalid Yusuf El-Zohry – Sohag Teaching Hospital (01118391123)


Ref MRCPass OE OE 2012 PasTest 2009 PassMedicine 2009 PasTest Exam ReviseMRCP 694
Elzohry MRCP Questions Bank (Part 1) – 2013 (For my personal use)

[ Q: 1539 ] PasTest 2009 - Basic The sinus node is supplied by the right
Science coronary artery in around 60% of people, the
AV node in around 90%.
You are reviewing a 52-year-old man who has
suffered a myocardial infarction. You suspect
occlusion of the posterior descending [ Q: 1540 ] PasTest 2009 - Basic
coronary artery. Science

In this case, which region of myocardium In performing a lumbar puncture, the


would you expect to be most affected? operator needs to be familiar with the
anatomy involved.
1- The right atrium
Which anatomical feature is relevant to this
2- The right ventricle
procedure?
3- The anterior septum
1- In the newborn baby, the spinal cord
4- The anterior left ventricular wall occupies the full length of the dural sac
5- The posterior portion interventricular 2- The dural sac in the adult terminates at the
septum and the posterior left ventricular lower end of the sacral canal
wall
3- The spinal cord in the normal adult
terminates anywhere from opposite the
Answer & Comments body of T12 to the body of L3; however,
Answer: 5- The posterior portion the commonest level is at the disc space
interventricular septum and the posterior left between L1 and L2
ventricular wall 4- The spinal cord in the average male is 12
inches (30 cm) in length
The coronary system consists of left and right
coronary arteries, which arise immediately 5- The extradural space comprises a thin layer
above the aortic valve. They are unique in that of avascular connective tissue
they fill during diastole, when not occluded by
valve cusps and when not squeezed by Answer & Comments
myocardial contraction.
Answer: 3- The spinal cord in the normal adult
The right coronary artery arises from the right terminates anywhere from opposite the body
coronary sinus, giving off branches supplying of T12 to the body of L3; however, the
the right atrium and right ventricle. It then commonest level is at the disc space between
continues as the posterior descending L1 and L2
coronary artery, which supplies the posterior
portion of the interventricular septum and the The spinal cord in both adult men and women
posterior left ventricular wall. is 18 inches (45 cm) in length. In the newborn
it terminates at L3. The dural sac in the adult
The left coronary artery divides into the left extends to the level of the second sacral
anterior descending (LAD) and circumflex segment. The spinal extradural space contains
arteries. The LAD runs in the anterior loose fat (which allows the ready diffusion of
interventricular groove and supplies the local anaesthetic in an extradural block),
anterior septum and the anterior left together with the extensive vertebral venous
ventricular wall. The left circumflex artery plexus of veins.
gives off branches that supply the left atrium
and left ventricle.

Dr. Khalid Yusuf El-Zohry – Sohag Teaching Hospital (01118391123)


Ref MRCPass OE OE 2012 PasTest 2009 PassMedicine 2009 PasTest Exam ReviseMRCP 695
Elzohry MRCP Questions Bank (Part 1) – 2013 (For my personal use)

[ Q: 1541 ] PasTest 2009 - Basic damage by compression or trauma.


Science Commoner causes include: surgery (e.g.
radical mastectomy, lymph node biopsy from
A 28 year old man, who is a keen bodybuilder,
axilla); stretch injury during sports (as in this
presents with a short history of left upper limb
case); viral/ post-infectious (brachial neuritis);
discomfort and difficulty in moving his
other causes of neuropathy (vascular, toxic
shoulder. On examination he is noticed to
etc.).
have winging of the left scapula. There is no
wasting of the shoulder girdle muscles. With Accessory nerve (XI) damage can also produce
stabilisation of the scapula, he has a full range scapular winging via weakness of trapezius,
of movement and is able to elevate the but this would be milder and would be
shoulder. Sensory testing is normal, as are expected to be associated with weakness of
upper limb reflexes. shoulder elevation, which this patient does
What is the likely anatomical origin of his not have. The other options listed would tend
problem? to be associated with other symptoms and
signs. Nerve conduction studies and
1- C3,4 nerve root electromyography would help confirm the
2- Long thoracic nerve diagnosis.
3- Diffuse left brachial plexus injury
[ Q: 1542 ] PasTest 2009 - Basic
4- Spinal accessory nerve (cranial nerve XI)
Science
5- C5,6 nerve root
A 28-year-old man presents with a septic
cavernous sinus thrombosis, with high fever,
Answer & Comments
orbital oedema and proptosis.
Answer: 2- Long thoracic nerve The primary source of infection would most
Seven muscles attach the scapula (shoulder likely arise from which site?
blade) to the chest wall and help maintain 1- The chin
normal scapular control. These muscles are
2- The occipital region
trapezius, levator scapulae, rhomboids major,
rhomboids minor, pectoralis minor, omohyoid 3- The skin over the parotid gland
and serratus anterior. The latissimus dorsi has 4- The pinna of the ear
a small attachment at the base of the scapula
5- The upper lip
but does not significantly contribute to
scapular stability.
Answer & Comments
Of these muscles, the serratus anterior and
the trapezius are the most important. A Answer: 5- The upper lip
winging scapula is nearly always associated The cavernous sinus lies on either side of the
with partial or complete paralysis of either of body of the sphenoid. The cavernous sinuses
these muscles. Weakness or paralysis of the receive venous blood from the facial veins (via
serratus anterior, secondary to palsy of the the superior and inferior ophthalmic veins) as
long thoracic nerve, is the commonest cause well as the sphenoid and middle cerebral
of winging. The long thoracic nerve (origin veins. Since the cavernous sinuses receive
C5,6 motor roots, with sometimes a blood via this distribution, infections of the
contribution from C4 +/- C7) is thin, fragile face including the nose, tonsils, and orbits can
and runs an anatomical course in the neck and spread easily by this route. Anteriorly, the
upper thorax that makes it susceptible to ophthalmic veins drain into the sinus and

Dr. Khalid Yusuf El-Zohry – Sohag Teaching Hospital (01118391123)


Ref MRCPass OE OE 2012 PasTest 2009 PassMedicine 2009 PasTest Exam ReviseMRCP 696
Elzohry MRCP Questions Bank (Part 1) – 2013 (For my personal use)

communicate with the anterior facial vein, year-old woman, following a suspected
which drains the face and upper lip - hence aneurysmal bleed.
the danger of spread of infection from this Which anatomical feature should be
locus. considered when interpreting the angiogram?
1- The middle cerebral artery is the largest
[ Q: 1543 ] PasTest 2009 - Basic
single component of the circle of Willis
Science
2- The posterior cerebral artery is clearly seen
A motor cyclist involved in a road traffic on a lateral carotid angiogram
accident sustained an injury to the brachial
plexus on the right side. He is found to have 3- The vertebral arteries meet at the foramen
weakness of right shoulder abduction and magnum to form the basilar artery
forearm flexion, as well as some sensory loss 4- The middle cerebral artery courses over the
over the lateral aspect of his upper arm. The lateral aspect of the temporal lobe of the
right biceps and brachioradialis reflexes are cerebrum
absent. 5- The middle meningeal artery is an
What is the likely level of maximal plexus extracranial branch of the internal carotid
injury? artery
1- C4,5 root
Answer & Comments
2- C5,6 root
3- C6,7 root Answer: 1- The middle cerebral artery is the
largest single component of the circle of Willis
4- C7,8 root
The middle meningeal artery is a branch of the
5- C8, T1 root
maxillary artery, one of the terminal branches
of the external carotid. The posterior cerebral
Answer & Comments artery arises from the termination of the
Answer: 2- C5,6 root basilar artery, which itself arises from the two
vertebral arteries that meet on the under
A C5/C6 lesion, Erb's palsy, produces sensory surface of the brainstem - so the posterior
loss over the lateral aspect of the upper arm cerebral artery can only be visualised by
(deltoid paralysis), with loss of shoulder vertebral angiography. The middle cerebral
abduction, and paralysis of the biceps, artery is indeed the largest component of the
brachialis and coracobrachialis. In addition to circle of Willis, being, in effect, the
loss of elbow flexion, the biceps is also a termination of the internal carotid. It passes
powerful supinator of the forearm, so the through the lateral sulcus of the cerebrum
forearm assumes a pronated position. A T1 between the temporal and frontal lobes.
lesion produces a claw hand, (Klumke's palsy).
Sympathetic chain injury results in a Horner's
[ Q: 1545 ] PasTest 2009 - Basic
syndrome, with ptosis of the upper eyelid and
Science
constriction of the pupil (meiosis) on the
affected side. A hypertensive, heavy smoking, 73-year-old
man suffers a massive cardiac infarct following
[ Q: 1544 ] PasTest 2009 - Basic occlusion of his anterior interventricular
Science artery, (anterior descending artery).
Angiography is performed to demonstrate the
A cerebral angiogram is performed on a 37- coronary vessels.

Dr. Khalid Yusuf El-Zohry – Sohag Teaching Hospital (01118391123)


Ref MRCPass OE OE 2012 PasTest 2009 PassMedicine 2009 PasTest Exam ReviseMRCP 697
Elzohry MRCP Questions Bank (Part 1) – 2013 (For my personal use)

Which anatomical relationship of these vessels 3- The transverse (horizontal) fissure of the
should be borne in mind? right lung corresponds to the right fifth
1- The anterior interventricular artery arises intercostal space
above the left posterior aortic cusp 4- The lower border of the lung on each side
2- The anterior interventricular artery supplies corresponds to the tenth rib in the mid-
almost all of the left ventricle axillary line

3- There is a rich collateral circulation 5- The lower border of the lung reaches the
between the right and left coronary twelfth rib posteriorly
arteries
Answer & Comments
4- The circumflex artery is the major branch of
the right coronary artery Answer: 2- The oblique fissure of the lung
5- The posterior interventricular artery is a corresponds to the medial border of the
branch of the circumflex artery scapula when the arm is fully abducted

The apex of the lung extends about 4 cm


Answer & Comments above the medial one-third of the clavicle. The
oblique fissure does indeed correspond
Answer: 2- The anterior interventricular artery
closely to the medial border of the scapula
supplies almost all of the left ventricle
when the arm is fully abducted. The
The left coronary artery arises above the left transverse fissure of the right lung
posterior aortic cusp. After a short course it corresponds to the level of the fourth rib. The
divides into an anterior interventricular and a lower border of the lung on each side
circumflex branch. The former is the main corresponds to the eighth rib in the mid-
arterial supply to the left ventricle. axillary line and the tenth rib posteriorly.
Unfortunately, there is only a poor collateral
supply between the branches of the two [ Q: 1547 ] PasTest 2009 - Basic
coronary arteries. The posterior Science
interventricular artery arises from the right
coronary artery. A 38-year-old builder's labourer sustained a
severe fracture of his left elbow, which
damaged the ulnar nerve behind the medial
[ Q: 1546 ] PasTest 2009 - Basic
epicondyle of the humerus. A month later, he
Science
still has a total ulnar nerve paralysis.
In the clinical examination of the chest, Which clinical sign is most likely to be present
accurate knowledge of the surface markings on examination?
of the lungs is essential.
1- Sensory loss over the ulnar 3‫ ½آ‬digits on the
Which of the following corresponds to the ulnar side of the hand
clinical situation?
2- Inability to grip a sheet of paper between
1- The apex of the lung corresponds precisely his fingers when the hand is placed flat on
to the upper border of the medial third of the table
the clavicle
3- Excessive sweating over the ulnar border of
2- The oblique fissure of the lung corresponds the left hand
to the medial border of the scapula when
the arm is fully abducted 4- Index and middle fingers on the affected
side are held in the claw position

Dr. Khalid Yusuf El-Zohry – Sohag Teaching Hospital (01118391123)


Ref MRCPass OE OE 2012 PasTest 2009 PassMedicine 2009 PasTest Exam ReviseMRCP 698
Elzohry MRCP Questions Bank (Part 1) – 2013 (For my personal use)

5- Marked wasting of the thenar eminence Answer & Comments


Answer: 1- The larynx is anaesthetic inferior to
Answer & Comments the vocal cord on the affected side
Answer: 2- Inability to grip a sheet of paper The recurrent laryngeal branch of the vagus
between his fingers when the hand is placed nerve (X) supplies all the muscles of the larynx
flat on the table apart from the cricothyroid muscle, which is
The ulnar nerve (usually) supplies sensation to supplied by the superior laryngeal branch of
the skin of the fifth and the ulnar side of the the vagus nerve. The recurrent nerve also
fourth finger, front and back. There is supplies sensory fibres to the larynx inferior to
sympathetic interruption, with absence of the vocal cords. The paralysed cord is seen to
sweating in the affected area. The thenar lie in the 'paralytic' position, slightly abducted
muscles are supplied by the median nerve and from the midline, and does not move on
are therefore spared. Although the fourth and phonation.
fifth digits are held in the clawed position
when the nerve is injured at the wrist, a high [ Q: 1549 ] PasTest 2009 - Basic
lesion paralyses the long flexors to these two Science
fingers and results in the loss of this sign. A
A 54-year-old woman has undergone some
test for paralysis of the palmar interossei,
blood tests as part of an employment health
supplied by the ulnar nerve, is the inability to
screen. She reports she is in good health and,
adduct the fingers and thus to be unable to
being very health conscious, takes regular
grip a sheet of paper between them.
vitamin and mineral supplements. She is
taking bendrofluazide 2.5 mg for hypertension
[ Q: 1548 ] PasTest 2009 - Basic and her blood pressure is 132/82 mmHg. The
Science only abnormality is a serum calcium
A patient has her inferior laryngeal nerve concentration of 2.94 mmol/l.
inadvertently divided during a partial Which of the following is the most likely
thyroidectomy. cause?
Which clinical features are likely to result from 1- Diuretic treatment
this?
2- High dietary calcium intake
1- The larynx is anaesthetic inferior to the
3- High dietary vitamin D intake
vocal cord on the affected side
4- Occult malignancy
2- The larynx is totally anaesthetic on the
affected side 5- Primary hyperparathyroidism

3- All the laryngeal muscles on the affected


side are paralysed Answer & Comments

4- All the laryngeal muscles are paralysed on Answer: 5- Primary hyperparathyroidism


the affected side, apart from the posterior
Thiazides can cause hypercalcaemia but it is
cricoarytenoid muscle
usually only mild. Vitamin D itself is
5- At laryngoscopy, the affected cord is seen physiologically inactive and, whereas 1-
to lie paralysed in the midline hydroxylated derivatives can be a cause of
hypercalcaemia, vitamin D - which has to be
metabolised to activate it - is less commonly
so. Intestinal absorption of calcium is subject

Dr. Khalid Yusuf El-Zohry – Sohag Teaching Hospital (01118391123)


Ref MRCPass OE OE 2012 PasTest 2009 PassMedicine 2009 PasTest Exam ReviseMRCP 699
Elzohry MRCP Questions Bank (Part 1) – 2013 (For my personal use)

to tight control, and a high intake does not and is also poorly tolerated. Fibrates are
cause hypercalcaemia. The two most common effective in combination with statins;
causes of hypercalcaemia are primary however, there is a theoretical increased risk
hyperparathyroidism and malignancy. In an of myositis when the drugs are used in
asymptomatic individual, primary combination, although this is not a problem in
hyperparathyroidism is the more likely cause. practice. A fibrate might be appropriate if the
triglyceride concentration is elevated or the
[ Q: 1550 ] PasTest 2009 - Basic HDL-cholesterol is low, but the treatment of
Science choice would be to add ezetimibe, an inhibitor
of cholesterol absorption from the gut.
A patient with familial hypercholesterolaemia
has a total cholesterol concentration of 10.2
[ Q: 1551 ] PasTest 2009 - Basic
mmol/l, LDL-cholesterol 8.1 mmol/l, HDL-
Science
cholesterol 1.2 mmol/l and fasting
triglycerides 1.9 mmol/l. He has a strong A 19-year-old man is admitted by ambulance
family history of premature myocardial after falling into a river. He is pulled out by
infarction. He is a non-smoker and two friends but is thought to have inhaled a
normotensive. He is given lifestyle and dietary significant quantity of water. On examination
advice and prescribed a statin. Some 2 months in the Emergency room his saturation is 90%
after being on the maximum dose of the on O2 by mask. He is drowsy but conscious,
statin, his total cholesterol concentration is with bradycardia and a temperature of 34.8oC.
6.8 mmol/l, LDL-cholesterol 5.2 mmol/l, HDL- Auscultation of his chest reveals wheeze and
cholesterol 1.3 mmol/l and fasting crackles consistent with fluid inhalation.
triglycerides 1.0 mmol/l. Which of the following is the most likely
What would be the most appropriate next step biochemical imbalance to be seen?
in his management? 1- Alkalosis on ABG measurement
1- Adding a bile-acid sequestrant to his 2- Low Urea
medication
3- Acidosis on ABG measurement
2- Adding a fibrate to his medication
4- Hypokalaemia
3- Adding ezetimibe to his medication
5- Hyponatraemia
4- Adding nicotinic acid to his medication
5- Continuing on the present medication with Answer & Comments
review in a further 2 months
Answer: 3- Acidosis on ABG measurement
Answer & Comments Intravascular volume depletion is common
whatever type of fluid has been aspirated. As
Answer: 3- Adding ezetimibe to his medication
such a low measured urea, sodium or
The effects of statins are usually maximal by 4 potassium is unlikely. Hypoxaemia and
weeks after an increase in dose, and a further acidosis are the most common findings apart
fall in cholesterol is unlikely to occur if the from volume depletion. Treatment of near
dose was increased 2 months ago. Bile-acid drowning centres on appropriate re-warming,
sequestrants are effective cholesterol- fluid resuscitation and correction of hypoxia.
lowering agents but are poorly tolerated 35-60% of patients with near drowning die
because of gastrointestinal side-effects. either en route to hospital or in the
Nicotinic acid often causes cutaneous flushing, Emergency department. Because of

Dr. Khalid Yusuf El-Zohry – Sohag Teaching Hospital (01118391123)


Ref MRCPass OE OE 2012 PasTest 2009 PassMedicine 2009 PasTest Exam ReviseMRCP 700
Elzohry MRCP Questions Bank (Part 1) – 2013 (For my personal use)

hypothermia and acidosis, the risk of cardiac 1- Hypoparathyroidism


arrhythmias is high. 2- Hyperparathyroidism
3- Pseudohypoparathyroidism
[ Q: 1552 ] PasTest 2009 - Basic
Science 4- Osteomalacia
5- Rickets
You are examining some strategies for
research into a possible metabolic defect. You
think this involves an abnormality of pyruvate Answer & Comments
kinase. Answer: 5- Rickets
In which of the following processes is pyruvate
The early stages of rickets are marked by
kinase the rate limiting step?
hypocalcaemia and hypophosphataemia. Later
1- Glycolysis a compensatory rise in PTH leads to
2- Hydrolysis normalised calcium levels. Rickets is not
uncommon in children of South Asian origin
3- Hydroxylation
who may have limited sun exposure and a diet
4- Carbonation lacking in vitamin D and calcium. Management
5- Dehydrogenation is with calcium and vitamin D
supplementation.
Answer & Comments
[ Q: 1554 ] PasTest 2009 - Basic
Answer: 1- Glycolysis Science
Pyruvate kinase is the rate limiting step in A 24-year-old woman who is 11 weeks into
glycolysis and gluconeogenesis. It catalyses her first pregnancy presents with severe
the transfer of a phosphate group from nausea and vomiting and weight loss. On
phosphoenolpyruvate to ADP, yielding a examination she looks tired and dehydrated.
molecule of pyruvate and a molecule of ADP. She looks like she has lost weight, but the
Deficient pyruvate kinase activity may result in thyroid does not feel abnormal to palpation
the development of hereditary haemolytic and there are no bruits.
anaemias.
Investigations;

[ Q: 1553 ] PasTest 2009 - Basic Hb 11.9 g/dl


Science WCC 4.6 x 109/L
You are reviewing the clinical chemistry PLT 170 x 109/L
results of a 9-year-old South Asian girl. Na+ 140 mmol/l
Investigations K+ 3.9 mmol/l
Na+ 139 mmol/l Creatinine 100 μmol/l
K+ 4.9 mmol/l TSH 0.3 U/l
Creatinine 67 μmol/l Free T3 7.4 pmol/l
Ca++2.05 mmol/l Free T4 29 pmol/l
PO4-0.58 mmol/l Thyroid autoantibodies negative
Which of the following is the most likely Which of the following is the most appropriate
diagnosis? next step?

Dr. Khalid Yusuf El-Zohry – Sohag Teaching Hospital (01118391123)


Ref MRCPass OE OE 2012 PasTest 2009 PassMedicine 2009 PasTest Exam ReviseMRCP 701
Elzohry MRCP Questions Bank (Part 1) – 2013 (For my personal use)

1- Start carbimazole The median nerve supplies the lateral two


2- Start propylthiouracil lumbricals, opponens pollicis, abductor pollicis
brevis and flexor pollicis brevis; the remainder
3- Arrange an FNA of the thyroid are served by the ulnar nerve. Global muscle
4- Arrange an ultrasound of the thyroid wasting of the hand indicates damage to both
5- Observe the median and ulnar nerves with damage to
the T1 nerve root. Isolated wasting of
abductor pollicis brevis occurs in association
Answer & Comments with median nerve damage from carpal tunnel
Answer: 5- Observe syndrome. More extensive wasting may
suggest a broader diagnosis such as
This woman has evidence of very mild syringomyelia or motor neurone disease.
biochemical hyperthyroidism against a
background of severe symptoms of
[ Q: 1556 ] PasTest 2009 - Basic
hyperemesis gravidarum. Normal thyroid on
Science
examination and negative thyroid
autoantibodies reduce the likelihood of a Which of the following statements best
thyroid condition being the cause of her characterises low-density lipoproteins (LDL)?
symptoms. β-HCG stimulates production of
1- 50% of their fat content is triglyceride
thyroid hormone in a similar way to TSH,
hence the biochemistry seen here, with a 2- Their concentration is highly correlated
suppressed TSH, T3 just above the normal with dietary cholesterol content
range and an elevated T4. Anti-thyroid drugs 3- They are involved in reverse cholesterol
have no value in the management of the transport
condition, and hyperemesis usually subsides
4- They are synthesised de novo in the liver
by the 20th week of the pregnancy.(
5- They contain apolipoprotein B-100
[ Q: 1555 ] PasTest 2009 - Basic
Science Answer & Comments

A 42-year-old man with a history of road Answer: 5- They contain apolipoprotein B-100
traffic accident and injury to his back and neck The major fat in low-density lipoproteins is
presents with global muscle wasting of the left cholesterol, but the concentration is
hand. determined mainly by the rates of hepatic
Which is the nerve or nerve root most likely to synthesis and peripheral clearance; dietary
be involved? saturated fat intake correlates more strongly
1- Radial nerve with LDL-cholesterol concentration than
dietary cholesterol intake. LDLs are formed
2- Median nerve from VLDL (very-low-density lipoprotein) and
3- Ulnar nerve IDL (intermediate-density lipoprotein) by
successive removal of triglyceride and
4- T1 nerve root
modification in the circulation. It is HDL-
5- C7 nerve root cholesterol that is involved in reverse
cholesterol transport. Each particle of LDL
Answer & Comments contains one molecule of apolipoprotein B-
100, the ligand for the LDL receptor.
Answer: 4- T1 nerve root

Dr. Khalid Yusuf El-Zohry – Sohag Teaching Hospital (01118391123)


Ref MRCPass OE OE 2012 PasTest 2009 PassMedicine 2009 PasTest Exam ReviseMRCP 702
Elzohry MRCP Questions Bank (Part 1) – 2013 (For my personal use)

[ Q: 1557 ] PasTest 2009 - Basic medication. Biochemical tests reveal a serum


Science alkaline phosphatase activity of 550 U/l (upper
limit of normal (ULN) 150 U/l); serum
A 75-year-old man has gangrene of the left
creatinine concentration is 132 mmol/l,
hallux. There are no pulses to feel below the
calcium 2.42 mmol/l, phosphate 1.21 mmo l/l,
rather weak femoral pulse on that side. A
albumin 41 g/l.
duplex scan reveals a block in the superficial
femoral artery. Which of the following is the most likely cause
of the high alkaline phosphatase?
Which statement pertaining to the arterial
system of the lower limb best accords with 1- Osteomalacia
usual clinical findings? 2- Osteoporosis
1- The common femoral artery divides into its 3- Paget's disease of bone
superficial and profunda branches a hand's
4- Primary hyperparathyroidism
breadth below the inguinal ligament
5- Renal osteodystrophy
2- The femoral vein lies on the lateral side of
the common femoral artery at the groin
Answer & Comments
3- The femoral artery passes into the popliteal
fossa, as the popliteal artery, by passing Answer: 3- Paget's disease of bone
between the adductor longus and magnus
Elevated serum alkaline phosphatase activity
4- The popliteal artery lies against the is seen most frequently in cholestatic
popliteal surface of the femur deep to the hepatobiliary disease and in bone disease in
popliteal vein, which itself lies deep to the which there is an increase in osteoblastic
tibial nerve activity. It is therefore not a feature of
5- The pulse of the posterior tibial artery is felt uncomplicated osteoporosis. Serum calcium
behind the lateral malleolus concentration is usually low-normal or low in
osteomalacia, and is elevated in primary
hyperparathyroidism. The elevated creatinine
Answer & Comments
indicates renal impairment, but not to an
Answer: 4- The popliteal artery lies against the extent likely to cause renal osteodystrophy of
popliteal surface of the femur deep to the a severity suggested by an alkaline
popliteal vein, which itself lies deep to the phosphatase level this high; furthermore, the
tibial nerve serum phosphate is usually elevated and
calcium is low in this condition. A high alkaline
The common femoral artery lies on the lateral
phosphatase level is characteristic of Paget's
side of the vein and divides 3 cm distal to the
disease, a common condition in the elderly
inguinal ligament. The superficial femoral
and not always symptomatic.
artery becomes the popliteal by passing
through the hiatus in the adductor magnus.
The posterior tibial pulse is sought behind the [ Q: 1559 ] PasTest 2009 - Basic
medial malleolus. Science
A 24-year-old woman undergoes resection of
[ Q: 1558 ] PasTest 2009 - Basic the terminal ileum with fashioning of an
Science ileostomy for Crohn's disease. Some 2 weeks
after surgery, she is making a good recovery,
A 75-year-old man is given a routine health
and is eating a high-energy, low-residue diet,
check by his family doctor. He has no specific
but has a high ileostomy volume, necessitating
complaints and is not on any regular

Dr. Khalid Yusuf El-Zohry – Sohag Teaching Hospital (01118391123)


Ref MRCPass OE OE 2012 PasTest 2009 PassMedicine 2009 PasTest Exam ReviseMRCP 703
Elzohry MRCP Questions Bank (Part 1) – 2013 (For my personal use)

intravenous fluid replacement. Her serum most frequently associated with which one of
calcium concentration is 1.82 mmol/l, the following?
phosphate 1.28 mmol/l, alkaline phosphatase 1- Carcinoid tumours
82 U/l (normal < 150), albumin 30 g/l,
creatinine 80 m mol/l. Prior to surgery, her 2- Lymphoma
serum calcium concentration was 2.18 3- Multiple myeloma
mmol/l, albumin 36 g/l. 4- Small-cell carcinoma of the bronchus
What is the most likely cause of her 5- Squamous-cell carcinoma of the bronchus
hypocalcaemia?
1- Formation of insoluble calcium salts in the Answer & Comments
intestine
Answer: 5- Squamous-cell carcinoma of the
2- Hypoalbuminaemia
bronchus
3- Hypomagnesaemia
Hypercalcaemia is a frequent complication of
4- Malabsorption of calcium cancer, and is most usually the result of the
5- Malabsorption of vitamin D secretion of parathyroid hormone-related
peptide. Squamous-cell carcinomas are
Answer & Comments particularly frequently responsible, but
hypercalcaemia is uncommon with other
Answer: 3- Hypomagnesaemia bronchogenic cancers. In myeloma, the
Impaired fat absorption can lead to the secretion of osteoclast-activating cytokines is
formation of insoluble calcium salts in the gut. the usual cause of hypercalcaemia, and in
Fat and calcium are absorbed in the proximal lymphomas, unregulated extrarenal
small intestine, so, too, is vitamin D. Although production of calcitriol can be responsible.
bile salts are absorbed distally, and impaired Carcinoid tumours sometimes secrete
absorption can lead to a secondary decrease vasopressin or ACTH, but rarely cause
in proximal fat absorption, this is unlikely to hypercalcaemia.
be responsible for hypocalcaemia developing
so quickly. The normal alkaline phosphatase [ Q: 1561 ] PasTest 2009 - Basic
level also militates against vitamin D Science
deficiency. Hypocalcaemia would normally be
Osteoporosis is most reliably diagnosed by
expected to stimulate parathyroid hormone
which one of the following techniques?
secretion and cause the plasma phosphate
concentration to fall (PTH is phosphaturic). 1- Dual-energy X-ray absorptiometry (DEXA)
Patients with ileostomies can lose large 2- Measurement of serum osteocalcin
amounts of magnesium through their stomas;
3- Measurement of urinary collagen
hypomagnesaemia impairs PTH secretion and
telopeptides
can cause hypocalcaemia that is resistant to
an increased provision of calcium. 4- Quantitative computed tomography (CT)
5- Quantitative ultrasonography
[ Q: 1560 ] PasTest 2009 - Basic
Science Answer & Comments
Hypercalcaemia in malignant disease Answer: 1- Dual-energy X-ray absorptiometry
secondary to the secretion of parathyroid (DEXA)
hormone-related peptide by the tumour is

Dr. Khalid Yusuf El-Zohry – Sohag Teaching Hospital (01118391123)


Ref MRCPass OE OE 2012 PasTest 2009 PassMedicine 2009 PasTest Exam ReviseMRCP 704
Elzohry MRCP Questions Bank (Part 1) – 2013 (For my personal use)

DEXA is currently the most precise and [ Q: 1563 ] PasTest 2009 - Basic
accurate method for diagnosing osteoporosis. Science
Quantitative CT of the lumbar spine is less
A 48-year-old man is referred by his GP with
precise (and more expensive, and involves a
suspected acromegaly.
greater exposure to radiation). Quantitative
ultrasound measurements on the calcaneum Which of the following would be the most
are being evaluated but have not been proven useful investigation to establish the diagnosis?
to be superior to DEXA. Biochemical indices of 1- Glucose tolerance test with measurement
bone formation (eg osteocalcin) and of growth hormone
breakdown (N- and C-terminal cross-linking
2- Measurement of serum basal growth
telopeptides of type-1 collagen) are of value in
hormone concentration
monitoring the response of patients to
treatment, but not for diagnosis. 3- Measurement of serum growth hormone
concentration during sleep
[ Q: 1562 ] PasTest 2009 - Basic 4- Measurement of serum insulin-like growth
Science factor I (IGF-1, somatomedin C)
Which of the following features most reliably 5- Measurement of serum growth hormone
suggests that a patient presenting with concentration following exercise
diabetes has type 1?
1- Family history of diabetes Answer & Comments

2- Hypertriglyceridaemia Answer: 1- Glucose tolerance test with


measurement of growth hormone
3- History of recent weight loss
4- Onset below 20 years of age Acromegaly is caused by excessive growth
hormone secretion after the fusion of the
5- Retinopathy
epiphyses, usually as a result of a pituitary
adenoma; ectopic secretion of growth
Answer & Comments hormone-releasing hormone (GHRH) is a rare
Answer: 3- History of recent weight loss cause. Isolated (basal, unstimulated) growth
hormone concentrations are often raised in
Weight loss in diabetes is highly suggestive of acromegaly, but because the hormone is
absolute insulin deficiency such as occurs in secreted sporadically, concentrations may not
type 1 diabetes, because lipolysis is inhibited be raised. The diagnostic test is to measure
by much lower concentrations of insulin than growth hormone during an oral glucose
are required to prevent hyperglycaemia. A tolerance test: in normal individuals, growth
family history of diabetes may be present in hormone secretion is suppressed; in
type 1 and type 2 diabetes, but is commoner acromegaly, there is failure of suppression or
in the latter. Although type 2 diabetes sometimes an increase in secretion.
typically presents in older people, it is
increasingly being diagnosed in obese Measurement of IGF-1 is valuable in
children. Diabetic retinopathy is never present monitoring patients following treatment but is
at diagnosis in type 1 diabetes. not a diagnostic test. Measurement of growth
Hypertriglyceridaemia can occur with poorly hormone during sleep or following exercise
controlled type 1 or type 2 diabetes. may be helpful in the diagnosis of growth
hormone deficiency.

Dr. Khalid Yusuf El-Zohry – Sohag Teaching Hospital (01118391123)


Ref MRCPass OE OE 2012 PasTest 2009 PassMedicine 2009 PasTest Exam ReviseMRCP 705
Elzohry MRCP Questions Bank (Part 1) – 2013 (For my personal use)

[ Q: 1564 ] PasTest 2009 - Basic [ Q: 1565 ] PasTest 2009 - Basic


Science Science
A 67-year-old woman presents to the clinic A 61-year-old patient who suffered a humeral
with generalised bony aches. She has Type 2 fracture after falling off a ladder presents for
diabetes with chronic renal impairment and review after being in a cast for the past 8
had a breast carcinoma excised some 6 years weeks. He presents with weakness in the
earlier. Medication includes ramipril, deltoid, and sensory loss over the deltoid
indapamide, gliclazide and pioglitazone. On region.
examination her BP is 155/92 mmHg, her BMI Which of the following is the most likely
is 31. General examination is unremarkable. underlying lesion?
Investigations 1- Brachial plexus injury
Hb 10.9 g/dl 2- Axillary nerve injury
WCC 4.5 x 109/L 3- Radial nerve injury
9
PLT 197 x 10 /L 4- Ulnar nerve injury
ESR 15 mm/hr 5- Neuralgic amyotrophy
Na+ 140 mmol/l
K+ 4.9 mmol/l Answer & Comments
Creatinine 210 μmol/l Answer: 2- Axillary nerve injury
Ca++ 2.85 mmol/l The axillary nerve supplies motor innervation
Alk P 178 U/l to the deltoid muscle, and carries sensory
fibres from the regimental badge area. It is
Which of the following is the most likely
likely that the axillary nerve injury occurred
diagnosis?
during the original fall and was not discovered
1- Metastatic breast carcinoma at the time of the initial injury. Axillary
2- Primary hyperparathyroidism neuropathy may also be caused by direct
pressure on the axilla, for instance from using
3- Secondary hyperparathyroidism
crutches.
4- Paget’s disease
5- Multiple myeloma [ Q: 1566 ] PasTest 2009 - Basic
Science
Answer & Comments Where would you visualise the azygous lobe
Answer: 3- Secondary hyperparathyroidism on an antero-posterior (A-P) chest X-ray?

The elevated calcium and slight increase in 1- Left lower zone


alkaline phosphatase seen here, coupled with 2- Left middle zone
her chronic renal failure is highly suggestive of 3- Left upper zone
secondary hyperparathyroidism. The low ESR
makes both multiple myeloma and metastatic 4- Right upper zone
carcinoma of the breast unlikely. PTH assay 5- Right lower zone
would be the investigation of choice, with
twice weekly low dose vitamin D the most Answer & Comments
appropriate therapeutic option.
Answer: 4- Right upper zone

Dr. Khalid Yusuf El-Zohry – Sohag Teaching Hospital (01118391123)


Ref MRCPass OE OE 2012 PasTest 2009 PassMedicine 2009 PasTest Exam ReviseMRCP 706
Elzohry MRCP Questions Bank (Part 1) – 2013 (For my personal use)

An azygous lobe is seen in about 0.5% of A small area along the upper part of the
routine chest X-rays and is a normal variant. It medial border of the left kidney is in relation
is seen as a ‘reverse comma sign' behind the with the left suprarenal gland, and close to the
medial end of the right clavicle. lateral border is a long strip in contact with
the renal impression on the spleen. A
[ Q: 1567 ] PasTest 2009 - Basic somewhat quadrilateral field, about the
Science middle of the anterior surface, marks the site
of contact with the body of the pancreas, on
Which of the following structures is located in the deep surface of which are the lienal
the anterior mediastinum on computed vessels. Above this is a small triangular
tomography (CT)? portion, between the suprarenal and splenic
1- Thymus areas, in contact with the postero-inferior
surface of the stomach. Below the pancreatic
2- Oesophagus
area, the lateral part is in relation with the left
3- Aorta colic flexure, the medial with the small
4- Heart intestine. The areas in contact with the
stomach and spleen are covered by the
5- Trachea
peritoneum of the omental bursa, while that
in relation to the small intestine is covered by
Answer & Comments the peritoneum of the general cavity; behind
Answer: 1- Thymus the latter are some branches of the left colic
vessels. The suprarenal, pancreatic, and colic
The anterior mediastinum is bordered areas are devoid of peritoneum.
anteriorly by the sternum and posteriorly by
the great vessels. It contains the thymus,
[ Q: 1569 ] PasTest 2009 - Basic
lymph nodes, fat, and vessels. Disorders of the
Science
anterior mediastinum are generally thymic,
thyroid (substernal goitre), teratoma (and What would be consistent with femoral nerve
other germ cell tumors), and lymphomas damage in a patient with pelvis trauma?
(Hodgkin's disease, non-Hodgkin's lymphoma). 1- Preserved knee reflex
2- Loss of sensation over the anterior-lateral
[ Q: 1568 ] PasTest 2009 - Basic
aspect of the thigh
Science
3- Loss of power in the biceps femoris muscle
Which organ lies anterior in direct contact
4- Loss of power in the peroneus muscle
with the left kidney without separation by
visceral peritoneum? 5- Reduced power on adduction
1- Spleen
Answer & Comments
2- Left suprarenal
3- Tail of the pancreas Answer: 2- Loss of sensation over the anterior-
lateral aspect of the thigh
4- Left psoas muscle
The femoral nerve may be damaged from
5- Splenic flexure
fractures of the pelvis or femur, or
dislocations of the hip, and hip or hernia
Answer & Comments surgery. It can also be involved in psoas
Answer: 3- Tail of the pancreas abscesses, thigh wounds and frequently in

Dr. Khalid Yusuf El-Zohry – Sohag Teaching Hospital (01118391123)


Ref MRCPass OE OE 2012 PasTest 2009 PassMedicine 2009 PasTest Exam ReviseMRCP 707
Elzohry MRCP Questions Bank (Part 1) – 2013 (For my personal use)

large psoas haematomas in patients with [ Q: 1571 ] PasTest 2009 - Basic


haemophilia and diabetic amyotrophy. Partial Science
lesions are common from thigh wounds with
A 40-year-old lady presents with numbness
the nerve to the quadriceps most frequently
and tingling in little finger of her right hand,
involved and causing great problems in
and a diagnosis of ulnar neuropathy is made.
walking with the knee often giving way,
especially when descending stairs. It leads to a Which muscle of her hand is most likely to be
loss of power in the knee extension. In affected?
addition ther is quadirceps wasting, loss of 1- Extensor digiti minimi
knee jerk and impaired sensatino over the
2- Lateral 2 lumbricals
front of the thigh.
3- Flexor pollicis longus
[ Q: 1570 ] PasTest 2009 - Basic 4- Flexor pollicis brevis
Science 5- Adductor pollicis
A 24-year-old woman is admitted with
dysarthria, tremor and parkinsonian Answer & Comments
symptoms. On examination you notice yellow
Answer: 5- Adductor pollicis
brown rings on examination of the eyes, seen
at the limbus of the corneae. Given the most A number of muscles of the hand are
likely diagnosis, which part of the brain is innervated by the ulnar nerve. These include
predominantly affected by her underlying the third palmar interosseus and adductor
condition? pollicis. The weakness of adductor pollicis
1- Cerebral cortex leads to weakness of pinch grip. Ulnar
neuropathy commonly occurs due to
2- Frontal lobes
entrapment at the elbow due either to injury
3- Basal ganglia or osteoarthritis. Other causes include those
4- Brainstem associated with mono-neuropathy such as
multiple sclerosis or diabetes mellitus. Surgical
5- Cerebellum
intervention is often successful where elbow
entrapment has occurred.
Answer & Comments
Answer: 3- Basal ganglia [ Q: 1572 ] PasTest 2009 - Basic
Science
This woman has Wilson's disease, the
appearance of her corneae being highly Which of the following is not typically a cause
suggestive of Kayser-Fleischer rings, of hypercalcaemia?
confirming the diagnosis. The basal ganglia are 1- Hyperparathyroidism
most affected, but changes in T2 signal on
2- Hypothyroidism
magnetic resonance imaging (MRI) scanning
are also seen in the cerebellum and cortical 3- Milk-alkali syndrome
atrophy may be seen in late stage disease. The 4- Sarcoid
disease is caused by a loss of ability to export
5- Squamous-cell carcinoma
copper into the bile and to incorporate it into
ceruloplasmin. Penicillamine is commonly
used as medical therapy of choice. Answer & Comments
Answer: 2- Hypothyroidism

Dr. Khalid Yusuf El-Zohry – Sohag Teaching Hospital (01118391123)


Ref MRCPass OE OE 2012 PasTest 2009 PassMedicine 2009 PasTest Exam ReviseMRCP 708
Elzohry MRCP Questions Bank (Part 1) – 2013 (For my personal use)

Two of commonest causes of hypercalcaemia over the medial aspect of the elbow which
in the western world are primary worsens on resisted wrist flexion and forearm
hyperparathyroidism and malignancy. In pronation. In severe cases there is weakness
primary hyperparathyroidism there is excess of muscles of the hand and loss of sensation in
production of parathyroid hormone (PTH); the ulnar nerve distribution. Medical
although usually from a benign adenoma, this intervention with appropriate physiotherapy
sometimes results from hyperplasia of the is the mainstay of treatment, rarely surgical
parathyroid glands and, in rare cases, a release is indicated, but is usually successful.
carcinoma. Thyrotoxicosis can cause
hypercalcaemia as well as osteoporosis. The [ Q: 1574 ] PasTest 2009 - Basic
milk-alkali syndrome can occur in patients Science
who suffer from dyspepsia and drink milk and
alkali-containing antacids, which may reduce A 64-year-old man is admitted with a severe
the renal excretion of calcium. Around one- haematemesis. Upper GI endoscopy identifies
fifth of those with sarcoid have increased a posterior gastric ulcer.
calcium levels. Various mechanisms cause Bleeding is most likely to having occurred from
raised hypercalcaemia of malignancy. which main vessel?
1- Splenic artery
[ Q: 1573 ] PasTest 2009 - Basic
2- Left gastroepiploic artery
Science
3- Inferior pancreaticoduodenal artery
A 22-year-old golfer presents for review. He
has noticed a decrease in his grip strength 4- Oesophageal branch of the left gastric
over the past few weeks and pain over his artery
elbow. On examination he has pain over his 5- Gastroduodenal branch of the right gastric
elbow which is worse on wrist flexion and artery
resisted forearm pronation. There is weakness
of the adductor policis muscle and loss of Answer & Comments
pinch grip and loss of sensation over the
lateral fingers of the hand. Answer: 1- Splenic artery
Which of the following is the most likely cause A posterior gastric ulcer may adhere to, and
for his injury? ulcerate, the splenic artery as this runs along
1- Lateral epicondylitis the upper border of the pancreas, resulting in
a drenching haemorrhage. A lesser curve
2- Radial neuropathy gastric ulcer may implicate the left gastric
3- Median neuropathy artery - the gastroepiploic vessels lie along the
4- Medial epicondylitis greater curve of the stomach. A posterior
duodenal ulcer may erode the gastroduodenal
5- Brachial plexus injury branch of the right gastric artery - ‘the ulcer of
duodenal haemorrhage' - the inferior
Answer & Comments pancreaticoduodenal artery supplies the lower
part of the second part of the duodenum, well
Answer: 4- Medial epicondylitis
clear of the site of ulceration. Oesophageal
Medial epicondylitis occurs with increased varices commonly extend into the upper
frequency in golfers and patients who have stomach and are, of course, venous in origin.
excessive use of the elbow joint such as
basketball players. Symptoms include pain

Dr. Khalid Yusuf El-Zohry – Sohag Teaching Hospital (01118391123)


Ref MRCPass OE OE 2012 PasTest 2009 PassMedicine 2009 PasTest Exam ReviseMRCP 709
Elzohry MRCP Questions Bank (Part 1) – 2013 (For my personal use)

[ Q: 1575 ] PasTest 2009 - Basic Answer & Comments


Science
Answer: 5- Inability to abduct the thumb
A 44-year-old woman presents complaining
The median nerve supplies all the muscles in
that she has trouble opening jars and holding
the anterior compartment of the forearm,
a pencil. On examination she has weakness of
apart from the flexor carpi ulnaris and the
the adductor pollicis muscle. There is also
flexor digitorum profundus to the ulnar two
sensory loss over the volar surface of the
fingers: so these two fingers can still be flexed.
hypothenar eminence.
The radial nerve supplies the extensors -
Which of the following is the likely site of the hence no wrist drop. The ulnar nerve supplies
lesion which has caused this clinical picture? the skin of the ulnar side of the hand, hence
1- Median nerve no anaesthesia there. It also supplies the
interossei muscles of the hand, which effect
2- Radial nerve
abduction and adduction of the fingers.
3- Ulnar nerve Absence of thumb abduction, due to paralysis
4- T7 of abductor pollicis brevis, is a good test for
median nerve paralysis.
5- Anterior interosseous nerve

[ Q: 1577 ] PasTest 2009 - Basic


Answer & Comments
Science
Answer: 3- Ulnar nerve
A 62-year-old man presents with lower back
The site of compression of the ulnar nerve is pain radiating into the posterior part of the
likely to be just proximal to, or within Guyon’s tops of both legs. He also reports trouble with
canal. Common causes of this type of injury difficulty starting and stopping his stream of
include a large, complex multi-lobulated urine and difficulty making it to the toilet
ganglion, and fracture repair, where trauma/ when he wants to pass stool. On examination
post-operative swelling leads to nerve he has local tenderness to palpation over the
compression. lower back. There is diminished light touch in
the perianal region and decreased anal tone.
[ Q: 1576 ] PasTest 2009 - Basic Where is the most likely cause of his
Science symptoms?
An anaesthetist performs a successful block of 1- Conus medullaris lesion
the median nerve at the elbow. 2- L1 disc lesion
Which neurological sign is likely to be present 3- Cauda equina syndrome
on examination?
4- T10 disc lesion
1- Inability to flex the fingers
5- Spinal meningioma
2- An obvious wrist drop deformity
3- The palm of the hand is totally Answer & Comments
anaesthetised
Answer: 3- Cauda equina syndrome
4- Inability to abduct and adduct the fingers
The clinical picture seen here is typical of
5- Inability to abduct the thumb
cauda equina syndrome with lower back pain
and saddle anaesthesia with bowel and
bladder disturbance, caused by compression

Dr. Khalid Yusuf El-Zohry – Sohag Teaching Hospital (01118391123)


Ref MRCPass OE OE 2012 PasTest 2009 PassMedicine 2009 PasTest Exam ReviseMRCP 710
Elzohry MRCP Questions Bank (Part 1) – 2013 (For my personal use)

of nerve roots below the end of the spinal Paralysis of the deltoid muscle causes
cord. MRI or CT scanning of the lower spine is weakness of shoulder abduction, particularly
the investigation of choice, with initial pain after 30 degrees of abduction. The
relief the cornerstone of management. Where supraspinatus initiates abduction of the
a cause for compression is identified, such as shoulder in the first 30 degrees of movement.
intervertebral disc herniation, neurosurgical Therefore, this part of shoulder abduction
intervention is of value. may be spared (especially in athletes
presenting sports injury to the axillary nerve,
[ Q: 1578 ] PasTest 2009 - Basic for example, who may have hypertrophied
Science supraspinati). However even if the
supraspinatus is fully functional, it is too weak
A 78-year-old man had poliomyelitis as a child, a muscle to be able to abduct the whole
which left him with total paralysis of the left weight of the arm, especially in an elderly man
deltoid muscle. with a longstanding neuromuscular deficit.
Which feature is most likely to be present on The deltoid, in addition to being the powerful
clinical examination? abductor of the elbow, also assists in flexion
and medial rotation (and extension and lateral
1- Anaesthesia over the ‘epaulette’ region of
rotation) of the shoulder by means of its
the left shoulder
anterior and posterior fibres, respectively.
2- The acromion process of the scapula forms Weakness of these movements compared to
the most lateral bony landmark of the left the normal side can be detected on careful
shoulder examination.
3- Drooping of the left shoulder compared to
the right side [ Q: 1579 ] PasTest 2009 - Basic
4- Detectable weakness in drawing the arm Science
forward and internally rotating the An 84-year-old man had his left sciatic nerve
shoulder when this is compared with the completely transected just inferior to the
right side buttock crease by a piece of shrapnel during
5- Abduction of the shoulder to 60o is likely to the D-day landings in 1944.
be preserved due to action of the intact Which sign is likely to be present on current
supraspinatus muscle on the left side neurological examination?
1- Complete anaesthesia below the knee
Answer & Comments
2- Spastic paralysis of the lower limb, with
Answer: 4- Detectable weakness in drawing increased ankle jerk
the arm forward and internally rotating the
3- Plantar flexed and everted foot
shoulder when this is compared with the right
side 4- Paralysed quadriceps femoris

Poliomyelitis only affects the anterior horn 5- Unimpaired hip abduction


cells of the spinal cord, so there is no sensory
loss. Although the shoulder appears flattened, Answer & Comments
due to deltoid wasting, the greater tubercle of
Answer: 5- Unimpaired hip abduction
the humerus remains the most lateral bony
landmark of the shoulder. It is paralysis of The saphenous branch of the femoral nerve -
trapezius that results in shoulder drop. the longest cutaneous nerve in the body - is
intact, so there is retention of normal

Dr. Khalid Yusuf El-Zohry – Sohag Teaching Hospital (01118391123)


Ref MRCPass OE OE 2012 PasTest 2009 PassMedicine 2009 PasTest Exam ReviseMRCP 711
Elzohry MRCP Questions Bank (Part 1) – 2013 (For my personal use)

sensation along the medial side of the anterior home. On examination there is weakness of
aspect of the leg down to the base of the the foot and ankle dorsiflexors and she has a
hallux. Peripheral nerve injuries result in so-called steppage gate. She has sensory loss
flaccid paralysis, with loss of stretch reflexes over the lateral portion of the leg extending
of the paralysed muscles. The foot is plantar onto the dorsum of the foot.
flexed due to gravity, (foot drop). However, it Which of the following is the most likely
is not everted (eversion is produced by the location of neurological injury?
peroneus longus and brevis, which are
paralysed), nor is it inverted, a function of the 1- Femoral nerve injury
paralysed long flexors of the foot. The 2- Common peroneal nerve injury
quadriceps is innervated by the intact femoral 3- Tibial nerve injury
nerve, and hip abduction, effected by the
gluteus medius and minimus, supplied by the 4- L3 nerve lesion
intact superior gluteal nerve, is unaffected. 5- L4 nerve lesion

[ Q: 1580 ] PasTest 2009 - Basic Answer & Comments


Science
Answer: 2- Common peroneal nerve injury
Which of the following is typically the earliest
lesion to develop in diabetic retinopathy? This woman has foot drop, a feature of
common peroneal nerve injury. It can be
1- Blot haemorrhages temporary, as in a “dead leg” because of a
2- Dot haemorrhages blow to the back of the knee, or it can be long-
lasting, as in this case. Causes include
3- Hard exudates
mononeuritis, (the cause here), compartment
4- Macular oedema syndrome, or a direct traumatic incident such
5- Soft exudates as a laceration. Physiotherapy is the mainstay
of treatment for foot drop, with the use of
Answer & Comments aids such as a foot drop splint.

Answer: 2- Dot haemorrhages


[ Q: 1582 ] PasTest 2009 - Basic
The earliest lesions to be detected in diabetic Science
retinopathy are usually dot haemorrhages A 25-year-old woman is referred to you for
(capillary microaneurysms) and venous treatment of her asthma. She is otherwise
dilatation. Hard exudates are characteristic of well and there is no significant previous
diabetic retinopathy but they, and blot medical history. During the consultation it
haemorrhages, usually appear later. These transpires that her sister died of cystic fibrosis
three are the components of ‘background' and she is worried about having a child
retinopathy. Soft exudates and macular affected with the same disease. Her chest X-
oedema are later, sight-threatening, ray is normal.
preproliferative, changes.
Assuming a population carrier frequency of 1
in 25, what is the chance of her having an
[ Q: 1581 ] PasTest 2009 - Basic
affected child?
Science
1- 1 in 10
A 48-year-old woman presents to the clinic
complaining that her left foot drags and gets 2- 1 in 50
caught when she tries to climb the steps at 3- 1 in 100

Dr. Khalid Yusuf El-Zohry – Sohag Teaching Hospital (01118391123)


Ref MRCPass OE OE 2012 PasTest 2009 PassMedicine 2009 PasTest Exam ReviseMRCP 712
Elzohry MRCP Questions Bank (Part 1) – 2013 (For my personal use)

4- 1 in 150 increased secretion of TSH. Patients with a


5- 1 in 200 slightly elevated TSH and low-normal
thyroxine are said to have ‘compensated' or
‘borderline' hypothyroidism. In some
Answer & Comments
individuals, it appears that this state can be
Answer: 4- 1 in 150 maintained without progression to frank
hypothyroidism. Triiodothyronine
Your patient would have a 2 in 3 chance of concentrations tend to fall later than
being a carrier, as she is not affected herself thyroxine concentrations in hypothyroidism;
(the history of asthma is coincidental). Her the concentration of thyroxine-binding
partner would have a 1 in 25 chance of being a globulin does not change significantly.
carrier. The chance of having an affected child
if both are carriers is 1 in 4. Therefore the
[ Q: 1584 ] PasTest 2009 - Basic
overall chance is 2/3 150/1 = 4/1 —‫ أ‬25/1 —‫أ‬.
Science
The commonest mistake (which also applies to
Botulinum toxin has which of the following
other autosomal-recessive diseases) is to
features?
assume that her chance of being a carrier is 1
in 2, on the premise that there is a 1 in 4 1- It is produced by a Gram-positive, aerobic
chance of being affected, 1 in 4 of not being a bacillus
carrier and a 1 in 2 chance of being a carrier. 2- The bacillus has 15 serotypes
As she is not affected, the chance of being a
3- Its main activity is at the presynaptic
carrier is in fact 2 in 3 and the chance of not
membrane
being a carrier is 1 in 3.
4- It may be used in the treatment of
[ Q: 1583 ] PasTest 2009 - Basic myasthenia gravis
Science 5- It may be used in the treatment of
blepharospasm
A 75-year-old woman is being followed by her
GP for suspected developing primary
hypothyroidism. Answer & Comments

Which of the following biochemical changes Answer: 5- It may be used in the treatment of
would you most expect to occur first? blepharospasm
1- Fall in serum free thyroxine Clostridium botulinum is a Gram-positive,
2- Fall in serum thyroxine-binding globulin spore-forming, obligate anaerobe. The bacillus
has seven serotypes, A to G. They have a wide
3- Fall in serum free triiodothyronine
range of therapeutic usage, from glabellar
4- Fall in serum total triiodothyronine lines, blepharospasm, spasticity, anismus, anal
5- Increase in serum TSH fissure to dystonia. However, myasthenia
gravis would be expected to worsen with such
treatment.
Answer & Comments
Answer: 5- Increase in serum TSH [ Q: 1585 ] PasTest 2009 - Basic
Hypothyroidism develops gradually, often Science
over many months or even years. In the early A 25-year-old patient presents with anaemia
stages, free thyroxine concentrations are and jaundice. A blood film shows
maintained in the normal range by the

Dr. Khalid Yusuf El-Zohry – Sohag Teaching Hospital (01118391123)


Ref MRCPass OE OE 2012 PasTest 2009 PassMedicine 2009 PasTest Exam ReviseMRCP 713
Elzohry MRCP Questions Bank (Part 1) – 2013 (For my personal use)

polychromasia, bite cells, reticulocytosis and 2- Exon deletion or duplication in the


Heinz bodies. dystrophin gene occurs in 60% of patients
What is the most probable diagnosis? 3- Prenatal diagnosis involves analysis of
1- Haemolytic uraemic syndrome restriction fragment length polymorphisms
(RFLPs)
2- Autoimmune haemolytic anaemia
4- The genetic defect affects mainly skeletal
3- Glucose-6-phosphate dehydrogenase muscle
deficiency
5- The majority of cases are due to new
4- Hereditary spherocytosis mutations
5- Paroxysmal nocturnal haemoglobinuria
Answer & Comments
Answer & Comments
Answer: 2- Exon deletion or duplication in the
Answer: 3- Glucose-6-phosphate dystrophin gene occurs in 60% of patients
dehydrogenase deficiency
Duchenne's muscular dystrophy is an X-linked
The blood picture, caused by haemolysis, is recessive disorder in which affected boys
consistent with G6PD deficiency. Glucose-6- develop progressive weakness of the limb-
phosphate dehydrogenase deficiency is the girdle muscles. Most muscular tissues,
commonest red cell enzyme defect. including cardiac tissues, are involved. An
Inheritance is sex-linked and the disease is abnormally high creatinine kinase level is
common in Africa, the Mediterranean and the found in all these patients. Since 60% of
Middle and Far East. The majority are patients have an exon deletion or duplication
asymptomatic until there is an oxidative crisis in the dystrophin gene, this can be tested
precipitated by drugs (eg primaquin, directly without the need for analysis of
sulfonamides, ciprofloxacin, quinidine, RFLPs.
probenecid), fava bean ingestion or illness.
Reticulocytosis is common and indicates [ Q: 1587 ] PasTest 2009 - Basic
active erythropoiesis. The presence of Heinz Science
bodies is characteristic of G6PD deficiency. In
hereditary spherocytosis, the red blood cells A middle-aged woman is referred to you by
are usually spherical in shape. This is an her GP with a 6-month history of increasing
autosomal dominant red cell membrane breathlessness on exertion. Her son is affected
defect where the RBCs are osmotically fragile. with Duchenne's muscular dystrophy, as was
Splenomegaly is common. Blood film shows her brother.
spherocytes. Hereditary elliptocytosis has the What would you be concerned about in her?
same clinical features as hereditary
1- Bronchiectasis
spherocytosis with the presence of elliptocytic
red cells on the blood film. 2- Cardiomyopathy
3- Emphysema
[ Q: 1586 ] PasTest 2009 - Basic 4- Respiratory muscle failure
Science
5- Pericardial effusion
In Duchenne's muscular dystrophy, which of
the following statements applies? Answer & Comments
1- Serum creatinine kinase is elevated in 30%
Answer: 2- Cardiomyopathy
of cases

Dr. Khalid Yusuf El-Zohry – Sohag Teaching Hospital (01118391123)


Ref MRCPass OE OE 2012 PasTest 2009 PassMedicine 2009 PasTest Exam ReviseMRCP 714
Elzohry MRCP Questions Bank (Part 1) – 2013 (For my personal use)

Duchenne's muscular dystrophy (DMD) is an The matrix metalloproteinases (MMPs) are a


X-linked condition that is typically diagnosed family of 24 proteolytic enzymes that share
in boys between the ages of 3 and 6 years. common characteristics. All are inhibited by
The boys have difficulty walking and develop specific tissue inhibitors of the
calf pseudohypertrophy. They are typically metalloproteinases (TIMPS). All contain a zinc
wheelchair-bound by early adolescence and atom. Each is involved in the degradation of at
are unlikely to live beyond 30 years, usually least one component of the extracellular
dying from cardiomyopathy or respiratory matrix and of basement membrane proteins
failure. Other complications include and bioactive mediators. All are secreted as a
contractures, scoliosis, intellectual impairment proenzyme, which in each case is activated by
and reduced night vision. The creatine cleavage of defined peptide sequences. All are
phosphokinase is grossly elevated and the involved in normal remodelling processes,
diagnosis can be made by muscle biopsy. such as embryonic development, postpartum
Genetic testing will detect around two-thirds involution of the uterus, bone and growth-
of mutations in the dystrophin gene. plate remodelling, ovulation and wound
healing. A range of MMP inhibitors is currently
Because both her son and brother are
under development for the treatment of solid
affected with DMD, this woman is an obligate
tumours.
carrier. Female carriers can develop a
proximal myopathy and there is a 10-20%
lifetime risk of cardiomyopathy. [ Q: 1589 ] PasTest 2009 - Basic
Science

[ Q: 1588 ] PasTest 2009 - Basic You are asked to see a 19-year-old man who
Science has had recurrent pneumothoraces. He is tall
with pes planus and has an increased arm
Which of the following statements is true
span to height ratio and upper segment to
about the matrix metalloproteinases, which
lower segment body ratios.
play a major role in pathological processes,
including rheumatoid arthritis, periodontitis, Which other feature would be most helpful in
vascular disease as well as tumour invasion making a diagnosis of Marfan's syndrome?
and metastasis? 1- Joint hypermobility
1- All are controlled by specific tissue 2- High arched palate
stimulators of the metalloproteinases
3- Mid-systolic click
2- All contain an iron atom
4- Arachnodactyly
3- Each is involved in the synthesis of at least
5- Early diastolic murmur
one component of the extracellular matrix,
basement membrane proteins and
bioactive mediators Answer & Comments

4- All are secreted as a proenzyme, which in Answer: 5- Early diastolic murmur


each case is activated by cleavage of
Marfan's syndrome is an autosomal-dominant
defined glycoprotein sequences
syndrome caused by mutations in the fibrillin-
5- All share sequence homologies 1 gene (FBN1) on chromosome 15q21 with an
incidence of 1-2 per 10,000. Some 25% are
Answer & Comments new mutations and the gene is large and
polymorphic, which make diagnostic genetic
Answer: 5- All share sequence homologies testing problematic. The diagnosis is therefore

Dr. Khalid Yusuf El-Zohry – Sohag Teaching Hospital (01118391123)


Ref MRCPass OE OE 2012 PasTest 2009 PassMedicine 2009 PasTest Exam ReviseMRCP 715
Elzohry MRCP Questions Bank (Part 1) – 2013 (For my personal use)

currently made on clinical criteria. These cover pathogenesis of RNA viruses, such as HIV,
the skeletal, eye, cardiac, respiratory and skin allowing their genetic code to be integrated
systems and are divided into major and minor into the host genome as DNA. It is also very
criteria. Although arachnodactyly, high arched important in molecular biology, allowing
palate, mitral valve prolapse and joint segments of DNA to be produced from mRNA.
hypermobility are all features of Marfan's
syndrome, they are relatively non-specific [ Q: 1591 ] PasTest 2009 - Basic
(minor criteria). On the other hand, an early Science
diastolic murmur indicates aortic valve
incompetence, which is likely to be secondary A 27-year-old woman presents with
to aortic root dilatation, a major feature of worsening back pain. As a child, she and her
Marfan's syndrome. This is the most serious sister had several admissions to hospital with
complication of Marfan's syndrome and bone fractures following minor trauma and
affected individuals require annual cardiology she has recently developed bilateral hearing
assessments. loss. Apart from some teeth discoloration and
mild joint laxity, examination was normal. A
Other major skeletal features include chest vertebral X-ray showed partial collapse of L3.
wall deformities (pectus carinatum or pectus
What is the most likely diagnosis?
excavatum, requiring surgery), a reduced
upper segment to lower segment ratio, an 1- Osteogenesis imperfecta
arm span to height ratio > 1.05, a scoliosis of > 2- Early-onset osteoporosis
20 degrees and pes planus. Lens dislocation
3- Ehlers-Danlos syndrome
(usually upwards) is found in 50-80% of
affected individuals. 4- Malignant bone metastases
5- Multiple myeloma
[ Q: 1590 ] PasTest 2009 - Basic
Science Answer & Comments
You are teaching molecular biology to a group Answer: 1- Osteogenesis imperfecta
of medical students, and one asks you about
how reverse transcriptase works. Osteogenesis imperfecta is a dominantly
inherited disorder of collagen, and this woman
How does reverse transcriptase work?
most probably has the type 1 (least severe)
1- It amplifies segments of DNA form. Type 2 is lethal in utero or shortly after
2- It cleaves specific portions of DNA birth and type 3 is the severe form where
affected individuals have hundreds of
3- It is involved in protein synthesis
fractures and usually become wheelchair-
4- It transcribes RNA from DNA bound. Clinical features include blue sclerae
5- It transcribes DNA from RNA (not always present, as in this case), early-
onset deafness and dentinogenesis
imperfecta.
Answer & Comments
Ehlers-Danlos syndrome is another
Answer: 5- It transcribes DNA from RNA
collagenopathy. The classical type is typically
Reverse transcriptase is a DNA polymerase associated with skin laxity and joint
enzyme that is involved in transcribing single hypermobility but not with susceptibility to
stranded portions of RNA into lengths of bone fractures.
double stranded DNA. Reverse transcriptase
has an important role to play in the

Dr. Khalid Yusuf El-Zohry – Sohag Teaching Hospital (01118391123)


Ref MRCPass OE OE 2012 PasTest 2009 PassMedicine 2009 PasTest Exam ReviseMRCP 716
Elzohry MRCP Questions Bank (Part 1) – 2013 (For my personal use)

The other conditions should be considered tubers, but these would not explain the
and excluded. However, none would explain history of sudden-onset headache.
her previous history of bone fractures, teeth
discoloration or hearing loss. [ Q: 1593 ] PasTest 2009 - Basic
Science
[ Q: 1592 ] PasTest 2009 - Basic
What is the main feature of DNA sequence
Science
polymorphisms that differentiates them from
A 38-year-old woman is sent as an emergency mutations?
to you with an acute-onset headache and 1- They are common in the population
deteriorating conscious level. Her husband
mentions that her brother has recently had a 2- They often cause serious medical conditions
kidney transplant, although he is not sure 3- They usually interfere with normal gene
why. function
What condition may be running in the family? 4- They are not found in certain ethnic groups
1- von Hippel-Lindau disease 5- They are evenly spread throughout all
2- Hereditary haemorrhagic telangiectasia genes

3- Polycystic kidney disease


Answer & Comments
4- Alport's syndrome
Answer: 1- They are common in the
5- Tuberous sclerosis
population

Answer & Comments A polymorphism is a DNA sequence variant


that is common in the population, whereas a
Answer: 3- Polycystic kidney disease mutation is a change in a DNA sequence away
Intracranial aneurysms occur in approximately from normal that results in an abnormal rare
10% of individuals with autosomal-dominant variant. However, these definitions are not
polycystic kidney disease (ADPKD), and absolute. The usual cut-off point between a
approximately 50% of patients with ADPKD mutation and a polymorphism is 1%.
have end-stage renal disease by age 60 years. Therefore, if the DNA sequence variant has a
It is a relatively common genetic disorder and frequency of 1% or higher, it is classed as a
there are two known ADPKD genes - PKD1, polymorphism, whereas it is regarded as a
accounting for about 85% of cases, and PKD2, mutation if the frequency is lower than 1%.
accounting for about 15% of cases. Polymorphic sequence variants are found in
all populations and usually do not cause
von Hippel-Lindau disease can cause renal clinically serious diseases. Some genes are
cysts, renal carcinoma and cerebellar more polymorphic than others. Many
haemangioblastomas, but a history of renal polymorphisms are found outside of genes
transplant would be unusual. Cerebral and have no effect on gene function. Others
atriovenous malformations can occur in may be found within genes, and modify gene
hereditary haemorrhagic telangiectasia and function to influence characteristics such as
these may result in intracerebral bleeding, but height and hair colour rather than being
the kidneys are not typically involved. Alport's pathogenic. However, polymorphic sequence
syndrome causes renal failure and deafness. variation does affect disease susceptibility and
Renal cysts can be found in tuberous sclerosis can also influence drug responses.
(TS). In addition, there is a high frequency of
subependymal glial nodules and cortical

Dr. Khalid Yusuf El-Zohry – Sohag Teaching Hospital (01118391123)


Ref MRCPass OE OE 2012 PasTest 2009 PassMedicine 2009 PasTest Exam ReviseMRCP 717
Elzohry MRCP Questions Bank (Part 1) – 2013 (For my personal use)

[ Q: 1594 ] PasTest 2009 - Basic 4- Microarray analysis


Science 5- Chromosome analysis
When is a gene mutation causing a particular
genetic disorder said to be highly penetrant? Answer & Comments
1- It affects a large number of the population Answer: 2- Linkage analysis
2- Affected individuals with the same
It is quite common for a gene mutation not to
mutation have a very similar phenotype
be found in a family even though the gene for
3- Individuals with the mutation invariably the condition is known. Often this is due to
develop the phenotype the limitations of mutation screening
4- The disorder only manifests early in life (although sometimes a mutation in a different
gene might cause the same phenotype). If a
5- It affects both sexes equally
reasonable number of samples are available
from affected and unaffected members of a
Answer & Comments family then linkage analysis can be performed
Answer: 3- Individuals with the mutation using microsatellite markers. By testing both
invariably develop the phenotype affected and unaffected members, the allele
carrying the mutation can be tracked through
The penetrance is the proportion of the family. DNA sequencing and Southern
individuals with the gene who show the blotting are mutation detection techniques
phenotype, eg Huntington's disease shows a and microarray analysis is a gene expression
high penetrance as individuals with the technique. Chromosome analysis is used to
mutation invariably develop the disorder. The look at gross chromosome structure rather
expression is the degree to which the disorder than gene analysis.
is expressed in the individual; for example,
neurofibromatosis type 1 has a very variable [ Q: 1596 ] PasTest 2009 - Basic
expression, in that individuals can manifest in Science
a variety of ways ranging from mild to severe.
A 25-year-old accountant is admitted to
hospital with severe abdominal pain, vomiting
[ Q: 1595 ] PasTest 2009 - Basic
and postural hypotension. She has had similar
Science
attacks in the past. A mid-stream urine sample
Multiple members from a large family suffer shows the presence of high levels of
from an autosomal-dominant disorder. aminolaevulinic acid (ALA) and
Although the gene for the condition is known, porphobilinogen (PBG).
a mutation has not been found in the family.
What is the most likely diagnosis?
One of the family members wants to know if
he will develop the disease as his father is 1- Hereditary coproporphyria
affected. Blood samples are available from 2- Porphyria cutanea tarda
both affected and unaffected members.
3- Acute intermittent porphyria
Which molecular technique is most useful in
4- Erythropoietic protoporphyria
this situation?
5- Variegate porphyria
1- DNA sequence analysis
2- Linkage analysis
Answer & Comments
3- Southern blotting
Answer: 3- Acute intermittent porphyria

Dr. Khalid Yusuf El-Zohry – Sohag Teaching Hospital (01118391123)


Ref MRCPass OE OE 2012 PasTest 2009 PassMedicine 2009 PasTest Exam ReviseMRCP 718
Elzohry MRCP Questions Bank (Part 1) – 2013 (For my personal use)

Acute intermittent porphyria (AIP) presents from his mother who must be a carrier (she
with gastrointestinal and neurological may even be homozygous as women in
symptoms. Central abdominal pain and bilious particular are often asymptomatic). In this
vomiting are characteristic. AIP occurs due to way, although it appears to be dominantly
the absence of porphobilinogen (PBG) inherited, it is in fact a recessive condition that
deaminase. This leads to elevated levels of 8- has been inherited in a pseudo-dominant
ALA and PBG. Attacks are intermittent, more manner. Pseudo-dominant inheritance is a
common in women and may be precipitated feature of diseases with a high carrier
by a number of drugs. There are no skin frequency or consanguineous families. Both
manifestations. Both variegate porphyria and mitochondrial and X-linked inheritance do not
hereditary coproporphyria are associated with exhibit male-male transmission and non-
photosensitive blistering skin lesions. paternity does not explain the inheritance
Erythropoietic protoporphyria and porphyria pattern.
cutanea tarda are cutaneous porphyrias. In
the majority of these patients subepidermal [ Q: 1598 ] PasTest 2009 - Basic
bullae and hypertrichosis develop. In Science
erythropoietic protoporphyria, the patient
develops a burning sensation affecting light- You review a 21-year-old man with albinism.
exposed parts of the skin. Urinary PBG and He is from Central/ South America and you
ALA levels are normal in both these believe that he has Hermansky-Pudlak
conditions. syndrome (HPS). You believe that this is due
to a defect in the transport of glycoproteins
from the Golgi body.
[ Q: 1597 ] PasTest 2009 - Basic
Science Which of the following best fits the position of
the Golgi body within the cell?
A 70-year-old Irish man is diagnosed with
haemochromatosis. Subsequently, his son is 1- In the nucleus
also diagnosed with the condition. Both are 2- Adjacent to the endoplasmic reticulum
homozygous for the common Northern 3- Moves within the cytoplasm
European mutation.
4- Sits across the cell membrane
What is the most likely explanation?
5- Just inside the cell membrane
1- Autosomal-dominant inheritance
2- Mitochondrial inheritance Answer & Comments
3- Non-paternity
Answer: 2- Adjacent to the endoplasmic
4- Pseudo-dominant inheritance reticulum
5- X-linked inheritance The Golgi body is one of the largest cellular
organelles and is found adjacent to the
Answer & Comments nucleus. It was first described by Camillo Golgi
in 1897 and is responsible for modification of
Answer: 4- Pseudo-dominant inheritance
proteins and lipids produced by other cellular
Haemochromatosis is an autosomal-recessive organelles. HPS results from a mutation in the
condition. However, the carrier frequency in AP3 gene responsible for trafficking specific
northern Europeans is high (1 in 10); in this glycoproteins from the Golgi body. It is
case the son must have inherited one mutant inherited in autosomal recessive fashion.
allele from his father and one mutant allele

Dr. Khalid Yusuf El-Zohry – Sohag Teaching Hospital (01118391123)


Ref MRCPass OE OE 2012 PasTest 2009 PassMedicine 2009 PasTest Exam ReviseMRCP 719
Elzohry MRCP Questions Bank (Part 1) – 2013 (For my personal use)

[ Q: 1599 ] PasTest 2009 - Basic oxygen and carbon dioxide can also diffuse
Science through the bilayer, passing easily through the
temporary small spaces between the tails of
Phase 0 of the cardiac action potential relates
the phospholipids.
to a:
1- Rapid efflux of calcium (Osmosis is the diffusion of water (not
particles) across a membrane, in which water
2- Rapid influx of calcium diffuses into a solution having a greater solute
3- Rapid influx of potassium concentration).
4- Rapid influx of sodium
[ Q: 1601 ] PasTest 2009 - Basic
5- None of the above
Science

Answer & Comments Which protein defect is responsible for


Marfan's syndrome?
Answer: 4- Rapid influx of sodium
1- Elastin
Phase 0: rapid sodium influx
2- Fibrillin
Phase 1: efflux of potassium 3- Myosin
Phase 2: slow influx of calcium 4- Actin
Phase 3: efflux of potassium 5- Collagen

Phase 4: sodium/calcium efflux, potassium


Answer & Comments
influx
Answer: 2- Fibrillin
[ Q: 1600 ] PasTest 2009 - Basic Marfan's syndrome is an archetypal defect of
Science connective tissue with a strong hereditary
By which process are particles moved along a basis causing characteristic skeletal,
concentration gradient across a selectively cardiovascular, and ocular disease. Patients
permeable membrane? with Marfan's syndrome are
disproportionately tall and thin with
1- Endocytosis
abnormally long extremities and, often, a
2- Diffusion cadaverous physique. Abraham Lincoln was
3- Exocytosis possibly affected. Marfan's syndrome is
caused by mutations in the human fibrillin
4- Osmosis
gene and is an autosomal dominant trait.
5- Phagocytosis
[ Q: 1602 ] PasTest 2009 - Basic
Answer & Comments Science
Answer: 2- Diffusion Where does RNA splicing occur?
Fat-soluble molecules, such as glycerol, can 1- Ribosome
diffuse through the membrane easily. They 2- Nucleus
dissolve in the phospholipid bilayer and pass
3- Golgi
through it in the direction of the
concentration gradient, from a high 4- Proteasome
concentration to a low concentration. Water,

Dr. Khalid Yusuf El-Zohry – Sohag Teaching Hospital (01118391123)


Ref MRCPass OE OE 2012 PasTest 2009 PassMedicine 2009 PasTest Exam ReviseMRCP 720
Elzohry MRCP Questions Bank (Part 1) – 2013 (For my personal use)

5- Peroxisome This is the standard method currently used for


cloning experiments. The most famous initial
Answer & Comments cloning success was that of Dolly the sheep,
cloned successfully be the Roslyn institute in
Answer: 2- Nucleus 1997. The technique is known as somatic
One of the most important stages in RNA nuclear transfer. Other potential methods
processing is RNA splicing. In many genes, the include parthenogenesis, a technique of
DNA sequence coding for proteins, or ‘exons', developing an embryo from unfertilised
may be interrupted by stretches of non-coding mammalian egg cells. So far this has resulted
DNA, called ‘introns'. In the cell nucleus, the in the development of embryos in mice, but as
DNA that includes all the exons and introns of yet not to full term.
the gene is first transcribed into a
complementary RNA copy called ‘nuclear [ Q: 1604 ] PasTest 2009 - Basic
RNA', or nRNA. In a second step, introns are Science
removed from nRNA by a process called RNA
The polymerase chain reaction (PCR) is
splicing. The edited sequence is called
becoming widely used in both research and
‘messenger RNA', or mRNA. The mRNA leaves
clinical medicine.
the nucleus and travels to the cytoplasm,
where it encounters cellular bodies called Which of the following statements is correct?
ribosomes. 1- The Taq polymerase used is of viral origin
2- After approximately 30 cycles, a million
[ Q: 1603 ] PasTest 2009 - Basic cDNA copies can theoretically be made
Science from a single target copy
A patient with type 1 diabetes presents to 3- Cycling of the reaction-mixture
clinic for review and says he has read about temperature enables sequential extension,
cloning as a cure for diabetes mellitus. He asks annealing and denaturation
you about the mechanism.
4- Specific sense and antisense primers that
Which of the following best describes the bind to part of the downstream target
mechanism currently used for cloning? sequence are used as the starting point of
1- An enucleated adult cell is fused with the the polymerase
nucleus from a donor cell 5- Reverse transcriptase-PCR (RT-PCR) is able
2- An enucleated oocyte is fused with the to identify the transcripts of a given gene
nucleus from a donor cell by detecting the messenger (m) RNA
coding for the gene
3- An enucleated adult cell is fused with the
nucleus from an oocyte
Answer & Comments
4- Genes are transfected into a donor oocyte
using a retrovirus Answer: 5- Reverse transcriptase-PCR (RT-PCR)
is able to identify the transcripts of a given
5- Oocytes are harvested from the donor and
gene by detecting the messenger (m) RNA
fused with adult cells from the donor
coding for the gene

Answer & Comments The Taq polymerase is derived from the


bacterium Thermus aquaticus. After
Answer: 2- An enucleated oocyte is fused with approximately 30 cycles, 109 cDNA copies can
the nucleus from a donor cell theoretically be made from a single target

Dr. Khalid Yusuf El-Zohry – Sohag Teaching Hospital (01118391123)


Ref MRCPass OE OE 2012 PasTest 2009 PassMedicine 2009 PasTest Exam ReviseMRCP 721
Elzohry MRCP Questions Bank (Part 1) – 2013 (For my personal use)

copy. Cycling of the reaction-mixture [ Q: 1606 ] PasTest 2009 - Basic


temperature enables sequential denaturation Science
(separation of the double-stranded template),
Why can trinucleotide repeat disorders
annealing (binding of the primers to their
become worse in successive generations?
targets) and extension (copying of the target
sequence by Taq polymerase in 5' to 3' 1- Modifying genes exacerbate the phenotype
direction). Specific sense and antisense 2- The phenotype is worse if the expansion is
primers that bind to part of the upstream inherited by a male
target sequence are used as the starting point
3- There is ascertainment bias with younger
of the polymerase.
family members diagnosed earlier

[ Q: 1605 ] PasTest 2009 - Basic 4- There is variable gene penetrance from


Science generation to generation
5- The repeat can expand from one generation
A 58-year-old man presents with progressively
to the next
worsening indigestion and weight loss. His GP
can feel an epigastric mass and arranges
upper gastrointestinal (GI) endoscopy. Biopsy Answer & Comments
of a suspicious lesion in the stomach reveals Answer: 5- The repeat can expand from one
numerous signet ring cells. generation to the next
Which of the following is the most likely
Anticipation is the phenomenon in which the
underlying diagnosis?
age of onset of a disorder is reduced or its
1- Gastric lymphoma severity increases in successive generations.
2- Oesophageal carcinoma Anticipation is due to an expansion in triplet
repeat size: the larger the repeat, the more
3- Gastric adenocarcinoma
unstable it is. Below a critical threshold
4- Gastric leiomyoma number of repeats, the expansion will be
5- Gastrinoma stable and non-pathogenic (eg below 35
repeats in Huntington's disease). Genetic
diseases caused by abnormal triplet repeat
Answer & Comments
expansion include Huntington's disease,
Answer: 3- Gastric adenocarcinoma myotonic dystrophy, fragile X, Friedreich's
ataxia, spinocerebellar ataxia and Kennedy's
Gastric adenocarcinoma cells may show a
disease. Although there is a general
number of different morphologies, each of
correlation with expansion size and severity of
which may be associated with a better or
phenotype, this cannot be used to accurately
worse prognosis. Classification is based on the
predict the severity on an individual basis.
most unfavourable microscopic element seen,
Anticipation is more likely to occur in
these are, in order of increasingly poor
myotonic dystrophy if the expansion is
prognosis, tubular, papillary, mucinous or
maternally inherited, while in anticipation is
signet-ring cells, undifferentiated lesions are
more likely to occur in Huntington's disease if
associated with the worst outcome. Tumours
the expansion is paternally inherited.
are also characterised according to
macroscopic appearance, ulcerative, polypoid, Modifying genes and variable penetrance may
scirrhous, superficial spreading or multi- account for some intrafamilial variation but
centric. would not explain the increasing severity with
successive generations. The phenotype can be

Dr. Khalid Yusuf El-Zohry – Sohag Teaching Hospital (01118391123)


Ref MRCPass OE OE 2012 PasTest 2009 PassMedicine 2009 PasTest Exam ReviseMRCP 722
Elzohry MRCP Questions Bank (Part 1) – 2013 (For my personal use)

worse in males for X-linked trinucleotide may be given for disseminated herpes
repeat disorders such as Fragile X but is not infection or in immunocompromised
true for autosomal trinucleotide repeat individuals.
disorders. Before the molecular mechanism
was understood, ascertainment bias was a [ Q: 1608 ] PasTest 2009 - Basic
seriously considered alternative explanation Science
and it can still be a contributing factor, eg a
person is more likely to be diagnosed earlier You are asked to help identify whether the
with Huntington's disease if there is a known child of a woman with a neuromuscular
family history. disorder is affected by the condition. A
specific DNA sequence associated with the
mutation which causes the disorder has been
[ Q: 1607 ] PasTest 2009 - Basic
identified.
Science
Which of the following methods is a technique
A 74-year-old man receives an acyclovir to identify a particular DNA sequence?
shingles treatment pack from his GP. He is a
retired chemist and is interested in asking you 1- Northern blot
about how acyclovir works. 2- Eastern blot
Which of the following best describes the step 3- Southern blot
required for acyclovir activation? 4- Western blot
1- Conversion to diphosphate form by viral 5- South-Western blot
thymidine kinase
2- Conversion to monophosphate form by Answer & Comments
viral thymidine kinase
Answer: 3- Southern blot
3- Metabolism by intracellular phosphatases
4- Reverse transcriptase activation Southern blotting was a technique developed
by Edwin Southern, a British biologist. It
5- Viral protease activity combines agarose gel electrophoresis for size
separation of DNA with use of a filter
Answer & Comments membrane for probe hybridisation. Northern
blotting studies sequences of RNA, whereas
Answer: 2- Conversion to monophosphate
Western blotting is used to look for specific
form by viral thymidine kinase
proteins. Southwestern blotting is used to
Acyclovir is a guanine analogue pro-drug used study DNA binding proteins.
primarily for the treatment of herpes zoster or
simplex infection. Conversion from a partial [ Q: 1609 ] PasTest 2009 - Basic
nucleoside structure to a monophosphate Science
form by viral thymidine kinase allows it to
incorporate into viral DNA resulting in chain You are asked to review a 17-year-old who
termination. Viral DNA polymerase is then suffers from leprachaunism. You understand
unable to function and replication of the that this occurs due to a mutation associated
Herpes virus is terminated. Peak plasma with the insulin receptor.
concentration is reached 1-2hrs after oral Where is the insulin receptor located?
administration of acyclovir and the drug is
1- Cell membrane
fairly rapidly eliminated, this necessitates
multiple daily dosing regimens. IV acyclovir 2- Nucleus/nuclear membrane

Dr. Khalid Yusuf El-Zohry – Sohag Teaching Hospital (01118391123)


Ref MRCPass OE OE 2012 PasTest 2009 PassMedicine 2009 PasTest Exam ReviseMRCP 723
Elzohry MRCP Questions Bank (Part 1) – 2013 (For my personal use)

3- Cytoplasm G2 is a gap phase when cells contain twice as


4- Endoplasmic reticulum much DNA as non-dividing cells. M is the
mitotic phase. G1 is a gap phase under the
5- Chromatin influence of the p53 gene. G0 is a resting stage
with non-dividing cells; S represents
Answer & Comments premitotic DNA synthesis. In normal tissues,
cells with significant damage to their DNA are
Answer: 1- Cell membrane
arrested at the G1 phase.
The insulin receptor is a trans-membrane
receptor. It belongs to the tyrosine kinase [ Q: 1611 ] PasTest 2009 - Basic
family of receptors which are involved in intra- Science
cellular signalling by phosphorylation of intra-
cellular proteins. These proteins include IRS-1, You are consenting a 19-year-old man, who is
which amongst other effects leads to up- thought to have a congenital neurological
regulation of the GLUT-5 glucose transporter degenerative disorder, for collection of
responsible for facilitating glucose entry into genetic material. He tells you he has heard
the cell. Leprachaunism is autosomal recessive about the human genome project.
and results from a mutation which leads to a Which of the following statements is true
non-functioning insulin receptor. concerning the project?
1- 10,000 genes have been identified
[ Q: 1610 ] PasTest 2009 - Basic
2- The project was completed in 2006
Science
3- 95% of the gene containing part of the
When planning the radiotherapy dosage for human DNA sequence was examined
solid tumours, it has to be remembered that
the division of cells is governed by the cell 4- 3.7 million human single nucleotide
cycle, the mechanism of which is disrupted by polymorphisms (SNPs) were mapped
radiation. 5- 10,000 full length human cDNAs were
Which of the following is true concerning the identified
cell cycle?
1- M represents the phase of premitotic DNA Answer & Comments
synthesis Answer: 4- 3.7 million human single
2- G1 is a gap phase under the influence of the nucleotide polymorphisms (SNPs) were
p53 gene mapped

3- S is the mitotic phase The human genome project set out to


4- G2 is a gap phase when cells contain half as sequence 30,000 human genes and this goal
much DNA as non-dividing cells was achieved in 2003. 99% of the gene
containing part of the human DNA sequence
5- In normal tissues, cells with significant was examined, with 15,000 full length human
damage to their DNA are arrested at the M cDNAs identified.The genetic code from a
phase number of organisms, including bacteria such
as E Coli, were also sequenced.
Answer & Comments
Answer: 2- G1 is a gap phase under the
influence of the p53 gene

Dr. Khalid Yusuf El-Zohry – Sohag Teaching Hospital (01118391123)


Ref MRCPass OE OE 2012 PasTest 2009 PassMedicine 2009 PasTest Exam ReviseMRCP 724
Elzohry MRCP Questions Bank (Part 1) – 2013 (For my personal use)

[ Q: 1612 ] PasTest 2009 - Basic 4- In the cytoplasm


Science 5- In the cell membrane
You are working on your PhD project into the
underlying basis for an inherited neurological Answer & Comments
condition. Part of your project involves
Answer: 4- In the cytoplasm
identifying a protein expressed in an abnormal
cell line. G-proteins are coupled to transmembrane
Which of the following is the appropriate receptors (G protein coupled receptors,
method for identifying such a protein? GPCRs). These are transmembrane proteins
that wind 7 times back and forth through the
1- Northern blot
plasma membrane. Their ligand-binding site is
2- Southern blot exposed outside the surface of the cell. Their
3- Eastern blot effector site extends into the cytosol. Many
ligands bind to the GPCR, including TSH, ACTH,
4- Western blot
serotonin and GABA. This ligand binding to the
5- RT-PCR external portion initiates production of a "2nd
messenger" within the cell. The most common
Answer & Comments of these 2nd messengers are cyclic AMP,
(cAMP) which is produced by adenylyl cyclase
Answer: 4- Western blot from ATP, and inositol 1,4,5-triphosphate
Western blotting uses gel electrophoresis to (IP3). The second messenger, in turn, initiates
separate proteins by polypeptide length. The a series of intracellular events such as
proteins are then transferred to a membrane phosphorylation and activation of enzymes
when they can be probed with antibodies to a release of Ca2+ into the cytosol from stores
specific protein. Southern blotting is used to within the endoplasmic reticulum. The best-
detect specific DNA abnormalities, and described human disease caused by G-protein
Northern blotting is the name of the mutation is pseudohypoparathyroidism. It is
technique for RNA detection. Far-Eastern caused by a mutation in the GNAS1 gene,
blotting is the name of the technique used for encoding the alpha subunit of the stimulatory
detecting lipid abnormalities. G-protein responsible for binding of
parathyroid hormone. This leads to a failure of
cyclic AMP production and a hormone
[ Q: 1613 ] PasTest 2009 - Basic
resistance syndrome. Several other human
Science
diseases may result from alpha subunit
You review a young man with hypocalcaemia mutation: these include resistance to
and notice on review of his hands that he has antidiuretic hormone (ADH), gonadotrophins,
a shortened fourth and fifth digit on each glucagon, adrenocorticotropic hormone
hand. You wonder if he may have (ACTH) and growth hormone. Standard
pseudohypoparathyroidism, caused by a G- treatment for pseudohypoparathyroidism
protein abnormality. involves supplementation with calcium and
Which of the following best describes the vitamin D.
location of G-proteins?
[ Q: 1614 ] PasTest 2009 - Basic
1- In the nucleus
Science
2- In the nuclear membrane
A woman has von Willebrand's disease and so
3- In the nucleolus does her partner. They wish to have children

Dr. Khalid Yusuf El-Zohry – Sohag Teaching Hospital (01118391123)


Ref MRCPass OE OE 2012 PasTest 2009 PassMedicine 2009 PasTest Exam ReviseMRCP 725
Elzohry MRCP Questions Bank (Part 1) – 2013 (For my personal use)

and seek genetic counselling. disorders. Despite numerous studies problems


How would the risk of disease transmission to associated with mitochondrial diseases have
her offspring be best described? not been completely solved yet. LHON
(Leber's Hereditary Optic Neuropathy) was the
1- All the children will have the disease first described disease associated with
2- All the children will be carriers hereditary point mutations in mtDNA. The
3- Half the children will be carriers disease is characterised by subacute loss of
binocular vision with a lesion of the central
4- Three-quarters of the children will have the field of vision, improper colour vision and
disease atrophy of optic nerve. Other examples
5- All the sons will be affected include Kearns-Sayre syndrome which is
caused both by deletions and duplications of
Answer & Comments mtDNA. The onset of disease symptoms is
observed before 20 year of age. Short stature,
Answer: 4- Three-quarters of the children will pigmentary retinal degeneration,
have the disease ophthalmoplegia, ptosis, ataxias, disturbances
Von Willebrand's is autosomal dominant. If in conduction in heart muscle, diabetes, and
hearing loss occur in the syndrome. The other
both parents have the disease, then three-
quarters of their offspring will have the example is CPEO (Chronic Progressive External
Ophthalmoplegia) which may occasionally
disease (on the assumption that both parents
occur as a result of de novo mutation, may be
are heterozygote - in the exam, this is a fair
assumption). There is no carrier state and, as maternally inherited (mt tRNA mutations) or
it is an autosomal disease, there is no sex can be of autosomally dominant inheritance.
Symptoms of CPEO are ptosis, myopathy,
difference.
depression, cataract and ketoacidosis. Alport's
syndrome, Noonan's syndrome, Fabry's
[ Q: 1615 ] PasTest 2009 - Basic disease and Marfan's syndrome are diseases
Science caused by mutations in nuclear and not
Which one of the following inherited diseases mitochondrial DNA.
is due to mutation in mitochondrial DNA?
1- Alport's syndrome [ Q: 1616 ] PasTest 2009 - Basic
Science
2- Leber's optic neuropathy
You are asked to review a young man with
3- Noonan's syndrome
short stature, short fifth metacarpals,
4- Fabry's disease subcutaneous calcification, intellectual
5- Marfan's syndrome impairment and hypocalcaemia.
Given the likely clinical diagnosis, mutation
Answer & Comments resulting in the dysfunction of what structure
is most likely?
Answer: 2- Leber's optic neuropathy
1- An ion-channel linked to the PTH receptor
Prevalence of mitochondrial diseases equals
1:10000 of live-born infants. Mutations of 2- A G-protein linked to the PTH receptor
mitochondrial DNA (mtDNA) are their most 3- A tyrosine kinase receptor
frequent cause. The diseases often result in 4- A tyrosine phosphatase receptor
encephalomyelopathy, cardiomyopathy,
vision disorders, dysacusis and metabolic 5- A serine kinase receptor

Dr. Khalid Yusuf El-Zohry – Sohag Teaching Hospital (01118391123)


Ref MRCPass OE OE 2012 PasTest 2009 PassMedicine 2009 PasTest Exam ReviseMRCP 726
Elzohry MRCP Questions Bank (Part 1) – 2013 (For my personal use)

Answer & Comments The term 'genomic imprinting' refers to the


dependence of phenotype on whether the
Answer: 2- A G-protein linked to the PTH
gene deletion is inherited from the mother or
receptor
father. Neurofibromatosis is an autosomal-
There are, broadly speaking, three types of dominant disorder with 95% penetrance. The
membrane-bound receptors, which can be gene is located on chromosome 17. Prader-
subclassified according to the mechanism they Willi syndrome is a result of a paternal
use to activate signalling molecules. They are deletion of a gene on chromosome 15;
ion channel-linked receptors, G protein-linked Angelman syndrome results if the same gene
receptors and enzyme-linked receptors. is deleted from the maternal side. In both
syndromes, 3-5% of cases are a result of
When the G protein-linked receptor is uniparental disomy (both chromosomes from
activated by a ligand it binds a trimeric the same parent). Huntington's chorea is a
complex (alpha, beta and gamma) which is result of a triplet codon repeat. There is a 35-
anchored to the inner surface of the plasma
90 repeat segment of CAG nucleotides on
membrane. This trimeric compound is called a chromosome 4p 16.3. It is rare in childhood,
GTP-binding protein or G-protein. The G- presenting later in adult life with dementia,
protein receptor then binds GTP (guanosine chorea and rigidity. Hurler's syndrome (type I)
triphosphate), and interacts with enzymes on is an autosomal-recessive disorder occurring
the inner membrane surface to activate one of due to a defect in chromosome 4p. Marfan's
a number of secondary messengers. syndrome is an autosomal-dominant disorder
Secondary messengers activated may include
affecting chromosome 15q.
cyclic AMP, calcium ions or inositol 1,4,5-
triphosphate/diacylglycerol.
[ Q: 1618 ] PasTest 2009 - Basic
Pseudohypoparathyroidism is caused by Science
mutation in a G-protein linked to the PTH
You are reviewing an 18-year-old woman with
receptor. Variable degrees of resistance may
cystic fibrosis.
also be seen to other G protein-linked
hormones including TSH, LH and FSH. How would the genetic inheritance of this
condition best be described?
[ Q: 1617 ] PasTest 2009 - Basic 1- It has an autosomal-dominant mode of
Science inheritance
Genomic imprinting is seen in which of the 2- It has an X-linked dominant mode of
following conditions? inheritance
1- Neurofibromatosis 3- It has an autosomal-recessive mode of
inheritance
2- Prader-Willi syndrome
4- It has an X-linked recessive mode of
3- Huntington's chorea
inheritance
4- Hurler's syndrome
5- It is a chromosomal disorder
5- Marfan's syndrome
Answer & Comments
Answer & Comments
Answer: 3- It has an autosomal-recessive
Answer: 2- Prader-Willi syndrome mode of inheritance

Dr. Khalid Yusuf El-Zohry – Sohag Teaching Hospital (01118391123)


Ref MRCPass OE OE 2012 PasTest 2009 PassMedicine 2009 PasTest Exam ReviseMRCP 727
Elzohry MRCP Questions Bank (Part 1) – 2013 (For my personal use)

Cystic fibrosis is an autosomal-recessive Glucose-6-phosphate dehydrogenase


disorder, the carrier frequency in Caucasians deficiency causes haemolysis when affected
being 1 in 22. There is a gene mutation on the individuals are exposed to certain drugs.
long arm of chromosome 7, with the
commonest abnormality being a deletion [ Q: 1620 ] PasTest 2009 - Basic
mutation at position 508 in the amino acid Science
sequence. This causes a defect in the
transmembrane conductance receptor (CFTR), A 20-year-old woman presents with
a chloride channel. The result is increased hypothyroidism. On further questioning it
tenacity and viscosity of secretions. transpires she has primary amenorrhoea. She
is also of relatively short stature compared to
Examples of autosomal-dominant disorders her sisters.
include neurofibromatosis and Marfan's
What is the most likely diagnosis?
syndrome. Examples of X-linked disorders
include Charcot-Marie-Tooth disease and 1- Turner's syndrome
Alport's syndrome. Examples of chromosomal 2- Down's syndrome
disorders include Down's syndrome (trisomy
3- Noonan's syndrome
21) and Patau's syndrome (trisomy 13).
4- XXX syndrome
[ Q: 1619 ] PasTest 2009 - Basic 5- Achondroplasia
Science
An 18-year-old army recruit undergoes Answer & Comments
preliminary health screening and is found to Answer: 1- Turner's syndrome
have a haemoglobin of 11.5 mg/dl (13-17).
Spherocytes are present in the peripheral Although Turner's syndrome (XO) and Down's
blood film. Investigation of his siblings reveals and Noonan's syndromes can be associated
that one of his two brothers also has with short stature and hypothyroidism,
spherocytic anaemia. Down's syndrome and Noonan's syndrome are
not associated with menstrual irregularities.
A defect in which of the following is most
Females with an extra X chromosome (XXX
frequently responsible for this condition?
syndrome) are usually tall, whereas individuals
1- Actin with achondroplasia are usually very short;
2- Ankyrin neither condition has an increased incidence
of hypothyroidism or menstrual irregularities.
3- Glucose-6-phosphate dehydrogenase
The other typical features of Turner's
4- Protein 4.1 syndrome are cardiac defects (eg coarctation
5- Spectrin of the aorta), congenital lymphoedema, neck
webbing, widely spaced nipples and cubitus
valgus. Noonan's syndrome is an autosomal-
Answer & Comments
dominant condition (so it affects both sexes),
Answer: 5- Spectrin and other typical features include pulmonary
stenosis, neck webbing and low-set
The diagnosis is hereditary spherocytosis, the
posteriorly rotated ears.
most frequent cause of which is a mutation in
the spectrin gene. Spectrin is a component of
the red cell membrane, as are actin, ankyrin
and protein 4.1, mutations in all of which are
less frequently responsible for the condition.

Dr. Khalid Yusuf El-Zohry – Sohag Teaching Hospital (01118391123)


Ref MRCPass OE OE 2012 PasTest 2009 PassMedicine 2009 PasTest Exam ReviseMRCP 728
Elzohry MRCP Questions Bank (Part 1) – 2013 (For my personal use)

[ Q: 1621 ] PasTest 2009 - Basic 4- MELAS (myopathy, encephalopathy, lactic


Science acidosis, stroke)
An 18-year-old woman presents with mild 5- Haemophilia type A
polyuria and polydipsia; her fasting blood
glucose concentration is 14 mmol/l. Her father Answer & Comments
and two of her three sisters have diabetes.
Answer: 3- Myotonic dystrophy
She responds well to treatment with insulin,
and is stabilised on a total dose of 14 units per Absence of male-to-male transmission is a
24 h. hallmark of X-linked genetic conditions. In X-
A mutation in which of the following is likely to linked conditions none of the male offspring
be responsible for her condition? of an affected male will be affected or be
carriers, whereas all his daughters will be
1- Glucokinase
carriers and themselves be at risk of having
2- Glutamic acid decarboxylase affected sons. In mitochondrial diseases
3- Hepatic nuclear factor-1a neither male nor female offspring of an
affected male will inherit as the mutated
4- Insulin
mitochondria are passed down in the female
5- Insulin promoter factor-1 egg.

Duchenne's muscular dystrophy and


Answer & Comments haemophilia type A are both examples of X-
Answer: 3- Hepatic nuclear factor-1a linked conditions, while MELAS is due to a
mitochondrial mutation. Cystic fibrosis (CF) is
Given the strong family history and low insulin an autosomal-recessive condition, so, in
requirement, the presumptive diagnosis is theory, a male with CF could have an affected
‘maturity onset diabetes of the young' son if his partner was a carrier. (The carrier
(MODY). Five variants of this condition have frequency in northern Europeans is
been described, involving mutations in various approximately 1 in 25.) However, in practice,
hepatic nuclear factors, glucokinase and males with CF are invariably infertile as
insulin promoter factor-1, but more than half congenital bilateral absence of the vas
of cases are associated with mutation in deferens (CBAVD) is a cardinal feature of CF.
hepatic nuclear factor-1a. Patients with type-1 Myotonic dystrophy is an autosomal-
diabetes often have islet cell antibodies in dominant condition and therefore there is a
their plasma at the time of diagnosis: their 50% risk to the male and female offspring of
antigens include insulin and glutamic acid an affected male, therefore this is the correct
decarboxylase but islet cell antibodies are not answer.
present in MODY.

[ Q: 1623 ] PasTest 2009 - Basic


[ Q: 1622 ] PasTest 2009 - Basic Science
Science
A couple attends genetic screening for an
In which of the following hereditary conditions inherited condition that has been observed in
will affected males have a significant risk of the husband's family. More severe cases of so-
fathering affected sons? called 'anticipation' have been observed.
1- Duchenne's muscular dystrophy What is the most likely condition?
2- Cystic fibrosis 1- Achondroplasia
3- Myotonic dystrophy

Dr. Khalid Yusuf El-Zohry – Sohag Teaching Hospital (01118391123)


Ref MRCPass OE OE 2012 PasTest 2009 PassMedicine 2009 PasTest Exam ReviseMRCP 729
Elzohry MRCP Questions Bank (Part 1) – 2013 (For my personal use)

2- Haemophilia B [ Q: 1624 ] PasTest 2009 - Basic


3- Marfan's syndrome Science

4- Myotonic dystrophy A patient with colon cancer presents with cafè


au lait spots around the nose and mouth,
5- Polycystic kidney disease
hand and feet and within the oral cavity.
What is the most likely diagnosis?
Answer & Comments
1- Fanconi's anaemia
Answer: 4- Myotonic dystrophy
2- Neurofibromatosis type 1
Anticipation is the tendency for a genetic
3- Neurofibromatosis type 2
disorder to become more severe and present
at an earlier age in successive generations. It is 4- Peutz-Jeghers' syndrome
usually the result of an unstable triplet repeat 5- Tuberous sclerosis
expansion in a gene and can show ‘parent of
origin' effect. Examples of genetic disorders
Answer & Comments
that show anticipation are fragile-X syndrome,
Huntington's disease, myotonic dystrophy Answer: 4- Peutz-Jeghers' syndrome
(MD) and some of the spinocerebellar ataxias.
The lentigines in patients with Peutz-Jeghers'
All these conditions are caused by the
syndrome are located primarily around the
expansion of a triplet repeat expansion within
nose and mouth, on the hands and feet, and
the gene. This repeat can undergo expansion
within the oral cavity. While the pigmented
during meiosis such that the children of an
macules on the face may fade with age, the
affected individual tend to inherit a larger
oral lesions persist. However, similar intra-oral
expansion that results in more severe disease.
lesions are also seen in Addison's disease and
Myotonic dystrophy is an autosomal dominant as a normal finding in darkly pigmented
disorder caused by the expansion of a individuals. Patients with this autosomal
cytosine-thymine-guanine (CTG) repeat that dominant syndrome (due to mutations in a
lies at the 3' end of the dystrophia myotonica- novel serine threonine kinase gene) have
protein kinase (DMPK) gene on 19q13.3. multiple benign polyps of the gastrointestinal
Normal individuals have fewer than 35 CTG tract, testicular tumours, and an increased risk
repeats. Affected individuals with MD have of developing gastrointestinal (primarily
more than 50 CTG repeats. A CTG expansion colon), breast, and gynaecologic cancers.
of 50-80 repeats is associated with early-onset
cataracts. Affected individuals with classical [ Q: 1625 ] PasTest 2009 - Basic
MD have larger CTG expansions. The most Science
severe form of MD is congenital MD, which
presents at birth with intrauterine growth A patient has been diagnosed with Marfan's
retardation, severe hypotonia, muscle syndrome.
weakness and feeding difficulties. Children Which gene mutation is responsible for this
with congenital MD have developmental delay condition?
and learning difficulties and develop myotonia
1- Fibrillin
in their teens. Affected individuals with
congenital MD have very large CTG expansions 2- Mitochondria
with 500-2000 repeats. Expansions of this size 3- Cyclooxygenase
are almost always maternally transmitted.
4- Hexosaminidase
5- Galactosidase

Dr. Khalid Yusuf El-Zohry – Sohag Teaching Hospital (01118391123)


Ref MRCPass OE OE 2012 PasTest 2009 PassMedicine 2009 PasTest Exam ReviseMRCP 730
Elzohry MRCP Questions Bank (Part 1) – 2013 (For my personal use)

Answer & Comments [ Q: 1627 ] PasTest 2009 - Basic


Science
Answer: 1- Fibrillin
You see a healthy 20-year-old man in your
Marfan's syndrome is characterised by a triad
clinic. There is no significant past medical
of features:
history and he plays rugby for Saracens rugby
long, thin extremities frequently associated club. He tells you that his brother died at the
with other skeletal changes, such as loose age of seven years with cystic fibrosis.
joints and arachnodactyly What are his chances of being a carrier?
reduced vision as the result of dislocations of 1- 1 in 4 (25%)
the lenses (ectopia lentis) 2- 1 in 3 (33%)
aortic aneurysms that typically begin at the 3- 1 in 2 (50%)
base of the aorta.
4- 2 in 3 (67%)
Most patients can be identified by detection 5- 3 in 4 (75%)
of mutations in the fibrillin gene.

Answer & Comments


[ Q: 1626 ] PasTest 2009 - Basic
Science Answer: 4- 2 in 3 (67%)

A patient has been investigated for loose Cystic fibrosis (CF) is an autosomal recessive
joints. He also complains of reduced vision. On disorder that is caused by mutations in the
examination he has very thin extremities and cystic fibrosis transmembrane conductance
arachnodactyly. regulator (CFTR) gene on 7q31.2. Affected
individuals have mutations in both alleles of
What is the most likely diagnosis?
this gene. The siblings of an affected individual
1- Marfan's syndrome have:
2- Tay-Sachs disease
a 1 in 4 (25%) chance of being affected with
3- Fabry disease this condition
4- Gaucher disease a 1 in 2 (50%) chance of being CF carriers
5- Niemann-Pick disease
a 1 in 4 (25%) chance of being neither affected
nor a carrier.
Answer & Comments
The proband in this question is a healthy 20-
Answer: 1- Marfan's syndrome year-old man and he is therefore unaffected
Marfan's syndrome is characterised by a triad with CF. When affected siblings are removed
of features: from the above equation then the unaffected
siblings have a 2 in 3 (67%) chance of being CF
long, thin extremities frequently associated carriers and 1 in 3 (33%) chance of not being
with other skeletal changes, such as loose carriers.
joints and arachnodactyly

reduced vision as the result of dislocations of [ Q: 1628 ] PasTest 2009 - Basic


the lenses (ectopia lentis) Science

aortic aneurysms that typically begin at the A 40-year-old man develops jerky movements
base of the aorta. affecting various parts of his body. His father

Dr. Khalid Yusuf El-Zohry – Sohag Teaching Hospital (01118391123)


Ref MRCPass OE OE 2012 PasTest 2009 PassMedicine 2009 PasTest Exam ReviseMRCP 731
Elzohry MRCP Questions Bank (Part 1) – 2013 (For my personal use)

died at the age of 55 and had been diagnosed 2- Polyarteritis nodosa


as having Huntington's disease. 3- Sarcoidosis
Which of the following genetic abnormalities 4- Diabetic ketoacidosis
is responsible for this condition?
5- Acute intermittent porphyria
1- Frame-shift mutation
2- Point mutation Answer & Comments
3- Splicing mutation
Answer: 5- Acute intermittent porphyria
4- Termination mutation
Acute intermittent porphyria (AIP) is the most
5- Triplet-repeat mutation common type of acute porphyria. It is five
times more common in females and presents
Answer & Comments between 15 and 30 years of age. The oral
contraceptive pill can precipitate an acute
Answer: 5- Triplet-repeat mutation
attack in AIP. Guillain–Barré syndrome
Huntington's disease, an autosomal-dominant commonly presents with headache, vomiting,
disease, is caused by a triplet-repeat pyrexia, back and limb pain followed by
mutation, with expansion (typically 40-55 paralysis. It usually follows 1 - 4 weeks after a
repeats) of a CAG triplet. This gives rise to a respiratory tract infection or diarrhoea. Renal
long sequence of glutamine residues in a functions are affected in over 75% of patients
protein of unknown function called with polyarteritis nodosa. Fever, malaise,
‘huntingtin'. Huntington's disease may weight loss and arthralgia are common
demonstrate ‘genetic anticipation' with an symptoms in this condition. Abdominal pain
increasingly early onset in succeeding may occur due to infarction or malabsorption
generations: this is due to an increase in the caused by chronic ischaemia. In sarcoidosis,
number of triplet repeats. Huntington's hypercalcaemia and hypercalciuria are
disease is inevitably fatal. common while abdominal pain is uncommon.
A 2- to 3-day history of gradual deterioration
The other types of mutation listed can all
with dehydration, acidosis and coma
cause mutations in proteins as a result of the
characterises diabetic ketoacidosis.
insertion of the wrong amino acid (point and
frame-shift mutations); incorporation of
amino acid sequences not normally present [ Q: 1630 ] PasTest 2009 - Basic
(splicing mutations) or premature or late Science
termination of transcription (termination A 7-year-old boy who came from India
mutations). presented with fever, maculopapular rash and
pharyngitis with greyish membranes
[ Q: 1629 ] PasTest 2009 - Basic surrounding the tonsils. There is regional
Science lymphadenopathy.

A 20-year-old woman, who was prescribed the What is the most likely diagnosis?
oral contraceptive pill a week earlier, develops 1- Infectious mononucleosis
central abdominal pain, vomiting and
2- Diphtheria
weakness in both lower limbs. Blood tests
show an elevated white cell count. 3- Rubella

What is the most probable diagnosis? 4- Measles

1- Guillain-Barrè syndrome 5- Streptococcus pyogenes

Dr. Khalid Yusuf El-Zohry – Sohag Teaching Hospital (01118391123)


Ref MRCPass OE OE 2012 PasTest 2009 PassMedicine 2009 PasTest Exam ReviseMRCP 732
Elzohry MRCP Questions Bank (Part 1) – 2013 (For my personal use)

Answer & Comments predisposition to colorectal cancer in the


absence of florid polyposis. There are two
Answer: 2- Diphtheria
main forms, Lynch I (no family history of other
In countries with active programs of cancers) and Lynch II (with other cancers in
immunisation, diphtheria has virtually the family, eg endometrial, ovarian, urinary
disappeared, but cases still do occur in the tract). It is diagnosed according to the
developing world. The most common ‘Amsterdam' criteria: families must have at
presentation is with pharyngitis, but any part least three relatives over two generations with
of the respiratory tract may be involved. Areas colorectal cancer, one a first-degree relative
of grey or white fibrinous exudates coalesce to of the other two, with at least one case being
produce a tough grey-yellow membrane that diagnosed below the age of 50 years. These
bleeds when removed. Cutaneous diphtheria criteria were originally devised for research
may occur; eventually the maculopapular rash purposes and less-stringent criteria are often
begins to ulcerate, with characteristic grey- used in clinical practice. At least six genes are
white exudates at the base of the ulcers. known to cause HNPCC and they are involved
Treatment is with diphtheria antitoxin and in repairing DNA-sequence mismatches.
benzylpenicillin iv for 1 week. Administration
Familial adenomatous polyposis is
of antitoxin has been associated with both the
characterised by profuse colonic
development of anaphylaxis and serum
adenomatous polyposis. Affected individuals
sickness after 2-3 weeks.
develop hundreds of polyps by their second
decade and have a greatly increased risk of
[ Q: 1631 ] PasTest 2009 - Basic colorectal cancer. Peutz-Jeghers' syndrome is
Science characterised by melanin flecks on the lips and
A 35-year-old man is referred to you with mucocutaneous borders with a
altered bowel habit and weight loss. One of hamartomatous polyposis, and is associated
his sisters had a hysterectomy for uterine with an increased risk of colorectal cancer. As
cancer aged 42, another sister had colorectal only a few polyps were found on colonoscopy
cancer aged 44 and his father died from both of these conditions would be less likely
colorectal cancer in his fifties. Colonoscopy than HNPCC. Neither neurofibromatosis type
shows a few polyps and a caecal mass. 1 or 2 nor von Hippel-Lindau disease are
associated with an increased risk of colorectal
Which genetic condition might be running in cancer.
the family?
1- Familial adenomatous polyposis [ Q: 1632 ] PasTest 2009 - Basic
2- Peutz-Jeghers' disease Science
3- Neurofibromatosis type 1 Which of these are well-recognised late
4- von Hippel-Lindau disease complications of trisomy 21 (Down's
syndrome)?
5- Hereditary non-polyposis colorectal cancer
1- Ischaemic heart disease
Answer & Comments 2- Addison's disease

Answer: 5- Hereditary non-polyposis 3- Alzheimer's dementia


colorectal cancer 4- Glioblastoma

Hereditary non-polyposis colorectal cancer 5- Cataracts


(HNPCC) is an autosomal-dominantly inherited

Dr. Khalid Yusuf El-Zohry – Sohag Teaching Hospital (01118391123)


Ref MRCPass OE OE 2012 PasTest 2009 PassMedicine 2009 PasTest Exam ReviseMRCP 733
Elzohry MRCP Questions Bank (Part 1) – 2013 (For my personal use)

Answer & Comments hypogonadism or infertility. Fragile X


syndrome is associated with macro-orchidism
Answer: 3- Alzheimer's dementia
and moderate to severe learning disability.
Individuals with Down's syndrome have a Kallmann's syndrome is associated with
greatly increased risk of developing anosmia and hypogonadotrophic
Alzheimer's dementia. Virtually 100% will hypogonadism (as opposed to
show neuropathological changes by the age of hypergonadotrophic hypogonadism in
35-40 years, and the average age of onset of Klinefelter's syndrome).
clinical features is 51-54 years.
Hypothyroidism is common in Down's [ Q: 1634 ] PasTest 2009 - Basic
syndrome and should always be considered as Science
a differential diagnosis. There is also an
increased risk of leukaemia. None of the other A 40-year-old man comes to see you because
conditions listed above are well-recognised he is worried about his family history of
late complications. Huntington's disease. His 45-year-old sister is
known to be affected and has been given a
molecular diagnosis, but neither parent is
[ Q: 1633 ] PasTest 2009 - Basic
affected and both have had a normal gene
Science
test.
A 20-year-old man is referred for investigation What is the most likely reason for this
of hypogonadism and infertility. He went inheritance pattern?
through normal puberty and there is no
1- Anticipation
significant family history. On examination he is
tall with gynaecomastia and small testes. He 2- A new mutation in the sister
has a normal sense of smell. 3- Non-paternity
Which condition would be high on your list of 4- Females are more often affected
differential diagnoses?
5- Non-penetrance in the parents
1- Klinefelter's syndrome
2- XYY syndrome Answer & Comments
3- Marfan's syndrome Answer: 3- Non-paternity
4- Kallmann's syndrome
Huntington's disease is an autosomal-
5- Fragile X syndrome dominant neurodegenerative condition
caused by a trinucleotide repeat expansion,
Answer & Comments and both sexes are equally affected. Less than
1% of cases are due to new mutations and
Answer: 1- Klinefelter's syndrome
they are virtually all due to the same
This is due to the addition of an extra X abnormal expansion. It is highly penetrant and
chromosome (XXY). It is the most common sex onset is typically in middle life. The expansion
chromosome abnormality (1 in 600 male is more unstable if transmitted through the
births) and the most common cause of male paternal line, leading to a younger age of
hypogonadism and infertility. Affected males onset in the subsequent generation
tend to be tall and can have mild learning (anticipation). However, as the father does
difficulties, although many have normal not carry the expansion this does not explain
intellect. XYY, Fragile X and Marfan's the situation in this family. Non-paternity
syndromes are not associated with would therefore have to be seriously

Dr. Khalid Yusuf El-Zohry – Sohag Teaching Hospital (01118391123)


Ref MRCPass OE OE 2012 PasTest 2009 PassMedicine 2009 PasTest Exam ReviseMRCP 734
Elzohry MRCP Questions Bank (Part 1) – 2013 (For my personal use)

considered. Of course, the situation would 6 of the b-globin chain is the genetic
have to be handled very delicately. Non- abnormality encountered in which one of the
paternity rates are notoriously difficult to following types of congenital haemolytic
measure. Many studies have found very anaemia?
variable results, from 1-30% or even higher in 1- Sickle cell anaemia
some age groups.
2- β-Thalassaemia

[ Q: 1635 ] PasTest 2009 - Basic 3- Hereditary spherocytosis


Science 4- Glucose-6-phosphate dehydrogenase
A colleague asks for your help in designing a (G6PD) deficiency
new diagnostic molecular assay where small 5- Methaemoglobinaemia
amounts of DNA need to be accurately
amplified and analysed. Answer & Comments
Which would be your technique of choice? Answer: 1- Sickle cell anaemia
1- Southern blotting
Haemolytic anaemia is either congenital or
2- Immunoprecipitation acquired: Congenital haemolytic anaemia
3- Polymerase chain reaction (PCR)
Membrane defect: hereditary spherocytosis
4- Western blotting
Enzyme defects: (G6PD) deficiency
5- Enzyme-linked immunoabsorbant assay
(ELISA) Haemoglobin defects: thalassaemia, sickle cell
anaemia
Answer & Comments Sickle cell anaemia is due to the inheritance of
Answer: 3- Polymerase chain reaction (PCR) a gene for a structurally abnormal b -globin
chain subunit of adult haemoglobin, HbS. The
PCR is a rapid and versatile method for abnormal haemoglobins are caused by amino
amplifying defined target DNA sequences. It is acid substitutions in their polypeptide chains.
one of the cornerstone techniques in The amino acid valine replaces glutamic acid
molecular genetics and facilitates the rapid at position 6 of the b -globin chain. b -
cloning and analysis of DNA. It uses a heat- Thalassaemia is due to failure to synthesise
stable polymerase enzyme originally derived beta chains. Hereditary spherocytosis and
from a species of thermophilic bacteria. A G6PD are not disorders of b -globin chain.
typical PCR reaction will takes 1-2 hours. It is a Methaemoglobin results from NADH-
fast, sensitive and robust technique and has methaemoglobin reductase deficiency which
been adapted for a multitude of different causes the iron atom to be oxidized to the
applications. Southern blotting is used in DNA ferric (Fe3+) form, rendering the molecule
analysis but does not amplify DNA. Western incapable of binding an oxygen molecule. It
blotting, immunoprecipitation and ELISA are causes persistent cyanosis without hypoxia. It
all techniques used in protein analysis. does not cause haemolysis.

[ Q: 1636 ] PasTest 2009 - Basic [ Q: 1637 ] PasTest 2009 - Basic


Science Science
The substitution of the amino acid valine A man in his mid-60s is referred to you
instead of the normal glutamic acid at position because he is worried about his family history

Dr. Khalid Yusuf El-Zohry – Sohag Teaching Hospital (01118391123)


Ref MRCPass OE OE 2012 PasTest 2009 PassMedicine 2009 PasTest Exam ReviseMRCP 735
Elzohry MRCP Questions Bank (Part 1) – 2013 (For my personal use)

of ischaemic heart disease. His father was a 2- An alternative diagnosis should be


non-smoker and died of a myocardial considered if chorea occurs with no
infarction aged 39 years. His maternal evidence of Kayser-Fleischer rings
grandfather and a paternal uncle both had 3- Chronic liver disease and autoimmune
myocardial infarcts in their 40s. haemolytic anaemia are recognised
What is his history most suggestive of? features
1- Hyperhomocysteinaemia 4- Raised serum copper levels evident at birth
2- Homocystinuria 5- Siblings with biochemical evidence of the
3- Familial hypercholesterolaemia disease are treated only when they
become symptomatic
4- Factor V Leiden deficiency
5- Haemochromatosis Answer & Comments
Answer: 2- An alternative diagnosis should be
Answer & Comments
considered if chorea occurs with no evidence
Answer: 3- Familial hypercholesterolaemia of Kayser-Fleischer rings

Familial hypercholesterolaemia affects around Wilson's disease is an autosomal recessive


1 in 500 of the population and is associated disease with a prevalence of 1 in 30,000. Its
with a significantly increased risk of clinical and pathological manifestation results
cardiovascular disease. Although there is from excessive accumulation of copper in
epidemiological evidence that elevated many tissues including the brain, liver, cornea
plasma homocysteine levels are associated and kidneys. Impaired biliary copper excretion
with cardiovascular disease, it is unlikely to be rather than enhanced absorption is the cause
responsible for the strong family history in the of copper accumulation. Neurological
question. Homocysteinuria is an autosomal- manifestations typically appear between the
recessive inborn error of methionine ages of 12 and 30 years and are almost
metabolism and is associated with a greatly invariably accompanied by the presence of
increased incidence of atheroma. Affected Kayser-Fleischer rings. The amount of copper
individuals often have a marfanoid habitus in the body at birth is normal. Evidence of
and downward lens dislocation is also a well- haemolysis in patients with chronic liver
recognised feature. However, the autosomal- disease represents a clue to the diagnosis of
recessive inheritance pattern would not fit Wilson's disease. The haemolysis is not
with this family history. Neither Factor V immune mediated. Asymptomatic siblings that
Leiden deficiency nor haemochromatosis are demonstrate biochemical evidence of the
associated with an increased risk of disease should receive treatment.
cardiovascular disease, although
cardiomyopathy is a feature of [ Q: 1639 ] PasTest 2009 - Basic
haemochromatosis. Science
A 19-year-old woman is referral to you with a
[ Q: 1638 ] PasTest 2009 - Basic
blood pressure of 180/130 mmHg. On
Science
examination she has cafè-au-lait patches and
Which one of the following statements BEST some axillary freckling. She required surgery
describes a patient with Wilson's disease? for scoliosis as a child.
1- The primary defect is believed to be What is the most likely underlying diagnosis?
enhanced intestinal absorption of copper

Dr. Khalid Yusuf El-Zohry – Sohag Teaching Hospital (01118391123)


Ref MRCPass OE OE 2012 PasTest 2009 PassMedicine 2009 PasTest Exam ReviseMRCP 736
Elzohry MRCP Questions Bank (Part 1) – 2013 (For my personal use)

1- Marfan's syndrome (raised connective tissue lesions usually on the


2- Neurofibromatosis type 1 lower back). Other features include cortical
tubers in the brain, learning disability, epilepsy
3- Hypertrophic cardiomyopathy and renal angiomyolipomas. Hypertrophic
4- Tuberous sclerosis cardiomyopathy can be a cause of
5- Cushing's disease hypertension and is associated with skin
lentigenes in the very rare LEOPARD
syndrome. Although scoliosis is a feature of
Answer & Comments Marfan's syndrome, this condition is not
Answer: 2- Neurofibromatosis type 1 associated with caf‫©أ‬-au-lait macules or
hypertension. Cushing's disease can cause
Neurofibromatosis type 1 (NF1) is an increased general skin pigmentation but not
autosomal-dominant condition diagnosed in caf‫©أ‬-au-lait macules.
an individual who has met two or more of the
following criteria:
[ Q: 1640 ] PasTest 2009 - Basic
1Six or more caf‫©أ‬-au-lait macules over 5 mm Science
in greatest diameter in prepubertal individuals
A 29-year-old man is referred to you with
and over 15 mm in greatest diameter in
lethargy, constipation and generalised bone
postpubertal individuals
pain. Some 5 years ago he had pituitary
2Two or more neurofibromas of any type or surgery for a prolactinoma. He mentions that
one plexiform neurofibroma his father had required surgery to his
parathyroid a few years ago and his brother is
3Freckling in the axillary or inguinal regions
currently in hospital with kidney stones.
4Optic glioma What is the most likely diagnosis?
5Two or more Lisch nodules (iris hamartomas) 1- Pseudohypoparathyroidism

6A distinctive osseous lesion, such as 2- Hypophosphataemia


sphenoid dysplasia or thinning of the long- 3- Familial hypocalciuric hypercalcaemia
bone cortex with or without pseudarthrosis
4- Multiple endocrine neoplasia type 1
7A first-degree relative (parent, sibling, or 5- Multiple endocrine neoplasia type 2
offspring) with NF1 as defined by the above
criteria
Answer & Comments
NF1 is due to mutations in the NF1 gene on
Answer: 4- Multiple endocrine neoplasia type
chromosome 17q11, and is one of the most
1
commonly dominantly inherited conditions.
Hypertension can be caused by renal artery The clinical features are very suggestive of
stenosis or phaeochromocytomas. Other hypercalcaemia and the previous history of a
complications include optic gliomas, vertebral pituitary tumour makes multiple endocrine
dysplasia, pseudarthrosis, intracranial neoplasia type 1 (MEN1) the most likely
tumours and malignant peripheral nerve- diagnosis. MEN1 is a dominantly inherited
sheath tumours. About half of the people with susceptibility to parathyroid adenomas,
NF1 have a learning disability. pituitary tumours and enteropancreatic
endocrine tumours due to mutations in the
Tuberous sclerosis is associated with
MEN1 gene. Conversely, multiple endocrine
hypopigmented macules, periungual fibromas,
neoplasia type 2 (MEN2) is a dominantly
facial angiofibromas and shagreen patches

Dr. Khalid Yusuf El-Zohry – Sohag Teaching Hospital (01118391123)


Ref MRCPass OE OE 2012 PasTest 2009 PassMedicine 2009 PasTest Exam ReviseMRCP 737
Elzohry MRCP Questions Bank (Part 1) – 2013 (For my personal use)

inherited susceptibility to medullary thyroid of neurological involvement. It can affect both


cancer, parathyroid adenomas and children and adults and mild forms can first
phaeochromocytomas due to mutations in the present in adulthood. It is most prevalent in
RET gene. MEN2 is subdivided into MEN2A the Ashkenazi Jewish population and can be
and MEN2B; MEN2B is associated with treated with enzyme replacement therapy.
mucosal neuromas of the lips and tongue, Types 2 and 3 are associated with neurological
distinctive facies with enlarged lips, involvement.
ganglioneuromatosis of the gastrointestinal
Tay-Sachs, Canavan's and Niemann-Pick's
tract and a ‘Marfanoid' body habitus.
disease are all also more common in the
Pseudohypoparathyroidism is characterised Ashkenazi Jewish population but all result in
by hypocalcaemia. Hypophosphataemia can severe intellectual impairment. Tay-Sachs and
be caused by primary or secondary Niemann-Pick's disease type A (which both
hyperparathyroidism . Familial hypocalciuric have a macular cherry-red spot that is a useful
hypercalcaemia is associated with moderate diagnostic aid) result in death in early
hypercalcaemia associated with an childhood, whereas the life expectancy for
inappropriate low rate of urinary calcium those with Canavan's disease can be into the
excretion. None of these conditions are teens. Galactosaemia is not more prevalent in
associated with pituitary tumours. the Ashkenazi Jewish population; it usually
presents in neonates with hypoglycaemia and
[ Q: 1641 ] PasTest 2009 - Basic acidosis and is associated with progressive
Science hepatosplenomegaly, cataracts and
intellectual impairment.
A 26-year-old teacher of Ashkenazi Jewish
descent presents with long-standing bone
[ Q: 1642 ] PasTest 2009 - Basic
pain. On examination there is marked
Science
hepatosplenomegaly.
Which genetic disorder should be included in A young man with severe learning difficulties
the list of differential diagnoses? and congenital heart disease is referred to you
for a cardiac assessment. He is accompanied
1- Tay-Sachs disease by his carer who says he has a chromosome
2- Canavan's disease abnormality.
3- Niemann-Pick's disease type A What type of abnormality is most likely to
4- Galactosaemia cause such a severe phenotype?
1- Balanced autosomal translocation
5- Gaucher's disease
2- Unbalanced autosomal translocation
Answer & Comments 3- Pericentric inversion
Answer: 5- Gaucher's disease 4- Paracentric inversion

Gaucher's disease is a recessively inherited 5- Sex chromosome aneuploidy


deficiency of the enzyme glucocerebrosidase,
which leads to an accumulation of Answer & Comments
glucosylceramide in lysosomes. It is the most
Answer: 2- Unbalanced autosomal
common lysosomal storage disease. Clinical
translocation
features include hepatosplenomegaly,
anaemia and osteopenia. Type 1 disease is the As a rule, the clinical effects of a chromosome
most common and is characterised by the lack abnormality reflect the amount of imbalance

Dr. Khalid Yusuf El-Zohry – Sohag Teaching Hospital (01118391123)


Ref MRCPass OE OE 2012 PasTest 2009 PassMedicine 2009 PasTest Exam ReviseMRCP 738
Elzohry MRCP Questions Bank (Part 1) – 2013 (For my personal use)

of genetic material. For example, all 4- Hereditary haemorrhagic telangiectasia


autosomal monosomies and most autosomal 5- Neurofibromatosis type 2
trisomies are incompatible with life, the
exceptions being trisomy 13 (Patau's
Answer & Comments
syndrome), trisomy 18 (Edward's syndrome)
and trisomy 21 (Down's syndrome): only the Answer: 3- von Hippel-Lindau syndrome
last of these carries a reasonable life
expectancy. Phaeochromocytomas are said to follow ‘the
rule of 10': 10% are malignant, 10% are
Inversions (where a piece of a chromosome is bilateral, 10% are extra-adrenal and 10% are
swapped around itself) often do not have a familial. Genetic conditions associated with
clinical phenotype but are commonly found as phaeochromocytomas include
an incidental finding. Occasionally the neurofibromatosis type 1 (NF1), multiple
inversion breakpoint is across a critical gene endocrine neoplasia type 2 (MEN2) and von
and causes a clinical disorder. Hippel-Lindau syndrome (VHL), all are
autosomal-dominant.
Balanced autosomal translocations are also
not normally associated with morbidity. These Individuals with NF1 can be diagnosed using
may only come to light when offspring are criteria based on the clinical features of caf‫©أ‬-
born with an unbalanced autosomal au-lait spots, axillary freckling, Lisch nodules
translocation that results in an abnormal copy and neurofibromas. The most serious
number for one or more chromosome regions complications in adults with NF1 are
(eg trisomic for one region and monosomic for hypertension due to either renal artery
another). Such chromosome abnormalities are stenosis or phaeochromocytomas and
generally associated with congenital malignant transformation of the
abnormalities, relatively short stature and neurofibromas.
intellectual impairment, although the exact
phenotype can vary widely. MEN2, due to mutations in the RET gene, is
characterised by medullary thyroid cancer,
Sex chromosome aneuploidies are associated parathyroid hyperplasia and
with comparatively less-severe phenotypes, phaeochromocytomas (type 2A) and rarely
eg Klinefelter's syndrome (XXY) and Turner's with a marfanoid habitus and mucosal
syndrome (XO). neuromas (type 2B).

Complications of the VHL syndrome include


[ Q: 1643 ] PasTest 2009 - Basic
phaeochromocytomas, renal-cell carcinomas
Science
and retinal and cerebellar
A 35-year-old man presents with severe haemangioblastomas, which can cause
hypertension and is found to have a unilateral intracerebral haemorrhage. VHL syndrome is
phaeochromocytoma. His father died from therefore the most likely genetic condition in
metastatic renal-cell carcinoma aged 48 and this case and the patient and other close
previously had surgery for a brain tumour. family members should be screened for
haemangioblastomas and renal-cell
What is the most likely underlying genetic
carcinomas in addition to
condition?
phaeochromocytomas. DNA analysis will
1- Neurofibromatosis type 1 detect VHL gene mutations in over 90% of VHL
2- Multiple endocrine neoplasia type 2A families.
3- von Hippel-Lindau syndrome

Dr. Khalid Yusuf El-Zohry – Sohag Teaching Hospital (01118391123)


Ref MRCPass OE OE 2012 PasTest 2009 PassMedicine 2009 PasTest Exam ReviseMRCP 739
Elzohry MRCP Questions Bank (Part 1) – 2013 (For my personal use)

Phaeochromocytomas are not associated with massive expansion in the triplet repeat that is
hereditary haemorrhagic telangiectasia (HHT) inherited from the affected mother.
or neurofibromatosis type 2 (NF2). However,
The other listed conditions are not associated
brain tumours (vestibular schwannomas,
with ptosis, although cardiac problems
gliomas, meningiomas) are a feature of NF2
(typically cardiomyopathy) can occur in
and cerebral arteriovenous malformations are
Duchenne's muscular dystrophy and its milder
sometimes seen in patients with HHT.
variant, Becker's muscular dystrophy.
Duchenne's muscular dystrophy usually
[ Q: 1644 ] PasTest 2009 - Basic results in death by the third decade.
Science Facioscapulohumeral dystrophy is
A 40-year-old man is referred to you with characterised by a pattern of muscle
breathlessness and bradycardia. During the weakness involving the face, scapula, upper
consultation you notice that he has a bilateral arm, lower leg and hip girdle. Spinal muscular
ptosis. He is estranged from his family and atrophy is an autosomal-recessive condition
never knew his father, but he does know that usually affecting children, but there is a milder
his sister has a muscle problem and lost a adult version (type IV) which causes a
child in infancy. proximal myopathy.

What diagnosis might you suspect in this


man? [ Q: 1645 ] PasTest 2009 - Basic
Science
1- Duchenne's muscular dystrophy
Angelman's and Prader-Willi syndromes both
2- Facioscapulohumeral dystrophy
involve defects in the same chromosome
3- Becker's muscular dystrophy region.
4- Myotonic dystrophy What is the best molecular explanation for
5- Spinal muscular atrophy differences in the phenotypes in these two
conditions?
Answer & Comments 1- It is sex-linked with the Prader-Willi
syndrome, occurring mainly in boys
Answer: 4- Myotonic dystrophy
2- It is due to anticipation, with successive
Myotonic dystrophy is an autosomal- generations becoming more severely
dominant neuromuscular disorder caused by affected
an expansion in a triplet repeat in the DMPK
3- It is related to the degree of X chromosome
gene. There is a rough correlation between
inactivation
the expansion size and the severity of the
phenotype. Clinical features include 4- It is due to modifying genes on other
myopathic facies, ptosis, myotonia, cataracts, chromosomes
testicular atrophy and diabetes mellitus. 5- It is due to the differential expression of
Cardiac conduction defects of varying degrees genes depending on parental origin
of severity are common. The disease exhibits
anticipation (a progressively severe phenotype
Answer & Comments
with each generation) due to the expansion of
the triplet repeat. It is possible his sister's Answer: 5- It is due to the differential
child had congenital myotonic dystrophy, a expression of genes depending on parental
very severe form of the condition due to a origin

Dr. Khalid Yusuf El-Zohry – Sohag Teaching Hospital (01118391123)


Ref MRCPass OE OE 2012 PasTest 2009 PassMedicine 2009 PasTest Exam ReviseMRCP 740
Elzohry MRCP Questions Bank (Part 1) – 2013 (For my personal use)

Imprinting is a mechanism that results in 4- They are proteins that attract phagocytes
differential gene expression according to 5- They perforate invading bacteria
parental origin. It is responsible for a number
of genetic syndromes, the best known of
Answer & Comments
which are the Angelman's and Prader-Willi
syndromes. Angelman's syndrome causes Answer: 4- They are proteins that attract
ataxia and severe learning disability and is phagocytes
caused by absent maternal expression of the
15q11-13 chromosome region. Conversely, Kinins are proteins that attract phagocytes,
the Prader-Willi syndrome (mild/moderate promote vasodilatation and increase the
learning disability, hyperphagia and obesity) is permeability of blood vessels. Complement
caused by absent paternal expression of the perforates invading bacteria, dilates blood
15q11-13 chromosome region. Other vessels, stimulates histamine release and
syndromes caused by imprinting attracts neutrophils.
abnormalities include the Beckwith-
Wiedermann syndrome (macroglossia, [ Q: 1647 ] PasTest 2009 - Basic
abdominal wall defects, hypoglycaemia, Science
visceromegaly) and the Russell-Silver
A patient in the intensive care unit following
syndrome (small stature). The exact
liver transplant surgery has a metabolic
mechanism of imprinting is not well
alkalosis.
understood but is thought to involve
differential gene methylation. Which of the following biochemical
abnormalities is most specifically indicative of
Both Angelman's and Prader-Willi syndromes this?
occur with equal frequency in both sexes.
1- Acidic urine
Anticipation is a completely different genetic
mechanism where trinucleotide repeat 2- High arterial blood pH (low hydrogen-ion
expansions cause increasingly severe concentration)
phenotypes in successive generations. 3- High arterial partial pressure of carbon
Variations in X chromosome inactivation are dioxide p(CO2)
thought to partly explain the mild phenotypes
4- High plasma bicarbonate concentration
sometimes seen in female carriers of X-linked
disorders. While modifier genes may well 5- Hypochloraemia
explain phenotypic variation between
individuals with the same syndrome, they do Answer & Comments
not explain the differences between the
Angelman's and Prader-Willi syndromes. Answer: 4- High plasma bicarbonate
concentration

[ Q: 1646 ] PasTest 2009 - Basic Arterial pH is increased in both metabolic and


Science respiratory alkalosis: plasma bicarbonate is
always increased in metabolic alkalosis and
Which of the following statements is true
can be low in chronic respiratory alkalosis. A
concerning kinins?
high p(CO2) can occur in metabolic alkalosis as
1- They are lipids a result of respiratory compensation, but it is
2- They promote vasoconstriction also a feature of respiratory acidosis. Although
the urine may become paradoxically acidic in
3- They decrease the permeability of blood
metabolic alkalosis, it is normally acidic,
vessels

Dr. Khalid Yusuf El-Zohry – Sohag Teaching Hospital (01118391123)


Ref MRCPass OE OE 2012 PasTest 2009 PassMedicine 2009 PasTest Exam ReviseMRCP 741
Elzohry MRCP Questions Bank (Part 1) – 2013 (For my personal use)

except sometimes immediately following a for digoxin toxicity.


meal. Hypochloraemia is present in metabolic Which of the following biochemical
alkalosis due to a loss of gastric acid, but may abnormalities, if present, would be most likely
not occur with alkalosis from other causes. to exacerbate the digoxin?
1- Hypokalaemia
[ Q: 1648 ] PasTest 2009 - Basic
Science 2- Hypocalcaemia

Which of following is true regarding gamma- 3- Hypomagnesaemia


glutamyl transferase (GGT)? 4- Hyponatraemia
1- It is increased in patients with fatty liver 5- Hypophosphataemia
disease
2- If elevated in prostate cancer it means liver Answer & Comments
metastases Answer: 1- Hypokalaemia
3- It is a better indicator of infectious hepatitis
Potassium depletion enhances the binding of
than of cholestasis
digoxin to the Na+/K+ pump, and this
4- It is only found in the liver increases the extent of inhibition of sodium
5- It is elevated in pregnancy transport, which can in turn cause digoxin
toxicity.
Answer & Comments Hypomagnesaemia may also have a
Answer: 1- It is increased in patients with fatty aggravating effect, but to a much less
liver disease important extent than potssium depletion.

Gamma-glutamyl transferase is elevated in a


[ Q: 1650 ] PasTest 2009 - Basic
number of hepatobiliary conditions (nearly all
Science
forms of biliary disease and cholestasis will be
associated with a raised GGT). These include: A 27-year-old woman presents with global
obstructive lesions of the biliary tree, weakness, lethargy and numbness of her
whatever the site alcoholic liver disease - not extremities, which has worsened over the past
specific for alcohol abuse but highly indicative 6 weeks. She admits to intermittent diarrhoea.
if very raised relative to transaminases or On examination in the Emergency Department
alkaline phosphatase; and infectious disease - she is markedly underweight with a body mass
a mild rise is only of particular use in index (BMI) of only 19. Blood testing reveals
distinguishing hepatic disease at times of hypokalaemia, raised chloride levels, low
childhood growth spurts and during serum bicarbonate and pH of 7.31.
pregnancy as the specificity of alkaline Which of the following diagnoses seems most
phosphatase (ALP) is reduced at this time (i.e. likely in this case?
if a patient has an isolated raised alkaline
1- Bartter's syndrome
phosphatase (high ALP with normal serum
aminotransferases) an elevated GGT suggests 2- Liddle's syndrome
the ALP is of liver origin). 3- Gitelman's syndrome
4- Laxative abuse
[ Q: 1649 ] PasTest 2009 - Basic
Science 5- Diuretic abuse

A 52 year old man is being treated in hospital

Dr. Khalid Yusuf El-Zohry – Sohag Teaching Hospital (01118391123)


Ref MRCPass OE OE 2012 PasTest 2009 PassMedicine 2009 PasTest Exam ReviseMRCP 742
Elzohry MRCP Questions Bank (Part 1) – 2013 (For my personal use)

Answer & Comments The half-life is defined as the time taken for
plasma levels of a particular chemical entity to
Answer: 4- Laxative abuse
fall by 50%. These results indicate that levels
This patient has a hypokalaemic, fall by 75% every 2 h. Hence the half-life is
hyperchloraemic metabolic acidosis. The most calculated at 1 h. It is worth noting that a
likely cause is gastrointestinal loss of halving of plasma levels may not indicate a
bicarbonate and potassium due to chronic reduction of 50% in activity of the agent, as a
severe laxative abuse. Whilst mild laxative number of agents are metabolized to
abuse results in alkalosis, more severe loss of products, which are also therapeutically or
bicarbonate eventually leads to acidosis. Her metabolically active.
markedly low BMI raises the suggestion of an
eating disorder. Although diuretic abuse [ Q: 1652 ] PasTest 2009 - Basic
would lead to marked potassium loss, the Science
appearance of alkalosis is more likely.
Diagnosis would be supported by urine testing You are asked to see a patient who had a
for metabolites of commonly used laxative chest drain removed 4 days ago. There
agents. appears to be some infection.
What are the stages in the cell biology of
[ Q: 1651 ] PasTest 2009 - Basic normal wound healing?
Science 1- Demolition is the first phase
You are trialing a new antihypertensive agent, 2- Maturation and remodelling can continue
'Wonderone'. As part of the drug- for up to a year
development programme you must assess the 3- Acute inflammation usually lasts for 6-12
half-life of Wonderone. Some results are sent hours
to you for calculation of the half-life:
4- Epithelial cell proliferation is the hallmark
15 mins after iv injection - Wonderone level of the demolition phase
150
5- Collagen deposition is the key process
2 h 15 mins after iv injection - Wonderone during demolition
level 37.5
4 h 15 mins after iv injection - Wonderone Answer & Comments
level 9.4.
Answer: 2- Maturation and remodelling can
Which of the following stems fits best with the continue for up to a year
half-life of this agent?
The first phase in healing by first intention is
1- 90 min
the phase of acute inflammation that lasts up
2- 15 min to 3 days, if uncomplicated. The initiating
3- 1 h factor appears to originate from platelets
activated by mature collagen exposed in the
4- 2 h
wound. Platelets first aggregate then release a
5- 3 h variety of active agents including lysosomal
enzymes, ATP, serotonin and wound
Answer & Comments cytokines. A fibrin clot develops, which
completes haemostasis and provides strength
Answer: 3- 1 h
and support to the wound. The surface dries
to form a scab. Platelets and macrophage

Dr. Khalid Yusuf El-Zohry – Sohag Teaching Hospital (01118391123)


Ref MRCPass OE OE 2012 PasTest 2009 PassMedicine 2009 PasTest Exam ReviseMRCP 743
Elzohry MRCP Questions Bank (Part 1) – 2013 (For my personal use)

factors cause local vasodilatation, which Answer & Comments


produces warmth and increases capillary
Answer: 5- T cells are mobile and contribution
permeability, allowing serum and white blood
to asthma is via cytokine production
cells to accumulate and cause swelling.
Cell type Classification Mode of action
After the initial acute inflammation,
Mobile/static*
macrophages become active as the main
agents of demolition, removing unwanted Neutrophils Granular leucocyte Phagocytosis
fibrin, dead cells and bacteria and creating Mobile
fluid-filled spaces for granulation tissue.
Macrophages also release factors that BasophilsGranular leucocyte Degranulation
stimulate the formation of new capillary buds Mobile
during this phase, and later they initiate and Eosinophils Granular leucocyte Degranulation
control fibroblast activity during repair. Within Mobile
the connective tissue, randomly orientated
collagen begins to form after a few days, Mast cells Granular leucocyte Degranulation
reaching a peak of activity after 5-7 days. Static

Epithelial cells at the edge of the wound start Macrophages Granular leucocyte Phagocytosis
to proliferate after 24 h and this phase can Both
last for up to 3 weeks. Monocytes Agranular leucocyte Phagocytosis
Finally, the phase of maturation and Mobile
remodelling lasts for up to 12 months, during NK cells Lymphocytes Phagocytosis Mobile
which time the tensile strength of the wound
increases and the random collagen is replaced B cells Lymphocytes Degranulation Mobile
by a more stable form orientated along lines
T cells Lymphocytes Cytokine production
of stress. Mobile

[ Q: 1653 ] PasTest 2009 - Basic *Mobile white blood cells circulate around the
Science body in the bloodstream; static white blood
cells circulate in a specific area, such as the
Concerning the respiratory cell biology of an lungs. NK, natural killer.
asthmatic individual, which of the following is
true?
[ Q: 1654 ] PasTest 2009 - Basic
1- Monocytes are granular and static Science
2- Eosinophils are agranular and mobile A 45-year-old man has severe pulmonary
3- Basophils are granular and their mode of emphysema. A diagnosis of α1-antitrypsin
action is phagocytosis deficiency is being considered.
4- B cells are mobile and their mode of action What is the genotype most typically
is phagocytosis associated with this condition?
5- T cells are mobile and contribution to 1- PiMM
asthma is via cytokine production 2- PiMZ
3- PiSS
4- PiSZ
5- PiZZ

Dr. Khalid Yusuf El-Zohry – Sohag Teaching Hospital (01118391123)


Ref MRCPass OE OE 2012 PasTest 2009 PassMedicine 2009 PasTest Exam ReviseMRCP 744
Elzohry MRCP Questions Bank (Part 1) – 2013 (For my personal use)

Answer & Comments side-effects of hypertension and hirsutism,


nor does it result in problems with lipid
Answer: 5- PiZZ
metabolism.
The function of α1-antitrypsin is to inhibit
neutrophil elastase and other proteases. The [ Q: 1656 ] PasTest 2009 - Basic
wild type is Pi (protease inhibitor) MM. The Science
gene displays considerable polymorphism,
with co-dominant inheritance. Most of the Which of the following is a feature of the early
variant proteins function normally but the asthma response?
PiZZ phenotype (approximately 1:2000 live 1- It occurs between 3-5 hours after an initial
births in Northern Europe) is associated with response to an allergen
deficiency of the protein and a risk of liver 2- Eosinophils are particularly important
disease in infants and emphysema in adults
(especially smokers). SS homozygotes and MS 3- Attraction of phagocytes is predominant
heterozygotes are not at increased risk, but 4- Mast-cell degranulation is seen in response
MZ heterozygotes have a slightly increased to the B-cell production of IgE
susceptibility to emphysema.
5- Usually resolves spontaneously after 6-8
hours
[ Q: 1655 ] PasTest 2009 - Basic
Science
Answer & Comments
In the selection of an optimum agent to
Answer: 4- Mast-cell degranulation is seen in
prevent rejection postrenal transplantation,
response to the B-cell production of IgE
which of the following cell biological principles
is correct? The early asthmatic response (EAR) is an
1- Daclizumab blocks the de novo pathway of episode of bronchoconstriction peaking 10-20
minutes after exposure to the provoking agent
purine synthesis
and resolving spontaneously after 1-2 hours.
2- Mycophenolate mofetil is a monoclonal The first inflammatory step is the appearance
antibody against IL-2 of an antigen (eg pollen) in the lung that
3- Basiliximab inhibits inosine monophosphate stimulates an immune response. Initial
dehydrogenase exposure to the antigen stimulates B cells to
produce IgE antibodies, which attack antigens
4- Tacrolimus is a calcineurin inhibitor
and become attached to the surface of mast
5- Sirolimus is a calcineurin inhibitor cells by high-affinity FceRI receptors. When
present, antigens will attach themselves to
Answer & Comments antibodies, forming antigen-antibody
complexes on the surface of the mast cell.
Answer: 4- Tacrolimus is a calcineurin inhibitor
These complexes cause the mast cell to
Mycophenolate mofetil blocks the de novo degranulate, releasing a range of
pathway of purine synthesis and inhibits inflammatory mediators, including histamine,
inosine monophosphate dehydrogenase. cytokines, prostaglandins and leukotrienes
Basiliximab and daclizumab are monoclonal with bronchoconstrictor effects. The antigen is
antibodies against IL-2 (interleukin-2) and they also presented to T cells via epithelial cells and
prevent T-cell proliferation. Tacrolimus and macrophages. The late asthmatic response
ciclosporin are both calcineurin inhibitors, but (LAR) occurs between 3 and 5 hours after an
not sirolimus. The latter does not have the initial response to an allergen. Eosinophils are

Dr. Khalid Yusuf El-Zohry – Sohag Teaching Hospital (01118391123)


Ref MRCPass OE OE 2012 PasTest 2009 PassMedicine 2009 PasTest Exam ReviseMRCP 745
Elzohry MRCP Questions Bank (Part 1) – 2013 (For my personal use)

particularly important and attraction of 1- It is uncharged


phagocytes is predominant. It usually resolves 2- It is depolarised
spontaneously after 6-8 hours.
3- It is negatively charged externally

[ Q: 1657 ] PasTest 2009 - Basic 4- It is positively charged externally


Science 5- It is unable to conduct impulses
Which of the following features applies to
acetylcholine-mediated transmission at the Answer & Comments
motor endplate? Answer: 4- It is positively charged externally
1- The synaptic fusion complex is made of
In its resting state, the inside of a neurone is
synaptobrevin, syntaxin and synaptosome-
rich in chloride ions and a lesser amount of
associated protein
potassium ions. The outside of the neurone is
2- Syntaxin forms a SNAP complex, together rich in sodium ions. The net result is that the
with NSF, Ca2+ and other proteins neurone is polarised, ie negative on the inside
3- Tetanus toxin (TeNT) cleaves specific sites and positive on the outside. This is called the
of SNAP-25 ‘resting potential'. When stimulated, the
permeability of the neurone changes, allowing
4- Botulinum toxin type B (BoNT/B) cleaves
sodium from the extracellular fluid to flood
syntaxin
into the neurone and potassium to move out.
5- Botulinum toxin type A (BoNT/A) cleaves This depolarisation causes the generation of
specific sites of synaptobrevin (VAMP) an action potential.

Answer & Comments [ Q: 1659 ] PasTest 2009 - Basic


Answer: 1- The synaptic fusion complex is Science
made of synaptobrevin, syntaxin and Degranulation of eosinophils allows which of
synaptosome-associated protein the following cellular processes?
The synaptic fusion complex is made up of 1- Fusion of the lysosomal membrane with the
synaptobrevin, syntaxin and synaptosome- plasma membrane
associated protein (SNAP-25). The fusion 2- Chemotaxis
complex serves to join the membranes of an
acetylcholine vesicle and nerve cell. 3- Ingestion within a phagosome
Synaptobrevin forms a SNAP complex, 4- Intracellular enzymatic degradation
together with NSF, Ca2+ and other proteins. 5- Endocytosis
Botulinum toxin type A (BoNT/A) and E
(BoNT/E) cleave specific sites of SNAP-25, eg
BoNT/A recognises Glu-Ala-Asn-Gln.Botulinum Answer & Comments
toxin type C (BoNT/C) cleaves syntaxin. Answer: 1- Fusion of the lysosomal membrane
BoNT/B, D, F, G and tetanus toxin (TeNT) with the plasma membrane
cleave specific sites of synaptobrevin (VAMP).
Basophils, eosinophils and mast cells contain
lysosomes and can release the contents of
[ Q: 1658 ] PasTest 2009 - Basic
these granules by degranulation. This allows
Science
them to act against larger infecting organisms
Which of the following statements pertains to such as protozoa and helminths, which cannot
a resting neurone? be engulfed by phagocytosis. Phagocytosis

Dr. Khalid Yusuf El-Zohry – Sohag Teaching Hospital (01118391123)


Ref MRCPass OE OE 2012 PasTest 2009 PassMedicine 2009 PasTest Exam ReviseMRCP 746
Elzohry MRCP Questions Bank (Part 1) – 2013 (For my personal use)

comprises chemotaxis, ingestion within a have ‘cherry-red' spots in their eyes. The
phagosome, intracellular enzymatic incidence of Tay-Sachs is particularly high
degradation and exocytosis. among people of Eastern European and
Askhenazi Jewish descent. Patients and
[ Q: 1660 ] PasTest 2009 - Basic carriers of Tay-Sachs disease can be identified
Science by a simple blood test that measures beta-
hexosaminidase A activity. Both parents must
The accumulation of gangliosidic GM2 in the carry the mutated gene in order to have an
central nervous system of individuals with Tay- affected child. In these instances, there is a
Sachs disease is attributed to: 25% chance with each pregnancy that the
1- Decreased lysosomal hydrolysis child will be affected with Tay-Sachs disease.
Prenatal diagnosis is available if desired.
2- Decreased Golgi stimulation
3- Increased permeability of the blood-brain
[ Q: 1661 ] PasTest 2009 - Basic
barrier
Science
4- Increased receptor-mediated endocytosis
Which of the following statements regarding
5- Increased synthesis in the endoplasmic the eukaryotic cell cycle is correct?
reticulum
1- M phase signifies meiosis

Answer & Comments 2- DNA is made in the G1 phase


3- DNA is made in the S phase
Answer: 1- Decreased lysosomal hydrolysis
4- G2 phase commences as G1 finishes
Tay-Sachs disease is a fatal genetic lipid
storage disorder in which harmful quantities 5- G2 determines variability in the length of
of a fatty substance called ganglioside GM2 the cell cycle in humans
build up in tissues and nerve cells in the brain.
The condition is caused by insufficient activity Answer & Comments
of an enzyme called beta-hexosaminidase A
Answer: 3- DNA is made in the S phase
that catalyses the biodegradation of acidic
fatty materials known as gangliosides. Mitosis occurs in the M phase of the
Gangliosides are made and biodegraded eukaryotic cell cycle and DNA is synthesised in
rapidly in early life as the brain develops. the S phase. G1 and G2 are gap phases. Cells
can enter the G0 phase from G1 if they are not
Infants with Tay-Sachs disease appear to
preparing for cell division. This may occur if
develop normally for the first few months of
the cell has reached its final differentiation.
life. Then, as nerve cells become distended
The G1 phase determines the variability of the
with fatty material, a relentless deterioration
cycle length.
of mental and physical abilities occurs. The
child becomes blind, deaf, and unable to
swallow. Muscles begin to atrophy and [ Q: 1662 ] PasTest 2009 - Basic
paralysis sets in. Other neurological symptoms Science
include dementia, seizures, and an increased A 42-year-old Afro-Caribbean man sustains a
startle reflex to noise. A much rarer form of myocardial infarction. He is discharged from
the disorder occurs in patients in their 20s and hospital 4 days later. His medication is aspirin,
early 30s, and is characterised by an unsteady simvastatin and a β-blocker. A month later, he
gait and progressive neurological is well and has routine blood tests in the
deterioration. Persons with Tay-Sachs also outpatient clinic. His serum creatine kinase

Dr. Khalid Yusuf El-Zohry – Sohag Teaching Hospital (01118391123)


Ref MRCPass OE OE 2012 PasTest 2009 PassMedicine 2009 PasTest Exam ReviseMRCP 747
Elzohry MRCP Questions Bank (Part 1) – 2013 (For my personal use)

(CK) activity is reported as being 400 U/l. His What is the most likely cause of the abnormal
CK 24 h after admission had been 620 U/l and findings?
had fallen to 415 U/l after a further 48 h. 1- Familial combined hyperlipidaemia
Which of the following is the most likely cause 2- Hepatic steatosis secondary to alcohol
of the high CK when he was reviewed in
outpatients? 3- Non-alcoholic steatohepatitis

1- Effect of simvastatin 4- Remnant hyperlipidaemia (familial


dysbetalipoproteinaemia, broad beta
2- Further myocardial infarction disease)
3- Heavy exercise 5- Valproate treatment
4- Racial variant
5- Undiagnosed hypothyroidism Answer & Comments
Answer: 2- Hepatic steatosis secondary to
Answer & Comments alcohol
Answer: 4- Racial variant The combination of hypertriglyceridaemia and
CK rises to a peak at about 24 h after a elevated g-glutamyltransferase activity is
myocardial infarction and usually returns to typical of alcohol-induced hepatic steatosis.
baseline by 72 h. The fact that this man's did Transaminase activity may also be elevated if
not would be consistent with an extension of there is hepatic inflammation. Non-alcoholic
his infarct, but serum CK activity in Afro- steatohepatitis is usually associated with
Caribbeans is often up to three times the obesity, and transaminase activity is usually
upper limit of normal for Caucasians. This is a elevated. Valproate is not an enzyme-inducing
more likely explanation than undiagnosed drug and does not cause an increase in g-
hypothyroidism (uncommon in a man of this glutamyltransferase (nor
age) or heavy exercise (which would be hypertriglyceridaemia); hepatotoxicity with
discouraged this soon after an infarct), though valproate is idiosyncratic, and causes an
both can cause increases in CK. So, too, can increase in transaminases. Remnant
statins, but there has been no increase above hyperlipidaemia is rare, and cholesterol and
the baseline inferred for this patient. triglyceride concentrations are typically much
higher than in this case. Familial combined
hyperlipidaemia can cause an elevated
[ Q: 1663 ] PasTest 2009 - Basic
triglyceride, cholesterol or both, but liver
Science
function tests are usually normal unless
A 25-year-old man is found to have a fasting hypertriglyceridaemia is severe.
serum triglyceride concentration of 4.2
mmol/l at a routine examination for life [ Q: 1664 ] PasTest 2009 - Basic
insurance. His serum cholesterol Science
concentration is 5.4 mmol/l; liver function
tests are normal apart from a g- A 55-year-old-woman presents with the
glutamyltransferase activity of 74 U/l (upper clinical features of Cushing's syndrome. She is
limit of normal, 55 U/l). He claims to be in on no medication. The results of routine
good health but has a history of epilepsy for biochemical investigations are normal. Her
which he is taking sodium valproate. His body 0900-h cortisol concentration is 800 nmol/l
mass index is 24 kg/m2. (normal 150-650) and ACTH 80 ng/ml (normal
< 50). Following dexamethasone 1 mg the

Dr. Khalid Yusuf El-Zohry – Sohag Teaching Hospital (01118391123)


Ref MRCPass OE OE 2012 PasTest 2009 PassMedicine 2009 PasTest Exam ReviseMRCP 748
Elzohry MRCP Questions Bank (Part 1) – 2013 (For my personal use)

previous evening, a repeat 0900-h cortisol Measurement of which of the following in a


concentration is 720 nmol/l. sample of faeces would be most appropriate
Which of the following is the most likely to prove this?
diagnosis? 1- Albumin
1- Adrenal adenoma 2- α1-antitrypsin
2- Adrenal carcinoma 3- Calmodulin
3- Cushing's disease (pituitary adenoma) 4- Calprotectin
4- Depression 5- Elastase
5- Ectopic ACTH secretion
Answer & Comments
Answer & Comments Answer: 2- α1-antitrypsin
Answer: 3- Cushing's disease (pituitary Albumin is degraded by proteases in the gut.
adenoma) However, α1-antitrypsin is a plasma protein
In Cushing's syndrome secondary to an that is resistant to degradation by proteases
adrenal tumour, it would be expected that the (it is a protease inhibitor) and its
high cortisol concentration would suppress measurement can indicate leakage of plasma
the secretion of ACTH. If the diagnosis is proteins into the gut. Faecal calprotectin is
uncertain however, a high dose increased in inflammatory bowel disease, only
dexamethasone test (2mg QDS for 48 hrs) may one of several causes of protein-losing
be added to the investigations. Ectopic ACTH enteropathy. Calmodulin is an intracellular
secretion stimulates adrenal cortisol calcium-binding protein. Elastase is measured
secretion, and is not suppressed either by as an index of pancreatic function.
cortisol or exogenous glucocorticoids.
However, patients usually present with weight [ Q: 1666 ] PasTest 2009 - Basic
loss, muscle weakness or glucose intolerance Science
rather than with the somatic features of
A 20-year-old man presents with mild jaundice
Cushing's, and ACTH concentrations tend to
following a flu-like illness. Following review by
be markedly higher than normal. An exception
a gastroenterologist, he has been told that a
is with ectopic ACTH secretion by carcinoid
diagnosis of Gilbert's syndrome is probable.
tumours, but this is relatively uncommon.
Depression can increase ACTH and cortisol Which of the following laboratory tests is most
secretion, but patients are not usually likely to confirm this diagnosis?
cushingoid. The presentation and hormonal 1- Absence of bilirubin in the urine
abnormalities are, however, typical of
2- Decreased serum haptoglobin
Cushing's disease.
concentration
3- Elevated serum aspartate aminotransferase
[ Q: 1665 ] PasTest 2009 - Basic
(transaminase, AST) activity
Science
4- Increased reticulocyte count
A patient with hypoalbuminaemia is
suspected of having a protein-losing 5- Increased urinary urobilinogen excretion
enteropathy.
Answer & Comments
Answer: 1- Absence of bilirubin in the urine

Dr. Khalid Yusuf El-Zohry – Sohag Teaching Hospital (01118391123)


Ref MRCPass OE OE 2012 PasTest 2009 PassMedicine 2009 PasTest Exam ReviseMRCP 749
Elzohry MRCP Questions Bank (Part 1) – 2013 (For my personal use)

In Gilbert's syndrome, the excess bilirubin is [ Q: 1668 ] PasTest 2009 - Basic


unconjugated, and does not appear in the Science
urine. The same is true for jaundice secondary
An overweight 32-year-old woman presents
to haemolysis. However, in haemolytic
with a short history of painless jaundice. There
jaundice, urinary urobilinogen is increased
is no previous history of illness and, apart
(increased production of bilirubin, and hence
from the jaundice, she has no signs of chronic
of urobilinogen), the reticulocyte count may
liver disease. Initial investigations reveal a
be elevated and serum haptoglobin
haemoglobin of 12.7 g/dl, MCV 105 fl, serum
concentration decreased. Haemolysis may
bilirubin 162 mmol/l, AST 145 U/l, alkaline
also cause a slight increase in serum
phosphatase 224 U/l, gamma-
aminotransferase (transaminase) activity.
glutamyltransferase 200 U/l.
Which of the following is the most likely
[ Q: 1667 ] PasTest 2009 - Basic
diagnosis?
Science
1- Alcoholic liver disease
A 71-year-old man presents with fatigue and
breathlessness. The results of initial 2- Autoimmune chronic hepatitis
investigations include a serum creatinine 3- Carcinoma of the head of the pancreas
concentration of 654 mmol/l.
4- Cholecystitis
Which of the following best suggests a specific
5- Hepatitis A infection
cause for his renal failure?
1- Anaemia
Answer & Comments
2- Bence Jones proteinuria
Answer: 1- Alcoholic liver disease
3- Hyperuricaemia
Jaundice with an elevation of both AST and
4- Hypocalcaemia
alkaline phosphatase suggests mixed
5- Metabolic acidosis hepatocellular damage and cholestatic liver
disease, typical of acute alcoholic hepatitis on
Answer & Comments a background of chronic liver disease (and is
not excluded by the lack of physical signs). The
Answer: 2- Bence Jones proteinuria high gamma-glutamyltransferase lends
Bence Jones proteinuria (excretion of support to this (although it may be increased
immunoglobulin light chains) suggests a in liver disease of any cause). Macrocytosis is
diagnosis of myeloma. This can cause renal typical of chronic excessive alcohol intake and
failure through various mechanisms, including is not a feature of the other conditions;
tubular obstruction by protein, infiltration by although were it not present, autoimmune
amyloid, etc. Anaemia (due to decreased liver disease would need to be considered.
production of erythropoietin and marrow In hepatitis A, AST is typically higher than
suppression), hyperuricaemia (due to alkaline phosphatase, while the reverse is true
decreased urate excretion), metabolic acidosis of pancreatic carcinoma. Chronic cholecystitis
(due to decreased renal acid excretion) and can cause jaundice but it would be unusual for
hypocalcaemia (secondary to decreased renal there to be no history of acute episodes.
synthesis of calcitriol (1,25-
dihydroxycholecalciferol)) are common in
patients with chronic renal failure, irrespective
of the cause.

Dr. Khalid Yusuf El-Zohry – Sohag Teaching Hospital (01118391123)


Ref MRCPass OE OE 2012 PasTest 2009 PassMedicine 2009 PasTest Exam ReviseMRCP 750
Elzohry MRCP Questions Bank (Part 1) – 2013 (For my personal use)

[ Q: 1669 ] PasTest 2009 - Basic In acute intermittent porphyria, a deficiency


Science of porphobilinogen deaminase leads to the
accumulation of porphyrin precursors in acute
Which one of the following statements best
attacks. These cause neurological symptoms,
applies to renal tubular acidosis type 4?
particularly autonomic. Abdominal pain,
1- It is an inherited condition nausea, vomiting, constipation occur in
2- It is a recognised complication of diabetes approximately 90% of patients. Hypertension,
mellitus tachycardia and motor neuropathies also
occur relatively frequently, but sensory
3- It is associated with hypokalaemia
neuropathies are uncommon.
4- It is due to decreased renal bicarbonate Neuropsychiatric manifestations include
reasborption depression, anxiety, fits and psychosis, but the
5- Renal calculi are a common presentation latter is rare.

Answer & Comments [ Q: 1671 ] PasTest 2009 - Basic


Science
Answer: 2- It is a recognised complication of
diabetes mellitus A 70-year-old man with symptoms of
prostatism has a serum prostate specific
Renal tubular acidosis, also known as antigen (PSA) concentration of 20 mg/l
‘hyporeninaemic hypoaldosteronism', is the (normal value, < 4 mg/l).
most common renal tubular acidosis. It is an
Which one of the following statements about
acquired condition, usually seen in patients
the clinical importance of this result is the
with mild chronic renal failure due to diabetes
most likely?
or tubulointerstitial renal disease. Decreased
renal bicarbonate absorption is the basis of 1- It could be elevated because a digital rectal
type-2 (proximal) RTA; renal calculi are examination was performed 48 h before
particularly associated with type-1 (distal) the blood sample was taken
RTA. 2- It could be elevated because of cancer
elsewhere in the urinary tract
[ Q: 1670 ] PasTest 2009 - Basic 3- It is diagnostic of prostatic cancer
Science
4- It is more likely to reflect prostatic cancer
A 19-year-old woman has been diagnosed as than benign prostatic hypertrophy
having acute intermittent porphyria. How is
5- It is normal for a man of this age
she most likely to have presented?
1- Acute abdomen
Answer & Comments
2- Hypertension
Answer: 4- It is more likely to reflect prostatic
3- Motor neuropathy cancer than benign prostatic hypertrophy
4- Psychosis
PSA is specific to the prostate, although not
5- Sensory neuropathy for cancer. Its concentration in the serum
increases with age and in benign prostatic
Answer & Comments hypertrophy, but a concentration this high is
more likely to be due to cancer than benign
Answer: 1- Acute abdomen disease. PSA increases only slightly, and
transiently, after digital rectal examination.

Dr. Khalid Yusuf El-Zohry – Sohag Teaching Hospital (01118391123)


Ref MRCPass OE OE 2012 PasTest 2009 PassMedicine 2009 PasTest Exam ReviseMRCP 751
Elzohry MRCP Questions Bank (Part 1) – 2013 (For my personal use)

[ Q: 1672 ] PasTest 2009 - Basic 3- Alkaline phosphatase activity


Science 4- Urea concentration
An elderly man with a history of prostatism 5- TSH (thyroid-stimulating hormone)
presents with acute retention of urine. His concentration
serum creatinine concentration is 520 mmol/l.
Which of the following additional abnormal Answer & Comments
serum biochemistry test results is most
Answer: 2- Glomerular filtration rate
suggestive of a chronic component to his renal
failure? The glomerular filtration rate declines with
1- Hyperkalaemia ageing beyond 40 years, but serum
concentrations of creatinine and urea are
2- Hyperuricaemia
usually not affected because of the
3- Hypocalcaemia concomitant decreases in muscle bulk and
4- Hyponatraemia protein turnover, respectively. Alkaline
phosphatase activity is often increased in
5- Low serum bicarbonate concentration
elderly people, but this is usually because of
subclinical Paget's disease or osteomalacia.
Answer & Comments Glucose tolerance tends to decline with ageing
Answer: 3- Hypocalcaemia (and the renal threshold for glucose to
increase). TSH concentrations are normal in
Hyperkalaemia and hyperuricaemia (due to older people except in hypothyroidism (which
decreased excretion), hyponatraemia (due may be subclinical or ‘compensated').
mainly to continued water intake with
decreased ability to excrete it) and metabolic [ Q: 1674 ] PasTest 2009 - Basic
acidosis occur in both acute and chronic renal Science
failure. The presence of hypocalcaemia in
renal failure suggests that this is, at least in A 25-year-old man is referred to a
part, of longstanding, and is due to decreased dermatologist with fleshy nodules on the
renal synthesis of calcitriol (1,25- backs of his elbows. He also has yellow linear
dihydroxycholecalciferol). deposits in his palmar creases. His fasting
serum lipids are: cholesterol 14.2 mmol/l,
triglycerides 16 mmol/l. Fasting plasma
[ Q: 1673 ] PasTest 2009 - Basic
glucose and thyroid function tests are normal.
Science
In addition to dietary and lifestyle advice,
A 68-year old man undergoes a routine health
what would be the most appropriate
check-up, which includes various laboratory
treatment?
investigations. He has abnormal results as
follows: fasting blood glucose 5.6 mmol/l; 1- A bile-acid sequestrant
urea 7.2 mmol/l; alkaline phosphatase activity 2- A fibrate
176 U/l (upper limit of reference range 150);
3- A statin
TSH 9.1 mU/l; creatinine clearance
(calculated) 70 ml/min. 4- Nicotinic acid
Which one of the following parameters can be 5- Omega-3 fish oils
regarded as a normal result of ageing?
1- Fasting blood glucose concentration Answer & Comments

2- Glomerular filtration rate Answer: 2- A fibrate

Dr. Khalid Yusuf El-Zohry – Sohag Teaching Hospital (01118391123)


Ref MRCPass OE OE 2012 PasTest 2009 PassMedicine 2009 PasTest Exam ReviseMRCP 752
Elzohry MRCP Questions Bank (Part 1) – 2013 (For my personal use)

The lipid deposits in the palmar creases, diuresis. ‘Normal saline' is the fluid of choice.
probable tuberous xanthomas on the elbows Bisphosphonates are effective, but must be
and elevations of cholesterol and triglyceride given intravenously to have a rapid effect. A
to approximately equal molar concentrations diuretic may be helpful once the patient has
are typical of remnant hyperlipidaemia been rehydrated, but this should be a loop
(familial dysbetalipoproteinaemia, broad beta diuretic, which increase calcium excretion, not
disease). Bile-acid sequestrants reduce a thiazide, as these reduce it. Corticosteroids
cholesterol but may increase triglyceride may occasionally be helpful but are not first-
concentrations. Statins are principally line treatment. Sodium phosphate infusion is
cholesterol-lowering agents. Nicotinic acid dangerous: it lowers the calcium
lowers cholesterol and triglycerides but is concentration rapidly, but risks causing
poorly tolerated, although derivatives may be metastatic calcification.
more acceptable to patients. Omega-3 fish oils
are relatively weak lipid-lowering agents. [ Q: 1676 ] PasTest 2009 - Basic
Fibrates are usually effective treatment for Science
this condition. Genetic predisposition to this
condition is approximately 100 times more Which of the following fluids would be the
prevalent than the condition itself. This most appropriate to replace the fluid being
dyslipidaemia is frequently expressed clinically lost in a patient with a paralytic ileus draining
because of an additional factor, for example 2 litres of fluid a day through a nasogastric
impaired glucose tolerance or frank diabetes, tube?
and such conditions should be sought and, if 1- Compound sodium lactate (Hartmann's
present, be treated appropriately. solution)
2- 5% dextrose
[ Q: 1675 ] PasTest 2009 - Basic
3- 10% dextrose
Science
4- 0.18% sodium chloride with 4% dextrose
A 56-year-old woman with known metastatic
(‘dextrose saline')
breast cancer presents to A&E with a calcium
concentration of 3.22 mmol/l. 5- 0.9% sodium chloride (‘normal saline')

Which of the following is the most appropriate


initial management? Answer & Comments

1- Intravenous hydrocortisone Answer: 5- 0.9% sodium chloride (‘normal


saline')
2- Intravenous infusion of 0.9% sodium
chloride (‘normal saline') In this situation, it is essential to supply
3- Intravenous infusion of sodium phosphate sufficient chloride ions to replace the chloride
being lost in the gastric fluid (gastric juice is
4- Oral bisphosphonate essentially dilute hydrochloric acid). If this is
5- Oral thiazide diuretic not done, a metabolic alkalosis can ensue. The
appropriate fluid is ‘normal saline'. The two
Answer & Comments dextrose solutions contain no chloride, and
‘dextrose saline' contains insufficient for this
Answer: 2- Intravenous infusion of 0.9% purpose. Hartmann's solution could
sodium chloride (‘normal saline') exacerbate any tendency to alkalosis as the
The priority in symptomatic hypercalcaemia is lactate it contains is metabolised to
to rehydrate the patient and establish a bicarbonate.

Dr. Khalid Yusuf El-Zohry – Sohag Teaching Hospital (01118391123)


Ref MRCPass OE OE 2012 PasTest 2009 PassMedicine 2009 PasTest Exam ReviseMRCP 753
Elzohry MRCP Questions Bank (Part 1) – 2013 (For my personal use)

[ Q: 1677 ] PasTest 2009 - Basic Answer & Comments


Science
Answer: 5- Tall, tented T wave
Which of the following tumour-associated
All these may be seen in hyperkalaemia
antigens is linked with the correct cancer?
except prominent U waves, which are typical
1- CA-125: Testicular teratoma of severe hypokalaemia. Although the pattern
2- CA 19-9: Pancreatic cancer with which they develop is variable, the
development of tall T waves typically precedes
3- Alpha-fetoprotein: Ovarian cancer
a reduction in P wave and prolongation of the
4- CEA: Hepatocellular carcinoma PR interval, while the sine-wave pattern is
5- HCG: Breast cancer typical of severe hyperkalaemia and is a
prearrest abnormality.
Answer & Comments
[ Q: 1679 ] PasTest 2009 - Basic
Answer: 2- CA 19-9: Pancreatic cancer Science
CA-125 is associated with ovarian epithelial A woman presented with diarrhoea that has
cancers. CA 19-9 with pancreatic and some persisted up to 2 weeks after
colorectal cancers. Alpha-fetoprotein is cholecystectomy.
associated with hepatocellular carcinoma,
What is the most likely cause of the diarrhoea?
hepatoblastoma, testicular teratoma and yolk
sac tumours. CEA is associated with colon 1- Salmonella spp.
carcinoma and some head and neck and lung 2- Yersinia
cancers. HCG is associated with germ-cell
ovarian tumours, choriocarcinoma, testicular 3- Campylobacter
teratoma and seminoma. 4- Bile acid malabsorption
5- Lactose malabsorption
[ Q: 1678 ] PasTest 2009 - Basic
Science Answer & Comments
A 48-year-old woman with known renal Answer: 4- Bile acid malabsorption
disease presents to A&E, and admission
biochemistry profile shows a serum potassium Bile acid malabsorption is an underdiagnosed
concentration of 5.9 mmol/l. Her ECG is cause of chronic diarrhoea and is a recognised
abnormal. cause of diarrhoea after cholecystectomy. Bile
acids are present in increased concentrations
Which ECG abnormality is most likely to have
in the colon (particularly deoxycholate and
occurred earliest as her hyperkalaemia
chenodeoxycholate) and lead to diarrhoea by
developed?
causing both increased colonic secretions and
1- Prolonged PR interval motility. Diagnosis is made with the selenium-
2- Prominent U wave 75-homocholic acid taurine (SeHCAT) test,
where radiolabelled bile acid analogue is
3- Reduced P wave
administered;% age retention at 7 days is
4- Sine-wave pattern calculated (less than 19% retention is
5- Tall, tented T wave abnormal). Treatment of bile acid
malabsorption is with cholestyramine, which
binds and inactivates bile acids in the colon.

Dr. Khalid Yusuf El-Zohry – Sohag Teaching Hospital (01118391123)


Ref MRCPass OE OE 2012 PasTest 2009 PassMedicine 2009 PasTest Exam ReviseMRCP 754
Elzohry MRCP Questions Bank (Part 1) – 2013 (For my personal use)

[ Q: 1680 ] PasTest 2009 - Basic is suffering from multiple sclerosis and are
Science considering prescribing beta-interferon for
her.
A 49-year-old woman presents to her
gynaecologist with CIN-3 changes. She has Which of the following best describes the
been reading on the Internet about how the action of beta-interferon?
human papillomavirus interferes with the 1- It increases MHC class-II expression on
programmed death of defective cells and may antigen-presenting cells
predispose to cancers.
2- It leads to increased MHC class-I expression
Which of the following protein names best
3- It is used in the treatment of hepatitis B
identifies the DNA-binding protein with which
the papillomavirus interferes? 4- It may be used as an adjunct in atypical
mycobacterial disease
1- p52
5- It activates macrophage and neutrophil
2- p51
intracellular killing
3- p53
4- p54 Answer & Comments
5- p55 Answer: 2- It leads to increased MHC class-I
expression
Answer & Comments
Alpha- and beta-interferon are produced
Answer: 3- p53 mainly in response to viral infection. They bind
the same cellular receptor and protect
p53 is a DNA-binding protein which induces
uninfected cells by inducing the upregulation
the expression of a number of genes that are
of molecules that inhibit the viral production
involved in programmed cell death. Clearly,
of RNA and DNA. They also induce MHC class-I
p53 gene mutations or products that interfere
expression leading to enhanced lysis of cells
with p53 protein functioning are implicated in
that are already infected with virus. In
carcinogenesis. Initial p53 gene expression
addition, they have antiproliferative actions.
causes genes involved in DNA repair to be
expressed. If DNA repair is too slow or gene Alpha-interferon is used in the treatment of
repair cannot be effected, then other genes chronic hepatitis B and C infections as well as
involved in apoptosis take over, and some forms of leukaemia. Beta-interferon is
programmed cell death ensues. used in the treatment of multiple sclerosis.
Gamma-interferon induces MHC class-II
Both the adenovirus E1B and papillomavirus
expression on antigen-presenting cells via a
E6 gene products bind p53 and interfere with
different receptor to alpha- and beta-
its functioning. Another protein involved in
interferon.
the cell cycle and triggering of apoptosis is the
RB protein, the functioning of which is
interfered with by a number of viruses [ Q: 1682 ] PasTest 2009 - Basic
including papillomavirus, adenovirus and the Science
SV40 virus. A 68-year-old man is admitted to hospital for
elective femoral angioplasty. On examination,
[ Q: 1681 ] PasTest 2009 - Basic he is found to have widespread
Science lymphadenopathy. Blood is taken for 'group
and save'. His cells are not agglutinated by
You are reviewing a 45-year-old woman who
either anti-A or anti-B; his serum does not

Dr. Khalid Yusuf El-Zohry – Sohag Teaching Hospital (01118391123)


Ref MRCPass OE OE 2012 PasTest 2009 PassMedicine 2009 PasTest Exam ReviseMRCP 755
Elzohry MRCP Questions Bank (Part 1) – 2013 (For my personal use)

cause agglutination of cells of blood group A 2- Myosin-binding protein C


or B. 3- Myosin light chains
Based on these data, which of the following is 4- Troponin I
most likely to be his genotype?
5- Troponin T
1- AB
2- AO Answer & Comments
3- BB
Answer: 5- Troponin T
4- BO
The diagnosis is hypertrophic cardiomyopathy.
5- OO This condition is usually familial with
autosomal-dominant inheritance. Mutations
Answer & Comments in all the proteins listed (and others) have
been described. The clinical and
Answer: 5- OO
morphological features vary according to the
If his cells are not agglutinated by anti-A or protein involved. Presenting features include
anti-B, he must be blood group O. The O gene chest pain, syncope, arrhythmias and sudden
is recessive and his genotype must therefore death, but sudden death with only mild
be OO. Group O individuals have naturally hypertrophy is particularly associated with
occurring antibodies to A and B blood group mutations in troponin T.
antigens, and it would therefore be expected
that his serum would agglutinate cells bearing [ Q: 1684 ] PasTest 2009 - Basic
these antigens. The fact that it does not Science
suggests that he may have a humoral immune
deficiency. The lymphadenopathy provides a A previously healthy 25-year-old man
clue to the likely cause of this: chronic presented with watery diarrhoea of 10 days'
lymphatic leukaemia is a relatively indolent duration. He had no significant past medical
condition characterised by generalised history. Examination showed significant
lymphadenopathy and may be associated with postural drop. Stool culture samples yielded
an immune paresis. Cryptosporidium parvum.
What is the next appropriate management
[ Q: 1683 ] PasTest 2009 - Basic after replacement with intravenous fluids?
Science 1- Albendazole
A 16-year-old youth collapses during a game 2- Co-trimoxazole
of football. He has previously been well. He 3- Metronidazole
has no carotid pulse and attempts at
cardiopulmonary resuscitation are 4- No specific therapy
unsuccessful. At postmortem, he is found to 5- Ciprofloxacin
have mild hypertrophy of the septum.
Microscopic examination shows myocyte Answer & Comments
disarray and disorganisation of myofibrils
within the mycoytes. Answer: 4- No specific therapy

Which of the following proteins is likely to be Cryptosporidium is found world wide,


abnormal in this condition? including the UK, where it has been
1- b-Myosin heavy chains demonstrated in supplies of drinking water.
The parasite is able to reproduce by sexual

Dr. Khalid Yusuf El-Zohry – Sohag Teaching Hospital (01118391123)


Ref MRCPass OE OE 2012 PasTest 2009 PassMedicine 2009 PasTest Exam ReviseMRCP 756
Elzohry MRCP Questions Bank (Part 1) – 2013 (For my personal use)

and asexual mechanisms and is transmitted by [ Q: 1686 ] PasTest 2009 - Basic


oocysts excreted in the faeces of the host. In Science
healthy individuals it is a self-limiting illness
Which of the following is the most likely
associated with watery diarrhoea, fever and
presentation of Staphylococcus aureus food
malaise that usually begin to resolve after 7-
poisoning?
10 days. In immunocompromised patients the
diarrhoea can be intractable and severe. 1- Severe vomiting 24-48 h after food
Diagnosis is made by microscopy of faeces; as ingestion
yet there is no effective antimicrobial 2- Watery diarrhoea without vomiting
intervention.
3- Dysentery

[ Q: 1685 ] PasTest 2009 - Basic 4- Severe vomiting 2-4 h after food ingestion
Science 5- Severe vomiting 6-12 h after food ingestion

A 42-year-old man is put on a proton-pump


inhibitor to suppress symptoms of Answer & Comments
oesophagitis. Answer: 4- Severe vomiting 2-4 h after food
The cell and membrane biology of the gastric ingestion
acid pump has which of the following
Some strains of S. aureus produce a heat-
features?
stable toxin (enterotoxin B), which causes
1- Histamine-stimulated acid production is massive secretion into the intestinal lumen
independent of the proton pump due predominantly to effects on the
2- The proton is exchanged with magnesium autonomic nervous system. It is a common
ions cause of gastrointestinal infection in Europe,
predominantly due to poor hygiene. Infection
3- Acetylcholine-stimulated acid production is
through dairy products such as fresh cream is
independent of the proton pump
a particular problem. Profuse vomiting begins
4- The proton pump spans the apical some 2-4 h after ingestion, and this is
membrane of the gastric parietal cell followed by watery diarrhoea, symptoms
5- The proton pump spans the basolateral usually resolve within 24 h. A similar
membrane of the gastric parietal cell syndrome is caused by Bacillus cereus, which
has two distinct subtypes, one causing
predominantly a diarrhoeal picture, the other
Answer & Comments
a picture characterised by vomiting.
Answer: 4- The proton pump spans the apical
membrane of the gastric parietal cell [ Q: 1687 ] PasTest 2009 - Basic
The H+-K+-ATPase is embedded in the apical Science
membrane. The channel is susceptible to A 22-year-old marathon runner is preparing
agents that bind to cysteine residues for a special event in 3 months' time and
(particularly 813 and 822). The proton pump is gradually increases his physical exercise.
the final common pathway of histamine and
Which of the following biochemical processes
acetylcholine-stimulated production, and
is likely to contribute most to energy creation
particularly explains the enhanced efficacy of
in his long-distance running?
proton-pump inhibitors in comparison to H2
antagonists or acetylcholine antagonists (eg 1- Ketogenesis
pirenzepine). 2- Creatine formation

Dr. Khalid Yusuf El-Zohry – Sohag Teaching Hospital (01118391123)


Ref MRCPass OE OE 2012 PasTest 2009 PassMedicine 2009 PasTest Exam ReviseMRCP 757
Elzohry MRCP Questions Bank (Part 1) – 2013 (For my personal use)

3- Lactate formation Answer & Comments


4- Fatty acid oxidation Answer: 5- Cephalexin
5- Glycogenesis
Cephalexin is the most likely of these
antibiotics to be associated with C. difficile
Answer & Comments diarrhoea. Oral agents that are most likely to
Answer: 4- Fatty acid oxidation be associated with C. difficile include
clindamycin, cephalosporins and augmentin.
Long-chain fatty acids and ketone bodies can Cephalosporins given intravenously are also
only be metabolised aerobically, and their likely to be associated with C. difficile. C.
catabolism takes place exclusively in the difficile is a common bacterium and is found
mitochondria. Two-carbon fragments are as a part of normal bowel flora in 3-5% of the
successively removed from long-chain fatty normal population. When it causes problems
acids (as long-chain fatty acid CoA) in a series with acute infection, two toxins are produced
of reactions known generically as b-oxidation (A is an enterotoxin and B is cytotoxic and
to form acetyl-CoA. Each turn of b-oxidation results in bloody diarrhoea). Symptoms range
generates sufficient reducing equivalents to from mild diarrhoea to severe colitis, when
regenerate five ATP molecules. Acetyl-CoA is the bowel mucosa may be covered by a
then oxidised through the tricarboxylic acid pseudomembrane. Antibiotics treatment of
cycle to regenerate more ATP, hence the choice is with oral metronidazole or
energy yield from long-chain fatty acid vancomycin.
oxidation is considerable.

Lipid-derived fuels are preferentially used by [ Q: 1689 ] PasTest 2009 - Basic


the heart. Their utilisation diminishes the use Science
of carbohydrate, thereby conserving glucose A 76-year-old diabetic woman presented with
for tissues that are obligatorily dependent on a non-healing ulcer on her right foot. Blood
carbohydrate as an energy source. culture samples grew methicillin-resistant
Staphylococcus aureus (MRSA).
[ Q: 1688 ] PasTest 2009 - Basic Which of the following antibiotics may be
Science considered in addition to iv vancomycin?
You review an elderly woman who has 1- Flucloxacillin
presented to the Emergency Department with
2- Metronidazole
dehydration resulting from severe diarrhoea.
She was prescribed antibiotics for a recent 3- Rifampicin
respiratory tract infection. 4- Ticarcillin
Which of the following drugs would be the 5- Ampicillin
most likely cause of Clostridium difficile
diarrhoea?
Answer & Comments
1- Penicillin V
Answer: 3- Rifampicin
2- Ciprofloxacin
Rifampicin may be used in combination for
3- Clarithromycin
MRSA infections. It should not be used as
4- Metronidazole monotherapy for MRSA, as resistance may
5- Cephalexin rapidly develop. Many hospital microbiology
departments recommend combination

Dr. Khalid Yusuf El-Zohry – Sohag Teaching Hospital (01118391123)


Ref MRCPass OE OE 2012 PasTest 2009 PassMedicine 2009 PasTest Exam ReviseMRCP 758
Elzohry MRCP Questions Bank (Part 1) – 2013 (For my personal use)

therapy with intravenous vancomycin and which follows days or months later, includes
teicoplanin. Other antibiotics to which there neurological symptoms, radiculopathies,
may be sensitivity include some macrolides, cardiac problems and arthritis. Isolation from
trimethoprim, sodium fusidate and blood cultures is rare, as is isolation from skin
aminoglycosides. Linezolid and combination lesions or (CSF). Clinical history with serology
therapy with quinupristin and dalfopristin are results are usually used to determine the
other possible options but these should be diagnosis. Early stage disease may be treated
reserved for patients with organisms that are with amoxicillin or doxycycline; later disease is
resistant to other combinations or for those usually treated with 2-4 weeks of iv
with tolerability issues to other combinations. benzylpenicillin or ceftriaxone.

[ Q: 1690 ] PasTest 2009 - Basic [ Q: 1691 ] PasTest 2009 - Basic


Science Science
A 45-year-old woman presented with pyrexia A 76-year-old lady presented with headache
of unknown origin. She had recently returned and neck stiffness. Lumbar puncture was done
from a hiking trip to America. Initially she and patient was started on iv ceftriaxone.
reported problems with a skin rash, this was Culture samples grew Listeria monocytogenes.
later followed by problems with arthralgia, What is the most appropriate treatment?
joint stiffness and pain. Blood, urine and
sputum were sent for culture. The 1- Change to ampicillin + gentamicin and treat
microbiological advice was to start a third- for 5-7 days total
generation cephalosporin. 2- Add ciprofloxacin
What is the most likely pathogen? 3- Continue iv ceftriaxone as monotherapy
1- Streptococcus pneumoniae 4- Change to ampicillin + gentamicin and treat
2- Legionella pneumophila for 10-14 days total

3- Borrelia burgdorferi 5- Add ampicillin to the ceftriaxone

4- Mycoplasma
Answer & Comments
5- Chlamydia pneumoniae
Answer: 4- Change to ampicillin + gentamicin
and treat for 10-14 days total
Answer & Comments
The recommended regime for meningitis
Answer: 3- Borrelia burgdorferi
caused by Listeria spp. is amoxicillin/ampicillin
The biggest clues here are the history of skin for 10-14 days. Listeria monocytogenes is an
rash and walking trip to the USA. Lyme environmental organism that is widely present
disease, a zoonosis affecting deer and other in soil and decaying matter and produces
wild animals, is caused by B. burgdorferi, a infection in both animals and man. Most
spirochaete. Infection is transmitted from commonly implicated foods that may cause
animal to man via a tick vector. The first stage infection include raw vegetables, soft cheeses
of the disease, occurring 7-10 days after and p‫أ‬¢t‫©أ‬s. The infection may cause serious
infection is characterised by erythaema problems in pregnant women, leading to
chronicum migrans. About 300 cases per year premature labour, septic abortion or stillbirth.
are reported in the UK. Initial stages of the Listeria in the elderly may result in
infection are associated with fever, headache, meningoencephalitis as in this case. Standard
myalgia and joint pain. The second stage, treatment is with ampicillin and gentamicin,

Dr. Khalid Yusuf El-Zohry – Sohag Teaching Hospital (01118391123)


Ref MRCPass OE OE 2012 PasTest 2009 PassMedicine 2009 PasTest Exam ReviseMRCP 759
Elzohry MRCP Questions Bank (Part 1) – 2013 (For my personal use)

as the organism is resistant to cephalosporins. [ Q: 1693 ] PasTest 2009 - Basic


Co-trimoxazole is an effective alternative Science
agent.
You see a 20-year-old girl with ataxia and
restricted eye movements in the clinic. She
[ Q: 1692 ] PasTest 2009 - Basic tells you that she developed bilateral hearing
Science loss at the age of nine years, retinitis
A man returning from holiday in South Africa pigmentosa at 10 years, and insulin-
presents with a painful ulcer over the sulcus of dependent diabetes mellitus at 15 years.
the penis with reactive inguinal lymph nodes. When she was 18 years old she had a
pacemaker inserted for complete heart block.
What is the most likely diagnosis?
What is your diagnosis?
1- Haemophilus ducreyi
1- Friedreich's ataxia
2- Primary syphilis
2- Kearns-Sayre syndrome
3- Herpes simplex
3- Myotonic dystrophy
4- Lymphogranuloma venereum
4- Spinocerebellar ataxia (SCA) type 7
5- Gonorrhoea
5- Usher syndrome
Answer & Comments
Answer & Comments
Answer: 1- Haemophilus ducreyi
Answer: 2- Kearns-Sayre syndrome
Chancroid is an acute sexually transmitted
infection (STI) caused by Haemophilus ducreyi, This patient's clinical features are typical of
it has very high prevalence in parts of Africa Kearns-Sayre syndrome, which is a
and Asia. The incubation period is 3-10 days in mitochondrial disorder characterised by
total. At the site of inoculation an rearrangements of mitochondrial DNA,
erythaematous papular lesion forms, which including deletions and duplications. The
later breaks down into an ulcer. The ulcer has clinical features that suggest a mitochondrial
a necrotic base, a ragged edge, bleeds easily disorder include any combination of
and causes pain. Co-infection with herpes and sensorineural hearing loss, retinitis
syphilis is common. Swabs should be taken pigmentosa, muscle weakness, ataxia,
from the ulcer; aspirates from local recurrent stroke, cardiomyopathy, insulin-
lymphadenopathy may also be useful in dependent diabetes mellitus, complete heart
forming a diagnosis. Gram stains of material block and lactic acidosis. The tissues involved
may show characteristic coccobacilli. Single- in mitochondrial disorders are those that
dose regimens include azithromycin 1 g as a depend heavily on oxidative phosphorylation
single dose or ceftriaxone 250 mg im. Other for energy production.
choices include a 3-day course of ciprofloxacin Kearns-Sayre syndrome is the only
or a 7-day course of erythromycin. Follow up mitochondrial disorder of the five possible
between day 3 and day 7 is advised. Partners answers provided. Other mitochondrial
should also be treated as asymptomatic disorders include myoclonic epilepsy and
carriage has been reported. ragged red fibres (MERRF); and mitochondrial
myopathy, encephalopathy, lactic acidosis and
stroke-like episodes (together known as
MELAS).

Dr. Khalid Yusuf El-Zohry – Sohag Teaching Hospital (01118391123)


Ref MRCPass OE OE 2012 PasTest 2009 PassMedicine 2009 PasTest Exam ReviseMRCP 760
Elzohry MRCP Questions Bank (Part 1) – 2013 (For my personal use)

[ Q: 1694 ] PasTest 2009 - Basic Answer & Comments


Science
Answer: 1- Helicase
Which of the following is a trinucleotide-
Nucleases are enzymes that degrade nucleic
repeat disorder?
acid molecules. Phosphodiesterases are a type
1- Amyotrophic lateral sclerosis of nuclease. Restriction endonucleases cut
2- Becker's muscular dystrophy DNA molecules at a limited number of specific
nucleotide sequences. Telomerases are
3- Kennedy's syndrome
enzymes that maintain the ends of eukaryotic
4- Sydenham's chorea chromosomes by synthesising telomeric
5- Wallenberg's syndrome repeat sequences.

Answer & Comments [ Q: 1696 ] PasTest 2009 - Basic


Science
Answer: 3- Kennedy's syndrome
Endothelin-1:
Amyotrophic lateral sclerosis is a form of
motor neurone disease where both upper and 1- Is a vascular cell-adhesion molecule
lower motor tracts are affected. Usually the 2- Is a polypeptide
arms have lower motor signs and the legs, 3- Is also known as ICAM-1
upper. Becker's muscular dystrophy is a milder
form than Duchenne's disease (in which 4- Is a heat-shock protein
dystrophin is undetectable). Kennedy's 5- Was formally known as IL-1
syndrome, also known as ‘X-linked bulbospinal
neuronopathy', is a trinucleotide-repeat Answer & Comments
disorder. Sydenham's chorea is sometimes
seen in patients with rheumatic fever and is Answer: 2- Is a polypeptide
also known as St Vitus' dance. Wallenberg's Endothelin-1 is a 21-amino acid polypeptide
syndrome is the ‘lateral medullary syndrome'. and is a highly potent vasoconstrictor. Levels
Other trinucleotide-repeat disorders include increase when endothelium is stressed, for
Huntington's chorea, fragile-X syndrome, example in trauma or oxidative stress. It plays
myotonic dystrophy and Friedreich's ataxia. a part in the modulation of vascular tone.
Endothelin-1 may also have a role in diseases
[ Q: 1695 ] PasTest 2009 - Basic such as Raynaud's. ICAM-1 is a cellular
Science adhesion molecule (I = intercellular), which is
increased during inflammation and by IL-1, an
Which enzyme breaks base pairs in a double-
interleukin. Heat-shock proteins (HSPs) are
stranded DNA molecule?
also increased during tissue stress and have
1- Helicase many different roles.
2- Nuclease
3- Phosphodiesterase [ Q: 1697 ] PasTest 2009 - Basic
Science
4- Restriction endonuclease
Which of the following neoplasms responds to
5- Telomerase
the specific tyrosine kinase inhibitor, imatinib?
1- Gastrointestinal stromal tumours
2- Acute myeloid leukaemia

Dr. Khalid Yusuf El-Zohry – Sohag Teaching Hospital (01118391123)


Ref MRCPass OE OE 2012 PasTest 2009 PassMedicine 2009 PasTest Exam ReviseMRCP 761
Elzohry MRCP Questions Bank (Part 1) – 2013 (For my personal use)

3- Philadelphia -ve chronic lymphoid cellular disassembly of the microtubule


leukaemia apparatus during mitosis. Imatinib is a specific
4- Acute lymphoid leukaemia tyrosine kinase inhibitor. 6-Mercaptopurine
(6-MP)-based analogues inhibit purine
5- Multiple myeloma synthesis.

Answer & Comments [ Q: 1699 ] PasTest 2009 - Basic


Answer: 1- Gastrointestinal stromal tumours Science

Specific tyrosine kinase inhibitors were Granuloma is seen in which of the following
developed after it was recognised that the conditions?
Philadelphia chromosome (translocation 9:22) 1- Syphilis
was associated with bcl-abl tyrosine kinase
2- Typhoid
overexpression. STI571, later known as
imatinib mesylate (Glivec), was therefore 3- Cholera
designed to treat chronic myeloid leukaemia, 4- Amoebiasis
but it was also found to be effective in
5- Shigellosis
gastrointestinal stromal tumours (GIST) -
previously known as leiomyosarcoma. Trials
also suggest that glivec may of benefit in some Answer & Comments
patients with CLL who are Philadelphia Answer: 1- Syphilis
chromosome positive.
A granuloma is a collection of macrophages:
giant cells as a nidus of chronic inflammation.
[ Q: 1698 ] PasTest 2009 - Basic
The centre may necrotise to form caseation,
Science
classically in tuberculosis. There is a long list of
Which of the following antineoplastic agents is infective and immunological conditions where
correctly paired with the site of action at the a granulomatous response may be seen,
cellular level? including tertiary syphilis, sarcoidosis and
1- Vinca alkaloids and abnormal microtubule Crohn's and Wegener's granulomatosis.
disassembly
[ Q: 1700 ] PasTest 2009 - Basic
2- Taxanes and abnormal microtubule
Science
assembly
3- Irinotecan and topoisomerase inhibition Which of the following statements is true
regarding the cellular and molecular
4- Imatinib mesylate and competitive
mechanisms that control apoptosis?
inhibition of nucleotide synthesis
1- The extrinsic pathway is initiated at the
5- Doxorubicin and competitive inhibition of
mitochondrial level
nucleotide synthesis
2- The intrinsic pathway is triggered at death
receptors on the cell surface
Answer & Comments
3- Abnormalities of caspase control are
Answer: 3- Irinotecan and topoisomerase associated with a range of lymphomas and
inhibition carcinomas
Vinca alkaloids inhibit the assembly of 4- Caspase 8 is an example of an executioner
microtubules, while taxanes inhibit the normal protein

Dr. Khalid Yusuf El-Zohry – Sohag Teaching Hospital (01118391123)


Ref MRCPass OE OE 2012 PasTest 2009 PassMedicine 2009 PasTest Exam ReviseMRCP 762
Elzohry MRCP Questions Bank (Part 1) – 2013 (For my personal use)

5- Death domains attract extracellular adapter lymphoma, and surviving with a number of
proteins carcinomas. Mutations of the p53 system are
seen in a variety of colorectal and other
Answer & Comments neoplasms. Finally, upregulation of other
mediating proteins, such as FLIP, can be seen
Answer: 3- Abnormalities of caspase control in certain tumours.
are associated with a range of lymphomas and
carcinomas
[ Q: 1701 ] PasTest 2009 - Basic
Apoptosis operates within two different Science
signalling pathways. The extrinsic pathway is
You are investigating periodic episodes of
initiated by ligand binding, the intrinsic
unexplained hypoglycaemia in a 23-year-old
pathway at the mitochondrial level. All death
nurse and request a C-peptide assay.
receptors (DR) are members of the tumour-
necrosis (TNF) superfamily and comprise a Which of the following statements is correct?
subfamily characterised by an intracellular 1- The level of C-peptide will be reduced in
domain, ie the death domain. Examples of insulinoma
known death receptors are CD95 (APO-1/Fas),
2- C-peptide cleavage is an example of post-
TRAIL (TNF- related, apoptosis-induced
transcriptional modification
ligand)-R1, TRAIL-R2, TNF-R1, DR3 and DR6.
3- C-peptide is attached to only the alpha
Death domains attract intracellular adaptor chain
proteins (eg CD95/FADD (MORT-1)), which in
turn attract procaspase 8. The latter is 4- The level of C-peptide will be unchanged in
factitious hypoglycaemia
modified into a complex of two small and two
large heterodimers, constituting active 5- C-peptide cleavage in an example of post-
caspase 8. This activates downstream translational modification
caspases, eg caspase 3 (a typical executioner
caspase) and results in the host cell Answer & Comments
undergoing characteristic apoptotic changes,
such as chromatin condensation and cell Answer: 5- C-peptide cleavage in an example
fragmentation (blebbing). of post-translational modification

Regulation of apoptosis is possible at various Post-transcriptional modification is what


levels. The number of death receptors can be happens to the mRNA, eg splicing. Post-
regulated (eg CD95 and TRAIL-R2 can be translational modification is what happens to
directly induced by p53); adapter proteins, eg the newly synthesised polypeptide hormone
the CD95–DISC complex, can be inactivated by precursor. The C-peptide connects both the
competing proteins such as FLIP; direct alpha and beta chains of insulin. The level of
inhibition of caspases and proteins of the BCL C-peptide is reduced in factitious
family can inhibit or stimulate apoptosis. hypoglycaemia, since exogenous insulin
suppresses the synthesis of the patient's own
A number of tumorigenesis mechanisms can insulin. C-peptide levels are increased in cases
result from disruption of the apoptosis of insulinoma.
pathway. At the mitochondrial level,
abnormalities of BCL-2 are associated with
[ Q: 1702 ] PasTest 2009 - Basic
follicular lymphoma and Apaf-1 with
Science
melanoma. Disruption of the caspase system
can also be seen, eg cIAP2 is seen in MALT You are reviewing a 54-year-old woman who

Dr. Khalid Yusuf El-Zohry – Sohag Teaching Hospital (01118391123)


Ref MRCPass OE OE 2012 PasTest 2009 PassMedicine 2009 PasTest Exam ReviseMRCP 763
Elzohry MRCP Questions Bank (Part 1) – 2013 (For my personal use)

is suffering from the blistering skin disorder 1- DNA gyrase


pemphigus vulgaris. 2- DNA ligase
How would the underlying pathological 3- DNA polymerase
process of this disorder best be described?
4- DNA photolyase
1- It is a disorder affecting ‘tight junctions'
between cells 5- DNA glycolyase

2- It is a disorder affecting claudins


Answer & Comments
3- It is a disorder affecting desmoglein-3
Answer: 2- DNA ligase
4- It is a disorder affecting desmoglein-1
5- It is a disorder affecting connexons DNA ligase synthesises phosphodiester bonds.

DNA gyrase is a type-2 topoisomerase of


Answer & Comments Escherichia coli.

Answer: 3- It is a disorder affecting DNA polymerase synthesises DNA on a


desmoglein-3 DNA/RNA template.

Adherent junctions are continuous on the DNA photolyase is a bacterial enzyme involved
basal side of cells. They contain cadherins and in photoreactivation repair.
are the major site of attachment of
DNA glycolyase takes part in base excision and
intracellular microfilaments. Intermediate
mismatch repair processes.
filaments are attached to desmosomes, which
are thickened areas of two apposed adjacent
cell membranes. In blistering skin disorders [ Q: 1704 ] PasTest 2009 - Basic
autoantibodies cause damage by attacking Science
proteins such as desmoglein-3 in pemphigus A 50-year-old obese patient has been
vulgaris, and desmoglein-1 in pemphigus complaining of gradually increasing firm
foliaceus. This leads to disruption of the nodules over the extensor surfaces of his
adherent junctions between cells and results fingers, hands and forearms. He also noticed
in blister formation. similar ear lesions.
Claudins are proteins involved in tight Which is the most likely enzyme when
junctions between cells. Tight junctions exist inhibited to cause a reduction of these
at the ends of margins adjacent to epithelial deposits?
cells (eg intestinal and renal cells) and form a
1- Urease
barrier to the movement of solutes and ions
across the epithelium. Connexons are protein 2- Xanthine oxidase
channels formed between two adjacent cells 3- Hypoxanthine phosphoribosyltransferase
to allow the passage of solutes up to a
4- Adenylate kinase
molecular weight of 1000 kDa.
5- Creatine kinase
[ Q: 1703 ] PasTest 2009 - Basic
Science Answer & Comments

Which enzyme synthesises phosphodiester Answer: 2- Xanthine oxidase


bonds as part of DNA replication, repair and Xanthine oxidase inhibitors are effective in
recombination processes? preventing acute gout. They act by reducing

Dr. Khalid Yusuf El-Zohry – Sohag Teaching Hospital (01118391123)


Ref MRCPass OE OE 2012 PasTest 2009 PassMedicine 2009 PasTest Exam ReviseMRCP 764
Elzohry MRCP Questions Bank (Part 1) – 2013 (For my personal use)

the serum urate concentration to a value Answer & Comments


below the solubility of sodium urate
Answer: 3- Ito cells
monohydrate in plasma so that tophaceous
deposits are mobilised and healing occurs. Ito cells (stellate cells) have a similar
This applies to the tophi in bones as well as morphology to fibroblasts but with the
elsewhere. addition of fat droplets, and are located within
the Disse space. A fine branching array of
[ Q: 1705 ] PasTest 2009 - Basic cytoplasmic processes circle sinusoids under
Science the endothelial cells. Ito cells contain most of
the body's stores of vitamin A.
The 3' to 5’ exonuclease activity possessed by
some DNA polymerases that enables the Retinoids are taken up from chylomicrons by
enzyme to replace misincorporated nucleotide specific receptors on hepatocytes and Ito cells,
is called what? and are stored within the latter. These cells
are central to the process of hepatic
1- Proofreading
fibrogenesis, responding to mediators
2- Replication released by parenchymal and Kupffer cells,
3- Recombination and causing their transformation into
myofibroblasts. Transforming growth factor-
4- Retrotransposition
beta (TGF-b ) initiates this process.
5- Splicing
The transformed Ito cell then stimulates the
production of extracellular matrix products
Answer & Comments
(collagen types I, III and IV, fibronectin,
Answer: 1- Proofreading laminin, chondroitin sulphate and hyaluronic
acid), in addition to products for matrix
Retrotransposition is transposition via an RNA degradation (collagenase, metalloproteinase
intermediate (transposition is the movement
and its inhibitor TIMP-1 (tissue inhibitor of
of a genetic element from one site to another metalloproteinase-1)).
in a DNA molecule). Splicing is the removal of
introns from the primary transcript of a The activation of Ito cells is also an important
discontinuous gene. mechanism for the control of sinusoidal
perfusion, through cytoskeletal actin within
[ Q: 1706 ] PasTest 2009 - Basic branching cellular processes beneath the
Science endothelium.

Following a liver biopsy on a 38-year-old


[ Q: 1707 ] PasTest 2009 - Basic
alcoholic man, the pathological examination
Science
reveals liver fibrosis.
Nitric oxide is derived from?
What is the most likely cell responsible for this
process? 1- Cyclic GMP
1- Hepatocytes 2- Endothelium-derived relaxing factor
2- Kupffer cells 3- GTN
3- Ito cells 4- l-Arginine
4- Endothelial cells 5- Nitrous oxide
5- Bile-duct epithelial cells

Dr. Khalid Yusuf El-Zohry – Sohag Teaching Hospital (01118391123)


Ref MRCPass OE OE 2012 PasTest 2009 PassMedicine 2009 PasTest Exam ReviseMRCP 765
Elzohry MRCP Questions Bank (Part 1) – 2013 (For my personal use)

Answer & Comments macrophage foam cells and smooth muscle


cells which migrate into the intima from the
Answer: 4- l-Arginine
media.
Nitric oxide (NO), which used to be known as
With time, these develop into raised fibrous
‘endothelium-derived relaxing factor', is a
(advanced) plaques, characterised by a dense
local cellular messenger. It is derived from l-
fibrous cap of connective tissue and smooth
arginine (an amino acid) by nitric oxide
muscle cells overlying a core containing
synthase. Its actions increase the levels of
necrotic material and lipid, mainly cholesterol
intracellular cGMP (cyclic guanosine
esters, which may form cholesterol crystals on
monophosphate), which has effects
histological section. The necrotic core is a
depending on which cell it is acting upon.
result of apoptosis and necrosis, increased
These include modulation of vascular tone
proteolytic activity and lipid accumulation.
(hence the therapeutic use of glyceryl
trinitrate, a synthetic compound) and Fibrous plaques also contain a large number
memory. NO has also been implicated in of macrophage foam cells, T cells and smooth
septic shock, adult respiratory distress muscle cells. This collection of cells,
syndrome (ARDS) and inflammation. Nitrous surrounding the necrotic core, promotes
oxide, also known as ‘laughing gas', is often plaque growth. The plaque undergoes
used in obstetrics and trauma for pain relief. vascularisation and microvessels develop in
connection with the artery's vasa vasorum.
[ Q: 1708 ] PasTest 2009 - Basic The new vessels provide a channel for the
Science access of inflammatory cells and may also lead
to intraplaque haemorrhage and thus weaken
A 50-year-old company director was admitted the plaque. Advanced atherosclerotic plaques
to hospital due to a myocardial infarction. He frequently accumulate calcium, due to the
was thrombolysed and received a coronary presence of proteins specialised in binding
artery bypass graft. The lesion leading to the calcium (osteocalcin, osteopontin, bone
myocardial infarction started many years ago morphogenic proteins).
with foam cells.
The advanced plaque is the substrate from
What is the most likely cell contributing to this
which the complicated plaque develops,
formation?
leading almost inevitably to clinical symptoms.
1- Endothelial cells The complicated plaque has a thin cap,
2- Fibroblasts especially at the shoulders or margins of the
lesion, and may contain ulcerations, fissures,
3- Lymphocytes
erosions or cracks. These provide sites of
4- Macrophages platelet adherence, aggregation and
5- Erythrocytes thrombosis. The thin fibrous cap may break or
tear leading to haemorrhage into the necrotic
core and thrombosis.
Answer & Comments
Answer: 4- Macrophages [ Q: 1709 ] PasTest 2009 - Basic
The earliest lesions of atherosclerosis are fatty Science
streaks. These consist of an accumulation of A 60-year-old obese smoker has been
lipid-engorged macrophages (foam cells). The admitted to hospital with chest pain due to
fatty streaks progress to intermediate lesions unstable angina. A nitrate infusion is started
(or transitional plaque), composed mainly of to relieve his chest pain.

Dr. Khalid Yusuf El-Zohry – Sohag Teaching Hospital (01118391123)


Ref MRCPass OE OE 2012 PasTest 2009 PassMedicine 2009 PasTest Exam ReviseMRCP 766
Elzohry MRCP Questions Bank (Part 1) – 2013 (For my personal use)

Which blood vessels are most sensitive to the of ATP.Riboflavin is absorbed from the upper
vasodilatatory effect of nitrates? gastrointestinal tract, there is no specific
1- Large arteries storage tissue, and it is excreted in the urine
either free or in small amounts of
2- Coronary arteries hydroxylated products.
3- Capillaries
4- Large veins [ Q: 1711 ] PasTest 2009 - Basic
Science
5- Pulmonary arteries
The gene for which of the following disorders
Answer & Comments is correctly paired with the stated
chromosome?
Answer: 4- Large veins
1- Duchenne muscular dystrophy: X
The antianginal and haemodynamic effects chromosome
are mediated predominantly by vasodilatation
2- Haemophilia A: Chromosome 11
of the venous system, leading to a fall in left
ventricular preload and cardiac work. 3- Variegate porphyria: X chromosome
4- Cystic fibrosis: Chromosome 1
[ Q: 1710 ] PasTest 2009 - Basic 5- Hereditary haemochromatosis :
Science Chromosome 7
For what metabolic process is riboflavin
required? Answer & Comments
1- The synthesis of the protein, collagen Answer: 1- Duchenne muscular dystrophy: X
2- The synthesis of glycogen from glucose chromosome
phosphate The gene for variegate porphyria (the PPOX
3- The synthesis of non-essential amino acids gene) has been located to chromosome 1. It
4- Prostaglandin synthesis has an autosomal-dominant pattern of
inheritance. Hereditary haemochromatosis is
5- The hydrogen-transfer chain in the an autosomal-recessive disorder with variable
mitochondria penetrance, and results from a mutation of
the HFE gene located on chromosome 6. This
Answer & Comments encodes a transport protein found in the
duodenum. 77.5% of patients have two copies
Answer: 5- The hydrogen-transfer chain in the
of the C282Y mutation, 4% have one copy,
mitochondria
while 6.5% have one or two copies of a
Riboflavin is a substituted alloxazine ring different mutation, H63D. Cystic fibrosis is
linked to ribotol, an alcohol derived from the caused by a mutation of the CFTR gene, which
pentose sugar ribose. It is light sensitive. is normally found on chromosome 7.
Dietary sources are liver, milk, cheese, eggs, Haemophilia, Duchenne and Becker forms of
some green vegetables, and beer. Other muscular dystrophy are all X-linked recessive
sources are yeast extracts (for example, conditions.
Marmite) and meat extracts (for example,
Bovril).Riboflavin is involved in redox [ Q: 1712 ] PasTest 2009 - Basic
processes involving the hydrogen-transfer Science
chain in the mitochondria and the production
Which one of the following muscles in the

Dr. Khalid Yusuf El-Zohry – Sohag Teaching Hospital (01118391123)


Ref MRCPass OE OE 2012 PasTest 2009 PassMedicine 2009 PasTest Exam ReviseMRCP 767
Elzohry MRCP Questions Bank (Part 1) – 2013 (For my personal use)

hand is supplied by the median nerve?


1- Lateral two interossei
2- Abductor pollicis brevis
3- Medial two lumbricales
4- Flexor pollicis longus
5- Extensor pollicis

Answer & Comments


Answer: 2- Abductor pollicis brevis

The median nerve supplies the following


structures in the hand:

The abductor pollicis brevis, flexor pollicis


brevis, opponens pollicis

The lateral two lumbricales

The skin of the lateral three and half fingers

The ulnar nerve supplies all the interossei and


the rest of the hand muscles.

Dr. Khalid Yusuf El-Zohry – Sohag Teaching Hospital (01118391123)


Ref MRCPass OE OE 2012 PasTest 2009 PassMedicine 2009 PasTest Exam ReviseMRCP 768

You might also like